You are on page 1of 266

© Star Publishing Pte Ltd. All rights reserved.

3A

Chow Wai Keung


General Editor: Ann Lui Yin Leng
Consultant: Prof Ling San
© Star Publishing Pte Ltd. All rights reserved.

Star Publishing Pte Ltd


115A Commonwealth Drive #05-12
Singapore 149596
Tel: (65) 64796800
Website: www.starpub.com.sg
Email: contactus@starpub.com.sg

© 2015 Star Publishing Pte Ltd

ISBN 978-981-4448-77-2

ALL RIGHTS RESERVED. No part of this


publication may be reproduced, stored in a
retrieval system, or transmitted in any form
or by any means, electronic, mechanical,
photocopying, recording or otherwise, without
the prior written permission of the publisher.

First published 2007


Second edition 2015
© Star Publishing Pte Ltd. All rights reserved.

Contents

Scheme-Of-Work 1

Notes On Teaching 21

Fully Worked Solutions


Chapter 1 Indices 31
Chapter 2 More about Quadratic Equations 56
Chapter 3 Linear Inequalities 104
Review Exercise 1 122
Chapter 4 Conditions of Congruence and Similarity 127
Chapter 5 Coordinate Geometry 163
Chapter 6 Functions and Graphs 198
Review Exercise 2 256

00 FWS3A(NA)Content.indd 3 12/11/14 4:03 PM


© Star Publishing Pte Ltd. All rights reserved.

Acknowledgements
The publisher would like to thank all the organisations and people who have given their
kind permission to reproduce their materials in this book.

The Geometer’s Sketchpad, Key Curriculum Press, 1150 65th Street, Emeryville, CA 94608,
www.keypress.com for permission to use the screenshots of activities created using
The Geometer’s Sketchpad.

While every effort has been made to contact the owners of copyrighted materials, we have
been unsuccessful in some cases. To these we offer our sincere apologies. We welcome any
information which would enable us to contact the copyright owners concerned. We will be
pleased to make the corrections in future editions of the book.

00 FWS3A(NA)Content.indd 4 12/11/14 4:03 PM


FWS3A(Exp_SOW(01-20).indd 1
[2nd Edition] Suggested Scheme of Work • Express

Strategies/Learning
Week Topic/Objectives Activities Resources Websites
Experiences (LE)

Term 1 Chapter 1 Chapters 1–6 in 3A


Week 1 Indices
1.1 – 1.2 Positive Indices and Laws
of Indices, Zero and Negative
Integral Indices
• state and apply the laws of To recognise laws of Indices for p.2 Class Activity 1 p.2–8 Textbook
indices positive integral indices E-Book http://www.homeschoolmath.
• state the definitions of zero LE1(b): Describe and compare Teacher’s Guide net/teaching/negative_zero_
and negative indices numbers written in index exponents.php
• simplify expressions involving form, e.g. “Which is greater,
integral indices 210 or 102 ?”, and explain how http://www.themathpage.
to multiply and divide such com/alg/negative-exponents.
numbers using the laws of htm
indices.
To describe and compare p.5 Class Activity 2
numbers written in index form
n
 a
To expand (ab)n and   , p.5 Class Activity 3
 b
where n is a positive integer
To explore the rules for zero p.8 Class Activity 4 p.8–11 Textbook
and negative integral indices. E-Book
Teacher’s Guide
© Star Publishing Pte Ltd. All rights reserved.

12/11/14 1:11 PM
2
Strategies/Learning
Week Topic/Objectives Activities Resources Websites
Experiences (LE)

Term 1 1.3 Fractional Indices

FWS3A(Exp_SOW(01-20).indd 2
Week 1 • state the definitions of Explaining the meaning of the p.12–16 Textbook http://education-portal.com/
fractional indices nth root of a number E-Book academy/lesson/working-
• simplify expressions involving Illustrating by using numerical with-fractional-powers.
Teacher’s Guide html#lesson
fractional indices examples, the relationship
between a and the nth root of a
positive number
Directing the above to the
definition of rational indices
Demonstrating the skills
of simplifying expressions
involving fractional indices

Term 1 1.4 – 1.5 Standard Form and


Week 2 Comparing Indices
• express numbers in standard LE N1(a): To investigate how p.19 Class Activity 5 p.16–23 Textbook http://www.youtube.com/
form standard form is used to watch?v=ceneATH5EZ8&
• understand the prefixes in SI represent very large and very hl=en-GB&gl=SG
units for very large and very small numbers in the real world
small numbers Conveying examples using the http://en.wikipedia.org/wiki/
prefixes in SI units for very Solar_System
• solve simple equations
involving indices large and very small numbers
http://www.
Introducing the technique p.24–25 Textbook enchantedlearning.com/
of equating indices to solve E-Book subjects/astronomy/planets/
equations with numbers having
the same base on both sides Teacher’s Guide
http://micro.magnet.
fsu.edu/primer/
java/scienceopticsu/
powersof10
p.23; Ex 1.4 (Qn 6)
NE Msg 1.
© Star Publishing Pte Ltd. All rights reserved.

12/11/14 1:11 PM
Strategies/Learning
Week Topic/Objectives Activities Resources Websites
Experiences (LE)

FWS3A(Exp_SOW(01-20).indd 3
Term 1 1.6 Calculation of Compound
Week 2 Interest
• Involving personal and LE N10(a): Examine and make p.154 in 3B p.25–31 Textbook
household finance (including sense of data in a variety of Problems in E-Book
simple and compound contexts (including real data Real-world Contexts
interest) presented in graphs, tables and B - Monthly Instalments Teacher’s Guide
formulae/equations)
Revision Ex 1
p.33–34
Go Further
p.34
Write in Your Journal
p.35

p.154

Term 1 Chapter 2
Week 3 More about Quadratic
Equations
2.1 – 2.2 Factorisation Method and
Graphical Method
• solve quadratic equations Revising the method of solving p.160 in 3B p.37–38 Textbook http://www.bbc.co.uk/
by factorisation method and a quadratic equation by Problems in schools/gcsebitesize/maths/
graphical method factorisation Real-world Contexts activities/flash/snap_
Illustrating the method of H - Skydiving Record p.39–47 Textbook factorising/snap_factorising.
solving a quadratic equation shtml
E-Book
by drawing its corresponding
graph Teacher’s Guide http://lgfl.skoool.co.uk/
viewdetails_ks4.aspx?id=390
p.160
© Star Publishing Pte Ltd. All rights reserved.

12/17/14 10:54 AM
4
Strategies/Learning
Week Topic/Objectives Activities Resources Websites
Experiences (LE)

Term 1 2.3 – 2.4 Completing the Square

FWS3A(Exp_SOW(01-20).indd 4
Week 3 Method and Quadratic Formula
• solve quadratic equations of LE N6(a): To investigate how b, p.48 Class Activity 1 p.47–52 Textbook http://www.univie.ac.at/
the form x2 + bx + c = 0 by c, p are related in the expansion future.media/moe/galerie/
the completing the square of (x + p)2 = x2 + bx + c gleich/gleich.html under
method Illustrating the method of Applet: Quadratic equations 1
• solve quadratic equations by solving a quadratic equation by
using the quadratic formula completing the square http://lgfl.skoool.co.uk/
keystage4.aspx?id=317
Deriving the quadratic formula p.53–56 Textbook
and illustrating how to apply it http://www.bbc.co.uk/
E-Book
to solve a quadratic equation schools/gcsebitesize/maths/
Teacher’s Guide activities/flash/snap_
LE N7(a): To discuss the cases p.55 Class Activity 2
when a quadratic equation factorising/snap_factorising.
ax2 + bx + c = 0 has two real shtml
distinct roots, two real equal
roots and no real roots

Term 1 2.5 – 2.6 Fractional Equations and


Week 4 Problems Involving Quadratic
Equations
• solve fractional equations Extending the method of p.57–60 Textbook http://lgfl.skoool.co.uk/
that can be transformed to solving quadratic equations to viewdetails_ks4.aspx?id=390
quadratic equations solve fractional equations that
• apply quadratic equations to can be transformed to quadratic
solve everyday problems equations
Demonstrating problem solving p.60–64 Textbook
involving application of E-Book
quadratic equations
Teacher’s Guide
© Star Publishing Pte Ltd. All rights reserved.

12/11/14 1:11 PM
Strategies/Learning
Week Topic/Objectives Activities Resources Websites
Experiences (LE)

FWS3A(Exp_SOW(01-20).indd 5
Term 1 2.7 Sketching the Graphs of
Week 5 Quadratic Functions
• sketching the graphs of LE N6(a): To explore the p.66–67 p.65–75 Textbook
quadratic functions given in characteristics of the quadratic Class Activity 3 E-Book
the form: graphs of y = (x – h)2 + k and
y = (x – p)2 + q y = – (x – h)2 + k, where h and k Teacher’s Guide
y = – (x – p)2 + q are constants
y = (x – a)(x – b) LE N6(a): To explore the p.69–70 Revision Ex 2
y = – (x – a)(x – b) characteristics of the quadratic Class Activity 4 p.78–79
graphs of y = (x – p)(x – q) and Go Further
y = – (x – p)(x – q), where p and p.80
q are constants
Write in Your Journal
LE N6(a): Use Graphmatica p.80
or other graphing software to
explore the characteristics of
various functions
LE N6(b): Work in groups to
match and justify sketches of
graphs with their respective
functions
© Star Publishing Pte Ltd. All rights reserved.

12/11/14 1:11 PM
6
Strategies/Learning
Week Topic/Objectives Activities Resources Websites
Experiences (LE)

Term 1 Chapter 3

FWS3A(Exp_SOW(01-20).indd 6
Week 6 Linear Inequalities
3.1 – 3.2 Basic Properties
of Inequalities and Linear
Inequalities in One Variable
• understand the basic To explore the addition and p.83 Class Activity 1 p.82–85 Textbook http://www.algebra-class.
properties of inequalities subtraction properties of p.85–88 Textbook com/inequalities.html
• solve linear inequalities in Inequalities
E-Book
one unknown Applying the properties to
solve linear inequalities in one Teacher’s Guide
• represent the solution of
a linear inequality on the unknown
number line Illustrating the representation
of the solution of an inequality
on a number line
LE N7(b): To compare p.86 Class Activity 2
the methods of solving a
linear inequality and the
corresponding linear equation,
and their solutions

Term 1 3.3 – 3.4 Simultaneous Linear


Week 7 Inequalities and Applications of
Linear Inequalities
• solve simultaneous linear Explaining the idea of p.86–91 Textbook http://www.algebra-class.
inequalities in one unknown simultaneous inequalities p.91–94 Textbook com/inequalities.html
• apply linear inequalities to Illustrating different cases of E-Book
solve everyday problems the solution of simultaneous
linear inequalities Teacher’s Guide
Developing the skill of problem p.94; Ex 3.4 (Qn 15)
solving involving application of NE Msg 6
inequalities Revision Ex 3
p.96–97
Go Further
p.97
Write in Your Journal
p.97
© Star Publishing Pte Ltd. All rights reserved.

12/11/14 1:11 PM
Strategies/Learning
Week Topic/Objectives Activities Resources Websites
Experiences (LE)

FWS3A(Exp_SOW(01-20).indd 7
Term 1 Revision for Chapter 1 to 3 p.98–99
Week 7 Review Exercise 1

Term 1 Chapter 4
Week 8 Conditions of Congruence
and Similarity
4.1 – 4.2 Congruent Triangles and
Similar Triangles
• state the conditions for two LE G2(a): To examine what p.102 Class Activity 1 p.101–111 Textbook http://www.youtube.com/
triangles to be congruent conditions are necessary for p.111–122 Textbook watch?hl=en-GB&
• determine if the two triangles congruence v=PO4zdUQyAjo&gl=SG
E-Book
are congruent LE G2(a): To determine what p.104 Class Activity 2
conditions are necessary for Teacher’s Guide
• state the conditions for two
triangles to be similar congruence
LE G2(a): To construct a right- p.107 Class Activity 3
angled triangle with a given
side and the hypotenuse, and
examine a condition for the
congruence of right-angled
triangles
LE G2(a): To construct p.112
two triangles with equal Class Activity 4 (GSP)
corresponding angles, and
determine a condition for
similar triangles
Le G2(a): To construct p.115
two triangles with sides in Class Activity 5 (GSP)
proportion, and determine a
condition for similar triangles

p.156 in 3B p.156
Problems in
Real-world Contexts
D - Scissor Lift
© Star Publishing Pte Ltd. All rights reserved.

12/17/14 10:45 AM
8
Strategies/Learning
Week Topic/Objectives Activities Resources Websites
Experiences (LE)

Term 1 4.3 Ratio of Areas of Similar Plane

FWS3A(Exp_SOW(01-20).indd 8
Week 9 Figures
• determine if two triangles are Prompting students to identify p.122–129 Textbook http://standards.nctm.org/
similar similar triangles and write E-Book document/eexamples/
• determine whether two plane down the reasons for similarity chap6/6.3/index.htm
Teacher’s Guide
figures or solids are similar Illustrating the applications
• state the relationship between of similar triangles to find
the ratio of areas and the unknown sides and angles
ratio of lengths of two similar To find the relationship p.123
plane figures between the ratio of sides and Class Activity 7 (GSP)
the ratio of areas of similar
plane figures

Term 1 4.4 Ratio of Volumes of Similar


Week 10 Solids
• determine whether two solids Demonstrating how to use the p.129–135 Textbook http://standards.nctm.org/
are similar above relationship to solve E-Book document/eexamples/
• state the relationship between application problems chap6/6.3/index.htm
Teacher’s Guide
the ratio of volumes and the To find the relationship p.129 Class Activity 8
ratio of lengths of two similar between the ratio of sides and
solids the ratio of their volumes
• apply the above property to Employing the above p.159 in 3B p. 159
solve problems relationship to solve application Problems in Revision Ex 4
problems Real-world Contexts p.137–139
G - Merlion Souvenirs
Go Further
p.140
Write in Your Journal
p.140
© Star Publishing Pte Ltd. All rights reserved.

12/11/14 1:11 PM
Strategies/Learning
Week Topic/Objectives Activities Resources Websites
Experiences (LE)

FWS3A(Exp_SOW(01-20).indd 9
Term 2 Chapter 5
Week 1 Coordinate Geometry
5.1 – 5.2 Length of a Line Segment
and Gradient of a Straight Line
• find the length of a line To find the length of a line p.142 Class Activity 1 p.142–149 Textbook www.active-maths.co.uk/
segment given the coordinates segment given the coordinates p.149–156 Textbook whiteboard/3geom/2points_
of its two end points of its two end points pythag.html
E-Book
• find the gradient of a straight Exploring the distance formula
line using the coordinates of for a line segment on a Teacher’s Guide
any two points on the line coordinate plane
Demonstrating the applications
of the formula
LE G6(a) & (b): To find the p.150 Class Activity 2
gradient of a line using the
coordinates of any two-points
on the line.
Applying the formula to find
the gradients of lines in various
positions
© Star Publishing Pte Ltd. All rights reserved.

12/11/14 1:11 PM
10
Strategies/Learning
Week Topic/Objectives Activities Resources Websites
Experiences (LE)

Term 2 5.3 – 5.4 Equation of a Straight

FWS3A(Exp_SOW(01-20).indd 10
Week 3 Line and Geometric Problems
Involving the Use of Calculators
• interpret and find the Introducing the gradient- p.156–161 Textbook http://www.math.com/school/
equation of a straight line intercept form of a straight line p.161–168 Textbook subject2/lessons/S2U4L2GL.
graph in the form y = mx + c Demonstrating the technique html
E-Book
• solve geometric problems of obtaining the equation of
involving the use of a straight line from the above Teacher’s Guide
coordinates form when sufficient conditions http://www.shodor.org/
are given interactivate/activities/
slopeslider/index.html

p.161; Example 10 and


Try It 10
NE Msg 1.
Teacher can add
activity related to the
MRT system.

Revision Ex 5
p.170-171
Go Further
p.172
Write in Your Journal
p.172
© Star Publishing Pte Ltd. All rights reserved.

12/11/14 1:11 PM
Strategies/Learning
Week Topic/Objectives Activities Resources Websites
Experiences (LE)

FWS3A(Exp_SOW(01-20).indd 11
Term 2 Chapter 6
Week 4 Functions and Graphs
6.1 Graphs of Power Functions
• draw the graph of the LE N6(a): To explore the p.174 Class Activity 1 p.174-182 Textbook http://www.regentsprep.org/
function y = axn for n = –2, characteristics of graphs of the E-Book regents/math/algebra/ac5/
–1, 0, 1, 2 and 3 power functions y = axn ,where TGraphFunc.htm
for n = 0, 1, 2 and 3 Teacher’s Guide
LE N6(a): To explore the p.178 Class Activity 2
characteristics of graphs of the
power functions y = axn,where
for n = –1, and –2,

Term 2 6.2 Graphs of Simple Sums of


Week 4 Power Functions
• draw the graph of the sum of Developing students’ skill in p.182–188 Textbook http://www.regentsprep.org/
not more than 3 of the power drawing the graphs of sums of E-Book regents/math/algebra/ac5/
functions power functions TGraphFunc.htm
Teacher’s Guide
Exploring various combinations
of power functions; graphing http://www.bbc.co.uk/
software may be employed schools/gcsebitesize/maths/
activities/flash/proportion_
graphs/proportion_graphs.
shtml
© Star Publishing Pte Ltd. All rights reserved.

11

12/11/14 1:11 PM
12
Strategies/Learning
Week Topic/Objectives Activities Resources Websites
Experiences (LE)

Term 2 6.3 – 6.4 Graphs of Exponential

FWS3A(Exp_SOW(01-20).indd 12
Week 5 Functions and Gradients of Curves
• draw the graph of an LE N6(a): To explore the p.190 Class Activity 3 p.189–195 Textbook http://www.bbc.co.uk/
exponential function y = kax, characteristics of graphs of the p.195–203 Textbook schools/gcsebitesize/maths/
where a is a positive integer exponential functions y = kax, activities/flash/proportion_
where k is a constant and a is a E-Book graphs/proportion_graphs.
• estimate the gradient of a
curve by drawing a tangent to positive integer. Teacher’s Guide shtml
the curve LE N6(b): To match and justify p.193 Class Activity 4
sketches of graphs with their http://www.regentsprep.org/
respective functions regents/math/algebra/ac5/
TGraphFunc.htm
Describing the idea of tangent
Demonstrating the technique of p.196 Class Activity 5
finding the gradient of a graph
by drawing a tangent
LE N6(a): To explore the
gradient of a curve at various
points on the curve

Term 2 6.5 Graphs in Practical Situations


Week 5 • interpret and analyse graphs Interpreting and analysing p.204–226 Textbook http://www.youtube.com/
in daily life data from tables and graphs, E-Book watch?v=wleTsegbZmQ
including distance-time graph
and speed-time graphs Teacher’s Guide
LE N6(b): To match graphs p.219 Class Activity 6
with descriptions of the real Revision Ex 6
situations that they represent p.229–232
Interpreting the solution in the Go Further
context of the problem p.233
Identifying assumptions made p.153 in 3B Write in Your Journal
and the limitations of the Problems in p.234
solution Real-world Contexts
A - Speed and Fuel
Efficiency of a Car
© Star Publishing Pte Ltd. All rights reserved.

12/16/14 4:25 PM
Strategies/Learning
Week Topic/Objectives Activities Resources Websites
Experiences (LE)

FWS3A(Exp_SOW(01-20).indd 13
Term 2 Review Exercise 2 p.235–236
Weeks Chapter 4 to 6
6–10 Revision
Mid year Examination

Term 3 Chapter 7 Chapters 7–10 in 3B


Week 1 Trigonometry
7.1 Further Problems of
Trigonometric Ratios of Acute
Angles
• state the definitions of Defining the trigonometric p.2-8 Textbook http://lgfl.skoool.co.uk/
trigonometric ratios of sine, ratios sine, cosine and tangent E-Book viewdetails_ks3.aspx?id=562
cosine and tangent for acute for acute angles
angles Teacher’s Guide http://lgfl.skoool.co.uk/
Demonstrating how to use a
• find a trigonometric ratio calculator to find the values of viewdetails_ks3.aspx?id=561
from a right-angled triangle trigonometric ratios, and how to
find the value of an angle when http://www.bbc.co.uk/
• obtain the values of schools/gcsebitesize/maths/
trigonometric ratios from a the value of a trigonometric
ratio of the angle is given activities/trigonometry.shtml
calculator
• use the trigonometric ratios Developing the skill of using http://lgfl.skoool.co.uk/
to calculate unknown sides trigonometric ratios to solve keystage4.aspx?id=317
and angles in a right-angled problems
triangle p.157 in 3B p.157
Problems in
Real-world Contexts
E - Singapore
Interlocking Puzzle
Coin Set (Require
the knowledge of Arc
Lengths and Sector
Areas)
© Star Publishing Pte Ltd. All rights reserved.

13

12/17/14 10:45 AM
14
Strategies/Learning
Week Topic/Objectives Activities Resources Websites
Experiences (LE)

Term 3 7.2 – 7.3 Sine and Cosine Ratios

FWS3A(Exp_SOW(01-20).indd 14
Week 1 of Obtuse Angles and Formula for
the Area of a Triangle
• state the definitions of sine Extending the definitions p.11 Class Activity 1 p.9–16 Textbook http://lgfl.skoool.co.uk/
and cosine functions for of sine and cosine to obtuse p.17–23 Textbook keystage4.aspx?id=317
obtuse angles angles
E-Book http://www.nipissingu.
• find the values of sine and Exploring the area formula of p.17 Class Activity 2
cosine for obtuse angles triangle, Area = bc sin A Teacher’s Guide ca/calculus/tutorials/
trigonometry.html
• find the area of a triangle Demonstrating the applications http://aleph0.clarku.
using the formula: of the above formula edu/~djoyce/java/trig/
Area = bc sin A area.html

Term 3 7.4 – 7.5 The Sine Rule and The


Week 2 Cosine Rule
• state the sine rule Deriving the sine rule using the p.23–30 Textbook http://lgfl.skoool.co.uk/
• apply the sine rule to solve area of a triangle p.30–37 Textbook keystage4.aspx?id=317
problems Demonstrating the applications E-Book
of the sine rule to solve http://www.bbc.co.uk/
• state the cosine rule Teacher’s Guide schools/gcsebitesize/maths/
problems, including the
• apply the cosine rule to solve ambiguous case shapeh/areaofatrianglerev1.
p.37; Ex 7.5 (Qn 16)
problems shtml
Deriving the formula of the NE Msg 1.
cosine rule
Demonstrating the applications p.34 Class Activity 3 Revision Ex 7
of the cosine rule to solve p.39-41
problems Go Further
LE G4(b): To use the sine rule p.42
and cosine rule to find the Write in Your Journal
relationships between the sides p.42
and angles of a triangle
© Star Publishing Pte Ltd. All rights reserved.

12/11/14 1:11 PM
Strategies/Learning
Week Topic/Objectives Activities Resources Websites
Experiences (LE)

FWS3A(Exp)_SOW(01-20).indd 15
Term 3 Chapter 8
Weeks Applications of
3–4 Trigonometry
8.1 – 8.2 Bearing Problems, Angles
of Elevation and Depression p.44–50 Textbook www.active-maths.co.uk/
• solve problems involving Discussing the measurement of p.50–55 Textbook whiteboard/3measure/meas_
bearings and navigation bearings bearing1.html
E-Book
• understand the idea of angles Applying the trigonometric
of elevation and depression rules to solve bearing problems Teacher’s Guide http://www.algebralab.
org/Word/Word.
• solve problems involving Introducing the idea of angles Chapter 8 opener;
aspx?file=Trigonometry_
angles of elevation and of elevation and depression NE Msg 1
AnglesElevDepression.xml
depression Demonstrating the skills in p.161 in 3B p.161
solving problems involving Problems in
angles of elevation and Real-world Contexts
depression I - Rooftop View

Term 3 8.3 Simple Three-Dimensional


Weeks Problems
5–6 • apply trigonometry to LE G4 (a): To visualise height, p.55 Class Activity 1 p.55–68 Textbook http://www.youtube.com/
solve simple 3-dimensional north direction, right-angled E-Book results?search_query=solving
problems in our daily life triangle, etc. from 2D drawings +3+dimensional+figure+mat
of 3D situations Teacher’s Guide h+problems
Revision Ex 8 (Videos on solving
p.70–72 3 Dimensional figures
related problems)
Go Further
p.73
Write in Your Journal
p.73
© Star Publishing Pte Ltd. All rights reserved.

15

12/17/14 12:02 PM
16
Strategies/Learning
Week Topic/Objectives Activities Resources Websites
Experiences (LE)

Term 3 Chapter 9

FWS3A(Exp_SOW(01-20).indd 16
Week 7 Arc Lengths and Sector
Areas
9.1 – 9.2 Arc Lengths, Sector Areas
and Radian Measure
• understand the relationship LE 5(a): To find the arc length p.79 p.79–84 Textbook http://www.coolmath.com/
between arc length and angle in a circle by considering it as a Class Activity 1 (GSP) p.85–92 Textbook reference/circles-geometry.
subtended by arc fraction of the circumference of html
the circle E-Book
• understand the relationship
between sector area of a Applying the above relationship Teacher’s Guide
circle and angle subtended by to solve application problems
arc LE 5(a): To find the sector area p.85
• calculate the arc length and of a circle by considering it as a Class Activity 2 (GSP)
sector area of a circle fraction of the area of the circle
• calculate the area of a Applying the above relationship
segment to find the areas of sectors and
the areas of segments
© Star Publishing Pte Ltd. All rights reserved.

12/11/14 1:11 PM
Strategies/Learning
Week Topic/Objectives Activities Resources Websites
Experiences (LE)

FWS3A(Exp_SOW(01-20).indd 17
Term 3 9.3 – 9.4 Radian Measure,
Week 8 Formulae in Radian Measure
• understand radian measure of Describing the definition of p.92–98 Textbook http://www.themathpage.
angle radian p.98–103 Textbook com/aTrig/arc-length.htm
• convert between radians and LE G5(b): To visualise the size p.93 Class Activity 3 E-Book
degrees of an angle of 1 radian, and
estimate the size of angles in Teacher’s Guide
• express arc length and sector
area formulae using radian radians
measure Showing the conversion Revision Ex 9
• apply the formulae to solve between radians and degrees p.105–106
problems Deriving the formulae for the Go Further
arc length and sector area when p.107
the angle at centre is in radians Write in Your Journal
Demonstrating the applications p.158 in 3B p.108
of the above formulae in Problems in
solving problems Real-world Contexts
F - 400 m Standard
Running Track p.158

p.162 in 3B p.162
Problems in
Real-world Contexts
J - Candle Pack
© Star Publishing Pte Ltd. All rights reserved.

17

12/11/14 1:11 PM
18
Strategies/Learning
Week Topic/Objectives Activities Resources Websites
Experiences (LE)

Term 3 Chapter 10

FWS3A(Exp_SOW(01-20).indd 18
Weeks Properties of Circles
9–10
10.1 – 10.2 Chords of a Circle,
Angles in a Circle
• recognise equal chords are LE G3(a): To investigate the p.111 Class Activity 1 p.110–119 Textbook http://learningcenter.
equidistant from the centre properties of chords using p.120–130 Textbook dynamicgeometry.com/
• recognise the perpendicular paper folding x38.xml
E-Book
bisector of a chord passes LE G3(a): To explore the p.115 Class Activity 2
through the centre of a circle property of equal chord in a Teacher’s Guide
• apply the above properties to circle using paper folding
solve problems Encouraging students to write
• recognise the angle at the down the reasons for steps in
centre is twice the angle at their workings
the circumference LE G3(b): To find the p.120
• recognise an angle in a relationship between the Class Activity 3 (GSP)
semicircle is a right angle angle at centre and angle at
circumference in a circle
• recognise the angles in the
same segment are equal LE G3(b): To find the property p.125
of angles in the same segment Class Activity 4 (GSP)
Grouping students and asking
them whether they can present
some proofs for the properties

Term 4 10.3 Angles in Opposite Segments


Week 1 • apply the above properties to LE G3(b): To find the p.131 p.131–136 Textbook www.mathsnet.net/dynamic/
solve problems relationship between the anlges Class Activity 5 (GSP) E-Book circle3.html
• state the properties of angles in the opposite segments of a
circle Teacher’s Guide
in opposite segments
Demonstrating the applications
of the properties to solve
problems
Showing the applications of
the above property in problem
solving
© Star Publishing Pte Ltd. All rights reserved.

12/11/14 1:11 PM
Strategies/Learning
Week Topic/Objectives Activities Resources Websites
Experiences (LE)

FWS3A(Exp)_SOW(01-20).indd 19
Term 4 10.4 Tangents to a Circle
Week 2 • draw a tangent to a circle Conveying the idea of tangent p.136–145 Textbook www.mathsnet.net/dynamic/
• understand that a tangent to to a circle E-Book circle1.html
a circle is perpendicular to LE G3(b): To find the p.139 Teacher’s Guide
the radius of the circle at the properties of tangents to a Class Activity 6 (GSP)
point of contact circle from an external point Revision Ex 10
• apply the above properties to Illustrating how to use those p.147-148
solve problems properties to solve problems Go Further
• recognise tangents from an p.149
external point are equal in Write in Your Journal
length p.149
• recognise the line joining an
external point to the centre
of a circle bisects the angle
between the tangents from the
external point
• apply the above properties to
solve problems

Term 4 Review Exercise 4 p.155 in 3B p.150–152


Weeks Chapter 9 to 10 Problems in p.155
3–5 Real-world Contexts
Revision
C - Food Waste in
End of Year Examination
Singapore
© Star Publishing Pte Ltd. All rights reserved.

19

12/17/14 12:17 PM
© Star Publishing Pte Ltd. All rights reserved.

20

FWS3A(Exp_SOW(01-20).indd 20 12/11/14 1:11 PM


© Star Publishing Pte Ltd. All rights reserved.

Notes On Teaching

Chapter 1 Indices

Suggested Approach
The idea of exponents has been introduced in Secondary 1. In this chapter, students will study the laws of indices
and the meaning of negative and rational indices. Activities to explore the laws of indices for numbers should be
provided prior to stating the laws to students.

It is advisable to introduce the laws one at a time; each law should be supported by immediate simple examples.
This would make it easier for students to master them individually. After teaching rational indices, some examples
involving the successive use of the laws will help in reinforcement.

1.1 Positive Indices and Laws of Indices


It would be better if simple numerical examples are used to illustrate the laws. Students should not go into
tedious manipulations of complicated expressions.

Some students may be confused with the addition and multiplication of indices. A common error could be
(2x)3 = 2x3 or (a3)4 = a7.

1.2 Zero and Negative Integral Indices


It should be emphasised that zero to the power of zero is undefined. Students should develop the habit of
expressing the answer in positive indices. Care must be taken to distinguish between (–3) –2 and –3–2.

1.3 Fractional Indices


In general, for fractional indices, the base should be positive. Some calculators cannot handle the calculation
4
3
of, say, (–5) .

1.4 Standard Form


In teaching standard form, students could be asked to quote some examples from other disciplines or books.
Some students may have difficulty in manipulating numbers in standard form using a calculator. More
hands-on experience is required to allow them to build up confidence.

1.5 Comparing Indices


Students should be reminded to compare the indices of two numbers only when the numbers are expressed
with the same base. Only simple equations are required in this section.

1.6 Calculation of Compound Interest


The concept of compound interest should be introduced after students have mastered the concept of simple
interest. We can then illustrate the calculation of compound interest as a repeat process of calculating simple
interest on an updated sum. It is suggested that teachers use a spreadsheet program to compare the calculation
of simple interest and compound interest. Students can explore to derive the compound interest formula from
the spreadsheet.

21

FWS3A(Exp)_NTeach(21-28).indd 21 12/10/14 5:54 PM


© Star Publishing Pte Ltd. All rights reserved.

Chapter 2 More about Quadratic Equations

Suggested Approach
The method of solving a quadratic equation by factorisation may be revised. Students can then be given examples,
such as x2 + 3x – 5 = 0, that cannot be solved by the factorisation method. Hence, the graphical method, completing
the square method and the quadratic formula can be introduced. After they become familiar with the different
techniques of solving quadratic equations, teachers may ask them to reflect what they have learnt and how they
would choose an effective method to solve a given quadratic equation.

Fractional equations that can be transformed into quadratic equations may be introduced by using practical examples.
Students should be reminded to check whether the roots obtained from a quadratic equation satisfy the original
fractional equation.

This chapter concludes with application problems requiring the solution of quadratic equations. Students may find
it more interesting if they can relate the problems to their experience and daily life. Interdisciplinary examples can
also be provided.

2.1 Factorisation Method


This section is a revision of solving a quadratic equation by factorisation learnt in Secondary 2. Students
should be reminded to move all the terms to the left hand side before factorisation. They should be aware that
applying factorisation in a smart way can yield a quicker solution.

2.2 Graphical Method


Students should understand that the roots of the quadratic equation ax2 + bx + c = 0 are the x-intercepts of the
graph of y = ax2 + bx + c. Sufficient examples should be provided to illustrate that a quadratic equation may
have two unequal real roots, two equal real roots or no real roots. In order to ease the job of the students, the
interval of values of x for the graph of y = ax2 + bx + c is given.

2.3 Completing the Square Method


Students should be aware that, in general, the roots of a quadratic equation obtained from the graphical method
are approximations only. The exact roots can be obtained by algebraic methods. They should understand the
idea of completing the square. It is sufficient for them to attempt problems like x2 + 3x – 5 = 0, where the
coefficient of x2 is unity.
2 2
Some common errors in completing the square of x2 + bx are: adding b2 instead of  b  and adding –  b 
 2  2
when b is negative.

2.4 Quadratic Formula


Students should understand that the quadratic formula is derived from the completing the square method. In
applying the formula, students should write down the values of a, b and c first before substituting them in
the formula. They should recognise that when the value under the square root sign is negative, the formula
yields no real roots.

When applying the quadratic formula, a common student error is to ignore the signs of the coefficients or
mishandle them.

Notes On Teaching
22

FWS3A(Exp)_NTeach(21-28).indd 22 12/10/14 5:54 PM


© Star Publishing Pte Ltd. All rights reserved.

2.5 Fractional Equations


1
In Secondary 1, students learnt the method of solving fractional equations, such as = 8, which can be
(3x – 4)
transformed into a linear equation. In this section, we include some examples involving fractional equations
that can be transformed into quadratic equations. To accomplish this, students should multiply both sides of
the original fractional equation by the LCM of the denominators of the terms in the equation.

2.6 Problems Involving Quadratic Equations


In illustrating a word problem example to students, ensure that they understand the problem and spend some
time in asking questions like “What should be taken as an unknown x?” and “What is the relationship between
x and certain quantities?”, etc. Some students may omit to state the unit of the unknown quantity.

2.7 Sketching the Graphs of Quadratic Functions


For the graph of y = ±(x – h)2 + k, students should recognise that the vertex is at (h, k) and the line of symmetry
is x = h. They should also be able to determine if the graph is open upwards or downwards. It is not necessary
to introduce the graphs of y = ± a(x – h)2 + k, where a ≠ 1.

For the graph of y = ± (x – p)(x – q), students should be able to find its x-intercepts and y-intercept. It is not
necessary to introduce the graphs of y = ± a(x – p)(x – q), where a ≠ 1.

23

FWS3A(Exp)_NTeach(21-28).indd 23 12/10/14 5:54 PM


© Star Publishing Pte Ltd. All rights reserved.

Chapter 3 Linear Inequalities

Suggested Approach
The theme of this chapter is to introduce the basic properties of inequalities and apply them to solving inequality
problems. Students may find it more interesting if they explore the basic properties by themselves. These properties
may also be demonstrated with numerical examples.

The solutions of linear inequalities in one unknown can be vividly represented on a number line with the use of
coloured chalk or shaded regions. In particular, for the solution of simultaneous inequalities in one unknown, students
can easily visualise the solution on a number line using shading.

3.1 Basic Properties of Inequalities


The property that “For any two numbers a and b, one and only one of the following relationships holds:
a , b, a = b or a . b.” is called the law of trichotomy. The property that “If a , b and b , c, then a , c.”
is called the transitive law.

Students should realise that transposing terms in an inequality is basically the same as that for an equality.
However, when an inequality is multiplied by a negative number, the inequality sign should be reversed.

3.2 Linear Inequalities in One Unknown


A varied form of linear inequalities involving ., ,, > and < should be introduced. Students should distinguish
whether the solution of an inequality includes an end point or not. Care must be taken when an inequality is
multiplied by a negative number.

3.3 Simultaneous Linear Inequalities


Some students may have difficulty in finding the solution for a system of simultaneous linear inequalities. It is
advisable to draw the individual solutions on a draft sheet and locate the common region of individual solutions.

When a , b, the simultaneous inequalities x , a and x . b have no solution. Some students may have the
wrong notion that the solution consists of two intervals.

3.4 Applications of Linear Inequalities


Students should realise the physical limitations of a variable in an inequality. For instance, some variables
may only take integer values or positive values.

Notes On Teaching
24

FWS3A(Exp)_NTeach(21-28).indd 24 12/10/14 5:54 PM


© Star Publishing Pte Ltd. All rights reserved.

Chapter 4 Conditions of Congruence and Similarity

Suggested Approach
The concepts of congruence and similarity were introduced in Secondary 2. This chapter will concentrate on the
congruence and similarity of triangles and their applications.

Students will discover the conditions of congruence and similarity through construction. For a condition of congruence,
each student should construct only one triangle for one set of conditions. Then congruent triangles in different
orientations and layouts could be used to consolidate the concept. Investigation using dilation and projection may
give a clearer idea of the conditions of similarity.

As the emphasis at this level is on the applications of congruence and similarity, we shall use the intuitive approach
to introduce the relationships of the ratio of areas of similar plane figures and the ratio of volumes of similar solids.
However, students should bear in mind the need for writing down the reasons in the working steps.

4.1 Congruent Triangles


The AAS condition of congruence of triangles is considered to be a variation of the ASA condition. Teachers
may help students understand that, when two pairs of corresponding angles of two triangles are equal, their
third pair of angles are also equal. Some applications of conditions for congruent triangles should be involved.

4.2 Similar Triangles


The conditions of similarity of triangles can be established by comparing triangles under dilation. Teachers
may ask students to compare and contrast between the conditions of similarity and congruence of triangles.
A variety of application problems may arouse students’ interest, and hence may provide them a better
understanding of this topic.

4.3 Ratio of Areas of Similar Plane Figures


Teachers may present some similar plane figures to students and ask them to explore the relationships between
their linear dimensions and areas. Some students may have difficulty in finding the ratio of linear dimensions
of two similar figures when the ratio of their areas are given.

Suppose D and E are points on the sides AB and AC of nABC respectively such that DE is parallel to the side AB.
A common error made by students is to assume AD : DB = DE : BC.

4.4 Ratio of Volumes of Similar Solids


Teachers may extend the idea of similar plane figures to similar solids. Students may get a better grasp of
the concept if numerical examples of similar cuboids, spheres and cones are considered. The relationship
between the ratio of volumes of similar solids and the ratio of their corresponding linear dimensions can then
be established by actual measurements.

25

FWS3A(Exp)_NTeach(21-28).indd 25 12/10/14 5:54 PM


© Star Publishing Pte Ltd. All rights reserved.

Chapter 5 Coordinate Geometry

Suggested Approach
Teachers could guide students in discovering the formula for the distance of two points on a coordinate plane. Some
activities can be used to bridge the definition of the gradient of a line, “rise over run”, to the gradient formula. It is
not necessary to mention the conditions for parallel lines and perpendicular lines.

We may use the intuitive approach to introduce the gradient-intercept form of a straight line. Using the gradient-
intercept form, students should be able to find the equation of a straight line when a point on the line and the gradient
of the line are given, or when two points on the line are given.

As an application, they may be asked to prove some geometric properties using the materials learned in this chapter.

5.1 Length of a Line Segment


The formula for the length of a line segment may be considered as an application of Pythagoras’ theorem.
Initially, students may be asked to find the distance between two horizontal points and the distance between
two vertical points. It would then be easier for them to discover the formula by themselves. They may be
reminded to leave the intermediate answers of distances in surd form, if appropriate, and asked to give the
final answers correct to 3 significant figures.

5.2 Gradient of a Straight Line


Students should note that any two points chosen on a straight line will give the same value for the gradient of
the line. In applying the formula, it does not matter which point is taken first. Students should be reminded
that the gradient of a vertical line is undefined.

5.3 Equation of a Straight Line


Students are expected to apply the gradient-intercept form to find the equation of a straight line from
various given conditions. They should also be able to find the gradient and intercepts of a line in the form
ax + by + c = 0.

5.4 Geometric Problems Involving the Use of Coordinates


Students are expected to use coordinates to solve some geometic problems in their daily life or prove some
geometric properties.

Notes On Teaching
26

FWS3A(Exp)_NTeach(21-28).indd 26 12/10/14 5:54 PM


© Star Publishing Pte Ltd. All rights reserved.

Chapter 6 Functions and Graphs

Suggested Approach
Students should already have the skill of drawing linear graphs and the skill of plotting some points and joining them
by a smooth curve to obtain a quadratic graph. We will extend these skills to the sketching of quadratic functions
of some particular forms.

Students are guided to understand the general properties of power functions and exponential functions through their
own exploration. It is desirable that students master the basic skill of drawing these functions by pencil and paper
before they proceed with any computer software for further exploration.

They should recognise that some optimization problems can be solved by drawing suitable graphs. They should also
know the limitation of accuracy of values obtained from a graph.

The idea of the tangent to a graph is introduced intuitively. Students should be encouraged to draw tangents to a
curve in various forms of curvature. As the formula for finding the gradient of a line has not been taught, the value
of the gradient of a tangent is found by using rise over run.

For the distance-time graphs and speed-time graphs, we shall use the straight line segment graphs to explain the
basic features of these graphs first. Then, we can discuss some close to real-life situations which involve curves in
these graphs.

Speed-time graphs can be confusing for students. They may need lots of practise on interpreting these graphs.

6.1 Graphs of Power Functions


1 1
Students should know that the graphs of y = and y = are undefined at x = 0. After sufficient practice,
x x2
they should be able to describe the shapes of the graphs of power functions. Some students may have difficulty
in deciding appropriate horizontal and vertical scales in drawing graphs.

6.2 Graphs of Sums of Power Functions


This section is confined to dealing with the sums of not more than three power functions. Students should
be reminded to determine the values of x for which a function is defined and to observe the behaviour of the
function at those points close to where the function is undefined.

6.3 Graphs of Exponential Functions


This section is confined to the treatment of the graph of exponential functions of the form y = kax, where
a is a positive integer greater than 1 and k is a constant. Students should know that, for k . 0, the graph of
y = kax grows rapidly for large positive values of x while the graph tends to zero when x tends to negative
infinity. Several exponential graphs can be drawn on the same diagram to compare the shapes of the graphs
for different values of k and a.

6.4 Gradients of Curves


Some students may have the misconception that a tangent to a curve always touches the curve only at one
point. They should recognise that the value of the gradient of a curve at a point obtained by the graphical
method is only an approximation.

27

FWS3A(Exp)_NTeach(21-28).indd 27 12/10/14 5:54 PM


© Star Publishing Pte Ltd. All rights reserved.

6.5 Graphs in Practical Situations


The main features of a distance-time-graph is the gradient of the graph, which represents speed. On the other
hand, the gradient of a speed-time graph represents acceleration while the area under a speed-time graph
represents the total distance travelled. Students are encouraged to draw both the distance-time graph and
speed-time graph of a situation and compare their features.

Notes On Teaching
28

FWS3A(Exp)_NTeach(21-28).indd 28 12/10/14 5:54 PM


© Star Publishing Pte Ltd. All rights reserved.

FULLY
WORKED
SOLUTIONS

29

01 FWS3A(Exp)_ch1.indd 29 12/16/14 12:48 PM


© Star Publishing Pte Ltd. All rights reserved.

Chapter 1 Indices
30

01 FWS3A(Exp)_ch1.indd 30 12/16/14 12:48 PM


© Star Publishing Pte Ltd. All rights reserved.

1 Indices

Class Activity 1
Objective: To recognise laws of indices for positive integral indices.
Express the following manipulations in index notation. The first one in each set has been done for you. Assume that m and n
are positive integers, m . n, and a  0.

1. (a) 72 3 73 = (7 3 7) 3 (7 3 7 3 7) = 75

(b) 23 25 = (2 3 2 2) 3 (2 2 2 2 2) =2
8
3 3 3 3 3 3

(c) a4 3 a3 = (a a a a) 3 (a a a) =a
7
3 3 3 3 3


(d) am 3 a2 = (a 3 a 3 a ... to m factors) 3 (a 3 a) =a
m+2


(a 3 a 3 a ... to m factors) 3

(e) a 3 a = (a 3 a 3 a ... to n factors) =a
m n m+n

53535353535
2. (a) 56 ÷ 52 = = 54
535

(b) 87 ÷ 84 = 8383838383838
=8
3

8383838

c 3c 3c 3c 3c   

(c) c ÷c = =c
5 3 c 3c 3c 2

b 3 b 3 b ... to m factors
(d) b ÷b =
m 2 b 3b
=b
m–2
, where m . 2.

a 3 a 3 a ... to m factors
, where m . n.
(e) am ÷ an = =a
a 3 a 3 a ... to n factors m–n

3. (a) (92)3 = 92 3 92 3 92 = 96

(b) (35)4 = 35 3 35 3 35 3 35 =3
20


(c) (x2)5 = x 3 x 3 x 3 x 3 x =x
2 2 2 2 2 7

m m m
(d) ( ym)3 = y 3 y 3 y =y
3m

m m m
(e) (am)n = a 3 a 3 a ... to n factors =a
mn

31

01 FWS3A(Exp)_ch1.indd 31 12/16/14 12:48 PM


© Star Publishing Pte Ltd. All rights reserved.

Class Activity 2
Objective: To describe and compare numbers written in index form.
1. (a) Describe in your own words the meanings of 210 and 102, and write down each number as a product of its base.
210 is read as “the 10th power of 2” or “2 to the power of 10”.

102 is read as “the 2nd power of 10” or “10 to the power of 2”.

210 = 2 3 2 3 2 3 2 3 2 3 2 3 2 3 2 3 2 3 2

102 = 10 3 10

(b) Split the factors of 210 in (a) into two equal groups and hence express 210 as a2, where a is an integer.
210 = (2 3 2 3 2 3 2 3 2) 3 (2 3 2 3 2 3 2 3 2) = (25)2 = 322, where a is 32.

(c) Without using a calculator, determine which number is greater, 210 or 102? Explain your answer.
210. It has a larger base.

2. Without using a calculator, determine which number is greater, 7 4 or 47? Explain your answer.
(Hint: 47 = (4 3 4) 3 (4 3 4) 3 (4 3 2) 3 (2 3 4).)
47 is greater. 47 = 162 3 82 = 1282. 74 = 72 3 72 = 492.

3. Without using a calculator, determine which number is greater, 36 or 93? Explain your answer.
(Hint: Express 93 as a product of 3.)
They are equal. 93 = (3 3 3)3 = 33 3 33 = 36.

4. Without using a calculator, determine which number is greater, 211 or 84? Explain your answer.
(Hint: Express 84 as a product of 2.)
84 is greater. 84 = (2 3 2 3 2)4 = 212.

5. Observing from the above results, if x and y are positive integers greater than 1 and x , y, is xy , yx or x y . yx ? Explain
your answer. (Use some numerical examples for x and y to see whether there are exceptional cases for your conclusion.)
If x = 2 and y = 3, then x y , y x ; if x = 2 and y = 4, then x y = y x. For other values of x and y, we have x y . y x since the number x y with a large index

is greater than the number of y x with a small index.

Class Activity 3 n
 a
Objective: To expand (ab)n and  
b
, where n is a positive integer.
Rewrite each of the following as a product or a quotient in index notation. The first one in each set has been done for you.
Assume that n is a positive integer, k ≠ 0, y ≠ 0 and b ≠ 0.
1. (a) (4 3 5)2 = (4 3 5) 3 (4 3 5) = 42 3 52

(b) (2 3 c)3 = (2 3 c) 3 (2 3 c) 3 (2 3 c) =2 c
3 3
3

(c) ( pq)4 = (pq) 3 (pq) 3 (pq) 3 (pq) =p q
4 4


(d) (ab)n = (ab) 3 (ab) 3 (ab) ... to n factors =a b
n n

Chapter 1 Indices
32

01 FWS3A(Exp)_ch1.indd 32 12/16/14 12:48 PM


© Star Publishing Pte Ltd. All rights reserved.

2
 2 2 2 22
2. (a)   = 3 =
3 3 3 3
2

3 3
7
(b)  7
= =
7 7 7
3 3
  k k k
k k
3

5 5
 x x x x x x x
(c)  y  = y
3 3 3 3
y y y y = 5
y
n a
n

(d)  a =
a
3
a
3
a
=
  b b b ... to n factors
b b
n

Class Activity 4
Objective: To explore the rules for zero and negative integral indices.
Task
Suppose a  0 and we assume the following:
• a 0, a –1, a –2, a –3, … exist
• the laws of indices are valid for these powers of a
• n is a positive integer

Let us perform the following calculations.

1. (a) By the laws of indices, By direct calculation,



25
25 ÷ 25 = 25 ÷ 25 = 2 3 2 3 2 3 2 3 2
2 3 2 3
5
2 23232
– = 1

=2
5 5


=2
0

\ 20 = 1

(b) By the laws of indices, By direct calculation,


3

a a 3a 3a
a3 ÷ a3 = 3 a3 ÷ a3 =
a a 3a 3a
– = 1

=a
3 3


=a
0

\ a0 = 1

73 73 73737
2. (a) 73 ÷ 75 = 5 5 =
7 7 737373737
– 1
= 7 =
3 5

7
2

=7
–2

1
∴ 7–2 =
7
2

– a4 a3a3a3a
(b) a4 ÷ a7 = a
4 7
=
a
7
a3a3a3a3a3a3a
–3
=a
= 1

a
3

∴ a –3 = 1
a
3

33

01 FWS3A(Exp)_ch1.indd 33 12/16/14 12:48 PM


© Star Publishing Pte Ltd. All rights reserved.

Class Activity 5
Objective: To investigate how standard form is used to represent very large and very small numbers in the real
world.

1. Copy and complete the following table. Discuss which representation you would use for each quantity.

Real-world Examples Expression in words Integer/Decimal Standard Form

World population in 2012 7.06 billion 7 060 000 000 7.06 3 109

Diameter of a hydrogen atom 0.1 nanometre 0.000 000 000 1 m 1 3 10 –10 m

Land area of Asia 44.6 square mega 44 600 000 km2 4.46 3 107 km
2

kilometre

Mass of 1 cm3 of salt 2.165 milli kilogram 0.002 165 kg 2.165 3 10 –3 kg

Electricity consumption in 41.8 gigakilowatt hour 4180 000 000 kWh 4.18 3 1010 kWh
Singapore in 2011
Wavelength of red light 0.72 micrometre 0.000 000 72 m 7.20 3 10 –7 m
Shortest distance between the
360 thousand km 360 000 km 3.06 3 10 –5 km
Moon and the Earth
Size of a human cell 2.5 micrometre 0.000 002 5 m 2.5 3 10 –6 m

2. Suggest one example of a very large number and one example of a very small number, and express the quantities in standard
form.
The age of the Earth, mass of a dust particle. 4.6 3 109, 7.53 3 10–10.

Chapter 1 Indices
34

01 FWS3A(Exp)_ch1.indd 34 12/16/14 12:48 PM


© Star Publishing Pte Ltd. All rights reserved.

Try It! Solution


Section 1.1 (a) 110 – 3 –2 = 1 –
1
2
3
1. Simplify the following. 1
(a) a10 3 a8 =1–
9
(b) (5x3y4) 3 (6xy7) 8
=
9
Solution
(a) a10 3 a8 = a10 + 8 = a18 (b) 5–9 3 56 = 5–9 + 6
(b) (5x3y4) 3 (6xy7) = (5 3 6)x3 + 1 y4 + 7 = 5–3
= 30x4y11 =
1
53
1
2. Simplify the following. =
125
(a) q17 ÷ q11
(b) (–30r 9s10) ÷ (–6r 8s3) (c) (2–9)5 ÷ (26) –7 = 2– 45 ÷ 2– 42
= 2– 45 + 42
Solution = 2–3
(a) q17 ÷ q11 = q17 – 11 = q6 =
1
3
2
(b) (–30r9s10) ÷ (–6r8s3) = –30
3 r9 – 8 s10 – 3 1
–6 =
8
= 5rs 7

 7
–1 1
(d)   =  
6
3. Simplify the following.
 6 7
(a) (b9)5 =
6
(b) (c6)8 ÷ (c3)10 7

Solution 6. Simplify the following and express your answers in


(a) (b ) = b
9 5 9×5
=b 45 positive index notation.
(a) (m – 4 n3) –5
(b) (c ) ÷ (c )
6 8 3 10
= c6 3 8 ÷ c3 3 10
= c48 – 30 (b) ( p6 q–2)( p–6 q–3)
= c18 (c) (5x– 4y8) ÷ (15x–3y6)

Solution
4. Simplify the following. (a) (m – 4 n3) –5 = m20 n–15
(a) ( p2 q5)4 3 ( p3 q)6 m 20
4 2 =
n
15
 3a 5   –9 a 3 
(b)  3  ÷  6 
b b
(b) ( p6 q–2)( p– 6 q–3) = p 0 q–5
1
Solution =
q5
(a) (p q )
2 5 4
3 ( p q) = p q
3 6 8 20
3 p q
18 6

1 –1 2
= p8 + 18 q20 + 6 (c) (5x– 4y8) ÷ (15x–3y6) = x y
3
= p26 q26 y 2
=
3x
4 2
 3a 5   –9 a 3  81a 20 81a 6
(b) ÷  6  = ÷ 12 Section 1.3
 b 3  b b
12
b

=
81 20 – 6 12 – 12
a b 7. Evaluate the following without using a calculator.
81 4 5
1024
(a) 16 (b)
=a 14 3125

Solution
Section 1.2 (a) Let r = 16
4

5. Evaluate the following without using a calculator. r4 = 16


(a) 110 – 3 –2 (b) 5–9 3 56 r4 = 24
\ r=2
–1
 7
i.e. 16 = 2
4
(c) (2–9)5 ÷ (26) –7 (d)
 6 

35

01 FWS3A(Exp)_ch1.indd 35 12/16/14 12:48 PM


© Star Publishing Pte Ltd. All rights reserved.

5
1024 11. Express each of the following as an integer or a decimal.
(b) Let x= . (a) 7.92 3 104
3125
1024 (b) 2.68 3 10 –5
x5 =
3125
45 Solution
x5 =
5
5
(a) 7.92 3 104 = 79 200
4 (b) 2.68 3 10 –5 = 0.000 026 8
\ x=
5
5
1024 4
i.e.
3125
= 12. Evaluate the following and express your answers in
5
standard form.
(a) 3.9 3 106 + 4.7 3 107
8. Evaluate the following without using a calculator. (b) 8.25 3 10 –13 – 6.01 3 10 –14
5 4

(a) 16 4 (b) 125 3
Solution
(a) 3.9 3 106 + 4.7 3 107
Solution = 0.39 3 107 + 4.7 3 107
5
= (0.39 + 4.7) 3 107
(a) 16 = ( 16 )5 4 4
= 5.09 3 107
= 25
(b) 8.25 3 10 –13 – 6.01 3 10 –14
= 32
= 8.25 3 10 –13 – 0.601 3 10 –13

4 = (8.25 – 0.601) 3 10 –13
3
(b) 125 3 = ( 125 ) – 4 = 7.649 3 10 –13
= 5– 4
=
1 13. Evaluate the following and express your answers in
54 standard form.
=
1 (a) (9.5 3 1013) 3 (4 3 10 –6)
625 (b) (2.38 3 10 –5) ÷ (1.4 3 104)

9. Simplify the following, expressing your answers in Solution


positive index notation. (a) (9.5 3 1013) 3 (4 3 10 –6)
5 1 = (9.5 3 4) 3 (1013 3 10 –6)
(a) (p )6 2
3 p 3 = 38 3 107
3 5 9 = 3.8 3 108

(b) (x y ) ÷ (x 3 y 2)
2 –4
(b) (2.38 3 10 –5) ÷ (1.4 3 104)
= (2.38 ÷ 1.4) 3 (10 –5 ÷ 104)
Solution = 1.7 3 10 –9
5 1 5 1

(a) ( p ) 6 2
p =p ×p
3 3 3
3
= p2 14. Evaluate the following, expressing your answers in
standard form.
3 5 9 3 5 9 2.37 3 10 – 6
– – –4 +
(a)
(b) (x y ) ÷ (x y
2 –4 3 2
) =x 2 3
y 2
4.18 3 10 5 + 9.08 3 10 3
1 1 8
– 7.38 10
(b)
3
=x y
6 2
–2
1.69 3 10
1

y 2
= 1
Solution
x6
2.37 3 10 – 6
(a) = 5.55 3 10 –2
Section 1.4 4.18 3 10 5 + 9.08 3 10 3
10. Express the following numbers in standard form.
(a) 378 000
8
7.38 10
(b) = 2.09 3 105
3

(b) 0.000 092 1.69 3 10 –2

Solution
(a) 378 000 = 3.78 3 105
(b) 0.000 092 = 9.2 3 10 –5

Chapter 1 Indices
36

01 FWS3A(Exp)_ch1.indd 36 12/16/14 12:48 PM


© Star Publishing Pte Ltd. All rights reserved.

15. The storage capacity of a memory card in a digital camera is Solution


16 GB. Suppose each digital photo takes up 4 MB, how Let $Pn be the sum at the beginning of the nth year,
many photos can be stored on the memory card? $In be the interest in the nth year and
$An be the amount after n years.
Solution
The required number of photos =
16 GB Year
4 MB Pn ($) In ($) An ($)
(n)
9
16 3 10
= 6 1 1500 1500 3 10% = 150 1500(1 + 10%)
4 3 10
= 4 3 10 3
2 1500(1 + 10%) 1500(1 + 10%) 3 10% 1500(1 + 10%)2
= 165
Section 1.5
3 1500(1 + 10%)2 1500(1 + 10%)2 3 10% 1500(1 + 10%)3
16. Find the value of x if 3x = 243. = 181.5

Solution Total interest in 3 years = $1996.50 – $1500


3x = 243 = $496.50
3x = 35 Amount after 3 years = $1500(1 + 10%)3
\ x = 5 = $1996.50
1
17. Find the value of x if 253x + 1 = . 20. A sum of $8400 is invested at the interest rate of 9% p.a. for
125
6 years. Find the total interest accumulated if it is
Solution
compounded
(a) yearly, (b) monthly.
1
253x + 1 =
125
1
(52)3x + 1 = 3
Solution
6
5
(a) Amount after 6 years = $8400 3  1 +
9 

5 =52(3x + 1) –3
100 
2(3x + 1) = –3 = $14 087.64
6x + 2 = –3 \ required compound interest
6x = –5 = $14 087.64 – $8400
5 = $5687.64
x=–
6
(b) Interest rate per month = 9% ÷ 12
3
= %
18. David borrowed $6000 from a bank at the interest rate 4
of 5% per annum. Interest is calculated using simple Number of months in 6 years = 12 3 6
interest. Find = 72
(a) the interest charged in 2 years, Amount after 72 months
(b) the amount to repay in 8 months. 72
= $8400 3  1 +
3 1 
3 
4 100 
Solution
= $14 385.44 (correct to 2 d.p.)
(a) Interest in 1 year = $6000 3 5% \ required compound interest
Interest in 2 years = $6000 3 5% 3 2 = $14 385.44 – $8400
= $600 = $5985.44
8
(b) 8 months = years
12
8 21. A company took up a loan from a bank at 9% p.a.
Interest in 8 months = $6000 5%
compound interest. The loan became $80 000 after
3 3
12
Amount = $6000 + $200 6 years. What was the original loan amount?
= $6200
Solution
Let the original loan amount be $x.
19. Farah borrows $1500 from a bank at the rate of 10% 6
per annum compound interest. Find the total interest x  1 +
9 
 = 80 000
100 
accumulated and the total amount she would owe the
80 000
bank after 3 years. x = 6
1.09
= $47 701.39 (correct to 2 d.p.)
The original loan amount is $47 701.39.

37

01 FWS3A(Exp)_ch1.indd 37 12/16/14 12:48 PM


© Star Publishing Pte Ltd. All rights reserved.

Exercise 1.1 Solution


Level 1 (a) 5a6 3 3a2 = (5 3 3)(a6 + 2)
= 15a8
1. Simplify the following, expressing your answers in index
notation. (b) (–8b3) 3 (4b9) = (–8 3 4)(b3 + 9)
(a) 35 3 37 (b) 210 ÷ 26 = –32b12
(c) (5 )
2 4
(d) (73)7 (c) 24c8 ÷ (– 6c5) = [24 ÷ (– 6)](c8 – 5)
(e) (2 3 7) 8
(f ) (32 3 11)4 = – 4c3
4 6
 3
(g)   (h)  7 
 3  (d) (–72d 17) ÷ (– 4d 13) = [(–72) ÷ (– 4)](d 17 – 13)
5 2
= 18d 4
Solution (e) (–2e3)4 = (–2)4 e12
(a) 35 3 37 = 35 + 7 = 312 = 16e12
(b) 210 ÷ 26 = 210 – 6 = 24 (f ) (–3r6 s4)5 = (–3)5r30 s20
(c) (52)4 = 52 3 4 = 58 = –243r30 s20
(d) (73)7 = 73 3 7 = 721
(e) (4 3 7)8 = 28 3 78
6
 –3u 2  (–3) 6 (u 2 ) 6
(g)   =
(f ) (32 3 11)4 = 38 3 114 v  v6
4
 3 35 729u 12
(g)   = =
v
6
5 135
6 3
76  4x 7  (4 3 ) x 21
( h)  7 
= (h) =
 2 3  2
18  8 
 y  y
24

64 x 21
=
2. Simplify the following. y
24

(a) a4 3 a7 (b) c 8 ÷ c5 Level 2


(c) (e4)5 (d) ( f 7)3 4. Compare each of the following pairs of numbers and
(e) ( pq)9 (f ) (r2 s3)4 state which is greater. Explain your answer.
6  v5 
11 (a) 718 and 714 (b) 36 and 53
(g) t
  (h)  4 (c) 2 and 3
12 8
(d) 433 and 922
u w 

Solution Solution
(a) a 3a =a
4 7
=a 4+7 11
(a) 718.
(b) c 8 ÷ c5 = c 8 – 5 = c 3 It has a greater index.
(c) (e4)5 = e4 3 5 = e20 (b) 36.
(d) ( f 7)3 = f 7 3 3 = f 21 36 = 32 3 3.
(e) ( pq)9 = p9 q9 It has a larger base.
(f ) (r2 s3)4 = (r2)4(s3)4 = r8s12 (c) 38.
6
(g) t =
t6 38 = (32)4 = 94.
  6
u u 212 = (23)4 = 84.
 v5 
11
v 5 × 11 v 55 It has a larger base.
( h) = =
 4
w  w
4 × 11
w
44
(d) 922.
433 = 22 3 11 3 3 = (23)22 = 822.
It has a larger base.
3. Simplify the following.
(a) 5a6 3 3a2
5. Simplify the following.
(b) (–8b3) 3 (4b9)
(a) a7b4 3 a5b6
(c) 24c8 ÷ (–6c5)
(b) c12 d 9 ÷ c10 d 8
(d) (–72d 17) ÷ (–4d 13)
(c) (3e2f ) 3 (4e9f 7)
(e) (–2e3)4
(d) (15g8h7) ÷ (5g2 h4)
(f ) (–3r6 s4)5
6 (e) (–18m3n5) ÷ (24m2 n)
 –3u 2 
(g)    8 p7   q9 
v  (f ) – 2  3 – 4 
3  q   p 
 4x 7 
(h)  8   3s 9   
(g) ÷  – 6s8
5
 y   3  
 t   t 

Chapter 1 Indices
38

01 FWS3A(Exp)_ch1.indd 38 12/16/14 12:48 PM


© Star Publishing Pte Ltd. All rights reserved.

(h) (u3v4)5 3 (u2v7)3 4 5 2


 2x 4   3x 2   6y5 
(d) − ÷  3  3
(i) (w3x4)6 ÷ (–w2 x3)5  y 

 y   x 
7
 2y3  2
(–2) 4 x 16 3 5x 10 6 2 y 10
( j) 
 z6 
 3  10  = ÷ 3
y4 y
15
 z  x
2
 4y 
 (–2) × 6  16 – 10 – 2 15 + 10 – 4
4 2
=   x y
Solution 35
(a) a7b4 3 a5 b 6 = a7 + 5 b4 + 6 =
64 4 21
xy
= a12 b10 27

(b) c12 d  9 ÷ c10 d  8 = c12 – 10 d  9 – 8


= c2 d Level 3
(c) (3e2f ) 3 (4e9f 7) = (3 3 4)e2 + 9f 1 + 7 7. A person sent 213 copies of junk mail via email. Each
= 12e11f 8 email consisted of 211 bytes of data. How many bytes of
data were sent altogether? Express your answer in index
(d) (15g8h7) ÷ (5g2 h4) = (15 ÷ 5)g8 – 2 h7 – 4 notation.
= 3g6 h3
 –18  3 – 2 5 – 1 Solution
(e) (–18m3n5) ÷ (24m2 n) = m n

24  The amount of data = 211 3 213 bytes
= – 3 mn4 = 224 bytes
4

(f )
 8 p7   q9 
= (–8)(–1)p7 – 4 q9 – 2 8. The number of thousands of bacteria in a culture after
– 2  3 – 4 
 q   p  n hours is 2n + 3.
= 8p3 q7 (a) Find the initial number of thousands of bacteria.
 3s 9    (b) After n hours, the bacteria are divided into 2 n – 1
(g) ÷  – 6s8 =  3  9–5 8–3
5

 3    – 6 
s t   groups. Find the number of thousands of bacteria
 t   t 
1 in each group.
= – s4t  5
2
Solution
(h) (u3v4)5 3 (u2v7)3 = u15v20 3 u6v21
(a) When n = 0,
= u21v41
number of thousands of bacteria at the beginning
(i) (w3x4)6 ÷ (–w2 x3)5 = w18 x24 ÷ (–w10 x15) = 2n + 3
= –w8 x9 = 20 + 3
 2y3 
7
 z6 
2 = 23
2 7 y 21 z 12
( j) 
 z 
 3  10  = 3 = 8
 4y  z 4 y
7 2 20
The initial number of bacteria is 8000.
= 8yz5
(b) When n = 2,
the bacteria are divided into 22 – 1 = 21 = 2 groups.
6. Simplify the following. Number of bacteria in each group
(a) 5a4 3 3a2 ÷ a3 = 8000 3 2
= 16 000
(b) (b3 c8) ÷ (b2 c5) 3 (b7c11)
(c) (–5mn3)3(–6m2 n9) ÷ (–3m2 n4)2
4 5 2 9. (a) Write down the values of the numbers 32, 33, 34, 35,
 3x 2 
(d)
 2x 4 
−  ÷  3  ×
 6y5 
  36, 37, 38 and 39.
 y   y  x  (b) Describe the pattern of the last digit of the number
3n when n = 1, 2, 3, ... .
Solution
(a) 5a4 3 3a2 ÷ a3 = (5 3 3)a4 + 2 – 3 Solution
= 15a3 (a) 32 = 9
(b) (b3 c8) ÷ (b2 c5) 3 (b7c11) = b3 – 2 + 7c8 – 5 + 11 33 = 27
= b8 c14 34 = 81
35 = 243
(c) (–5mn3)3(–6m2 n9) ÷ (–3m2 n4)2 36 = 729
= (–5)3m3n9(–6)m2 n9 ÷ (–3)2 m4 n8 37 = 2187
= (–53 3 –6 ÷ 32)m3 + 2 – 4 n9 + 9 – 8 38 = 6561
250 39 = 19 683
= mn10
3 (b) The pattern of the last digit is 9, 7, 1, 3 when
n = 1, 2, 3, 4 and repeats itself when n = 6, 7, 8, 9.

39

01 FWS3A(Exp)_ch1.indd 39 12/16/14 12:48 PM


© Star Publishing Pte Ltd. All rights reserved.

10.
Rohanna received an email informing her that good  1
–3

fortune would come to her if she forwarded the email (b)   = 43


4
to 5 friends. Consider that Rohanna sent the email to 5 = 64
of her friends in the first round, while these friends sent 
–1
2
(c)
7
the email to 5 of their own friends in the second round   + 90 = +1
7 2
and so on. Let y be the maximum possible number of 9
persons receiving the email in the nth round. =
2
(a) Find the value of y when
(d) 8– 6 3 84 = 8–2
(i) n = 2,
1
(ii) n = 3. = 2
8
(b) Express y in terms of n.
1
(c) Find the least value of n when y is more than one =
64
million.
(e) 7–10 ÷ 7–10 = 70
Solution
=1
(a) (i) When n = 2, y = 5 3 5 = 25. (f ) (411)0 = 1
The maximum possible numbers of persons (g) (3 –9)4 × (3 –5) –7 = 3 –36 3 335
receiving the email in the 2nd round is 25. = 3 –1
(ii) When n = 3, y = 5 3 5 3 5 = 125. 1
=
The maximum possible number of persons 3
receiving the email in the 3rd round is 125.
(h) (25) – 6 ÷ (2–9)3 = 2–30 ÷ 2–27
(b) The expression is y = 5n.
= 2–3
(c) 5n . 1 000 000.
1
\ n , 10 = 3
2
The least value of n is 9. 1
=
8
11. If (am)n = a231, where m and n are positive integers greater
than 1, and m  n, find two possible sets of values of m 2. Simplify the following, expressing your answers in
and n. positive index notation.
Solution (a) a2 ÷ a5 (b) b3 3 b– 4
(am)n = a231 (c) (3 –2 e4) –1 (d) ( f –5g 3)2
amn = a231
–2
 5 –3 
\ mn = 231 (e)  2  (f ) (  p–2 q3)(  p– 4 q–3)
 h 
mn = 3 3 7 3 11 (g) (r 3s–1)(r–9s6) (h) (x4y3) ÷ (x–5y8)
Hence, the possible values of m and n are:
(i) m = 3, n = 77; Solution
(ii) m = 7, n = 33; (a) a2 ÷ a5 = a –3
(iii) m = 11, n = 21. 1
= 3
a
(b) b3 3 b– 4 = b–1
Exercise 1.2 =
1
b
Level 1
1. Evaluate the following without using a calculator. (c) (3 e ) = 32 e– 4
–2 4 –1

9
 1
–2 = 4
(a) 130 – 2–3 (b)   e
(d)
4
 2
–1 ( f –5g3)2 = f –10g6
(c)   + 90 (d) 8–6 3 84 g6
7 =
f
10
(e) 7 –10 ÷ 7 –10 (f ) (411)0 –2
 5 –3  56
(g) (3 – 9)4 3 (3 –5) – 7 (h) (25) – 6 ÷ (2– 9)3 (e)  2  =
 h  h–
4

Solution
= 15 625h4

(a) 130 – 2–3 = 1 –


1 (f ) ( p–2 q3)( p– 4 q–3) = p– 6 q0
3
2 1
1 =
=1– p6
8
7
=
8

Chapter 1 Indices
40

01 FWS3A(Exp)_ch1.indd 40 12/16/14 12:48 PM


© Star Publishing Pte Ltd. All rights reserved.

(g) (r3s–1)(r–9s6) = r–6 s5  x3 


–4
 x –12 
s 5 (h) (x–2y) –5 3  2 = (x10y–5) 3  –8 
= y   y 
r
6
= x–2y3
(h) (x y ) ÷ (x y ) = x y
4 3 –5 8 9 –5
=
y3
x x
9 2
=
y
5

Level 3
Level 2 4. In a graphic animation, the length and breadth
3. Simplify the following, expressing your answers in of a rectangle at time t seconds are a 4 t –2 cm and
positive index notation. a5t–1 cm respectively. Find the area of the rectangle at time
(a) 1 ÷ (52 a – 6) –2 t seconds. Express your answer in positive index notation.
(b) (3 – 4 b5) –1(3 –1b2)2
Solution
( 4 c 3 d –1 ) 4
(c) Area of the rectangle at time t seconds
2 –1 c –2 d 5 = (a4t  –2)(a5t  –1) cm2
–3 –1
 e3   e–4  = a9t  –3 cm2
(d)  2 3   a9
 f   f  = cm2
t
3

(e) (m n ) ÷ (m n )
3 –4 3 –5 ­– 6

(f ) (3p–3 q–1)2 3 (– 4p3 q–2)2 5. Is (–9)0 = –90 ? Explain your answer.


(r –3 s – 4 )–3
(g)
Solution
–2 –5 – 6
(r s )
 x3 
–4 (–9)0 = 1
(h) (x–2y) –5 3  2 –90 = –1
y 
\ (–9)0  –90
Solution
(a) 1 ÷ (52 a –6) –2 = 1 ÷ (5– 4 a12) 6. Suppose a . 0 and n is a positive integer.
625 –n
=  1
Is   =
1
? Explain your answer.
a 12  a a –n
(b) (3 – 4 b5) –1(3 –1b2)2 = (34 b–5)(3­–2 b4) Solution
= 32 b–1 –n
9  1  = an
=  a
b
( 4 c 3 d –1 ) 4 4 4 c 12 d – 4 1
= an
(c) = a –n
2 1c 2 d 5
– –
2 –1 c –2 d 5  1
–n
1
512c 14 \   =
=
–n
a a
d
9

–3 –1
 e3   e–4  e
–9
e4
(d)  2 3   = 3
 f   f  f
–6
f
–1
Exercise 1.3
e –5
= Level 1
f
–7

1. Evaluate the following without using a calculator.


f7
=
(a) (b)
3
e
5
289 125
(e) (m n ) ÷ (m n ) = m n ÷ m n
4
(c) (d)
3 –4 3 –5 – 6 9 –12 –5 – 6 5
81 243
= m14 n–6
(e) ( 121 ) 3
(f ) ( 3
216 )
2
m 14
= 5 4
n
6 1 16
(g) (h)
32 625
(f ) (3p–3 q–1)2 3 (– 4p3 q–2)2 = (32p–6 q–2) 3 [(– 4) pq ]
2 6 –4

= 144p 0 q– 6
Solution
= 144
q 6 (a) Let r = 289 .
–3 – 4 –3
(r s ) r s9 12 r2 = 289
(g) = r2 = 172
r s
–2 –5 – 6 12 30
(r s )
1 \ r = 17
=
r 3 s 18 i.e. 289 = 17

41

01 FWS3A(Exp)_ch1.indd 41 12/16/14 12:48 PM


© Star Publishing Pte Ltd. All rights reserved.

3
(b) Let r = 125 .
4 4

r3 = 125 (f ) 125 3 = (53) 3


r3 = 53 = 54
\ r=5 = 625
3
i.e. 125 = 5 –
3

3

(c) Let r = 81 .
4 (g) 16 4 = (24) 4

r4 = 81 = 2–3
1
r4 = 34 = 3
2
\ r=3
1
i.e. 81 = 3
4 =
8
5
(d) Let = 243 .
2 2
r – –

r5 = 243 (h) 1331 = (11 ) 3 3 3

r5 = 35 = 11–2
1
\ r = 3 = 2
5 11
i.e. 243 = 3 1
=
121
(e) ( 121)3 = ( 112 )3
4 4
= 113 – –
(i) 512 = (2 ) 9 9 9
= 1331
= 2­– 4
3 3
(f ) ( 216 )2 = ( 6 3 )2 =
1
= 62 24
= 36 =
1
5 16
5
1 1
(g)
1
= = 1 –
1
32 25 2  100  2
–   10  2  2
( j)   =   
4
16
4
2
4
2 169    13  
(h) = = –1
625 5
4
5  10 
=  
13
13
2. Evaluate the following without using a calculator. =
10
1 1

(a) 144 (b) 27 3


2
3. Express the following in index notation.
1 1
(a) (b)
3
(c) 625 4
(d) 32 5
a a2
3 4
(c) (d)
4 6
(e) 49 2 (f ) 125 3 a8 a9
1 1

3

2
(e) (f )
(g) 16 (h) 1331 a
5 4
4 3
a3
1 1
4 1 (g) (h)

( a 2 )6
3
9 – ( a )5
 100 
(i) 512 ( j)  
2
169 
Solution
Solution 1

(a) a =a
1 1 2

(a) 144 = (12 ) = 12


2 2 2
2

(b) a =a
1 1 3 2 3

(b) 27 = (3 ) = 3
3 3 3
8
1 1
(c) a =a
4 8 4
(c) 625 = (5 ) = 5 4 4
4

1 1 = a2
(d) 32 = (2 ) = 2
5 5 5 9

(d) = a6
6
a
9
3 3

(e) 49 = (7 ) 2 2
2 3

= 73 = a 2
= 343

Chapter 1 Indices
42

01 FWS3A(Exp)_ch1.indd 42 12/16/14 12:48 PM


© Star Publishing Pte Ltd. All rights reserved.

1 1 6
(e) =
2 4 3
– –
(f ) (g h ) = gh
5
a
5 1 3 5 2
a 5
1 g
– =
=a
6
5
h5
1 1
(f ) 4 = 3 Level 2
a 3
a4
3 5. Simplify the following, expressing your answers in
positive index notation.

= a 4
1 1

(g)
1
5
= 1 (a) (a4b3) 2 × (a2b–5) 3
1
( a) 1 1
(a 2 ) 5 – –
(b) (m–7n 2 2
) × (m3n–3) 3

= 1

1 1
(p 2q )2
5 3
a 2 (c) 1
5
p 4q
–5

= a 2
1 1
– –
r 3
s
4
1 (d)
(h)
1
= (r s
1
2 – 2 –3
)
( a 2 )6
3 2

(a ) 3 6
1 1
3 –
(e) (u v )
3 3 2
= 1 2
u v 3
a4 1 1 1
= a – 4
– –
(f ) (36p q ) (49p 3 q4)
2 4 2 2

4 3
4. Simplify the following, expressing your answers in (g) u2
positive index notation. 1 1 1 1 1
– –
(h) (4x 3 y 2 ) 3 ÷ (32–1x2y 2 )
1 1 3 2 3

(a) (b)
2 3
a a b b
10 5
3 3
1 1 3 7
– –
(c) c ÷c (d) d ÷d
8
4 2 6 Solution
1 1 3 2 5
1 2 4 3 –
(a) (a4 b3) 2 (a2 b–5) 3 = (a2 b 2 ) (a 3 b )
– – 3
(e) (e f )
–3 4 2
(f ) (g h ) 3 5 2 3 3
8 1

Solution =a b 3 6

1 1 1 1 8
+
(a) a =a
2 3
a 3 2 3
=
a3
1
5
b6
=a 6
1 1
– –
3 2 3
+
2 (b) (m n ) –7 2 2
3 (m n ) 3 –3 3
= (m –14 n–1) 3 (m –1n)
(b) b b =b
10 5
= m –15n 0
10 5
3
7
= 115
=b 10
1 1 1 3
m
– –
(p q )
3 2
p q
(c)
2
=
4 2
1 1 1 1
– – –
(c)
1

c ÷c =c
4 8 4 8 1
p 4q
–5
p q 4 –5
3
– 1 13
=c 8 –
1 =p 2
q2
= 3 13
c8 q2
= 1
3

7 3 7
+
p2
(d) d ÷d2 6
=d 2 6

1

1

1

1
8 r 3
s
4
r s
(d) =
3 4

=d 3 1
2 – 2 –3
3
(r s ) r s
–6 2
1 3
– 17 7
(e) (e–3f 4) 2
=e f 2 –2 –
=r3s 4
3
17
e
2

= = r 3

f2 7
s4

43

01 FWS3A(Exp)_ch1.indd 43 12/16/14 12:48 PM


© Star Publishing Pte Ltd. All rights reserved.

3 –
1 1 1 2 (c) (i) When x = 4096, x = 212.
3 13
(e) u v × (u v ) = (u v) × (u v )
2 3 2 3 3 –1 3

1 2 Volume of the cuboid = (2 ) 12 12

= (uv ) × (u v ) 3 –1 3 = 213
= 8192 cm3
= u 0v (ii) Total surface area of the cuboid
=v 5 3 7

1 1 1 1 1 1 = 2[(212) 6 + (212) 4 + (212) 12 ]


– – –
(f ) (36p q ) (49p 3 q4)
2 4 2 2
= (36 pq2)(49 2 2
p q )
6 –2 = 2(210 + 29 + 27)
7 = 2(1024 + 512 + 128)
=
6
p 6
q0 = 3328 cm2
7
7 2

=
6 6
p 7. Find the values of p and q such that a p × a q = a 3 and
7 –
5

a ÷a =a .
4 p q 3
4 3 2
(g) u2 = u 3

2 1 Solution
= (u 3 ) 4 p=
1
, q=
1
or p = 0, q =
2
1 3 3 3
= u 6

1 1 1 1 1

(h)

(4x y ) ÷ (32 x y )
3 2 3 –1 2 2

3 Exercise 1.4
2 1 1 5 2 1 Level 1
– – –
= (2 x y ) ÷ (2 x 3 9 6 3 3
y 6
) 1. Express the following numbers in standard form.
5 1 (a) 83 700
= 2 x y –1 9 3 (b) 720 000
5 1
(c) 96 200 000
(d) 1 450 000 000
= x y
1 9 3
2 (e) 0.000 16
(f ) 0.000 028
(g) 0.000 009 5
Level 3 (h) 0.000 000 030
1 1 1

6.
A closed cuboid is x 2 cm long, x 3 cm wide and x 4 cm Solution
high. (a) 83 700 = 8.37 × 104
(a) Express the volume of the cuboid in terms of x. (b) 720 000 = 7.2 × 105
(b) Express the total surface area of the cuboid in terms (c) 96 200 000 = 9.62 × 107
of x. (d) 1 450 000 000 = 1.45 × 109
(c) Given that x = 4096, find (e) 0.000 16 = 1.6 × 10 – 4
(i) the volume of the cuboid, (f ) 0.000 028 = 2.8 × 10 –5
(ii) the total surface area of the cuboid. (g) 0.000 009 5 = 9.5 × 10 – 6
(h) 0.000 000 030 = 3.0 × 10 –8
Solution
1 1 1

(a) Volume of the cuboid = (x 2 )(x 3 )(x 4 ) 2. Express the following numbers as integers.
1 1 1
+ +
(a) 9.8 3 103
=x 2 3 4 (b) 5 3 104
6 +4+3 (c) 7.23 3 106
=x 12
(d) 1.06 3 108
13

= x 12 cm3 Solution
(a) 9.8 3 103 = 9800
(b) Total surface area of the cuboid
1 1 1 1 1 1 (b) 5 3 104 = 50 000
= 2(x 2 x 3 + x 2 x + x 3 x )
4 4 (c) 7.23 3 106 = 7 230 000
5 3 7 (d) 1.06 3 108 = 106 000 000
= 2(x 6 + x 4 + x 12 ) cm2

Chapter 1 Indices
44

01 FWS3A(Exp)_ch1.indd 44 12/16/14 12:48 PM


© Star Publishing Pte Ltd. All rights reserved.

3. Express the following numbers as decimals. 6. In 2010, the United States of America consumed
(a) 4 3 10 –3 1.915 3 107 barrels of oil per day and Singapore consumed
(b) 3.6 3 10 –5 1.08 3 106 barrels of oil per day. Find the difference in
(c) 1.58 3 10 –6 oil consumption per day between the United States of
(d) 2.07 3 10 –10 America and Singapore.

Solution Solution
(a) 4 3 10 –3 = 0.004 Difference in oil consumption
(b) 3.6 3 10 –5 = 0.000 036 = 1.915 3 107 – 1.08 3 106
(c) 1.58 3 10 – 6 = 0.000 001 58 = 1.807 3 107
(d) 2.07 3 10 –10 = 0.000 000 000 207
7. The diameter of the Earth is 1.275 3 107 metres.
4. Evaluate the following using a calculator and express
(a) What is the circumference of the equator in
your answers in standard form.
kilometres? (Take r = 3.142)
(a) 3.6 3 104 + 4.7 3 105
(b) The radius of the Sun is about 6.96 3 108 m.
(b) 6.8 3 10 –9 + 9 3 10 –7
By how many times is the radius of the Sun more
(c) 4 3 106 – 9.8 3 105
than the radius of the Earth? Give your answer
(d) 5.4 3 10 –11 – 6.6 3 10 –12
correct to the nearest integer.
(e) (5 3 106) 3 (3 3 108) volume of the Sun
(f ) (4 3 10 –5) 3 (1.7 3 109) (c) Find the value of .
volume of the Earth
(g) (2 3 10 –3) ÷ (8 3 10 –7)
Give your answers in standard form, correct to
(h) (3.4 3 10 – 4) ÷ (1.7 3 105)
3 significant figures, where appropriate.
Solution
Solution
(a) 3.6 3 104 + 4.7 3 105 = 5.06 3 105
(a) Diameter of the Earth = 1.275 3 107 m
(b) 6.8 3 10 –9 + 9 3 10 –7 = 9.068 3 10 –7
= 1.275 3 104 km
(c) 4 3 106 – 9.8 3 105 = 3.02 3 106
Circumference of the equator
(d) 5.4 3 10 –11 – 6.6 3 10 –12 = 4.74 3 10 –11
= r 3 D
(e) (5 3 106) 3 (3 3 108) = 1.5 3 1015
= 3.142 3 1.275 3 104
(f ) (4 × 10 –5) 3 (1.7 × 109) = 6.8 3 104
= 4.01 3 104 km (correct to 3 s.f.)
(g) (2 3 10 –3) ÷ (8 × 10 –7) = 2.5 3 103
(h) (3.4 3 10 – 4) ÷ (1.7 3 105) = 2 3 10 –9 (b) Radius of the Earth = 1.275 3 107 ÷ 2
= 6.375 3 106 m
Level 2
Radius of the Sun
=
6.96 3 10 8
6
Radius of the Earth 6.375 3 10
5. Evaluate the following using a calculator and express
your answers in standard form. = 1.09 3 102
(a) (1.3 3 107)2 The radius of the Sun is 109 times more than the
(b) (5 3 10 –6 )3 radius of the Earth.
Volume of the Sun
4
rr 3
(c) 1.96 3 10 8 (c) = 3

Volume of the Earth


4
3
rr 3
(d)
3
2.16 10 –10 (6.96 3 10 8 ) 3
3 = 6 3
(6.375 3 10 )
(2 3 10 –5 ) 3 (6 3 10 7 )
(e) –8
= 1.30 3 106 (correct to 3 s.f.)
4 3 10
8 3 10 – 4 + 1 3 10 –5
(f ) 8. Assume that the Earth is a sphere of radius 6378.5 km.
5 3 10 6 – 5 3 10 5

Solution
(a) (1.3 3 107)2 = 1.69 3 1014 equator
(b) (5 3 10 – 6)3 = 1.25 3 10 –16

(c) 1.96 310 8 = 1.4 3 104
(a) Find the circumference of the equator in metres,
(d)
3
2.16 10 –10 = 6 × 10 – 4
expressing your answer in standard form.
3

(e)
(2 3 10 –5 ) 3 (6 3 10 7 )
= 3 × 1010 (b) The speed of light in air is 3 3 108 m/s. Find the
time taken for a light beam to travel a distance
–8
4 3 10
8 × 10 – 4 + 1 × 10 –5 equal to the circumference of the equator.
(f ) 5 × 10 6 – 5 × 10 5
= 1.8 × 10 –10 Give your answers correct to 3 significant figures.

45

01 FWS3A(Exp)_ch1.indd 45 12/16/14 12:48 PM


© Star Publishing Pte Ltd. All rights reserved.

Solution Solution
(a) Circumference of the equator (a) 2x = 32
= 2r 3 6378.5 km 2 x = 25
= 40 077 km \ x = 5
= 40 077 3 103 m (b) 3x = 729
= 4.01 3 107 m (correct to 3 s.f.) 3x = 36
(b) The required time \ x = 6
= (4.0077 3 107) ÷ (3 3 108) (c) 4x = 64
= 1.3359 3 10 –1 4x = 43
= 0.134 s (correct to 3 s.f.) \ x = 3
(d) 7x = 343
Level 3
7x = 73
9. In the year 2012, the population in Singapore was \ x = 3
about 5.3 3 106. If each person, on average, consumed
0.3 kg of meat a day, how many kilograms of meat were
Level 2
consumed in Singapore in a day in 2012?
2. Solve the following equations.
Solution (a) 4x – 3 = 256
The amount of meat required = 0.3 3 (5.3 3 106) (b) 23x + 1 = 128
= 1.59 3 106 kg (c) 59 – x = 625
1
(d) 3x =
243
10. The Punggol Waterway contains 5 3 105 m3 of water. If 1
one drop of water is 0.5 cm3, find the number of drops (e) 322x – 1 =
8
of water contained in the waterway.
(f ) 10001 + 2x = 0.1
Solution
Solution
5310 5 m 3
Number of drops of water = –6 3 (a) 4x – 3 = 256
0.5310 m
= 1 3 1012 4x – 3 = 44
The number of drops of water contained in the waterway \ x – 3 = 4
is 1 3 1012. x = 7
(b) 23x + 1 = 128
11. The number of atoms in 12 g of Carbon-12 is 23x + 1 = 27
6.022 3 1023. This number is called the Avogadro’s \ 3x + 1 = 7
number. Find the mass of one atom of Carbon-12. 3x = 6
x = 2
Solution (c) 59 – x = 625
Mass of one atom =
12 59 – x = 54
\ 9 – x = 4
23
6.022310
= 1.99 3 10 –23 g x = 5
1
(d) 3x =
12. An atom of an element consists of protons, neutrons and 243

electrons. An electron has a mass of 9.109 56 3 10 –31 kg 3x =


1
which is 5 3 10 –4 times the mass of a proton. Find the
5
3
mass of a proton, expressing your answer in standard \ 3x = 3 –5
form correct to 3 significant figures. x = –5
1
Solution (e) 322x – 1 =
8
The mass of a proton = (9.109 56 3 10 –31) ÷ (5 3 10 – 4) 1
= 1.82 3 10 –27 kg
(25)2x – 1 = 3
2
25(2x – 1) = 2–3
Exercise 1.5
10x – 5 = –3
Level 1 10x = 2
1
1. Solve the following equations. x =
5
(a) 2x = 32 (b) 3x = 729
(c) 4x = 64 (d) 7x = 343

Chapter 1 Indices
46

01 FWS3A(Exp)_ch1.indd 46 12/16/14 12:48 PM


© Star Publishing Pte Ltd. All rights reserved.

(f ) 10001 + 2x = 0.1 (c) 3 –n + 7 =


1
1 2187

(103)1 + 2x = 1
10 3 –n + 7 = 7
10 = 10 3(1 + 2x) –1 3
\ –n + 7 = –7
3 + 6x = –1
n = 14
6x = – 4
2
x = –
3 6. The solution of the equation 4x = 2n is an integer x. Find
two possible values of the constant n.
3. Solve the following equations.
(a) 42x = 81 – x Solution
(b) 272x + 1 = 81x 4x = 2n
22x = 2n
Solution 2x = n
n
(a) 42x = 81 – x \ x=
2
(22)2x = (23)1 – x
4x = 3 – 3x If x is an integer, then n must be even.
\ some possible values of n are – 4, –2, 0, 2, 4.
7x = 3
3
x=
7
Exercise 1.6
(b) 272x + 1 = 81x
(33)2x + 1 = 34x Level 1
6x + 3 = 4x 1. Copy and complete the following table on simple interest.
2x = –3
3 Interest,
x = – Interest rate,
2 Principal, Time, I = P 3 r% 3 n or Amount,
(p.a.)
P= I n= I I=A–P A=P+1
r% = I
Level 3 r% 3 n
P 3n
P 3 r%

4. The number of bacteria in a colony is 2t + 1, where t is the


number of hours after the first observation. After how (a) $3000 4% 2 years
3000 3 4% 3 2 3000 + 240
many hours will there be 1024 bacteria? = $240 = $3240

Solution 6800 + 357


(b) $6800 7% 9 months 6800 3 7% 3
9
2 = 1024
t+1
12 = $7157
= $357
2t + 1 = 210
\ t + 1 = 10 960
t = 9 (c) 8% 3 5 8% 5 years $960
2400 + 960
i.e. the required time is 9 hours. = $2400
= $3360

5. Consider the sequence 729, 243, 81, 27, … . 700


5000 + 700
(d) $5000 3.5% 5000 3 3.5% $700
(a) Write down the 5th and 6th terms of the sequence. = 4 years = $5760
(b) Express the nth term of the sequence in terms
of n. 8 months
480
1
(c) If the nth term of the sequence is , find the 8 7680 – 7200
2187 (e) $7200 7200 3
12 $7680
value of n. = 0.1 = 10%
= $480

Solution
(f) $1500 6% 3 years $270 $1770
(a)
1
The 5th term = 27 3 =9
3
1
The 6th term = 9 3 =3
3
n–1
 1
(b) The nth term = 729 3 
 3
= 36 3 3 –n + 1
= 3 –n + 7

47

01 FWS3A(Exp)_ch1.indd 47 12/16/14 12:48 PM


© Star Publishing Pte Ltd. All rights reserved.

2. Copy and complete the following table on compound interest.

Interest rate
Principal Interest Amount
(p.a.) Compounded Time, n r
P= An
I = An – P An = P  1+ r 


n
r  r% 100 
 1+ 
100 

3
 6 
$2977.54 – $2500 2500  1+ 100 
(a) $2500 6% Yearly 3 years
= $477.54
= $2977.54

5
 7 
3800  1+ 100 
$5329.70 – $3800
(b) $3800 7% Yearly 5 years
= $477.54 = $5329.70
(correct to 2 d.p.)

4
 1 
2900  1+ 100 
$3017.75 – $2900
(c) $2900 12% Monthly 4 months
= $117.75 = $3017.75
(correct to 2 d.p.)

36
 8 
3 years 8700  1+
12(100) 
$11051.06 – $8700 
(d) $8700 8% Monthly = (3 3 12)
= $2351.06
months = $11 051.06
(correct to 2 d.p.)

5408
2 years $5408 – $5000
(e) 4% Yearly $5408
2
 4 
 1+  = $408
100 
= $5000
(f ) 1582
P = 12
 0.5 
 1+ 
100  $1582 – $1490.094
= $1490.094 6% Monthly 1 year = $91.91 $1582
= $1490.09 (correct to 2 d.p.)

(correct to 2 d.p.)

Level 2 4. A sum of money is deposited at 7% per annum simple


3. Mrs Lee deposited $3500 at 5.5% per annum simple interest. If the yield after 3 years is $7744, find the
interest for 18 months. Find the interest earned and the original sum of money.
amount in her account after 18 months.
Solution
Solution Let $P be the original sum of money, and $I be the
18 interest accumulated
18 months = years Hence, I = P 3 7% 3 3
12
Interest earned after 18 months I = 0.21P.......................... (1)
18 and 7744 = I + P .......................... (2)
= 3500 5.5%
Substitute (1) in (2):
3 3
12
= $288.75 7744 = 0.21P + P
Amount in her account after 18 months = 1.21P
= 3500 + 288.75 7744
= $3788.75 \ the original sum, P =
1.21
= $6400

Chapter 1 Indices
48

01 FWS3A(Exp)_ch1.indd 48 12/16/14 3:26 PM


© Star Publishing Pte Ltd. All rights reserved.

5. Kumar borrowed $1800. He had to repay $1989 after (b) From (a), substitute r = 10 into (1):
9 months, including the simple interest charged to him. 1
Find the interest rate per annum of the loan. 3740 = P  1 + 10 
 100 

= 1.1P
Solution
Interest accumulated after 9 months = $1989 – $1800 ∴ original loan, P =
3740
= $189 1.1
9
= $3400
9 months = years
12
Let r% be the interest rate per annum
8. A sum of money grows to $3800 in 3 years at 6% per
9
Hence, 189 = 1800 3 r% 3 annum compound interest. Find the sum of money if the
12
Interest rate (p.a.) = r% interest is compounded
(a) yearly,
189
= 9 (b) monthly.
1800 3
12

= 0.14 Solution
= 14% (a) Let P be the original sum of money.
3
Hence, 3800 = P  1 + 6 
 100 
6. Meiyi has $7200, and she is deciding between depositing
it in a bank that pays 3% per annum compound interest ∴ the original sum of money,
3800
compounded annually, and a finance company that pays P=
(1 + )
3
3% per annum simple interest. If the money is deposited for
6
100
4 years, find the difference in interest Meiyi will get
= $3190.55 (correct to 2 d.p.)
from the bank and the finance company.
6
(b) Interest rate per month =
Solution 12
Simple interest earned = 7200 3 3% 3 4 = 0.5%
= $864 Number of months in 3 years = 3 3 12
 3 
4 = 36
Amount after 4 years = 7200  1 + 36
 100  Hence, 3800 = P  1 +
 0.5 

= $8103.663  100 
Required compound interest ∴ original sum of money,
= $8103.663 – $7200 P=
3800

(1 + )
36
= $903.663 0.5

= $903.66 (correct to 2 d.p.) 100

= $3175.45 (correct to 2 d.p.)


Difference = $903.663 – $864
= $39.66 (correct to 2 d.p.) Level 3
9. Ravi invested $78 000 partly at 7% p.a. simple interest
7. A loan grows to $3740 after 1 year and $4114 after 2 years and partly at 5% p.a. simple interest. He received a total
with compound interest which is computed annually. Find interest of $4800 after a year. How much did he invest
(a) the interest rate per annum, at 7% p.a. simple interest?
(b) the original loan.
Solution
Solution Let $S be the amount he invested at 7% p.a. simple
(a) Let $P be the original loan, and r% be the interest interest,
rate per annum. and $C be the amount he invested at 5% p.a. compound
 r 
1
interest.
Hence, 3740 = P  1 + .. ……(1)
 100  Hence, S + C = 78 000..................... (1)
r  
2
Also, interest received on S is
and 4114 = P  1 +
...……(2)
100   S 3 7% 3 1 = 0.07S
(2) ÷ (1):
4114
=1+
r and interest received after 1 year on C is
3740 100 C 3 5% = 0.05C
r 4114 Hence, 0.07S + 0.05C = 4800
= –1
100 3740
1.4S + C = 96 000............. …….(2)
1
= (2) – (1) : 0.4S = 18 000
10
18 000
 1
∴ Interest rate (p.a.), r = 100   ∴ amount invested at the simple interest, S =
10 0.4
= 10% = $45 000

49

01 FWS3A(Exp)_ch1.indd 49 12/16/14 3:27 PM


© Star Publishing Pte Ltd. All rights reserved.

10. The simple interest earned on a sum of money for (b) On 31st March 2016, the account has earned interest
4 years at 7.5% p.a. exceeds that on the same sum for for 3 consecutive years’ deposits, but the 4th year’s
3
1
years at 8% p.a. by $90. deposit has not yet been made.
2 ∴ amount of money David would get if he
(a) Find the original sum of money. withdraws all the money in the account on 31st
(b) If the original sum of money accumulates to March 2016 is
$4612.50 in 5 months at simple interest, find the 2500 (1 + 7%) + 2500 (1 + 7%)² + 2500 (1 + 7%)³
interest rate per annum. = $8599.8575
Solution = $8599.86 (correct to 2 d.p.)
(a) Let $P be the original sum of money.
Interest earned 4 years at 7.5% is 12.
Meijun borrowed $3600 from a bank at the interest rate
P 3 7.5% 3 4 = 0.3P of 15% p.a. compounded monthly. She repaid $1200 at
and interest earned after 3 years at 8% p.a. is the end of the first month, $1300 at the end of the second
P 3 8% 3 3.5 = 0.28P month, and made a final settlement payment at the end
Hence, 0.3P – 0.28P = 90 of the third month.
0.02P = 90 (a) How much did she owe the bank just after
∴ original sum of money, P =
90
(i) the first payment,
0.02
(ii) the second payment?
= $4500
(b) How much was the final settlement payment?
(b) Interest accumulated after 5 months (c) If Meijun had repaid $1300 at the end of the first
= $4612.50 – $4500 month and $1200 at the end of the second month,
= $112.50 would the final settlement payment at the end of
5
5 months = year the third month be the same as the previous case?
12
Explain briefly.
Let r% be the interest rate per annum.
5
Hence, 112.50 = 4500 3 r% 3 Solution
12
15
112.50 Interest rate per month =
∴ interest rate (p.a.) = r% = 12
4500 × 5
12 = 1.25%
= 0.06 therefore,
= 6% (a) (i) amount owed just before first payment
 1.25 
= 3600  1 +
11.
On 1st April 2013, David deposited $2500 into a Gold-  100 
saver account with a bank. Subsequently, on 1st April of = $3645
2014 and 2015, he deposited $2500 into the same account. amount owed just after first payment
This account paid compound interest at 7% p.a. = $3645 – $1200
(a) Find the accumulated amount in David’s Gold-saver = $2445
account immediately after the interest payment and (ii) amount owed just before second payment
the annual deposit on  1.25 
(i) 1 April 2014, (ii) 1 April 2015. = 2445  1 +
 100 
(b) If David were to withdraw all the money from the = $2475.5625
account on 1st April 2016, how much money would amount owed just after second payment
he get? = $2475.5625 – $1300
Solution = $1175.5625
(a) = $1175.56 (correct to 2 d.p.)

Total amount (b) Amount owed just before the final settlement payment
Number of
Total amount immediately after = 1175.5625  1 +
 1.25 
years after 100 
Date accrued from the interest 
1st deposit
deposits payment and = $1190.26 (correct to 2 d.p.)
(1 Apr 2013)
annual deposits i.e. the final settlement payment was $1190.26.

(i) 1 April 1 2500(1 + 7%) $2675 + $2500 (c)


The final settlement payment would not be the
2014 = $2675 = $5175 same.
Explanation:
(ii) 1 April 2 2500(1 + 7%) + By increasing the first payment to $1300, the
$5537.25 + $2500
2015 2500(1 + 7%)2 resulting decrease in interest would be compounded
= $8037.25
= $5537.25 twice: before second payment and before final
settlement.
Chapter 1 Indices
50

01 FWS3A(Exp)_ch1.indd 50 12/16/14 12:49 PM


© Star Publishing Pte Ltd. All rights reserved.

By decreasing the second payment to $1200, the (c) (2a3b4)3 = 23 a9 b12


resulting increase in interest would be compounded = 8a9 b12
only once: before final settlement. (–2 p 3 q 2 ) 2 (–2) 2 p 6 q 4
Hence, there would be a resultant decrease in the (d) =
(4 p q )
2 5 3
4 p q
3 6 15

total amount of interest that Meijun needs to pay


1
by the time of the final settlement. =
16q
11

13. Nursarah deposited $4000 at r % p.a. simple interest


for n years. If the amount accumulated after n years is 2. Evaluate the following without using a calculator.
$5920, find two possible sets of values of r and n. (a) 10 –2 + 100 + 102 (b) 3 –1 – 4 –2
–3 3 –2
 4  2  9
Solution (c)   (d)   ÷  
5 3 4
Interest accumulated = $5920 – $4000
= $1920 Solution
∴ 1920 = 4000 3 r% 3 n 1
1920 (a) 10 –2 + 100 + 102 = + 1 + 100
r% = 4000 × n
100
1
= 101
= 12 100
25 n 1
(b)
1
r 3 –1 – 4 –2 = –
= 12 3 16
100 25 n 13
12 =
rn = (100) 48
25 –3
 4 53 125
= 48 (c)   = 3 =
5 4 64
Two possible sets of values of r and n are:
r = 1, n = 48;
3 –2
 2  9 23 42
(d)   ÷   = ÷
r = 2, n = 24 3 4 3
3
9
2

(generally, r and n can be any pair of positive real numbers =


23 34
3 4
such that rn = 48) 3
3
2
3
=
14.
2
A sum of money is invested at 7% per annum compound
interest. Find the minimum number of years that the sum
of money has to be invested for the accumulated interest 3. Simplify the following, expressing your answers with
to be more than twice the initial sum of money. positive indices.
(a) (x3y–1)(x– 4y5)
Solution
(b) (–2x4y3)0(–3x– 4y5) –2
Let $P be invested for n years to achieve the aim.
n (6 p 2 q –3 ) 5
P  1 +
7 
. P + 2P (c)
(–3 pq )
–5 3
 100 
3 4
1.07n . 3 (d)
 x4 
 –2  ÷
 x –2 
 3 
By trial and error, 1.0716 = 2.952 and 1.0717 = 3.159. y   y 
\ the minimum number of years is 17 years.
Solution
(a) (x3y–1)(x– 4y5) = x–1y4
Revision Exercise 1 y4
=
x
1. Simplify the following, expressing your answers with
positive indices. (b) (–2x4y3)0(–3x– 4y5) –2 = (1)[(–3) –2 x8y–10]
(a) (x3y)(x4y2) (b) (a5b6) ÷ (a3b8) x
8
=
9y
10
(–2 p q )
3 2 2

(c) (2a3b4)3 (d)


(4 p q )
2 5 3
(6 p q ) 6 5 p 10 q –15
2 –3 5

(c) =
(–3 pq ) (–3) p q
–5 3 3 3 –15

Solution
= –288p7
(a) (x3y)(x4y2) = x3 + 4 y1 + 2
= x7y3
3 4
 x4   x –2  x 12 x –8
(d)  –2  ÷  3  = ÷
(b) (a5b6) ÷ (a3b8) = a5 – 3b6 – 8 y   y  y
–6
y
12

= a2 b–2 x
12
= 3 y
12

a2
= y
–6
x
–8
2
b
= x20y18

51

01 FWS3A(Exp)_ch1.indd 51 12/16/14 12:49 PM


© Star Publishing Pte Ltd. All rights reserved.

4. Evaluate the following without using a calculator. 3



4

1 2

(c) (m –1n )4(m –2 n ) = (m – 4 n6)(mn )


3 2 3 2 3

(a) (b) 256


4 2
1296 20


4

2 = m –3n 3
(c) 128 (d) 1000
7 3 20

n3
=
m
3

Solution 1 1 1 1 1
– –
(d) (a + a )(a – a ) = a + a 0 – a 0 – a –1
2 2 2 2
(a) 1296 = (6 ) 4
=6
4 4

1
3 3 =a–
a
(b) 256 = (2 ) = 212 = 4096
2 8 2

6.
Let x = 9 230 000, y = 471 000 and z = 0.000 005.
4

1
(c) 128 =
7
4 (a) Express x, y and z in standard form.
(b) Find the values of the following, expressing your
7 7
(2 )

=
1
answers in standard form.
(ii) xz
4
2 (i) x + y
1
= (iii) z3 (iv) y
16
z
2

1 Solution
(d) 1000 =
3
2
3 3
(a) x = 9 230 000
(10 )
= 9.23 3 106
=
1 y = 471 000
= 4.71 3 105
2
10
= 1 z = 0.000 005
100
= 5 3 10 – 6
(b) (i) x + y = 9.23 3 106 + 4.71 3 105
5. Simplify the following, expressing your answers with
= 9.23 3 106 + 0.471 3 106
positive indices.
= 9.701 3 106
(ii) xz = (9.23 3 106) × (5 3 10 – 6)
3
(a) a2 ÷ a5
1
= 46.15 3 100
(b) (8a)

3
×
4
81 = 4.615 3 101
a3 (iii) z = (5 3 10 – 6)3
3
3

4

1
= 53 3 10 –18
(c) (m –1n )4(m –2 n )
2 3 2
= 125 3 10 –18
1

1 1

1 = 1.25 3 10 –16
(d) (a + a )(a – a )
2 2 2 2
4.71 3 10 5
(iv) –6
y
=
5 3 10 z
Solution
2 5

= 0.942 3 1011
3
= 9.42 3 1010
(a) a ÷ a =a ÷a
5
2 3 2

2 5

=a 3 2
7. Solve the following equations.

11 (a) 42 – 3x = 512 (b) 243x = 9x + 6
=a 6
1
1
(c) 36x – 1 =
= 11
216

a6
1 1
1 Solution
(b) (8a)

3
3
4
81
= (2 a) 3

3
 34  4
3  3
(a) 42 – 3x = 512
a3 1
a  (22)2 – 3x = 29
– 3 24 – 6xx = 29
=2 a –1 3
3 3
4 – 6x = 9
a4
3 1 6x = –5
= × 5
\ x=–
3 1
2 +
a 4 3
6
3
= 13

2 a 12

Chapter 1 Indices
52

01 FWS3A(Exp)_ch1.indd 52 12/16/14 12:49 PM


© Star Publishing Pte Ltd. All rights reserved.

(b) 243x = 9x + 6 10. A ream of A4 office paper consists of 500 sheets and its
35x = 32x + 12 height is 5 cm.
5x = 2x + 12 (a) Find the thickness of a sheet of the A4 paper in m,
3x = 12 expressing your answer in standard form.
\ x = 4 (b) A sheet of A4 paper has an area of
1
m2. It
16
(c)
36x – 1 =
1
is known that 1 m 2 of the paper has a mass
216
of 80 g. Find the mass of a sheet of the A4 paper
(62)x – 1 =
1
in kg, expressing your answer in standard form.
63
62x – 2 = 6 –3 Solution
2x – 2 = –3 (a) 5 cm = 5 3 10 –2 m
2x = –1 Thickness of a sheet of A4 paper
1
\ x =– =
5 3 10 –2
2
500
= 1 3 10 – 4 m
8.
A well-known formula in physics is E = mc . It is given
2
(b) 80 g = 80 3 10 –3 kg
that m = 2.5 3 10 –9 and c = 3 3 108. Mass of a sheet of A4 paper
(a) Express m as a decimal. 1
(b) Express c as an integer. = (80 3 10 ) –3
3 16
(c) Find the value of E, expressing your answer 1
(i) in standard form, = 5 3 10 kg –3

(ii) as an integer.
11. The distance between the Earth and the Moon is
Solution
384 400 km.
(a) m = 2.5 3 10 –9
(a) Express the distance in standard form.
= 0.000 000 002 5
(b) In 1969, the spacecraft Apollo 11 launched
(b) c = 3 3 108 from the Earth. It took 3 days 3 hours and
= 300 000 000 49 minutes to reach the Moon. Find the average
(c) (i) E = (2.5 3 10 –9) 3 (3 3 108)2 speed of the spacecraft in km/h, expressing your
= (2.5 3 10 –9) 3 (32 3 1016) answer in standard form.
= (2.5 3 9) × 107
= 22.5 3 107 Solution
= 2.25 3 108 (a) 384 000 km = 3.844 3 100 000
(ii) E = 225 000 000 = 3.844 3 105 km
(b) 3 days 3 hours and 49 minutes
9. (a) Find the number of seconds in a day.  49 
=  3 3 24 + 3 + 60  hours
(b) Suppose there are 365.24 days in a year. Find the
number of seconds in a year correct to 3 significant =
49
75 hours
figures, expressing your answer in standard form. 60
384 400
(c) Sulin just turned 15 years old. How many seconds Average speed of the spacecraft = 49
75
has she been living? Give your answer in standard 60
form, correct to 3 significant figures. = 5.07 3 103 km/h

Solution
12. Population Reference Bureau reported that the world
(a) Number of seconds in a day
population was 7.06 3 10 9, the population in Asia
= 24 3 60 3 60
was 4.26 3 109 and that the population in Europe was
= 86 400
7.40 3 108 in mid-2012.
(b) Number of seconds in a year (a) How many more people were there in Asia than in
= 86 400 3 365.24 Europe? Give your answer in standard form, correct
= 31 556 736 to 3 significant figures.
= 3.16 3 107 (correct to 3 s.f.) (b) Express the population in Europe as a percentage
(c) Number of seconds that Sulin has been living of the world population.
= 31 556 736 3 15
= 4.73 3 108 (correct to 3 s.f.) Solution
(a) 4.26 3 109 – 7.40 3 108 = 3 520 000 000
There were 3.52 3 109 more people in Asia than
in Europe.

53

01 FWS3A(Exp)_ch1.indd 53 12/16/14 12:49 PM


© Star Publishing Pte Ltd. All rights reserved.

7.40 × 10 8 15. Madam Aminah plans to make a monthly deposit of


(b) 100% = 10.5 % (correct to 3 s.f.)
7.06 × 10
9 3
$2500 to the bank on the first day of each month for 3
The population in Europe was 1.05 3 101% of the months. The interest is calculated at the rate of 6% per
world population. annum compounded monthly. Find the total amount that
she will get at the end of the third month.
13. Mrs Tan deposits a sum of $7200 in a bank. The simple
Solution
interest for 9 months is $216.
6
(a) Find the interest rate. Interest rate per month =
12
(b) Find the amount after 3 years. = 0.5%
(c) If the interest is compounded annually at the same
The deposits made on the first day of April, May and
rate, find the amount after 3 years.
June will have earned 3, 2 and 1 months of compound
Solution interest respectively by the end of June.
9 \ Total amount that she will get at the end of June
(a) 9 months = year = $2500(1 + 0.5%)3 + $2500(1 + 0.5%)2 + $2500(1 + 0.5%)
12
Let r% be the interest rate per annum. = $7575.25 (correct to 2 d.p.)
9
Hence, 216 = 7200 3 r% 3
12
16. Mr Singh borrowed a sum of money from a bank at the
216
r% = 9
interest rate of 10% per annum compounded annually.
7200 3
12 He had to repay an amount of $10 648 after 3 years.
= 0.04 (a) Find the sum of money that Mr Singh borrowed.
\ The interest rate per annum is 4%. (b) The sum borrowed was for his business which gains
(b) Interest accumulated after 3 years 15% per annum at simple interest. How many years
= $7200 3 4% 3 3 will he take to earn back the interest paid to the
= $864 bank?
Total amount after 3 years
Solution
= $7200 + $864
= $8064 (a) Let $P be the sum of money that Mr Singh
borrowed.
(c) Using compound interest,
amount after 3 years Hence, 10 648 = P(1 + 10%)3
= $7200(1 + 4%)3 10 648
= $8099.02 (correct to 2 d.p.) \ The sum of money borrowed, P =
1.10 3
= $8000
14. A sum of money is deposited in a bank and it earns (b) Interest paid to the bank = $10648 – $8000
interest at 5% per annum for 3 years. If the difference
= $2648
between the compound interest, compounded yearly, and
the simple interest earned on the sum is $36.60, find the Let n years be the time taken to earn back the
sum. above interest.
Hence, 2648 = 8000 3 15% 3 n
Solution \ Time taken to earn back the interest,
Let $P be the sum of money. n=
2648
Simple interest earned after 3 years 8000 3 15%
= P 3 5% 3 3
=2
31
years
= 0.15P 150
Compound interest earned after 3 years = 2.21 years (correct to 3 s.f.)
= Amount after 3 years – $P
3
= P  1 + 5 
–P
 100 
= 0.1576P (correct to 4 d.p.)
Difference between compound interest and simple interest
= 36.60
\ 0.1576P – 0.15P = 36.6
P = $4800
The required sum of money is $4800.

Chapter 1 Indices
54

01 FWS3A(Exp)_ch1.indd 54 12/16/14 12:49 PM


© Star Publishing Pte Ltd. All rights reserved.

Go Further Solution
1. The diagram shows the cross-section of a piece of optical (a) The resident population of Singapore in 2013
fibre. It is formed by two circles with the same centre = 1.175 3 106 3 3.47
and diameters 50 micrometres and 125 micrometres = 4 000 000 (correct to nearest million)
respectively. The inner circle is called the core and the (b) In 2014,
shaded part is called the cladding. number of resident households
1.9
= 3 (1.175 3 106) + 1.175 3 106
100
= 1.197 3 106
Cladding
In 2015,
Core
number of resident households
1.9
= 3 1.197 3 106 + 1.197 3 106
100
= 1.22 3 106 (correct to 3 s.f.)

50
(c)
Difference between the number of households in
Singapore and Hong Kong
= 2.417 3 106 – 1.175 3 106
= 1.242 3 10­6
125

(a) Find the area of the cladding in m2.


(b) If the optical fibre is 2 km long, find
(i) the volume of the core in m3,
(ii) the volume of the cladding in m3.
Express your answers in standard form.

Solution
(a) Radius of the bigger circle = (125 3 10 –6) ÷ 2
= 6.25 3 10 –5 m
Radius of the core = (50 3 10 –6) ÷ 2
= 2.5 3 10 –5 m
Area of the cladding
= r(6.25 3 10 –5)2 – r(2.5 3 10 –5)2
= 1.227 3 10 –8 – 1.96 3 10 –9
= 1.03 3 10 –8 m2
(b) (i) Length of the optical fibre = 2000 m
Volume of the core = 1.96 3 10 –9 3 2000
= 3.93 3 10 –6 m3
(ii) Volume of the cladding = 1.03 3 10 –8 3 2000
= 2.06 3 10 –5 m3

2. In Singapore, the number of resident households in 2013


was 1.175 3 106. The average number of persons in a
household was 3.47.
(a) Estimate the resident population of Singapore in
2013. Give your answer to the nearest million.
(b) If the number of households increases by 1.9% per
year, find the number of households in Singapore
in 2015. Give your answer in standard form correct
to 3 significant figures.
(c) The number of households in Hong Kong in 2013
was 2.417 3 106. Find the difference between the
number of households in Singapore and Hong Kong.

55

01 FWS3A(Exp)_ch1.indd 55 12/16/14 12:49 PM


© Star Publishing Pte Ltd. All rights reserved.

2 More about Quadratic


Equations

Class Activity 1
Objective: To investigate how b, c and p are related in the expansion of (x + p)2 = x2 + bx + c.

1. Expand the following perfect square expressions and complete the table. An example has been done for you.

Perfect square, Expansion, Coefficient of Constant term, b


2

 
(x + p)2 x2 + bx + c the x-term, b c 2

2
 2
(x + 1)2 x 2 + 2x + 1 2 1  2  =1

2
 4
(x + 2)2 x2 + 4x + 4 4 4   =4
2

2
 6
(x + 3)2 x 2 + 6x + 9 6 9  2  =9

2
 8
(x + 4)2 x 2 + 8x + 16 8 16  2  = 16

2. Expand the following perfect square expressions and complete the table. An example has been done for you.

Perfect square, Expansion, Coefficient of Constant term, b


2

 
(x + p)2 x2 + bx + c the x-term, b c 2

2
 –2 
(x – 1)2 x 2 – 2x + 1 –2 1  2  =1

2
 –4 
(x – 2)2 x 2 – 4x + 4 –4 4  2  =4

2
 6
(x – 3)2 x2 – 6x + 9 –6 9  –  =9
2
2
 –8 
(x – 4)2 x 2 – 8x + 16 –8 16  2  = 16

3. Observe the patterns of the perfect squares in Questions 1 and 2. If (x + p)2 = x2 + bx + c, express
(a) c in terms of b, (b) p in terms of b.
2
b b
c=   p=
 2 2

Chapter 2 More about Quadratic Equations


56

02 FWS3A(Exp)_ch2.indd 56 12/16/14 12:49 PM


© Star Publishing Pte Ltd. All rights reserved.

4. You are given the algebra discs for the expression x2 + 8x in the diagram below.

x2 x x x x

x 3 x 4

x x x 2
4x
?
x 4 4x ?

(a) How many 1-discs are needed to make the x2-disc and x-discs into a square array?
16

(b) What is the resulting perfect square expression?


x 2 + 8x + 16 = (x + 4)2

5. (a) You are given the algebra discs for the expression x2 – 4x. How many 1-discs are needed to add so that all discs
form a square array?
2
 –4 
Number of 1-discs needed to add to x 2 – 4x to form a square array =   = 4
 2 

(b) What is the resulting perfect square expression?


2
 –4 
x 2 – 4x +   = x – 4x + (–2)2 = (x – 2)2
 2 

The resulting perfect square expression is (x – 2)2.

6. Fill in the constant term that should be added to each of the following to make it a perfect square quadratic expression.
2 2
 3
(a) (b)
 10 
x2 + 10x +  2  x2 + 3x +  2 

2 2

(c) (d)
 –8   –5 
x2 – 8x +  2 
x2 – 5x +  2 

Class Activity 2
Objective: To discuss the cases when a quadratic equation ax2 + bx + c = 0 has two real distinct roots, two real equal
roots and no real roots.
1. Copy and complete the following table.

Equation ax2 + bx + c = 0 b2 – 4ac Roots of the equation

(a) x2 – 2x – 8 = 0 (–2)2 – 4(1)(–8) = 36 2 real distinct roots

(b) 2x + 5x – 1 = 0
2 2
5 – 4(2)(–1) = 33 2 real distinct roots

(c) 4x2 + 12x + 9 = 0 122 – 4(4)(9) = 0 2 real equal roots

(d) 9x2 – 30x + 25 = 0 302 – 4(9)(25) = 0 2 real equal roots

(e) 4x2 + 7x + 6 = 0 72 – 4(4)(6) = – 47 no real roots

(f ) 3x – x + 2 = 0
2 2
(–1) – 4(3)(2) = –23 no real roots

57

02 FWS3A(Exp)_ch2.indd 57 12/16/14 12:49 PM


© Star Publishing Pte Ltd. All rights reserved.

2. What is the sign of b2 – 4ac when a quadratic equation ax2 + bx + c = 0 has two distinct real roots?
Positive

3. Explain why there are no real roots to a quadratic equation ax2 + bx + c = 0 when b2 – 4ac is negative.
Square root of a negative number is not a real number. Hence, ax 2 + bx + c = 0 has no real roots when b 2 – 4ac is negative.

4. Under what condition will a quadratic equation ax2 + bx + c = 0 have two equal real roots? Explain your answer.
–b ± 0 –b
A quadratic equation ax 2 + bx + c = 0 has two real equal roots when b 2 – 4ac = 0 since 0 = 0. Hence, x = = .
2a 2a

There is only one real value of x, i.e. the two roots are repeated or equal.

Class Activity 3
Objective: To explore the characteristics of the quadratic graphs of y = (x – h)2 + k and y = –(x – h)2 + k, where h and
k are constants.

1.

(a) Using The Geometer’s Sketchpad, draw the graphs of y = x2, y = x2 – 4, y = (x – 3)2 + 2 and y = (x + 5)2 – 3 on the
same screen.

(b) Study the graphs, copy and complete the following table. Then consider the graph of y = (x – h)2 + k, where h and
k are constants.

Is the turning point


Line of Turning
Function y-intercept maximum
symmetry point
or minimum?

y = x2 0 x=0 (0, 0) Minimum


y = x2 – 4 –4 x=0 (0, – 4) Minimum
y = (x – 3)2 + 2 11 x=3 (3, 2) Minimum
y = (x + 5) – 3
2
22 x = –5 (–5, –3) Minimum
y = (x – h) + k
2 2
h +k x=h (h, k) Minimum

Chapter 2 More about Quadratic Equations


58

02 FWS3A(Exp)_ch2.indd 58 12/16/14 12:49 PM


© Star Publishing Pte Ltd. All rights reserved.

2.

(a) Using The Geometer’s Sketchpad, draw the graphs of y = –x2, y = –(x + 7)2, y = –(x + 3)2 + 4 and y = –(x – 4)2 – 1
on the same screen.

(b)
Study the graphs, copy and complete the following table. Then consider the graph of y = – (x – h)2 + k, where h and
k are constants.

Line of Turning Is the turning point
Function y-intercept
symmetry point maximum or minimum?

y = –x2 0 x=0 (0, 0) Maximum


y = –(x + 7)2
– 49 x = –7 (–7, 0) Maximum

y = –(x + 3)2 + 4 –5 x = –3 (–3, 4) Maximum

y = –(x – 4)2 – 1 –17 x=4 (4, –1) Maximum

y = –(x – h) + k
2 2
–h + k x=h (h, k) Maximum

Class Activity 4
Objective: To explore the characteristics of the quadratic graphs of y = (x – p)(x – q) and
y = –(x – p)(x – q), where p and q are constants.

1.

Using The Geometer’s Sketchpad, draw the graphs of y = (x – 1)(x – 4), y = (x + 2)(x – 1), y = –(x + 3)(x + 5) and
y = –(x + 1)(x – 3) on the same screen.

59

02 FWS3A(Exp)_ch2.indd 59 12/16/14 12:49 PM


© Star Publishing Pte Ltd. All rights reserved.

2. Observe the points of intersection of each of the above graph with the x-axis and complete the following table.

Points of intersection
Graph y-intercept Line of symmetry
with the x-axis

y = (x – 1)(x – 4) (1, 0) and (4, 0) 4 x = 2.5

y = (x + 2)(x – 1) (–2, 0) and (1, 0) –2 x = –0.5

y = –(x + 3)(x + 5) (–3, 0) and (–5, 0) –15 x = –4

y = –(x + 1)(x – 3) (–1, 0) and (3, 0) 3 x=1

3. (a) Does the graph of y = (x – p)(x – q), where p and q are constants, open upward or downward?
Opens upward

(b) At which points does the graph of y = (x – p)(x – q) cut the x-axis?
(p, 0) and (q, 0)

(c) If the line of symmetry is x = h, what is the relationship between h, p and q?


p +q
h=
2

4. (a) Does the graph of y = –(x – p)(x – q), where p and q are constants, open upward or downward?
Opens downward

(b) At which points does the graph of y = –(x – p)(x – q) cut the x-axis?
(p, 0) and (q, 0)

(c) If the line of symmetry is x = h, what is the relationship between h, p and q?


p +q
h=
2

Chapter 2 More about Quadratic Equations


60

02 FWS3A(Exp)_ch2.indd 60 12/16/14 12:49 PM


© Star Publishing Pte Ltd. All rights reserved.

Try It! (b)


Section 2.1    y
1. Solve the equation 2x + 9x – 5 = 0.
2
y = x2 + x – 4
8
Solution
2x2 + 9x – 5 = 0 6
(2x – 1)(x + 5) = 0
2x – 1 = 0 or x + 5 = 0 4
1
x= or x = –5
2
2

2. Solve the equation (9x – 1)(x – 2) = –7x – 2.


x
–3 –2 –1 O 1 2 3
Solution
(9x – 1)(x – 2) = –7x – 2 –2
9x2 – 19x + 2 = –7x – 2
9x2 – 12x + 4 = 0 –4
(3x – 2)2 = 0
\ 3x – 2 = 0
2
x=
3 (c) From the graph, the solutions of x2 + x – 4 = 0 are
x = –2.6 and x = 1.6 (correct to 1 d.p.).
3. Solve the equation 2x(5x + 3) = 9(5x + 3).
5. (a) Copy and complete the following table.
Solution
2x(5x + 3) = 9(5x + 3) x –5 –4 –3 –2 –1 0 1
2x(5x + 3) – 9(5x + 3) = 0 y = x + 4x + 4
2

(5x + 3)(2x – 9) = 0
5x + 3 = 0 or 2x – 9 = 0 (b) Draw the graph of y = x2 + 4x + 4 for –5 < x < 1
x=–
3
or x=
9 using a scale of 2 cm to 1 unit on the x-axis and
5 2 1 cm to 1 unit on the y-axis.
(c) Use the graph to solve the equation x2 + 4x + 4 = 0.

Section 2.2 Solution


(a)
4. (a) Copy and complete the following table. x –5 –4 –3 –2 –1 0 1
y = x + 4x + 4
2
9 4 1 0 1 4 9
x –3 –2 –1 0 1 2 3
y = x2 + x – 4 (b)
y
(b) Draw the graph of y = x + x – 4 for –3  x  3
2

using a scale of 2 cm to 1 unit on the x-axis and 10


1 cm to 1 unit on the y-axis.
y = x2 + 4x + 4
(c) Hence solve the equation x2 + x – 4 = 0 graphically. 8

Solution
6
(a)
x –3 –2 –1 0 1 2 3
4
y = x2 + x – 4 2 –2 –4 –4 –2 2 8

x
–5 –4 –3 –2 –1 O 1

61

02 FWS3A(Exp)_ch2.indd 61 12/16/14 12:49 PM


© Star Publishing Pte Ltd. All rights reserved.

(c) From the graph, the root of x2 + 4x + 4 = 0 is Solution


x = –2. (a) y = –2t2 – t + 15

t 0 0.5 1 1.5 2 2.5 3


6. (a) Copy and complete the following table.
y 15 14 12 9 5 0 –6
x –2 –1 0 1 2 3
y = x2 – x + 3
y
(b) Draw the graph of y = x – x + 3 for –2  x  3
2

using a scale of 2 cm to 1 unit on the x-axis and 15


1 cm to 1 unit on the y-axis.
(c) Use the graph to solve the equation x2 – x + 3 = 0.
y = –2t2 – t + 15
Solution
(a) 10
x –2 –1 0 1 2 3
y = x2 – x + 3 9 5 3 3 5 9

(b) y 5

10
y = x2 – x + 3

8 t
O 1 2 3

4 –5

(b) From the graph,


x
–2 –1 O 1 2 3 (i) the distance between A and B = 15 m.
(ii) the parcel reached point A at 2.5 seconds.

(c) From the graph, the equation x2 – x + 3 = 0 has no


8. (a)
Draw the graph of y = 3x2 – 2x + 4 for –2  x  3
real roots.
using a scale of 2 cm to 1 unit on the x-axis and
2 cm to 5 units on the y-axis.
7. A parcel is sliding down from the top B of a ramp (b) Use the graph to find the values of x for which
AB to the ground. Its distance, y m, from the lowest (i) 3x2 – 2x + 4 = 12,
point A, at time t seconds is given by the equation (ii) 3x2 – 2x – 3 = 0,
y = –2t2 – t + 15. (iii) 3x2 – 2x + 4 = x + 6.
B
Solution
y (a) y = 3x2 – 2x + 4

A C x –2 –1 0 1 2 3

(a)
Draw the graph of y = –2t2 – t + 15 for 0  t  3, y 20 9 4 5 12 25
using the scale of 2 cm to 1 unit for the t-axis and
1 cm to 2 units on the y-axis.
(b) Use your graph to find
(i) the distance between A and B,
(ii) the time when the parcel reached point A.

Chapter 2 More about Quadratic Equations


62

02 FWS3A(Exp)_ch2.indd 62 12/16/14 12:49 PM


© Star Publishing Pte Ltd. All rights reserved.

y x2 – 12x – 5
2 2
y = 3x2 – 2x + 4 
= x2 – 12x +  −
12 


–  −
12 
 –5
25 2 2
= x2 – 12x + (–6)2 – 41
20 = (x – 6)2 – 41
(b) x2 + 3x + 1
15 The constant term that will make x2 + 3x into a
y = 12  3
2
perfect square is   .
2
10 6
y=x+
x2 + 3x + 1
y=7 2 2
5 = x2 + 3x +
 3
  –
 3
  +1
2 2
2
x
=  x +
3 5
–2 –1 O 1 2 3  –
2 4

10. Solve the equation x2 + 2x – 15 = 0 by completing the
(b) (i) 3x2 – 2x + 4 = 12 square.
i.e. y = 12
Draw the line y = 12 on the graph in (a). Solution
Hence, the required roots are x = –1.3 and
x2 + 2x – 15 = 0
x = 2. x2 + 2x = 15
(ii) 3x2 – 2x – 3 = 0  2
2
 2
2
x2 + 2x + = 15 +
3x2 – 2x = 3  2   2 

3x2 – 2x + 4 = 3 + 4 x2 + 2x + 1 = 15 + 1
i.e. y =7 (x + 1)2 = 16
Draw the line y = 7 on the graph in (a).
x + 1 = ± 16
Hence, the required roots are x = –0.7 and
x + 1 = – 4 or x + 1 = 4
x = 1.4.
x = –5 or x=3
(iii) 3x2 – 2x + 4 = x + 6
i.e. y =x+6
11. Solve the equation x2 – 3x – 5 = 0 by completing the
Draw the graph of y = x + 6 on the graph
square.
in (a).
Solution
x –1 0 3
x2 – 3x – 5 = 0
y 5 6 9 x2 – 3x = 5
2 2
 3  3

x2 – 3x +   =5+  
From the graph, the line y = x + 6 intersects 2 2
the curve y = 3x2 – 2x + 4 at x = –0.5 and 2
 x –
3 29
x = 1.5 (correct to 1 d.p.).  =
2 4
Hence, the required roots are x = –0.5 and
x = 1.5. x–
3
=–
29
or x –
3
=
29
2 4 2 4

Section 2.3 x=
3

29
or x=
3
+
29

9.
2 4 2 4
Express each of the following in the form (x + h) + k, 2

where h and k are constants. x = –1.19 or x = 4.19


(a) x2 – 12x – 5 (correct to 3 s.f.)
(b) x2 + 3x + 1

Solution
(a) x2 – 12x – 5
The constant term that will make x2 – 12x into a
2
 12 
perfect square is  −  .
2

63

02 FWS3A(Exp)_ch2.indd 63 12/16/14 12:49 PM


© Star Publishing Pte Ltd. All rights reserved.

12. Solve the equation x2 + x + 1 = 0 by completing the square. Section 2.5


3x – 1 x–3
Solution 16. Solve the equation = .
2x + 5 x+4
x +x+1=0 2

x2 + x = –1 Solution
2 2
 1  1 3x – 1 x–3
x +x+
2
 2 
=  2 
–1 =
2x + 5 x+4
 1
2
3 (3x – 1)(x + 4) = (x – 3)(2x + 5)
 x +  =–
2 4 3x2 + 11x – 4 = 2x2 – x – 15
1 3 x2 + 12x + 11 = 0
x + =± − (x + 1)(x + 11) = 0
2 4
1 3
x + 1 = 0 or x + 11 = 0
x=– ± − \ x = –1 or x = –11
2 4
Since the square root of a negative number does not exist,
the equation has no real roots. x+2 1
17. Solve the equation + = 2.
x x–2

Section 2.4
Solution
13. Solve the equation 4x2 + 7x – 3 = 0, giving your answers x+2 1
correct to 2 decimal places. + =2
x x–2
( x – 2)(x + 2) + x
Solution =2
4x + 7x – 3 = 0
2 x ( x – 2)
From the equation, a = 4, b = 7 and c = –3. x2 – 4 + x = 2x(x – 2)
–b ± b 2 – 4 ac
x2 – 4 + x = 2x2 – 4x
x= x2 – 5x + 4 =0
2a
(x – 1)(x – 4) =0
x – 1 = 0 or x – 4 = 0
2
–7 ± 7 – 4(4)(–3)
=
2(4) \ x = 1 or x= 4
–7 ± 97
=
8 x 1 5
= –2.11 or 0.36 (correct to 2 d.p.) 18. Solve the equation – = .
x–2 x+3 ( x – 2)( x + 3)

14. Solve the equation (9x + 5)(x + 2) = –x – 6 using the Solution


quadratic formula.
x

1
=
5
x–2 x+3 ( x – 2)( x + 3)
Solution
x(x + 3) – 1(x – 2) = 5
(9x + 5)(x + 2) = –x – 6 x2 + 3x – x + 2 = 5
9x2 + 23x + 10 = –x – 6 x2 + 2x – 3 = 0
9x2 + 24x + 16 = 0 (x + 3)(x – 1) = 0
x =
–24 ± 24 2 – 4(9)(16) x = –3 (rejected) or x = 1
2(9) \ x= 1
–24 ± 0
=
18
4 3x − 1 5
= – 19. Solve the equation 2 + = .
3 x +1 x–2

15. Solve the equation 3x2 + 11 = 5x. Solution


3x − 1 5
Solution 2+ =
x +1 x–2
3x2 + 11 = 5x 2(x + 1)(x – 2) + (3x – 1)(x – 2) = 5(x + 1)
3x2 – 5x + 11 = 0 2x2 – 2x – 4 + 3x2 – 7x + 2 = 5x + 5
–(–5) ± (–5) 2 – 4(3)(11) 5x2 – 14x – 7 = 0
x=
2(3) –(–14) ± (–14) 2 – 4(5)(–7)
x=
5± –107 2(5)
=
6 14 ± 336
=
–107 is not a real number, 10
\ the equation has no real roots. \ x = –0.433 or x = 3.23 (correct to 3 s.f.)

Chapter 2 More about Quadratic Equations


64

02 FWS3A(Exp)_ch2.indd 64 12/16/14 12:50 PM


© Star Publishing Pte Ltd. All rights reserved.

Section 2.6 x – 15 = 0 or x + 18 = 0
20. A rectangular swimming pool x = 15 or x = –18 (rejected)
measures 20 m by 16 m. A \ x = 15
path of uniform width is built The cyclist’s speed is 15 km/h.
16
around the pool. If the area of
the path is 100 m2, find the Section 2.7
width of the path, giving your 20
23. Sketch the graph of y = (x – 2)2 + 3.
answer correct to 3 significant
figures. Solution
y = (x – 2)2 + 3
Solution The minimum point is (2, 3).
Let x m be the width of the path. The line of symmetry is x = 2.
(20 + 2x)(16 + 2x) – (20)(16) = 100 When x = 0, y = (0 – 2)2 + 3
320 + 72x + 4x2 – 320 = 100 = 7
4x2 + 72x – 100 = 0 The diagram below shows the graph of
x2 + 18x – 25 = 0 y = (x – 2)2 + 3.
–18 ± 18 2 – 4(1)(–25) y
x =
2(1)
x=2
–18 ± 424
=

2
= 1.30 or –19.3 (rejected) (correct to 3 s.f.) y = (x – 2)2 + 3
7
\ x = 1.30 6
The width of the path is 1.30 m.
4

21. Mrs Li spent $300 to buy some premium tea leaves. 2 (2, 3)
If the price of the tea leaves had been reduced by $50 x
per kg, she could have bought 0.5 kg more. Find the O 1 2 3
original price per kg of the tea leaves.

Solution
Let the price of the tea leaves be $x/kg. 24. Sketch the graph of y = –x2 + 6x – 11.
300 300
– = 0.5 Solution
x – 50 x
300x – 300(x – 50) = 0.5x(x – 50) y = –x2 + 6x – 11
300x – 300x + 15 000 = 0.5x2 – 25x = – (x2 – 6x) – 11
x2 – 50x – 30 000 = 0  2  –6  
2 2
 –6 
= –  x – 6 x +   –    – 11
x2 – 50x – 30 000 = 0  2 2

(x – 200)(x + 150) = 0 = – (x – 6x + 9) + 9 – 11
2

x – 200 = 0 or x + 150 = 0 = – (x – 3)2 – 2


x = 200 or x = –150 (rejected) \ the maximum point is (3, –2).
\ x = 200 The line of symmetry is x = 3.
The price of the tea leaves is $200/kg.
When x = 0, y = – (0 – 3)2 – 2
= –11
22. A cyclist cycles at a constant speed from P to Q. The The diagram below shows the graph of
distance between P and Q is 60 km. If he had increased y = –x2 + 6x – 11.
his speed by 3 km/h, the time taken would have been y
40 minutes less. Find his speed.

Solution
Let the cyclist’s speed be x km/h.
60 60 40 x
– = O 1 2 3 4 5
x x+3
(3, _ 2)
60
60( x + 3) – 60x 2
= 5
x ( x + 3) 3
540 = 2x(x + 3)
10 y = –x2 + 6x – 11
x2 + 3x – 270 = 0
11
(x – 15)(x + 18) = 0

x=3
65

02 FWS3A(Exp)_ch2.indd 65 12/16/14 12:50 PM


© Star Publishing Pte Ltd. All rights reserved.

25. Sketch the graph of y = (x + 2)(x – 1) indicating clearly (b) By symmetry, the maximum point is at
its intercepts with the axes. Find the coordinates of its x=
1
(0 + 60) = 30.
turning point. 2
When x = 30,
Solution y = 60(30) – 302 = 900
y = (x + 2)(x – 1) i.e. the maximum point is (30, 900).
y = [x – (–2)](x – 1) \ 30 vases must be produced.
\ the x-intercepts are –2 and 1. The maximum profit is $900.
When x = 0, y = (0 + 2)(0 – 1) = –2
\ the y-intercept is –2.
The following diagram shows the graph of
y = (x + 2)(x – 1).

x = –0.5
y
y = (x + 2)(x – 1)
4

_3 _2 _1 O x
1 2
_2

_4

By symmetry, the minimum point is at


1
x= (–2 + 1) = – 0.5.
2
When x = – 0.5, y = (– 0.5 + 2)(– 0.5 – 1) = –2.25
\ the minimum point is (– 0.5, –2.25).

26. Suppose the weekly profit y (in dollars) for producing


and selling x vases is given by y = 60x – x2.
(a) Sketch the graph of y = 60x – x2 for x  0.
(b) How many vases must be produced and sold in
order to maximise the weekly profit? What is the
maximum weekly profit?

Solution
(a) y = 60x – x2 = –x(x – 60)
The x-intercepts are 0 and 60.
The diagram below shows the graph of y = 60x – x2.

1000 y = 60x _ x2

500

x
O 10 20 30 40 50 60

Chapter 2 More about Quadratic Equations


66

02 FWS3A(Exp)_ch2.indd 66 12/16/14 12:50 PM


© Star Publishing Pte Ltd. All rights reserved.

Exercise 2.1 ( j) 14x2 – 29x = 15


Level 1 14x2 – 29x – 15 = 0
(7x + 3)(2x – 5) = 0
1. Solve the following equations.
7x + 3 = 0 or 2x – 5 = 0
(a) x2 – 4x + 3 = 0 (b) x2 – 2x – 8 = 0 3 5
(c) 2x2 – 3x + 1 = 0 (d) 15x2 + 8x + 1 = 0 x=– or x=
7 2
(e) 2x2 + x – 6 = 0 (f ) 3x2 – 2x – 5 = 0
(g) 8x2 + 10x + 3 = 0 (h) 4x2 + 12x + 9 = 0
Level 2
(i) 25x2 – 16 = 0 ( j) 14x2 – 29x = 15
2. Solve the following equations.
Solution (a) x2 – 2x = 5x – 12
(a) x2 – 4x + 3 = 0 (b) x(x + 11) = 2(x + 5)
(x – 1)(x – 3) = 0 (c) 3x(2x + 5) = 4(2x + 5)
x – 1 = 0 or x – 3 = 0 (d) 36 – 25x2 = 0
x = 1 or x=3 (e) (7x + 2)(2x – 3) = 5x – 6
(f ) (x + 1)2 = 9(x + 1)
(b) x2 – 2x – 8 = 0 (g) (2x – 1)2 – 4(2x – 1) – 5 = 0
(x + 2)(x – 4) = 0 (h) 2(3x + 4)2 – 5(3x + 4) + 3 = 0
x + 2 = 0 or x – 4 = 0
x = –2 or x=4 Solution
(c) 2x – 3x + 1 = 0
2 (a) x2 – 2x = 5x – 12
(2x – 1)(x – 1) = 0 x2 – 7x + 12 = 0
2x – 1 = 0 or x – 1 = 0 (x – 3)(x – 4) = 0
1 x – 3 = 0 or x – 4 = 0
x= or x=1
2 x = 3 or x=4
(d) 15x2 + 8x + 1 = 0 (b) x(x + 11) = 2(x + 5)
(3x + 1)(5x + 1) = 0 x2 + 11x = 2x + 10
3x + 1 = 0 or 5x + 1 = 0 x2 + 9x – 10 = 0
x =–
1
or x=–
1 (x + 10)(x – 1) = 0
3 5 x + 10 = 0 or x – 1 = 0
(e) 2x2 + x – 6 = 0 x = –10 or x=1
(x + 2)(2x – 3) = 0 (c) 3x(2x + 5) = 4(2x + 5)
x + 2 = 0 or 2x – 3 = 0 3x(2x + 5) – 4(2x + 5) = 0
x = –2 or x=
3 (2x + 5)(3x – 4) = 0
2 2x + 5 = 0 or 3x – 4 = 0
(f ) 3x2 – 2x – 5 = 0 x=–
5
or x=
4
(x + 1)(3x – 5) = 0 2 3
x + 1 = 0 or 3x – 5 = 0 (d) 36 – 25x2 = 0
x = –1 or x=
5 25x2 – 36 = 0
3 (5x + 6)(5x – 6) = 0
(g) 8x2 + 10x + 3 = 0 5x + 6 = 0 or 5x – 6 = 0
(4x + 3)(2x + 1) = 0 x=–
6
or x=
6
4x + 3 = 0 or 2x + 1 = 0 5 5

x=–
3
or x=–
1 (e) (7x + 2)(2x – 3) = 5x – 6
4 2 14x2 – 17x – 6 = 5x – 6
(h) 4x2 + 12x + 9 = 0 14x2 – 22x = 0
(2x + 3)2 = 0 2x(7x – 11) = 0
2x + 3 = 0 x = 0 or 7x – 11 = 0
11
x=–
3 x = 0 or x=
2 7

(i) 25x2 – 16 = 0 (f ) (x + 1)2 = 9(x + 1)


(5x + 4)(5x – 4) = 0 (x + 1) – 9(x + 1) = 0
2

5x + 4 = 0 or 5x – 4 = 0 (x + 1)[(x + 1) – 9] = 0
4 4 x + 1 = 0 or x – 8 = 0
x=– or x=
5 5 x = –1 or x=8

67

02 FWS3A(Exp)_ch2.indd 67 12/16/14 12:50 PM


© Star Publishing Pte Ltd. All rights reserved.

(g) (2x – 1)2 – 4(2x – 1) – 5 = 0 5. When a trolley slides down a runway, its distance
[(2x – 1) + 1][(2x – 1) – 5] = 0 travelled, s m, at time t seconds is given by the formula
2x(2x – 6) = 0 s = 10t + 2t2. Find the time that the distance travelled is
x = 0 or 2x – 6 = 0 48 m.
x = 0 or x= 3
Solution
(h) 2(3x + 4)2 – 5(3x + 4) + 3 = 0
s = 10t + 2t 2
[2(3x + 4) – 3][(3x + 4) – 1] = 0
When s = 48
(6x + 8 – 3)(3x + 4 – 1) = 0
10t + 2t 2 = 48
6x + 5 = 0 or 3x + 3 = 0
2t 2 + 10t – 48 = 0
5
x = – or x = –1 t 2 + 5t – 24 = 0
6
(t + 8)(t – 3) = 0
t+8=0 or t–3=0
Level 3 t = –8 (rejected) t=3
\ the distance travelled is 48 m at time 3 seconds.
3. The sum of the areas of two squares is 269 cm2. The
side of one square is 3 cm longer than that of the other
square. Find the length of a side of the smaller square. 6. Tiles are laid to form the following patterns.

x x+3
Pattern 1 Pattern 2 Pattern 3

Solution (a) Draw the tiles for pattern 4.


Let x cm be the length of a side of the smaller square. (b) Write down the number of tiles for pattern n in
x2 + (x + 3)2 = 269 terms of n and explain your answer.
x + x2 + 6x + 9 = 269
2
(c) Find the value of n if pattern n has 153 tiles.
2x2 + 6x – 260 = 0
x2 + 3x – 130 = 0 Solution
(x – 10)(x + 13) = 0 (a)
x – 10 = 0 or x + 13 = 0
x = 10 or x = –13 (rejected)
\ x = 10
The length of a side of the smaller square is 10 cm.

4. The three sides of a right-angled triangle are (x + 2) cm,


(2x + 3) cm and (3x – 1) cm. Find the value of x. Pattern 4

(b)
Let the number of tiles for pattern n be an2 + bn + c,
where a, b and c are constants.
3x _ 1
x+2 When n = 1,
a(1)2 + b(1) + c = 1
a + b + c = 1.................... (1)
2x + 3
When n = 2,
Solution a(2)2 + b(2) + c = 3
By Pythagoras’ Theorem, 4a + 2b + c = 3.................... (2)
(3x – 1)2 = (x + 2)2 + (2x + 3)2 When n = 3,
9x2 – 6x + 1 = x2 + 4x + 4 + 4x2 + 12x + 9 a(3)2 + b(3) + c = 6
4x2 – 22x – 12 = 0 9a + 3b + c = 6.................... (3)
2x2 – 11x – 6 = 0 (2) – (1) 3a + b = 2.................... (4)
(x – 6)(2x + 1) = 0 (3) – (2) 5a + b = 3....................(5)
x – 6 = 0 or 2x + 1 = 0 (5) – (4) 2a = 1
1
x= 6 or x= –
1
(rejected) \ a=
2 2
\ x= 6

Chapter 2 More about Quadratic Equations


68

02 FWS3A(Exp)_ch2.indd 68 12/16/14 12:50 PM


© Star Publishing Pte Ltd. All rights reserved.

Putting a =
1
into (4): Exercise 2.2
2
 1 Level 1
3   + b = 2
2 1. (a) Copy and complete the following table.
b=
1
2 x –1 0 1 2 3 4 5
1 1
Putting a = and b = into (1): y = x2 – 4x + 4
2 2

1 1
+ +c=1
2 2 (b) Taking 2 cm to represent 1 unit on the x-axis and
\ c=0 1 cm to represent 1 unit on the y-axis, draw the
Hence the number of tiles for pattern n is given by graph of y = x2 – 4x + 4 for –1  x  5.
n2 +
1 1
n. (c) Us e yo u r g r a p h t o s o l ve t h e e q u a t i o n
2 2 x2 – 4x + 4 = 0.
1 1
(c) n2 + n = 153
2 2 Solution
n + n = 306 2
(a)
n2 + n – 306 = 0 x –1 0 1 2 3 4 5
(n – 17)(n + 18) = 0
y = x2 – 4x + 4 9 4 1 0 1 4 9
n – 17 = 0 or n + 18 = 0
n = 17 or n = –18 (rejected)
(b)
\ n = 17
y

7.
Sumin solved the equation 3x(x + 4) = x(x + 1) as follows. 10
Is her solution correct? Explain your answer. y = x2 – 4x + 4
3x(x + 4) = x(x + 1) 8
3(x + 4) = x + 1
3x + 12 = x + 1
2x = –11 6
11
\ x= –
2 4

Solution
No. The root x = 0 was missed out. 2
Sumin cancelled x in the second step.
x
–1 O 1 2 3 4 5
8.Peter solved the equation (5x – 2) = (2x + 1) as follows.
2 2

Is his solution correct? Explain your answer.


(5x – 2)2 = (2x + 1)2 (c) The graph of y = x2 – 4x + 4 touches the x-axis at
Taking square roots of both sides, x = 2 only. Hence, the root of x2 – 4x + 4 = 0 is
5x – 2 = 2x + 1 x = 2.
3x = 3
\ x = 1.
2. (a) Copy and complete the following table.
Solution x –1 0 1 2 3 4 5
(5x – 2)2 = (2x + 1)2
When taking square roots of both sides, we should have
y = 3x – x
2

5x – 2 = ±(2x + 1).

\ Peter has missed out one root. His solution is not
(b) Draw the graph of y = 3x – x2 for –1  x  5
correct.
using a scale of 2 cm to 1 unit on the x-axis and
The roots of this equation should be 1 cm to 1 unit on the y-axis.
x = 1 or x=
1
. (c) Use your graph to solve the equation 3x – x2 = 0.
7

69

02 FWS3A(Exp)_ch2.indd 69 12/16/14 12:50 PM


© Star Publishing Pte Ltd. All rights reserved.

Solution (b)
(a)
x –1 0 1 2 3 4 5 y

y = 3x – x2 –4 0 2 2 0 –4 –10 y = x2 – 2x + 2
10

(b) 8
y
6
4
4
2
2
x
–1 O 1 2 3 4 5
x
–2 –1 O 1 2 3 4
–2

–4 (c) The graph of y = x2 – 2x + 2 does not meet the


x-axis.
   –6 Hence, the equation x2 – 2x + 2 = 0 has no real
roots.
y = 3x – x2
–8
Level 2
4. Taking 2 cm to represent 1 unit on the x-axis and
–10 1 cm to represent 1 unit on the y-axis, draw the graph of
y = x2 – 2x – 3 for –2  x  4.
(a) Use your graph to solve the equation x2 = 2x + 3.
(c) From the graph, the roots of 3x – x 2 = 0 are (b) By drawing suitable straight lines on the same axes,
x = 0 and x = 3. find the values of x for which
(i) x2 – 2x = 6,
(ii) x2 – x – 3 = 0.
3. (a) Copy and complete the following table.
Solution
x –2 –1 0 1 2 3 4 y = x2 – 2x – 3
y = x2 – 2x + 2 x –2 –1 0 1 2 3 4

(b) Draw the graph of y = x2 – 2x + 2 for –2  x  4 y 5 0 –3 –4 –3 0 5

using a scale of 2 cm to 1 unit on the x-axis and


1 cm to 1 unit on the y-axis. y
(c) Use your graph to solve the equation x2 – 2x + 2 = 0.
6
Solution y = x2 – 2x – 3
(a)
x –2 –1 0 1 2 3 4 4
y=3
y = x – 2x + 2 10
2
5 2 1 2 5 10
2

x
–2 –1 O 1 2 3 4

–2

–4
y = –x

Chapter 2 More about Quadratic Equations


70

02 FWS3A(Exp)_ch2.indd 70 12/16/14 12:50 PM


© Star Publishing Pte Ltd. All rights reserved.

(a) x2 = 2x + 3 (a) 2x2 + 5x = 1


x – 2x – 3 = 0
2
2x2 + 5x – 1 = 0
i.e. y=0 i.e. y =0
From the graph, the required roots are x = –1 and From the graph, the required roots are x = –2.7 and
x = 3. x = 0.2.
(b) (i) x2 – 2x = 6 (b) (i) 2x2 + 5x – 5 = 0
x2 – 2x – 3 = 6 – 3 2x2 + 5x – 1 – 4 = 0
i.e. y=3 2x2 + 5x – 1 = 4
Draw the line y = 3 on the graph above. i.e. y =4
Hence, the required roots are x = –1.65 and Draw the line y = 4 on the graph above.
x = 3.65. Hence, the required roots are x = –3.25 and
(ii) x2 – x – 3 = 0 x = 0.75.
x2 – x – x – 3 = –x (ii) 2x2 + x – 6 = 0
x2 – 2x – 3 = –x 2x2 = –x + 6
i.e. y = –x 2x + 5x – 1 = –x + 6 + 5x – 1
2

Draw the line y = –x on the graph above. i.e. y = 4x + 5


Draw the line y = 4x + 5 on the graph above.
x –22 0 4
x –3 0 2
y = –x 2 0 –4
y –7 5 13
Hence, the required roots are x = –1.3 and
x = 2.3. Hence, the required roots are x = –2 and
x = 1.5.
5.
Using a scale of 2 cm to 1 unit on the x-axis and 1 cm to
2 units on the y-axis, draw the graph of y = 2x2 + 5x – 1 6.
Draw the graph of y = 3x2 – 2x + 4 for –2  x  3
for – 4  x  2. using a scale of 2 cm to 1 unit on the x-axis and 2 cm to
(a) Use your graph to solve the equation 2x2 + 5x = 1. 5 units on the y-axis.
(b) By drawing suitable straight lines on the same axes, (a) Use your graph to solve the equation 3x2 = 2x – 4.
find the values of x for which (b) By drawing suitable straight lines on the same axes,
(i) 2x2 + 5x – 5 = 0, find the values of x for which
(ii) 2x2 + x – 6 = 0. (i) 3x2 – 2x – 10 = 0,
(ii) 3x2 – 5 = 0.
Solution
y = 2x2 + 5x – 1 Solution
y = 3x2 – 2x + 4
x –4 –3 –2 –1 0 1 2
x –2 –1 0 1 2 3
y 11 2 –3 –4 –1 6 17
y 20 9 4 5 12 25

y
y
20
y = 3x2 – 2x + 4
25
15
y = 2x2 + 5x – 1 20
10
15 y = 14
y=4 5
10
x
–4 –3 –2 –1 O 1 2 y = –2x + 9
5
–5
y = 4x + 5
x
–2 –1 O 1 2 3

71

02 FWS3A(Exp)_ch2.indd 71 12/16/14 12:50 PM


© Star Publishing Pte Ltd. All rights reserved.

(a) 3x2 = 2x – 4 (a) x2 + 2x – 6 = 0


3x – 2x + 4 = 0
2
Multiply equation by (–1):
i.e. y=0 –x2 – 2x + 6 = 0
The graph of y = 3x2 – 2x + 4 does not meet the 6 – 2x – x2 = 0
x-axis. i.e. y=0
Hence the equation 3x2 = 2x – 4 has no real roots. Hence, the required roots are x = –3.65 and
x = 1.65.
(b) (i) 3x2 – 2x – 10 = 10
3x2 – 2x = 10 (b) (i) x2 + 2x – 2 = 0
3x2 – 2x + 4 = 10 + 4 x2 + 2x = 2
i.e. y = 14 –x2 –2x = –2
Draw the line y = 14 on the graph above. 6 – 2x – x2 = 6 – 2
Hence, the required roots are x = –1.5 and i.e. y=4
x = 2.2. Draw the line y = 4 on the graph above.
(ii) 3x2 – 5 = 0 Hence, the required roots are x = –2.7 and
3x2 = 5 x = 0.7.
3x – 2x + 4 = 5 – 2x + 4
2
(ii) 3 – x – x2 = 0
i.e. y = –2x + 9 –x2 = x – 3
Draw the line y = –2x + 9 on the graph above. 6 – 2x – x2 = 6 – 2x + x – 3
i.e. y =3–x
x –2 0 2
Draw the line y = 3 – x on the graph above.
y 13 9 5
x –3 0 2
Hence, the required roots are x = –1.3 and y 6 3 1
x = 1.3.
Hence, the required roots are x = –2.3 and
7.
Using a scale of 2 cm to 1 unit on the x-axis and 1 cm x = 1.3.
to 1 unit on the y-axis, draw the graph of y = 6 – 2x – x2
for – 4  x  2. 8.
Draw the graph of y = –3x2 + 6x + 2 for –2  x  4
(a) Use your graph to solve the equation x2 + 2x – 6 = 0. using a scale of 2 cm to 1 unit on the x-axis and 2 cm
(b) By drawing suitable straight lines on the same axes, to 5 units on the y-axis.
find the values of x for which (a) Use your graph to solve the following equations
(i) x2 + 2x – 2 = 0, (i) 3x2 – 6x – 2 = 0,
(ii) 3 – x – x2 = 0. (ii) 7 + 6x – 3x2 = 0.
(b) On the same axes, draw the graph of 5x – y = 4.
Solution (c) (i) Write down the x-coordinate of the points at
y = 6 – 2x – x2 which the two graphs intersect.
x –4 –3 –2 –1 0 1 2 (ii) These values of x are the solutions of the
equation –3x2 + bx + c = 0. Find the values
y –2 3 6 7 6 3 –2 of b and c.

y Solution
y = –3x2 + 6 + 2
8
x –2 –1 0 1 2 3 4

6 y –22 –7 2 5 2 –7 –22

y=4
4

2
y=3–x

_4 x
–3 –2 –1 O 1 2

–2
y = 6 – 2x – x2

  

Chapter 2 More about Quadratic Equations


72

02 FWS3A(Exp)_ch2.indd 72 12/16/14 12:50 PM


© Star Publishing Pte Ltd. All rights reserved.

y (b)
On the same axes, draw the graph of x + 2y = 4.
(c)
(i) Write down the x-coordinate of the points at
5x – y = 4
5 which the two graphs intersect.
(ii) Find the equation, in the form x2 + bx + c = 0,
which is satisfied by the values of x found
x
–2 –1 O 1 2 3 4 in part (c)(i).
_5 y = –5
Solution
1
y= x2 + 2x
2
–10
x –6 –5 –4 –3 –2 –1 0 1 2
–15 y 6 2.5 0 –1.5 –2 –1.5 0 2.5 6

–20
y
y = –3x2 + 6x + 2
–25 y = 1 x2 + 2x 6
2

   5

(a) (i) 3x2 – 6x – 2 = 0 4


Multiply equation by (–1):
–3x2 + 6x + 2 = 0
3
i.e. y=0
Hence, the required roots are x = –0.3 and
x = 2.3. 2

(ii) 7 + 6x – 3x2 = 0 x + 2y = 4
–3x2 + 6x + 2 + 5 = 0 1
–3x2 + 6x + 2 = –5
i.e. y = –5 _6 _5 _4 _3 _2 _1 O
x
1 2
Draw the line y = –5 on the graph above.
Hence, the required roots are x = –0.8 and _
1 y = –1
x = 2.8.
(b) 5x – y = 4 _2
y = 5x – 4
x –2 0 2
1
  (a) x2 + 2x + 1 = 0
y –14 –4 6 2
1
x2 + 2x = –1
Draw the line 5x – y = 4 on the graph above. 2
i.e. y = –1
(c) (i)
The two graphs intersect at the points where Draw the line y = –1 on the graph above.
x = –1.25 and x = 1.6. Hence, the required roots are x = –3.4 and x = –0.6.
(ii) Where the two graphs y = –3x2 + 6x + 2 and
y = 5x – 4, intersect, (b) x + 2y = 4
–3x2 + 6x + 2 = 5x – 4 2y = –x + 4
–3x2 + x + 6 = 0 y = – x + 2
1
2
The values of x in part (c)(i) are the solutions
of the equation –3x2 + x + 6 = 0. x –6 0 2
Hence, b = 1 and c = 6.
y 5 2 1
1

9. Draw the graph of y = x2 + 2x for –6  x  2 using Draw the line x + 2y = 4 on the graph above.
2
a scale of 2 cm to 1 unit on the x-axis and 1 cm to 1 unit (c) (i) The two graphs intersect at the points where
on the y-axis. x = –5.7 and x = 0.7.
(a) Use your graph to solve the equation
1
x2 + 2x + 1 = 0.
2

73

02 FWS3A(Exp)_ch2.indd 73 12/16/14 12:50 PM


© Star Publishing Pte Ltd. All rights reserved.

(ii) Where the two graphs, (b) When x = 0, y = –160. This means that before any
1 1
y = x2 + 2x and y = – x + 2, intersect, vase is produced, Sam has incurred a negative
2 2 earning of $160. This amount of money is spent

1 2 1
– x + 2x = – x + 2 to purchase materials to make his vases.
2 2
1 2 5 (c) (i) Maximum value of y occurs when x = 12.
x + x – 2 = 0
2 2 \ for maxium daily earning, the number of
Multiplying the equation by 2, we have vases Sam should produce is 12.
(ii) Draw the line y = 100 on the graph in (a).
x2 + 5x – 4 = 0.
From the graph, for y > 100, we have
Hence, the equation x2 + 5x – 4 = 0 is satisfied 8.3 < x < 15.8.
by the values of x found in (c)(i). \ the possible numbers of vases produced
such that the daily earning is at least $100
are 9, 10, 11, 12, 13, 14 and 15.
10.
Sam’s daily earning, $y, of producing x vases in a day
is given by y = –2x2 + 48x – 160. Level 3
(a) Draw the graph of y = –2x 2 + 48x – 160 for
11. A dolphin leapt out of the sea, dived back into the sea
0  x  22 using a scale of 1 cm to represent
and then leapt out again. It was observed that the height,
2 vases on the x-axis and 2 cm to represent $50
h metres, of the dolphin’s leap above the sea level at time
on the y-axis.
t seconds, is given by h = t 2 – 5t + 1 for 0  t  5.
(b) Find the value of y when x = 0 and interpret its
(a) D r aw t h e g r a ph of h = t 2 – 5t + 1 for
physical meaning.
0  t  5 using a scale of 2 cm to 1 unit on the
(c) Use your graph to find
horizontal axis and 1 cm to 1 unit on the vertical axis.
(i) the number of vases Sam should produce so
(b) Determine graphically the times at which the
that his daily earning is a maximum,
dolphin was at sea level.
(ii) the possible numbers of vases produced such
that the daily earning is at least $100. Solution
(a)
Solution x 0 1 2 3 4 5
(a) y = –2x + 48x – 160
2
h = t 2 – 5t + 1 1 –3 –5 –5 –3 1

x 0 2 4 6 8 10 12
h

y –160 –72 0 56 96 120 128 2

t
x 14 16 18 20 22 O 1 2 3 4 5
_2
y 120 96 56 0 –72 h = t 2 _ 5t + 1
_4
y _6

150

y = 100 (b) From the graph, the roots of t  2 – 5t + 1 = 0 are
100 t = 0.2 or t = 4.8 (correct to 1 d.p.).
The times that the dolphin was at sea-level are
50 0.2 second and 4.8 seconds.

x 12. The cost, $C1, of making a hanging ornament of height


O 4 8 12 16 20 24 x cm using material A is given by C1 = 2x2 + 7x + 3.
_50 The cost, $C 2 , of making the ornament of height
x cm using material B is given by C2 = x2 + 10x + 11.
Let y = C1 – C2.
_100
y = –2x2 + 48x – 160 (a) Express y in terms of x.
(b) Draw the graph of y against x for 0  x  8 using
_150 a scale of 1 cm to 1 unit on the x-axis and 1 cm to
5 units on the y-axis.
(c) Hence find the value of x such that the costs of
making the ornament from material A and from
material B are the same.
Chapter 2 More about Quadratic Equations
74

02 FWS3A(Exp)_ch2.indd 74 12/16/14 12:50 PM


© Star Publishing Pte Ltd. All rights reserved.

Solution  8
2

(a)
C1 = 2x2 + 7x + 3 (c)
x2 – 8x +  –  = (x – 4)2
 2
C2 = x2 + 10x + 11 \ the required term = 16.
y = C1 – C2
= 2x2 + 7x + 3 – x2 – 10x – 11
2 2
 7
=  x –
7
(d) x2 + 7x +  2  
y = x2 – 3x – 8 2
49
(b) \ the required term = .
x 0 1 2 3 4 5 6 7 8 4

y
2 2
–8 –10 –10 –8 –4 2 10 20 32  1
=  x +
1
(e) x2 + x + 
 2  2
1
y \ the required term = .
4
35 2 2
x2 – 11x +  –
11 
=  x –
11 
(f )  
2 2
30
121
y = x2 – 3x – 8 \ the required term = .
4
25

20 2. Express each of the following in the form (x + h)2 + k,


where h and k are constants.
15 (a) x2 + 2x + 5
(b) x2 – 6x – 3
10 (c) x2 + 5x – 6
(d) x2 – 9x + 4
5
Solution
2 2
 2  2
O 1 2 3 4 5 6 7 8
x (a) x2 + 2x + 5 = x2 + 2x +  2  –  2  +5
_5 = (x + 1)2 + 4
2 2
 6  6
_10 (b) x2 – 6x – 3 = x2 – 6x +  –  –  –  –3
2 2
= (x – 3) – 12 2

2 2
 5  5
(c) x2 + 5x – 6 = x2 + 5x +  –  –6
(c) When the costs C1 and C2 are the same, 
2

2
C1 – C2 = 0  5
2
1
i.e. y =0 =  x +  – 12
2 4
The graph y = x2 – 3x – 8 intersects the x-axis at
x = 4.7.  9
2
 9
2
(d) x2 – 9x + 4 = x2 – 9x +  –  –  –  +4
Hence, the required value of x is 4.7. 2 2
2
 9
=  x –
1
– 16
2  4
Exercise 2.3
Level 1
3. Solve the following equations, giving your answers correct
1. Find the constant term that must be added to each of the to 3 significant figures where necessary.
following expressions to make it a perfect square. (a) (x – 1)2 = 25
(a) x2 + 2x (b) x2 – 4x (b) (x + 3)2 = 49
(c) x – 8x
2
(d) x2 + 7x 2
 1
(e) x + x
2
(f ) x2 – 11x (c) 
x +  =1
2
2
Solution 
–  = 9
5
2 (d)  x
 2 2 4
(a) x2 + 2x +  2  = (x + 1)2
(e) (x + 4) = 72

\ the required term = 1. (f ) (x – 3)2 = 10


2
 4
(b) x2 – 4x +  –  = (x – 2)2
2
\ the required term = 4.

75

02 FWS3A(Exp)_ch2.indd 75 12/16/14 12:50 PM


© Star Publishing Pte Ltd. All rights reserved.

Solution (b) x2 – 8x – 12 = 0
(a) (x – 1) = 25 2
x2 – 8x = 12
x – 1 = ± 25  8
2
 8
2
x2 – 8x +  –  = 12 +  – 
x – 1 = –5 or x – 1 = 5  2  2
\ x = – 4 or x=6 (x – 4)2 = 28
(b) (x + 3)2 = 49 x – 4 = ± 28
x + 3 = ± 49 x – 4 = – 28 or x – 4 = 28
x + 3 = –7 or x + 3 = 7 x = 4 – 28 or x = 4 + 28
\ x = –10 or x=4 \ x = –1.29 or x = 9.29 (correct to
2 3 s.f.)
1 
 (c)
 x + =1 (c) x2 – 3x + 1 = 0
2
1 x2 – 3x = –1
x+ =± 1 2 2
 3  3
2
x2 – 3x + –
 2 
= –1 + –
 2 
1 1
x+ = –1 or x + =1
2 2  3
2
5
3 1  x –  =
\ x=– or x= 2 4
2 2
3 5
 5
2 x– =±
 x – 2 
9
(d) = 2 4
4
3 5 3 5
5 9 x= – or x = +

x– =± 2 2 2 2
2 4 \ x = 0.382 x = 2.62 (correct to 3 s.f.)

x–
5
=–
3
or x –
5
=
3
2 2 2 2 (d) x2 + 5x – 2 = 0
\ x=1 or x=4 x2 + 5x = 2
2 2
(e) (x + 4) = 72
x2 + 5x +  5
= 2 + 
5
 2  
2
x+4=± 7 2
 x + 
5 33
=
x = – 4 – 7 or x = – 4 + 7 2 4

\ x = –6.65 or x = –1.35 (correct to 3 s.f.) x +


5

33
2 4
(f ) (x – 3)2 = 10
5 33 5 33
x – 3 = ± 10 x=– – or x = – +
2 2 2 2
x = 3 – 10 or x = 3 + 10 \ x = –5.37 or x = 0.372 (correct to 3 s.f.)
x = – 0.162 or x = 6.16 (correct to 3 s.f.)

5. By completing the square, solve the following equations,


4. By completing the square, solve the following equations, giving your answers correct to 2 decimal places.
giving your answers correct to 3 significant figures where (a) x2 – 12x + 26 = 0
necessary. (b) x2 – 9x – 21 = 0
(a) x2 + 4x + 2 = 0 (c) x2 + 7x – 5 = 0
(b) x2 – 8x – 12 = 0 (d) x2 – x – 1 = 0
(c) x2 – 3x + 1 = 0
(d) x2 + 5x – 2 = 0 Solution
(a) x2 – 12x + 26 = 0
Solution x2 – 12x = –26
(a) x2 + 4x + 2 = 0  12 
2
 12 
2
x2 + 4x = –2
x2 – 12x +  – = –26 +  –
2  2 
2 2
 4  4
x2 + 4x +   = –2 +   (x – 6)2 = 10
2 2
(x + 2)2 = 2 x – 6 = ± 10
x+2=± 2 x = 6 – 10 or x = 6 + 10
\ x = 2.84 or x = 9.16 (correct to 2 d.p.)
x + 2 = – 2 or x + 2 = 2
x = –2 – 2 or x = –2 + 2
\ x = –3.41 or x = –0.586 (correct to
3 s.f.)

Chapter 2 More about Quadratic Equations


76

02 FWS3A(Exp)_ch2.indd 76 12/16/14 12:50 PM


© Star Publishing Pte Ltd. All rights reserved.

(b) x2 – 9x – 21 = 0 (b) x2 – 4 = 5x
x2 – 9x = 21 x2 – 5x = 4
2 2 2 2
 9  9  5  5
x2 – 9x +  – = 21 +  – x2 – 5x +  – = 4 +  –
2  2  2  2 
2 2
 9  5
 x
165
 x
41
– = – =
2  4 2  4
9 165 5 41
x– =± x– =±
2 4 2 4
9 165 9 165 5 41 5 41
x= – or x = + x= – or x = +
2 4 2 4 2 4 2 4
\ x = –1.92 or x = 10.92 (correct to 2 d.p.) \ x = –0.702 or x = 5.70 (correct to 3 s.f.)

(c) x2 + 7x – 5 = 0 (c) x(x – 5) = 7(x + 1)


x2 + 7x = 5 x2 – 5x = 7x + 7
 7
2
 7
2 x2 – 12x = 7
x2 + 7x +   =5+   2 2
2 2  12   12 
x2 – 12x +  – = 7 +  –
2 2  2 
 x +
7 69
=

2 4 (x – 6)2 = 43
7 69 x – 6 = ± 43
x+ =±
2 4 x = 6 – 43 or x = 6 + 43
7 69 7 69 \ x = – 0.557 or x = 12.6 (correct to 3 s.f.)
x=– – or x = – +
2 2 2 2
\ x = –7.65 or x = 0.65 (correct to 2 d.p.) (d) (x – 1)(x + 8) = 1 – 6x
x2 + 7x – 8 = 1 – 6x
(d) x2 – x – 1 = 0 x2 + 13x = 9
x2 – x = 1  13 
2
 13 
2
x2 + 13x +  =9+
 2   2 
2 2
 1  1
x2 – x +  – = 1 + –
 2   2   13 
2
205
2  x + =

x–  =
1 5  2  4
 2 4
13 205
1 5 x+ =±
x– =± 2 4
2 4
13 205 13 205
x =
1

5
or x =
1
+
5 x =– – or x = – +
2 2 2 2 2 2 2 2
\ x = – 0.62 or x = 1.62 (correct to 2 d.p.) \ x = –13.7 or x = 0.659 (correct to 3 s.f.)

Level 2 (e) (x – 4)2 = 25x


x – 8x + 16 = 25x
2

6. By completing the square, solve the following equations, x2 – 33x = –16


giving your answers correct to 3 significant figures where 2 2
 33   33 
necessary. x2 – 33x +  – = –16 +  –
2  2 
(a) x2 + 18x = 13 2
(b) x2 – 4 = 5x

 x –
33 
=
1025
2 
(c) x(x – 5) = 7(x + 1) 4

(d) (x – 1)(x + 8) = 1 – 6x x+
33

1025
(e) (x – 4)2 = 25x 2 4
(f ) (3 – x)2 = –1 33 1025 33 1025
x= – or x = +
2 4 2 4
Solution \ x = 0.492 or x = 32.5 (correct to 3 s.f.)
(a) x2 + 18x = 13
 18 
2
 18 
2 (f ) (3 – x)2 = –1
x + 18x + 2
 2  = 13 +  2 
\ 3 – x = ± –1
(x + 9) = 94 2
Since the square root of a negative number is not
x + 9 = ± 94 a real number, the equation has no real roots.
x = –9 – 94 or x = –9 + 94
\ x = –18.7 or x = 0.695 (correct to
3 s.f.)

77

02 FWS3A(Exp)_ch2.indd 77 12/16/14 12:50 PM


© Star Publishing Pte Ltd. All rights reserved.

Level 3 11. What can you say about the roots of the equation
7. The cost of a square artwork of length x cm is $(x2 + 4x). (x – 1)2 = –1?
If an artwork costs $300, what is its length?
Solution
Solution (x – 1)2 = –1
x + 4x = 300 2
x – 1 = ± –1
x2 + 4x + 22 = 300 + 22
But –1 is not a real number.
(x + 2)2 = 304
\ the equation has no real roots.
x + 2 = ± 304
x = –2 + 304 or x = –2 – 304 (rejected)
\ x = 15.4 (correct to 3 s.f.) Exercise 2.4
The length of the artwork is 15.4 cm. Level 1
1. Solve the following equations using the quadratic formula,
8. When a stone is thrown vertically up, its height, h m, above giving your answers correct to 2 decimal places where
the ground at time t seconds is given by h = 20t – 5t2 necessary.
for 0 < t < 4. Find the possible times at which the stone (a) x2 – 12x + 36 = 0
is 10 m above the ground. Give your answer correct to (b) 2x2 + 6x + 1 = 0
2 decimal places. (c) 3x2 – 2x – 5 = 0
Solution
(d) 5x2 – x + 4 = 0
20t – 5t 2 = 10 (e) 7x2 + x – 2 = 0
t 2 – 4t = –2 (f ) 8x2 + 9 = 0
2 2
 4  4
t 2 – 4t +  –  = –2 +  –  Solution
2 2
(t – 2)2 = 2 (a) x2 – 12x + 36 = 0
t–2=± 2 12 ±
2
12 – 4(1)(36)
x=
t = 2 – 2 or t = 2 + 2 2(1)
\ t = 0.59 or t = 3.41 (correct to 2 d.p.)
=
12
Hence, the possible times at which the stone is 10 m 2
above the ground are 0.59 second and 3.41 seconds. \ x=6

(b) 2x2 + 6x + 1 = 0
9. The area, A cm2, of a rectangle is given by A = x(20 – x),
–6 ± 6 2 – 4(2)(1)
where x cm is its length. Find the length of the rectangle x=
2(2)
if its area is 80 cm 2. Give your answer correct to
3 significant figures.
=
– 6 ± 28
4
Solution –6 – 28 – 6 ± 28
x= or x =
x(20 – x) = 80 4 4
20x – x2 = 80 \ x = –2.82 or x = –0.18 (correct to 2 d.p.)
x2 – 20x = –80
2 2 (c) 3x2 – 2x – 5 = 0
x2 – 20x +  –
20 
= –80 +  –
20 
 2   2  2±
2
2 – 4(3)(–5)
(x – 10)2 = 20 x=
2(3)
x – 10 = ± 20
=
2 ±8
6
x = 10 – 20 or x = 10 + 20
\ x = –1 or x = 1.67 (correct to 2 d.p.)
\ x = 5.53 or x = 14.5 (correct to 3 s.f.)
Since x is the length of the rectangle, x . 20 – x. (d) 5x2 – x + 4 = 0
Hence, the length of the rectangle is 14.5 cm.
1± 12 – 4(5)(4)
x=
2(5)
10. Find a quadratic equation whose roots are
1± –79
2 – 3 and 2 + 3 .
=
10
Solution \ the equation has no real roots.
The required quadratic equation is:
(x – 2)2 = 3
x – 4x + 4 = 3
2

i.e. x2 – 4x + 1 = 0.
Chapter 2 More about Quadratic Equations
78

02 FWS3A(Exp)_ch2.indd 78 12/16/14 12:50 PM


© Star Publishing Pte Ltd. All rights reserved.

(e) 7x2 + x – 2 = 0 (c) (3x – 2)2 + 7 = 0


9x2 – 12x + 4 + 7 = 0
–1 ± 12 – 4(7)(–2)
x= 9x2 – 12x + 11 = 0
2(7)
12 ± 12 2 – 4(9)(11)
–1 + 57 x=

= 2(9)
14
12 ± –252
x=
–1 – 57
or x =
–1 + 57
=
14 14 18

\ x = – 0.61 or x = 0.47 (correct to 2 d.p.) \ the equation has no real roots.

(f ) 8x2 + 9 = 0 (d) x(1 – 2x) + 9 = 0


0± 2
0 – 4(8)(9) x – 2x2 + 9 = 0
x = 2x2 – x – 9 = 0
2(8)
1± 12 – 4(2)(–9)

=
–288 x=
16 2(2)

\ the equation has no real roots. 1± 73

=
4

1– 73 1+ 73
x= or x =
Level 2 4 4
2. Solve the following equations using the quadratic formula, \ x = –1.89 or x = 2.39 (correct to 3 s.f.)
giving your answers correct to 3 significant figures where (e) (1 – x)(4 + x) = 5
necessary. 4 – 3x – x2 = 5
(a) x + 7x(1 – x) = 3x – 26 x2 + 3x + 1 = 0
(b) (4x –5)(x + 2) = 4x + 1 −3 ± 32 – 4(1)(1)
(c) (3x – 2)2 + 7 = 0 x=
2(1)
(d) x(1 – 2x) + 9 = 0 −3 ± 5
(e) (1 – x)(4 + x) = 5
=
2
3
(f ) x2 = x+2 −3 – 5 −3 + 5
4 x= or x =
2 2
Solution \ x = –2.62 or x = –0.382 (correct to 3 s.f.)
(a)
x + 7x(1 – x) = 3x – 26 3
x + 7x – 7x2 = 3x – 26 (f ) x2 = x +2
4
7x2 – 5x – 26 = 0 4x = 3x + 8 2

5± 2
(–5) – 4(7)(–26)
4x2 – 3x – 8 = 0
x= 3± (–3) 2 – 4(4)(–8)
2(7)
x=
2(4)
5± 753

=
14 –3 ± 137

=
5– 753 5+ 753 8
x= or x =
14 14
3– 137 3+ 137
\ x = –1.60 or x = 2.32 (correct to 3 s.f.) x= or x =
8 8

(b) (4x –5)(x + 2) = 4x + 1 \ x = –1.09 or x = 1.84 (correct to 3 s.f.)


4x2 + 3x – 10 = 4x + 1
4x2 – x – 11 = 0 3. Solve the following equations.

2
(–1) – 4(4)(–11) (a) (x – 2)(x + 1) = 8
x=
2(4) (b) 3x2 – 4 = 7x
1± 177 (c) (x – 2)(x + 2) = 4(x – 2)

=
8 (d) 5x(x + 1) = 3(x – 2)
1– 177 1± 177 (e) (2x + 1)2 = (x – 3)2
x= or x =
2
8 8
(f ) x(x – 6) = (x2 + 1)
\ x = –1.54 or x = 1.79 (correct to 3 s.f.) 3

79

02 FWS3A(Exp)_ch2.indd 79 12/16/14 12:50 PM


© Star Publishing Pte Ltd. All rights reserved.

Solution Level 3
(a) (x – 2)(x + 1) = 8 4. It is given that the sum of the first n positive even integers
x2 – x – 2 = 8 is equal to n(n + 1). That is
x – x – 10 = 0
2
2 + 4 + 6 + … + 2n = n(n + 1).
x=
1± 12 – 4(1)(–10) (a) Find the value of 2 + 4 + 6 + … + 1000.
2(1) (b) If the sum of the first n positive even integers is
1± 41 6006, find the value of n.

=
2
1– 41 1+ 41 Solution
x = or x = (a) Given that
2 2
2 + 4 + 6 + ... + 2n = n(n + 1).
\ x = –2.70 or x = 3.70 (correct to 3 s.f) Let 1000 = 2n
Hence, n = 500
(b) 3x – 4 = 7x
2
\ 2 + 4 + 6 + ... + 1000
3x2 – 7x – 4 = 0
= 500(500 + 1)
7 2 – 4(3)(–4)
x=

= 250 500
2(3)
(b) n(n + 1) = 6006
7± 97
= n2 + n – 6006 = 0
6
–1 ± 12 – 4(1)(–6006)
7– 97 7+ 97 n=
x = or x = 2(1)
6 6
\ x = – 0.475 or x = 2.81 (correct to 3 s.f.)
=
–1 ± 24 025
2
(c) (x – 2)(x + 2) = 4(x – 2) –1 + 24 025 –1 – 24 025
n= or n = (rejected)
(x – 2)(x + 2) – 4(x – 2) = 0 2 2
(x – 2)(x + 2 – 4) = 0 \ n = 77
(x – 2)(x – 2) = 0
\ x=2
5. A ball is thrown from the top of a building. Its vertical
(d) 5x(x + 1) = 3(x – 2) distance, h metres, from the ground after time t seconds,
5x2 + 5x = 3x – 6 is given by h = 50 + 10t – 5t 2.
5x + 2x + 6 = 0
2

2
–2 ± 2 – 4(5)(6)
x=
2(5)
–2 ± –116

=
110
h
\ the equation has no real roots.

(e) (2x + 1)2 = (x – 3)2


4x2 + 4x + 1 = x2 – 6x + 9 (a) Find the height of the building.
3x2 + 10x – 8 = 0 (b) When is the ball at the same level as the top of the
(x + 4)(3x – 2) = 0 building again?
x + 4 = 0 or 3x – 2 = 0 (c) When does the ball reach the ground?
2 (d) When is the ball 30 m above the ground?
\ x = –4 or x=
3
Solution
2
(f ) x(x – 6) = (x2 + 1) (a) h = 50 + 10t – 5t  2
3
3x(x – 6) = 2(x2 + 1) When t = 0,
3x2 – 18x = 2x2 + 2 h = 50 + 10(0) – 5(0)2
x – 18x – 2 = 0
2 = 50
18 ±
2
(–18) – 4(1)(–2)
\ the height of the building is 50 m.
x=
2(1) (b) When h = 50,
18 ± 332 50 = 50 + 10t – 5t  2

= 5t   – 10t = 0
2
2
5t(t – 2) = 0
18 – 332 18 + 332
x= or x = t = 0 or t = 2
2 2
\ the required time is 2 seconds.
\ x = –0.110 or x = 18.1 (correct to 3 s.f.)

Chapter 2 More about Quadratic Equations


80

02 FWS3A(Exp)_ch2.indd 80 12/16/14 12:50 PM


© Star Publishing Pte Ltd. All rights reserved.

(c) When h = 0, 10 + 180 100 10 – 180 100


x = or x=
0 = 50 + 10t – 5t  2 6 6
t   – 2t – 10 = 0
2
(rejected)
2± 2
2 – 4(1)(–10) x = 72 (to the nearest km/h)
t=
2(1) Hence, the estimated speed of the car is
2± 44 72 km/h.

= (ii) Yes, the car exceeded the speed limit as
2
2+ 44 2– 44
72 km/h more than the speed limit 60 km/h.
t = or t= (rejected)
2 2
= 4.32 (correct to 3 s.f.)
\ the ball reaches the ground after 4.32 seconds. Exercise 2.5
Level 1
(d) When h = 30,
30 = 50 + 10t – 5t  2 1. Solve the following equations.
1
5t   – 10t – 20 = 0
2
(a) x+ =2
x
t  2 – 2t – 4 = 0 x –1 x
(b) =
2 4 x+3
2± 2 – 4(1)(–4)
t= x –1 x–6
2(1) (c) =
2x – 5 x+2
2± 20

= x 24
2 (d) +1=
2 x
2+ 20 2– 20
t = or t= (rejected) (e)
x–2
=1–
1
2 2 5 2x – 3
= 3.24 (correct to 3 s.f.) 1 1 5
\ t he ball is 30 m above the ground after (f ) x+3
+ =
x–3 8
3.24 seconds. ( x – 2)( x – 3) 5
(g) =
( x – 1)( x – 5) 3
6.
The braking distance, d m, of a car after the brakes are (h)
7–x

1
=
1
applied for a car travelling at the speed of x km/h can x+2 x –1 3
be modelled by the equation d = 0.006x2 – 0.02x.
(a) What is the braking distance if the speed of the Solution
car is 60 km/h? Give your answer correct to the (a) x +
1
=2
nearest metre. x
(b) In a 60 km/h speed limit zone, a car caused an x + 1 = 2x 2

accident. The police estimated that its braking x2 – 2x + 1 = 0


distance was 30 m. (x – 1)2 = 0
(i) What is the estimated speed of the car before \ x=1
the brakes were applied? Give your answer x –1 x
to the nearest km/h. (b) =
4 x+3
(ii) Did the car exceed the speed limit? Give your (x – 1)(x + 3) = 4x
reason. x2 + 2x – 3 = 4x
x2 – 2x – 3 = 0
Solution
(x + 1)(x – 3) = 0
(a) d = 0.006x2 – 0.02x
x + 1 = 0 or x – 3= 0
When x = 60,
\ x = –1 or x=3
d = 0.006(60)2 – 0.02(60)
= 20.4 (c)
x –1
=
x–6
= 20 (correct to the nearest m) 2x – 5 x+2
Hence, the required braking distance is 20 m. (x – 1)(x + 2) = (x – 6)(2x – 5)
x2 + x – 2 = 2x2 – 17x + 30
(b) (i) 0.006x – 0.02x = 30 2
x2 – 18x + 32 = 0
6x2 – 20x – 30 000 = 0
(x – 2)(x – 16) = 0
3x2 – 10x – 15 000 = 0
x – 2 = 0 or x – 16 = 0
x =
10 ± (–10) 2 – 4(3)(–15 000) \ x = 2 or x = 16
2(3)
10 ± 180 100
=
6

81

02 FWS3A(Exp)_ch2.indd 81 12/16/14 12:50 PM


© Star Publishing Pte Ltd. All rights reserved.

x 24 Level 2
(d) +1=
2 x
2. Solve the following equations, giving your answers correct
x2 + 2x = 48 to 3 significant figures where necessary.
x2 + 2x – 48 = 0 x x+7
(x + 8)(x – 6) = 0 (a) + =5
x–2 x+2
x + 8 = 0 or x – 6= 0 3 2 x + 10
\ x = –8 or x=6 (b) – = 1
x –1 x + 2x – 3
2
3
x–2 1 4x – 3 x–5
(e) =1– (c) =2–
5 2x – 3 x–5 x–7
(x – 2)(2x – 3) = 5(2x – 3) – 5 1
(d)
4
2x2 – 7x + 6 = 10x – 15 – 5 – =5
x+3 x–3
2x2 – 17x + 26 =0
1 2 3
(x – 2)(2x – 13) =0 (e) + + =0
x x –1 x +1
x–2 = 0 or 2x – 13 = 0
5 2 3
\ x =2 or x=
13 (f ) – =
x – 15 x+6 x–9
2
1 1 5
(f ) x + 3 + = Solution
x–3 8
x x+7
( x – 3) + (x + 3) 5 (a) + =5
= x–2 x+2
( x + 3)(x – 3) 8
x(x + 2) + (x + 7)(x – 2) = 5(x – 2)(x + 2)
2x
=
5
x2 + 2x + x2 + 5x – 14 = 5x2 – 20
x –9
2
8
3x2 – 7x – 6 = 0
16x = 5x2 – 45 (3x + 2)(x – 3) = 0

5x – 16x – 45 = 0
2
\ x=–
2
or x = 3

(5x + 9)(x – 5) = 0 3
5x + 9 = 0 or x–5 =0
3 2 x + 10
\ x=–
9
or x =5 (b) – = 1
x –1 x + 2x – 3
2

5 3
3 2 x + 10 1
– =
( x – 2)( x – 3) 5 x –1 ( x – 1)( x + 3) 3
(g) =
( x – 1)( x – 5) 3 1 2
3(x + 3) – (2x + 10) = (x + 2x – 3)
3(x – 2)(x – 3) = 5(x – 1)(x – 5) 3
3(x2 – 5x + 6) = 5(x2 – 6x + 5) x–1=
1 2
(x + 2x – 3)
3x2 – 15x + 18 = 5x2 – 30x + 25 3
2x2 – 15x + 7 = 0 3x – 3 = x2 + 2x – 3
(2x – 1)(x – 7) = 0 x2 – x = 0
2x – 1 = 0 or x – 7 = 0 x(x – 1) = 0
1 x = 0 or x = 1 (rejected)
\ x= or x=7 \ x=0
2
4x – 3 x–5
(h)
7–x

1
=
1 (c) =2–
x–5 x–7
x+2 x –1 3
(4x – 3)(x – 7) = 2(x – 5)(x – 7) – (x – 5)(x – 5)
(7 – x )( x – 1) – (x + 2) 1 2 – 31x + 21 = 2x2 – 24x + 70 – x2 + 10x – 25
= 4x
( x + 2)(x – 1) 3
3x2 – 17x – 24 = 0
8x – x 2 – 7 – x – 2 1
= 17 ± (–17) 2 – 4(3)(–24)
x2 + x – 2 3 x=
2(3)
3(–x2 + 7x – 9) = x2 + x – 2
–3x2 + 21x – 27 = x2 + x – 2
=
17 ± 577
6
4x2 – 20x + 25 = 0
(2x – 5)2 = 0 x=
17 – 577
or x =
17 + 577
5 6 6
\ x= \ x = –1.17 or x = 6.84 (correct to 3 s.f.)
2

Chapter 2 More about Quadratic Equations


82

02 FWS3A(Exp)_ch2.indd 82 12/16/14 12:50 PM


© Star Publishing Pte Ltd. All rights reserved.

3 1
(d)
1

4
=5 (b) +5 =
x+3 x–2 x +1
x–3
x – 3 – 4(x + 3) 3(x + 1) + 5(x – 2)(x + 1) =x–2
=5
( x + 3)( x – 3) 3x + 3 + 5x2 – 5x – 10 =x–2
–3x – 15 = 5(x2 – 9) 5x2 – 3x – 5 =0
–3x – 15 = 5x2 – 45 3± (–3) 2 – 4(5)(–5)

5x2 + 3x – 30 = 0 x=
2(5)
2
–3 ± 3± 109
x=
3 – 4(5)(–30)

=
2(5) 10
3– 109

=
–3 ± 609 x= or x = 3+ 109
10 10 10

–3 + 609 –3 – 609 \ x = –0.744 or x = 1.34 (correct to 3 s.f.)


\ x = or x =
10 10 4 1
= 2.17 or = –2.77 (correct to 3 s.f.) (c) =6 +
x–3 x+3

1 2 3
4(x + 3) + (x – 3) = 6(x – 3)(x + 3)
(e) + + =0 4x + 12 + x – 3 = 6x2 – 54
x x –1 x +1
(x – 1)(x + 1) + 2x(x + 1) + 3x(x – 1) = 0 6x2 – 5x – 63 = 0
x2 – 1 + 2x2 + 2x + 3x2 – 3x = 0 5± (–5) 2 – 4(6)(–63)
6x2 – x – 1 = 0 x=
2(6)
(3x + 1)(2x – 1) = 0 5± 1537
1 1
=
\ x = – or x= 12
3 2
5– 1537 5+ 1537
5 2 3 x= or x =
(f )
  
– = 12 12
x – 15 x+6 x–9 \ x = –2.85 or x = 3.68 (correct to 3 s.f.)
5(x + 6)(x – 9) – 2(x – 15)(x – 9)
= 3(x – 15)(x + 6) 7 x
(d) – –2=0
x2 – x – 6 x+2
5x2 – 15x – 270 – 2x2 + 48x – 270
= 3x2 – 27x – 270 7 x
– –2=0
( x + 2)( x – 3) x+2
60x = 270
9 7 – x(x – 3) – 2(x + 2)(x – 3) = 0
\ x = 7 – x2 + 3x – 2x2 + 2x + 12 = 0
2
3x2 – 5x – 19 = 0
3. Solve the following equations, giving your answers correct
2
5± (–5) – 4(3)(–19)
x=
to 3 significant figures where necessary. 2(3)
5± 253
(a) x+
1
=4
=
x+3 6
1 5– 253 5+ 253
(b)
3
+5= x= or x =
x–2 x +1 6 6
4 1 \ x = –1.82 or x = 3.48 (correct to 3 s.f.)
(c) + =6
x–3 x+3
7 x Level 3
(d) – –2=0
x2 – x – 6 x+2 25
4. The sum of a number and its reciprocal is . Find the
12
possible numbers.
Solution
1
x+(a) =4
x+3 Solution

x(x + 3) + 1 = 4(x + 3) Let x be the required number.

x2 + 3x + 1 = 4x + 12 1 25
x+ =
x2 – x – 11 = 0 x 12

x=
1± (–1) 2 – 4(1)(–11) 12x2 + 12 = 25x
2(1) 12x2 – 25x + 12 = 0

=
1± 45 (4x – 3)(3x – 4) = 0
2 3 4
\ x= or x=
1– 45 1+ 45
x= or x = 4 3
2 2 3 4
Hence, the possible numbers are and .
\ x = –2.85 or x = 3.85 (correct to 3 s.f.) 4 3

83

02 FWS3A(Exp)_ch2.indd 83 12/16/14 12:50 PM


© Star Publishing Pte Ltd. All rights reserved.

5. Durians are sold at $y per kg in March. In April, the  7200


+ 20  (m – 6) = 600
(c) 7200 –  m
price rises by $2 per kg. The mass of durians that can 
1
be bought for $45 in April is kg less than the mass 7200 – 7200 +
43 200
– 20m + 120 = 600
2
m
that can be bought for $40 in March.
(a) Express, in terms of y, the mass of durians for $40
43 200
– 20m – 480 = 0
m
in March.
m + 24m – 2160 = 0
2
(shown)
(b) Express, in terms of y, the mass of durians for $45
in April. (d) m + 24m – 2160 = 0
2

(c) From the given information, form an equation in (m – 36)(m + 60) = 0


y and show that it reduces to y2 + 12y – 160 = 0. m = 36 or m = –60
(d) Solve the equation y2 + 12y – 160 = 0.
(e) m = –60 (rejected)
(e) Hence find the price of durians in March.
Hence, the original number of students, m = 36.
\ the original monthly fee of each student is
Solution
 7200 
40 $  = $200.
(a) Mass of durians for $40 in March = kg 36 
y
45
(b) Mass of durians for $45 in April = kg 7. A tin of 144 sweets is divided equally among x children.
y+2
If 4 children left the group, each of the remaining children
40 45 1
(c) – = will have 6 sweets more. Find the value of x.
y y+2 2
1
40(y + 2) – 45y = y(y + 2) Solution
2
144 144
1 2 – =6
40y + 80 – 45y = (y + 2y) x–4 x
2
144x – 144(x – 4) = 6x(x – 4)
2(80 – 5y) = y2 + 2y 576 = 6x2 – 24x
160 – 10y = y2 + 2y 6x2 – 24x – 576 =0
y + 12y – 160 = 0 (shown)
2
x2 – 4x – 96 =0
(d) y2 + 12y – 160 = 0 (x – 12)(x + 8) =0
(y – 8)(y + 20) = 0 x = 12 or x = –8 (rejected)
\ y = 8 or y = –20 \ x = 12
(e) y = –20 (rejected)
Hence the price of durians in March is $8 per kg. 8. Consider the following solution of a fractional equation.

4
6. Mrs Kwon runs a yoga class of m students. Her monthly 5
– =
10
.......................(1)
x–2 x x ( x – 2)
income is $7200.
(a) Express the monthly fee of each student in terms Multiplying both sides by x(x – 2), we have
of m.  5 4 10
(b) When 6 students drop the class, she increases the x(x – 2) 3  x–2 – = x(x – 2) 3 .....(2)
 x  x ( x – 2)
monthly fee of each student by $20. Express her 5x – 4(x – 2) = 10................................(3)
new monthly income in terms of m. 5x – 4x + 8 = 10................................(4)
(c) If her monthly income in (b) is $600 less than ∴ x = 2 ................................(5)
her original income, write down an equation
to represent the information, and show that it (a) Is the solution x = 2 valid?
simplifies to m2 + 24m – 2160 = 0. (b) Among equations (1) to (5), which equations are
(d) Solve the equation m2 + 24m – 2160 = 0. invalid when x = 2?
(e) Find the original monthly fee of each student.
Solution
Solution
(a) The solution x = 2 is not valid. This is because
 7200 
(a) Monthly fee of each student = $ the denominator of
5
in the original equation
m  x–2
 7200 is zero when x = 2.
(b) New monthly income = $  + 20  (m – 6)
 m 
(b) Equations (1) and (2) are invalid when x = 2.

Chapter 2 More about Quadratic Equations


84

02 FWS3A(Exp)_ch2.indd 84 12/16/14 12:50 PM


© Star Publishing Pte Ltd. All rights reserved.

Exercise 2.6 5. Two square tiles A and B are such that the side of B is
Level 1 5 cm longer than the side of A. The sum of the areas of
the tiles is 1525 cm2. Let x cm be the length of the side
1. The product of two consecutive positive integers is 1260.
of tile A.
Find the integers.
(a) Express the length of the side of tile B in terms
Solution of x.
Let x be the smaller integer. (b) Express the area of tile B in terms of x.
x(x + 1) = 1260 (c) Find the value of x.
x2 + x – 1260 = 0
(x – 35)(x + 36) = 0 Solution
x = 35 or x = –36 (rejected) (a) Length of the side of tile B = (x + 5) cm
\ x = 35 (b) Area of tile B = (x + 5)2 cm2
The integers are 35 and 36. (c) x2 + (x + 5)2 = 155
x2 + x2 + 10x + 25 = 1525
2. The sum of the squares of two consecutive positive odd 2x2 + 10x – 1500 = 0
numbers is 650. Find the numbers. x2 + 5x – 750 = 0
(x – 25)(x + 30) = 0
Solution x = 25 or x = –30 (rejected)
Let x be the smaller number. \ x = 25
x2 + (x + 2)2 = 650
x + x2 + 4x + 4 = 650
2

2x2 + 4x – 646 = 0 6.
A rectangle is 24 cm long and 17 cm wide. When its
x2 + 2x – 323 = 0 length decreases by x cm and its breadth increases by
(x – 17)(x + 19) = 0 x cm, its area is increased by 12 cm2.
x = 17 or x = –19 (rejected) (a) Express, in terms of x,
\ x = 17 (i) the new length,
The two odd numbers are 17 and 19. (ii) the new breadth,
(iii) the new area of the rectangle.
(b) Find the possible values of x.
3. The height of a triangle is 3 cm less than its base. The area
of the triangle is 90 cm2. Find the base of the triangle. Solution
Solution (a) (i) New length = (24 – x) cm
Let the base of the triangle be x cm. (ii) New breadth = (17 + x) cm
1 (iii) New area = (24 – x)(17 + x) cm2
x(x – 3) = 90
2 (b) (24 – x)(17 + x) = (24)(17) + 12
x2 – 3x – 180 = 0 408 + 7x – x2 = 408 + 12
(x – 15)(x + 12) = 0 x2 – 7x + 12 = 0
x = 15 or x = –12 (rejected) (x – 3)(x – 4) = 0
\ x = 15 \ x = 3 or x = 4
The base of the triangle is 15 cm.
7. The sum of the ages of Mingfa and his father is 34 years.
4. The perimeter of a rectangle is 68 cm and its area is In 4 years’ time, the square of Mingfa’s age will be equal
253 cm2. Suppose the length of the rectangle is x cm. to his father’s age. Find Mingfa’s present age.
(a) Express the breadth of the rectangle in terms of x.
(b) Find the dimensions of the rectangle. Solution
Let Mingfa’s present age be x years.
Solution
His father’s present age = (34 – x) years.
 68
(a) Breadth =  2 – x  (x + 4)2 = (34 – x) + 4

= (34 – x) cm x2 + 8x + 16 = 34 – x + 4
x2 + 9x – 22 = 0
(b) x(34 – x) = 253 (x – 2)(x + 11) = 0
34x – x2 = 253 x = 2 or x = –11 (rejected)
x2 – 34x + 253 = 0 \ Mingfa’s present age is 2 years.
(x – 11)(x – 23) = 0
x = 11 or x = 23
When x = 11, breadth = 34 – 11 = 23 cm.
When x = 23, breadth = 34 – 23 = 11 cm.
\ the dimensions of the rectangle are 23 cm by
11 cm.
85

02 FWS3A(Exp)_ch2.indd 85 12/16/14 12:50 PM


© Star Publishing Pte Ltd. All rights reserved.

8. A piece of wire is 76 cm long. It is cut into two unequal 11. The distance s metres travelled by a car in time t seconds
parts and each part is bent into a square. The sum of the after the first observation is given by s = 3t 2 + 4t. Find
areas of the two squares is 205 cm2. Find the length of the time taken for the car to travel 100 m.
the shorter part of the wire.
Solution
Solution s = 3t  2 + 4t
Let x cm be the length of the shorter part. When s = 100,
x
2
76 – x 
2 100 = 3t  2 + 4t
  +   = 205 3t   + 4t – 100 = 0
2
4 4 
–4 ± 4 2 – 4(3)(–100)
x2 5776 – 152x + x 2 t=
+ = 205 2(3)
16 16
–4 ± 1216
2x – 152x + 5776 = 3280
2
=
6
2x2 – 152x + 2496 = 0 t = 5.15 or t = – 6.48
(rejected)
x2 – 76x + 1248 = 0 (correct to 3 s.f.)
(x – 24)(x – 52) = 0 \ the required time taken is 5.15 seconds.
x = 24 or x = 52 (rejected)
\ the length of the shorter part is 24 cm. 12. A rod AB, 65 cm long, leans A
against a vertical wall. Its
lower end B is 25 cm from
Level 2 the wall. When it slides 65
9. The sum of the squares of three consecutive positive down to the position DE, it D
even numbers is 2360. Find the smallest number. is found that AD = BE. Find
(a) the length of AC, C
25 B E
Solution (b) the length of AD.
Let x be the smallest number.
x2 + (x + 2)2 + (x + 4)2 = 2360 Solution
x2 + x2 + 4x + 4 + x2 + 8x + 16 = 2360 (a) By Pythagoras’ Theorem,
3x2 + 12x – 2340 = 0   AC2 = AB2 – BC2
x2 + 4x – 780 = 0 = 652 – 252
(x – 26)(x + 30) = 0 = 3600
x = 26 or x = –30 (rejected)
\ AC = 3600
\ the smallest number is 26.
= 60 cm
(b) Let AD be x cm.
10. On a 21 cm by 29 cm rectangular sheet of paper, the Then CD = (60 – x) cm
typing area is a rectangle of area 380 cm2. There are CE = (25 + x) cm
margins of equal width on all four sides of the paper. CD2 + CE2 = DE2
Find the width of a margin, giving your answer correct (60 – x)2 + (25 + x)2 = 652
to the nearest 0.1 cm. 3600 – 120x + x2 + 625 + 50x + x2 = 4225
2x2 – 70x = 0
2x(x – 35) = 0
x = 35 or x = 0 (rejected)
typing 29 \ the length of AD is 35 cm.
area

13. A shop owner spent $540 to purchase a stock of computer


21 keyboards. If the price of each keyboard had been reduced
by $2, he could have bought 3 more keyboards. Find the
Solution price of one keyboard.
Let x cm be the width of a margin.
(29 – 2x)(21 – 2x) = 380 Solution
609 – 100x + 4x2 = 380 Let the price of a keyboard be $x.
4x2 – 100x + 229 = 0
540

540
=3
100 ±
2
100 – 4(4)(229) x–2 x
x= 540x – 540(x – 2) = 3x(x – 2)
2(4)
1080 = 3x2 – 6x
100 ± 6336
= 3x – 6x – 1080 = 0
2
8
x2 – 2x – 360 = 0
x = 2.6 or x = 22.4 (rejected) (correct to 1 d.p.)
(x – 20)(x + 18) = 0
\ the width of a margin is 2.6 cm.
x = 20 or x = –18 (rejected)
\ the price of a keyboard is $20.
Chapter 2 More about Quadratic Equations
86

02 FWS3A(Exp)_ch2.indd 86 12/16/14 12:50 PM


© Star Publishing Pte Ltd. All rights reserved.

14. The distance between two stations, A and B, is 100 km. Solution
If the average speed of a bus is increased by 5 km/h, Let Ada and Bob be 10 km apart at t p.m. .
the time taken by the bus to travel from A to B would be By Pythagoras’ Theorem,
10 minutes less. Find the original average speed of the bus. (4t)2 + [4(t – 1)]2 = 102
16t2 + 16(t2 – 2t + 1) = 100
Solution 16t2 + 16t2 – 32t + 16 = 100
Let the original speed of the bus be x km/h. 32t2 – 32t – 84 = 0
100 100 10 8t2 – 8t – 21 = 0
– =
x x+5 60 8± 8 2 – 4(8)(–21)
1 t =
100(x + 5) – 100x = x(x + 5) 2(8)
6
3000 = x2 + 5x =
8± 736
x + 5x – 3000 = 0
2 16

–5 ± 5 2 – 4(1)(–3000) t = 2.196 or t = –1.196 (rejected)


x= 2.196 hours = 2 hours 12 minutes
2(1)
(correct to the nearest minute)
=
–5 ± 12 025
\ they were 10 km apart at 2.12 p.m.
2
x = 52.3 or x = –57.3 (rejected) (corrected to 3 s.f.)
\ the required speed is 52.3 km/h. 17. A group of people hired a boat for $1200 and each agreed
to share the cost equally. Five people did not turn up, so
each person who went on the trip had to pay $8 more.
15. T h e f ig u r e s h ows a s q u a r e Find the number of people in the group who went on the
cardboard. A 6-cm square is cut trip.
from each of its corners and the
remaining cardboard is then folded Solution
along the dotted lines to form a Let the number of people who went on the trip be x.
square tray. If the volume of the 1200
6 1200
tray is 2000 cm3, find the length – =8
x x+5
6
of a side of the cardboard. 1200(x + 5) – 1200x = 8x(x + 5)
6000 = 8x(x + 5)
Solution 750 = x2 + 5x
Let the length of a side of the cardboard be x cm. x + 5x – 750 = 0
2

6(x – 12)2 = 2000 (x – 25)(x + 30) = 0


(x – 12)2 =
1000 x = 25 or x = –30 (rejected)
3 \ 25 people went on the trip.
1000
x – 12 = ±
3
18. The numerator of a positive fraction is 2 less than
x = 12 +
1000
or x = 12 –
1000 its denominator. When both the numerator and the
3 3
denominator are increased by 3, the new fraction is
x = 30.3 or x = – 6.26 (correct to 3 s.f.) 1
greater than the original one by . Find the original
\ the required length is 30.3 cm. 18
fraction.
Level 3
16. At noon, Ada left junction T and walked due north. At Solution
1.00 p.m., Bob left junction T and walked due east. Both Let the denominator of the fraction be x.
of them walked at a speed of 4 km/h. At what time would ( x − 2) + 3 x–2 1
– =
x+3 x
they be 10 km apart? Give your answer correct to the 18

nearest minute. x ( x + 1) – (x – 2)(x + 3) 1


=
x ( x + 3) 18
1

x2 + x – x2 – x + 6 = x(x + 3)
18
Ada 1 2
6 = (x + 3x)
18
x2 + 3x – 108 = 0
(x – 9)(x + 12) = 0
Bob
x = 9 or x = –12 (rejected)
T
The numerator = 9 – 2 = 7
7
\ the original fraction is .
9

87

02 FWS3A(Exp)_ch2.indd 87 12/16/14 12:50 PM


© Star Publishing Pte Ltd. All rights reserved.

19.
A shop owner has 80 copies of a computer game. Based The maximum total amount of sale is $5120.
on past experience, if the price is set at $60 per copy, Hence, the total amount of sale is maximum when
all of them will be sold. For each $5 increase in price, the price per copy is $80.
an additional 4 copies of the game will be unsold.
(a) If the price is set at $70 per copy,
20.
The capacity of the petrol tank of a car is 60 litres. The
(i) find the number of copies that will be sold,
petrol consumption rate of the car is x litres per 100 km
(ii) find the total sales amount.
on city roads and (x – 2) litres per 100 km on highways.
Let $(60 + 5n) be the price per copy of the game, where
(a) For a full tank of 60 litres of petrol, express, in
n is a positive integer.
terms of x, the number of kilometres that the car
(b) Express the number of copies sold in terms of n.
can drive on
(c) Express the total amount of sale in terms
(i) city roads, (ii) highways.
of n.
(b) For 60 litres of petrol, the car can drive 200 km
(d) Find the possible prices per copy if the total amount
longer on highways than on city roads.
of sale is $5100.
(i) Form an equation in x and show that it reduces
(e) What should the price per copy be in order to get
to x2 – 2x – 60 = 0.
the maximum total amount of sale?
(ii) Solve the equation x2 – 2x – 60 = 0.
Solution (iii) Find the distance that the car can drive on
(a) (i) Increase in price = $10 city roads with 40 litres of petrol.
10
No. of copies unsold = 4 3
5 Solution
=8 (a) No, of kilometres that the car can drive on
No. of copies sold = 80 – 8 60
= 72 (i) city roads = 3 100
x
(ii) Total sales amount = $70 3 72 6000
= $5040 =
x
(b) Number of copies sold = 80 – 4n (ii) highways =
60
3 100
x–2
(c) Total amount of sale
= $(60 + 5n) 3 (80 – 4n) =
6000
x–2
= $(4800 + 160n – 20n2)
(d) 4800 + 160n – 20n2 = 5100 (b) (i)
6000

6000
= 200
20n2 – 160n + 300 = 0 x–2 x
n2 – 8n + 15 = 0 6000x – 6000(x – 2) = 200x(x – 2)
(n – 3)(n – 5) = 0 12 000 = 200x2 – 400x
n = 3 or n = 5 200x – 400x – 12 000 = 0
2

When n = 3, x2 – 2x – 60 = 0 (shown)
60 + 5n = 60 + 5(3) (ii)
x – 2x – 60 = 0
2

= 75 2± (–2) 2 – 4(1)(–60)
When n = 5, x=
2(1)
60 + 5n = 60 + 5(5)
2± 244
= 85
=
2
Hence, the possible prices per copy are $75 and
$85. x=
2+ 244
or x =
2– 244
2 2
(e) We can set up a table as follow to observe the \ x= 8.81 or x = –6.81 (correct to 3 s.f.)
relationship between the price per copy and the
total amount of sale. (iii) x = –6.81 (rejected)
Hence the petrol consumption rate of the car
Price per copy No. of copies Sales is 8.81 litres per 100 km on city roads.
($) sold amount ($)
\ d istance travelled on city roads with
60 80 4800 40 litres of petrol
65 76 4940 40
70 72 5040
= 3 100
8.8102
75 68 5100 = 454 km (correct to 3 s.f.)
80 64 5120
85 60 5100
90 56 5040
95 52 4940

Chapter 2 More about Quadratic Equations


88

02 FWS3A(Exp)_ch2.indd 88 12/16/14 12:50 PM


© Star Publishing Pte Ltd. All rights reserved.

Exercise 2.7 (d) y = (x + 2)2 – 5


Level 1 The turning point is (–2, –5).
When x = 0, y = (0 + 2)2 – 5 = –1
1. State the turning point and sketch the graph of each of
the following functions. y
(a) y = x2 + 2
(b) y = (x + 1)2
(c) y = (x – 3)2 + 4
(d) y = (x + 2)2 – 5 y = (x + 2) 2 _ 5
(e) y = (x + 3)2 + 2 _4 _3 _2 _1 O x
(f ) y = (x – 4)2 _1 1

Solution
(a) y = x2 + 2 ( _2, _5)
The turning point is (0, 2).

y
(e) y = (x + 3)2 + 2
y = x2 + 2 The turning point is (–3, 2).
When x = 0, y = (0 + 3)2 + 2 = 11

2 y

_2 _1 O x
1 2
y = (x + 3)2 + 2
11
10
(b) y = (x + 1) 2

The turning point is (–1, 0).


When x = 0, y = (0 + 1)2 = 1
5
y

(–3, 2)
y = (x + 1) 2 x
–7 –6 –5 –4 –3 –2 –1 O 1
2
1 (f ) y = (x – 4)2
_3 _2 _1 O x The turning point is (4, 0).
2
When x = 0, y = (0 – 4)2 = 16
y

(c) y = (x – 3)2 + 4
The turning point is (3, 4).
When x = 0, y = (0 – 3)2 + 4 = 13 16
y = (x _ 4)2
y
8
15
y = (x _ 3) 2 + 4
10 x
O 2 (4, 0) 6 8
5
(3, 4)
x
O 1 2 3 4 5 6 2. State the line of symmetry and sketch the graph of each
of the following functions.
(a) y = –x2 – 3
(b) y = –(x – 4)2
(c) y = –(x + 1)2 – 2
(d) y = –(x – 2)2 + 3
(e) y = –(x + 2)2
(f ) y = –(x + 3)2 + 1

89

02 FWS3A(Exp)_ch2.indd 89 12/16/14 12:50 PM


© Star Publishing Pte Ltd. All rights reserved.

Solution (d) y = – (x – 2)2 + 3


(a) y = –x – 3
2
The line of symmetry is x = 2.
The line of symmetry is x = 0. The turning point is (2, 3).
The turning point is (0, –3). When x = 0, y = – (0 – 2)2 + 3 = –1
y y
(2, 3)
3
_2 _1 O x y = _(x _ 2) 2 + 3
1 2
_ 3 (0, –3)

y = _ x2 _ 3 O
x
_1 1 2 3 4

x=2

(b) y = – (x – 4)2
The line of symmetry is x = 4. (e) y = –(x + 2)2
The turning point is (4, 0). The line of symmetry is x = –2.
When x = 0, y = – (0 – 4)2 = –16 The turning point is (–2, 0).
y When x = 0, y = – (0 + 2)2 = –4
x=4

y
x
O 2 4 6 8
(–2, 0)
x
y = _(x _ 4) 2 O 1
_ 16

_4
y = _(x + 2) 2

(c) y = – (x + 1)2 – 2 x = –2
The line of symmetry is x = –1.
The turning point is (–1, –2).
When x = 0, y = – (0 + 1)2 – 2 = –3 (f ) y = –(x + 3)2 + 1
The line of symmetry is x = –3.
y The turning point is (–3, 1).
When x = 0, y = – (0 + 3)2 + 1 = –8

_3 _2 _1 x y
O 1
x = –3
( 1, _2) –1
_
–2
_3 y = _(x + 1) 2 – 2 (–3, 1)
x
–6 –5 –4 –3 –2 –1 O

–4
x = –1
–8

y = –(x + 3)2 + 1

Chapter 2 More about Quadratic Equations


90

02 FWS3A(Exp)_ch2.indd 90 12/16/14 12:50 PM


© Star Publishing Pte Ltd. All rights reserved.

3. State the x-intercepts, the y-intercept and sketch the graph The maximum point is at x =
1
(–2 – 3) = –2.5.
of each of the following functions. 2
(a) y = (x – 1)(x – 4) When x = –2.5, y = – (–2.5 + 2)(–2.5 + 3)
(b) y = (x + 3)(x – 1) = 0.25
(c) y = –(x + 2)(x + 3) \ the maximum point is (–2.5, 0.25).
(d) y = –x(x + 5) y

Solution (_ 2.5, 0.25)


(a) y = (x – 1)(x – 4) _4 _3 _2 _1 x
O 1
The x-intercepts are 1 and 4.
When x = 0, y = (0 – 1)(0 – 4) = 4 y = _ (x + 2)(x +3)
\ the y-intercept is 4. _6
1
The minimum point is at x = (1 + 4) = 2.5.
2
When x = 2.5, y = (2.5 – 1)(2.5 – 4)
(d) y = –x(x + 5)
= –2.25
The x-intercepts are 0 and –5.
\ the minimum point is (2.5, –2.25).
When x = 0, y = – 0 × (0 + 5) = 0
\ the y-intercept is 0.
y 1
The maximum point is at x = (–5 + 0) = –2.5.
2
4 When x = –2.5, y = – (–2.5)(–2.5 + 5)
y = (x _ 1)(x _ 4) = 6.25
\ the maximum point is (–2.5, 6.25)
y

x
O 1 2 3 4 5 (_ 2.5, 6.25)
6

(2.5, _2.25)

_6 _5 _4 _3 _2 _1 x
O 1
y = _ x(x + 5)
(b) y = (x + 3)(x – 1)
The x-intercepts are –3 and 1.
When x = 0, y = (0 + 3)(0 – 1) = –3 Level 2
\ the y-intercept is –3. 4. For each of the following,
1
The minimum point is at x = (–3 + 1) = –1. (i) express the function in the form y = (x – h)2 + k
2
When x = –1, y = (–1 + 3)(–1 – 1) = – 4 or y = – (x – h)2 + k,
\ the minimum point is (–1, – 4). (ii) state the coordinates of the turning point of the
function,
y
(iii) write down the equation of the line of symmetry
of the graph,
(iv) sketch the graph of the function.
y = (x + 3)(x _ 1) (a) y = x2 + 2x + 3
(b) y = –x2 + 8x – 5
x (c) y = 5x – x2
_4 _3 _2 _1 O 1 2 (d) y = x2 – 7x + 6
(e) y = x2 + 4x + 4
(f ) y = –x2 – 10x – 25
_3


_ _ Solution
( 1, 4) (a) (i) y = x2 + 2x + 3
= x2 + 2x + 1 + 2
(c) y = – (x + 2)(x + 3) = (x + 1)2 + 2
The x-intercepts are –2 and –3. (ii) The minimum point is (–1, 2).
When x = 0, y = – (0 + 2)(0 + 3) = – 6 (iii) The line of symmetry is x = –1.
\ the y-intercept is – 6. (iv) When x = 0, y = 02 + 2(0) + 3 = 3

91

02 FWS3A(Exp)_ch2.indd 91 12/16/14 12:50 PM


© Star Publishing Pte Ltd. All rights reserved.

y
(d) (i) y = x 2 – 7x + 6
2 2
 7  7
y= x2 + 2x + 3
= x2 – 7x +   –   +6
2 2
3 2
=  x –
7
 – 6.25
2
( _ 1, 2) (ii) The minimum point is (3.5, – 6.25).
(iii) The line of symmetry is x = 3.5.
_3 _2 _1 x
O 1 2 (iv) When x = 0, y = 02 – 7(0) + 6 = 6
y

x = –1
6 y = x 2 _ 7x + 6

(b) (i) y = –x2 + 8x – 5


= – (x2 – 8x) – 5
= – (x2 – 8x + 42) + 42 – 5 _2 x
O 2 4 6
= – (x – 4)2 + 11
(ii) The maximum point is (4, 11).
(iii) The line of symmetry is x = 4.
(iv) When x = 0, y = – 02 + 8(0) – 5 = –5 (3.5, _ 6.25)
x = 3.5
y
(4, 11) (e) (i) y = x2 + 4x + 4
10 = (x + 2)2
(ii) The minimum point is (–2, 0).
y = _x 2 + 8x _ 5 (iii) The line of symmetry is x = –2.
(iv) When x = 0, y = – 02 + 4(0) + 4 = 4
y
x x = 2.5
O 2 4 6 8

_5

x
x=4 –1 O 2.5 6

–6 y = –6 – 5x + x2
(c) (i) y = 5x – x2
y = –(x2 – 5x)
 2
 5 
2
 5

= –  x 2 – 5x +    +  
 2  2
2
 5
= –  x –
+
25
(f ) (i) y = –x2 – 10x – 25
2  4
= –(x2 + 10x + 25)
(ii) The maximum point is (2.5, 6.25). = –(x + 5)2
(iii) The line of symmetry is x = 2.5. (ii) The maximum point is (–5, 0).
(iv) When x = 0, y = 5(0) – 02 = 0. (iii) The line of symmetry is x = –5.
y
(iv) When x = 0, y = – 02 –10(0) – 25 = –25
y
(2.5, 6.25)
6 y = 5x _ x 2

(–5, 0)
x
O
x
O 1 2 3 4 5

x = 2.5
–25

y = –x2 – 10x – 25
x = –5

Chapter 2 More about Quadratic Equations


92

02 FWS3A(Exp)_ch2.indd 92 12/16/14 12:50 PM


© Star Publishing Pte Ltd. All rights reserved.

5.
For each of the following, (iii) The maximum point is at x =
1
(1 + 2)
(i) express the function in the form 2
y = (x – p)(x – q) or y = – (x – p)(x – q), = 1.5.
(ii) sketch the graph of the function, When x = 1.5, y = –(1.5 – 1)(1.5 – 2)
(iii) find the coordinates of the turning point of the = 0.25
function. ∴ the maximum point is (1.5, 0.25).
(a) y = x2 + 3x – 4 (c) (i) y = –x2 – 2x – 1
(b) y = –x2 + 3x – 2 = –(x2 + 2x + 1)
(c) y = –x2 – 2x – 1 = –(x + 1)2
(d) y = x2 – 4 (ii) The graph touches the x-axis at (–1, 0).
(e) y = –6 – 5x + x2 When x = 0, y = – 02 – 2(0) – 1 = –1
(f ) y = 6x – x2 – 9
y
x = –1
Solution
(a) (i) y = x2 + 3x – 4
_3 _2 _1 x
= (x + 4)(x – 1) O 1
(ii) The x-intercepts are – 4 and 1.
When x = 0, y = 02 + 3(0) – 4 = – 4 _1

y
y = _x 2 _ 2x _ 1
y= x2 + 3x _ 4
(iii) ∴ the maximum point is (–1, 0).
(d) (i) y = x2 – 4
x = (x + 2)(x – 2)
_4 O 1 (ii) The x-intercepts are –2 and 2.
_4 When x = 0, y = 02 – 4 = – 4
y

x = –1.5
y = x2 _ 4
1
(iii) The minimum point is at x = (– 4 + 1) x
2 _2 _1 O 1 2
= –1.5.
When x = –1.5,
y = (–1.5 + 4)(–1.5 – 1)
= – 6.25 _4
∴ the minimum point is (–1.5, – 6.25).

1
(b) (i) y = –x + 3x – 2 2 (iii) The minimum point is at x = (–2 + 2)
2
= –(x2 – 3x + 2) = 0.
= –(x – 1)(x – 2) ∴ the minimum point is (0, – 4).
(ii) The x-intercepts are 1 and 2.
When x = 0, y = – 02 + 3(0) – 2 = –2 (e) (i) y = –6 – 5x + x2
y = x2 –5x – 6
x = 1.5 = (x + 1)(x – 6)
(ii) The x-intercepts are –1 and 6.
When x = 0, y = –6 –5(0) + 02 = –6
x y
O 1 2 3
x = 2.5

y = _ x 2 + 3x _ 2
_2
x
–1 O 2.5 6
–6 y = –6 – 5x + x2


93

02 FWS3A(Exp)_ch2.indd 93 12/16/14 12:50 PM


© Star Publishing Pte Ltd. All rights reserved.

1
(iii) The minimum point is at x = (– 4 + 1) y
2
= –1.5.
When x = –1.5,
y = (–1.5 + 4)(–1.5 – 1) 1
= – 6.25
∴ the minimum point is (–1.5, – 6.25).
x
(f ) (i) y = 6x – x2 – 9 –1 O 1
= –(x2 – 6x + 9)
= –(x – 3)2 y = 1 – x2
(ii) The graph touches the x-axis at (3, 0). –1
When x = 0, y = 6(0) – 02 – 9 = –9
y
x=3
(b) The line of symmetry of the graph is x = 0.

x 7. The diagram shows the graph of y = x2 – 2x – 8. It cuts


O 3
the x-axis at A and B, and the y-axis at C. Find
(a) the coordinates of C,
(b) the coordinates of A and B,
(c) the coordinates of the minimum point of the graph.
–9 y
y = 6x – x2 – 9

x
(iii) The maximum point is (3, 0). A O B

6. (a) Sketch the graph of y = 1 – x2 on the axes provided. y = x 2 – 2x – 8


(b) Write down the equation of the line of symmetry
of the graph. C

y
Solution
(a) y = x2 – 2x – 8
1 At C, x = 0
\ y = 02 – 2(0) – 8
_ x = –8
1 O 1 The coordinates of C are (0, –8).
_ (b) When y = 0,
1
x2 – 2x – 8 = 0
(x + 2)(x – 4) = 0
x = –2 or x = 4
\ the coordinates of A = (–2, 0) and B = (4, 0).
Solution
1
(a) y = 1 – x2 (c) The minimum point is at x = (–2 + 4) = 1.
2
= (1 + x)(1 – x) When x = 1, y = 1 – 2(1) – 8 = –9
2

The x-intercepts are –1 and 1. \ the minimum point is (1, –9).


When x = 0, y = 1 – 02 = 1
1
The maximum point is at x = (–1 + 1) = 0 Level 3
2
i.e. (0, 1) is the maximum point. 8. The difference between two numbers is 7. Suppose the
smaller number is x and the product of the two numbers
is y.
(a) Express the other number in terms of x.
(b) Express y in terms of x.
(c) Sketch the graph of y against x.
(d) Hence find the minimum value of the product of
the two numbers. What are these two numbers?
Chapter 2 More about Quadratic Equations
94

02 FWS3A(Exp)_ch2.indd 94 12/16/14 12:50 PM


© Star Publishing Pte Ltd. All rights reserved.

Solution 10. In a building, an arch ABC is supported by two


(a) The other number is x + 7. vertical columns OA and DC of equal heights,
(b) y = x(x + 7) where O and D are points on the level ground. The
equation of the arch can be modelled by the equation
(c) The x-intercepts of the graph are 0 and –7.
y = 2 + 4x – x2, where O is taken to be the origin, OD
1
The minimum point is at x = (–7 + 0) is the x-axis, and the unit of length is metre.
2
= –3.5. (a) What is the height of the column OA?
When x = –3.5, (b) Find the distance OD.
y = –3.5 3 (–3.5 + 7) = –12.25 (c) If B is the highest point of the arch, find the
∴ the minimum point is (–3.5, –12.25). coordinates of B.
(d) Sketch the graph of the arch ABC.
y
B

A C

_8 _6 _4 _2 x
O

O D
_ 10
Solution
(_ 3.5, _ 12.25) (a) When x = 0,
y = 2 + 4(0) – 02
=2
(d) The minimum product of the two numbers is –12.25. \ the height of the column OA is 2 m.
The corresponding numbers are
–3.5 and –3.5 + 7, (b) Distance OD = AC
i.e. –3.5 and 3.5. Equation of the line AC is y = 2.
Putting y = 2 into y = 2 + 4x – x2,
we have 2 = 2 + 4x – x2
9. The revenue y (in dollars) for selling x items is given by x – 4x = 0
2

y = 40x – x2. x(x – 4) = 0


(a) Sketch the graph of y = 40x – x2 for x  0. x = 0 or x = 4
(b) How many items should be sold in order to have (point A) (point C)
the maximum revenue? What is the maximum \ distance AC = 4 – 0
revenue? = 4 m
Hence, distance OD is 4 m.
Solution
(a) y = 40x – x2 (c) Maximum point B of y = 2 + 4x – x2 is at
= –x(x – 40) x=
1
(0 + 4)
The x-intercepts of y = 40x – x2 are 0 and 40. 2
1 =2
The maximum point is at x = (0 + 40) When x = 2,
2
= 20. y = 2 + 4(2) – 22
When x = 20, y = –20(20 – 40) = 400 =6
\ the coordinates of B are (2, 6).
y
(d) The diagram below shows the graph of the arch
(20, 400) ABC.
400
y = 40x _ x 2 y

B(2, 6)

x
O 10 20 30 40 y = 2 + 4x – x2

(b) The maximum point of the graph is (20, 400). C(4, 2)


2 A
Hence, 20 items should be sold.
The maximum revenue is $400.
x
O 2 4
95

02 FWS3A(Exp)_ch2.indd 95 12/16/14 12:50 PM


© Star Publishing Pte Ltd. All rights reserved.

11. (a) Find three quadratic functions such that the graph (c) The diagram below shows the graph of the equation
of each function has the turning point at (3, –1). in (b).
(b) Sketch the graphs of the functions that you have y
found in (a) on the same coordinate plane. (20, 5)
1 x(x – 40)
y = – 80
Solution
(a)
Some quadratic functions with the turning point at
(3, –1) are
x
1. y = (x – 3)2 – 1, O 20 40
2. y = –(x – 3)2 – 1,
3. y = 2(x – 3)2 – 1.
(b) y Revision Exercise 2
y = 2(x _ 3)2_ 1 1. Solve the following equations by factorisation.
8 (a) 2x2 + x – 15 = 0
(b) 3x2 – 20x – 7 = 0
y =(x _ 3)2_ 1 (c) 9x2 – 25 = 0
x
O 1 2 3 4 5 (d) (x + 3)(x – 2) = 5x(x – 2)

Solution
y = _ (x _ 3)2_ 1
_ 10 (a) 2x2 + x – 15 = 0
(x + 3)(2x – 5) = 0
5
\ x = –3 or x =
2

(b) 3x2 – 20x – 7 = 0


12. A golf ball is hit from a point O on a horizontal field. (3x + 1)(x – 7) = 0
It flies in a trajectory which can be represented by a 1
quadratic function. The ball reaches its highest point \ x = – or x = 7
3
5 m above the field and at a horizontal distance 20 m
(c) 9x2 – 25 = 0
away from O.
(3x + 5)(3x – 5) = 0
(a) How far away from O will the ball land? 5 5
(b) Let O be the origin, x m be the horizontal distance \ x = – or x =
3 3
of the ball from O and y m be the height of the ball
above the field. If the equation of the flight of the (d) (x + 3)(x – 2) = 5x(x – 2)
ball is represented by the equation y = a(x – p)(x – q), (x + 3)(x – 2) – 5x(x – 2) = 0
where a, p and q are constants, determine the (x – 2)(x + 3 – 5x) = 0
equation. (x – 2)(– 4x + 3) = 0
(c) Sketch the graph of the equation in (b). \ x =
3
or x=2
4

Solution
(a) By symmetry of the trajectory, 2. Solve the following equations by completing the square.
the required distance = 2 3 20 = 40 m (a) x2 – 6x + 4 = 0
(b) (0, 0) and (40, 0) are points on the graph. (b) x2 + 4x – 3 = 0
Hence,   y = a(x – 0)(x – 40) (c) x2 + 15x + 8 = 0
= ax(x – 40) (d) x2 – 7x – 2 = 0
The maximum point is (20, 5).
Solution
∴ 5 = a 3 20 3 (20 – 40)
1 (a)
x2 – 6x + 4 = 0
a= – x2 – 6x = – 4
80
∴ the equation of the trajectory is x2 – 6x + 32 = – 4 + 32

1
y = – x(x – 40). (x – 3)2 = 5
80 x – 3 = ± 5
x =3– 5 or x = 3 + 5
\ x = 0.764 or x = 5.24 (correct to 3 s.f.)

Chapter 2 More about Quadratic Equations


96

02 FWS3A(Exp)_ch2.indd 96 12/16/14 12:50 PM


© Star Publishing Pte Ltd. All rights reserved.

(b)
x2 + 4x – 3 = 0 y
x2 + 4x = 3
x2 + 4x + 22 = 3 + 22 4
(x + 2)2 = 7 y = x2 + 3x – 6
x + 2 = ± 7 2
x = –2 – 7 or x = –2 + 7
x
\ x = – 4.65 or x = 0.646 (correct to 3 s.f.) _5 _4 _3 _2 _1 O 1 2

(c) x2 + 15x + 8 = 0 _2
x2 + 15x = –8 y = –3

 15 
2
 15 
2
_4

x2 + 15x +  2  = –8 +  2 

 15 
2
193 _6
 x + =
2  4
_8
15 193 y = 2x – 1
x+ =±
2 4

193 193
(b) x2 + 3x – 6 = 0
15 15
x=– – or x = – +
2 4 2 4
i.e y =0
\ x = –14.4 or x = –0.554 From the graph, the roots of the equation
x2 + 3x – 6 = 0 are
(d) x2 – 7x – 2 = 0
x = – 4.4 or x = 1.4 (correct to 1 d.p.).
x2 – 7x = 2
 7
2
 7
2 (c) (i) x2 + 3x – 3 = 0
x2 – 7x +  –  = 2 +  –  x2 + 3x – 3 – 3 = –3
2 2
2
x2 + 3x – 6 = –3

x –
7
=
57
i.e. y = –3
 
2 4
Draw the line y = –3 on the graph in (a).
7 57 Hence, the required roots are x = –3.8 or
x– =±
2 4 x = 0.8.
7 57 7 57 (ii) x2 + x – 5 = 0
x = – or x = + x2 = –x + 5
2 4 2 4
\ x = – 0.275 or x = 7.27 (correct to 3 s.f.) x2 + 3x –6 = –x + 5 + 3x – 6
i.e. y = 2x – 1
Draw the line y = 2x – 1 on the graph in (a).
3. (a)
Draw the graph of y = x2 + 3x – 6 for –5  x  2
using a scale of 2 cm to 1 unit on the x-axis and x –3 0 2
1 cm to 1 unit on the y-axis.
(b) Use your graph to solve the equation x2 + 3x – 6 = 0. y –7 –1 3
Give your answers correct to 1 decimal place.
(c) By drawing suitable straight lines on the same axes, Hence, the required roots are x = –2.8 or
find the values of x for which x = 1.8.
(i) x2 + 3x – 3 = 0,
(ii) x2 + x – 5 = 0. 4. Solve the following equations using the quadratic formula,
giving your answers correct to 3 significant figures.
Solution (a) x2 + 13x – 88 = 0 (b) 3x2 + 12x + 1 = 0
(a) (c) 4x – 5x + 9 = 0
2
(d) 5x2 – 8x – 2 = 0
x –5 – 4 –3 –2 –1 0 1 2
y = x + 3x – 6
2
4 –2 –6 –8 –8 –6 –2 4 Solution
(a) x2 + 13x – 88 = 0
–13 ± 132 – 4(1)(–88)
x=
2(1)
–13 + 521

=
2
–13 – 521 –13 + 521
x= or x =
2 2
\ x = –17.9 or x = 4.91 (correct to 3 s.f.)

97

02 FWS3A(Exp)_ch2.indd 97 12/16/14 12:50 PM


© Star Publishing Pte Ltd. All rights reserved.

(b) 3x2 + 12x + 1 = 0 3 2 3


(d) – =
–12 ± 2
12 – 4(3)(1) 7x – 2 7x + 3 4
x= 3(7 x + 3) – 2(7x – 2)
2(3) 3
=
(7 x – 2)(7x + 3) 4
–12 ± 132
= 7 x + 13 3
6 =
49 x + 7 x – 6
2
–12 – 132 –12 + 132 4
x= or x =
6 6 4(7x + 13) = 3(49x2 + 7x – 6)
\ x = –3.91 or x = –0.0851 (correct to 3 s.f.) 28x + 52 = 147x2 + 21x – 18

147x2 – 7x – 70 = 0
(c) 4x2 – 5x + 9 = 0 21x2 – x – 10 = 0
x=
5± 5 2 – 4(4)(9)
(3x + 2)(7x – 5) = 0
2(4) 2 5
\ x = – or x=
5± –119 3 7

=
8
\ the equation has no real roots. 6. Solve the following equations.
(a) 3x2 + 7x + 1 = 0
(d) 5x2 – 8x – 2 = 0
(b) (2x – 1)2 = (x + 1)2
x=
8± 8 2 – 4(5)(–2) (c) (x + 3)(2x – 9) = x(x + 2)
2(5) 1 2 3
(d) 3x
+ 3x – 1
+ =0
104 8± 3x + 1
=
10
8 + 104 8– 104 Solution
x= or x = (a) 3x2 + 7x + 1 = 0
10 10
\ x = 1.82 or x = – 0.220 (correct to 3 s.f.) –7 ± 7 2 – 4(3)(1)
x=
2(3)
5. Solve the following fractional equations. –7 ± 37
x +1 8
=
(a) = 6
2 x +1
10 \ x = –2.18 or x = – 0.153 (correct to 3 s.f)
(b) 10(x – 2) + x–2
= 29
x x +1
(b) (2x – 1)2 = (x + 1)2
(c) x +1
+ =3 (2x – 1) – (x + 1)2 = 0
2
x
3 2 3 [(2x – 1) – (x + 1)][(2x – 1) + (x + 1)] = 0
(d) – =
7x – 2 7x + 3 4 (x – 2)(3x) = 0
\ x = 0 or x = 2
Solution
(a)
x +1
=
8 (c) (x + 3)(2x – 9) = x(x + 2)
2 x +1 2x2 – 3x – 27 = x2 + 2x
(x + 1)2 = 16 x2 – 5x – 27 = 0
x+1 = ± 16 5± 5 2 – 4(1)(–27)
x+1 = ±4 x=
2(1)
\ x = –5 or x = 3 5± 133
10 =
(b) 10(x – 2) + x–2
= 29 2
\ x = –3.27 or x = 8.27 (correct to 3 s.f.)
10(x – 2)2 + 10 = 29(x – 2)

10(x – 2) – 29(x – 2) + 10
2
=0
1 2 3

[5(x – 2) – 2][2(x – 2) – 5] =0 (d) + + =0
3x 3x – 1 3x + 1
(5x – 12)(2x – 9) =0 2(3x + 1) + 3(3x – 1)
1
12 9 + =0
\ x = or x= 3x (3x – 1)(3x + 1)
5 2
15 x – 1 1
x x +1 =–
(c) x + 1 + =3 9x – 1
2
3
x
3x(15x – 1) = – (9x2 – 1)
x2 + (x + 1)2 = 3x(x + 1)
45x2 – 3x = –9x2 + 1
x + x2 + 2x + 1 = 3x2 + 3x
2
54x – 3x – 1
2
=0
x2 + x – 1 = 0
(9x + 1)(6x – 1) =0
–1 ± 12 – 4(1)(–1)
x= \ x = –
1
or x=
1
2(1) 9 6
–1 ± 5

=
2
\ x = –1.62 or x = 0.618 (correct to 3 s.f.)
Chapter 2 More about Quadratic Equations
98

02 FWS3A(Exp)_ch2.indd 98 12/16/14 12:51 PM


© Star Publishing Pte Ltd. All rights reserved.

7. (a)
Draw the graph of y = 5 – 2x – 2x2 for –3 < x < 2 Draw the line y = –x – 3 on the graph in (a).
using a scale of 2 cm to 1 unit on the x-axis and
1 cm to 1 unit on the y-axis. x –3 0 2
(b) Use you r g raph to f i nd t he solut ions of y 0 –3 –5
5 – 2x – 2x2 = 0.
(c) By drawing suitable straight lines on the same axes, Hence, the required roots are x = –2.3 or
find the values of x which satisfy the following x = 1.8.
equations
(i) 1 – 2x – 2x2 = 0,
(ii) 8 – x – 2x2 = 0. 8. The diagram shows the plan of a garden. It consists of
a rectangular lawn measuring 2x m by x m, and two
Solution rectangular paths of 1.5 m wide along the shorter sides.
(a) y = 5 – 2x – 2x2 If the total area of the garden is 189 m 2, find the
dimensions of the lawn.
x –3 –2 –1 0 1 2

y –7 1 5 5 1 –7
x

y
1.5 2x 1.5

6
Solution
y=4 Area of lawn + Area of paths = 189
4 2x2 + 2(1.5x) = 189
2x2 + 3x – 189 = 0
2 y = 5 – 2x – x2 (x – 9)(2x + 21) = 0
21
x = 9 or x = – (rejected)
2
_3 _2 _1 x \ x= 9
O 1 2
2x = 18
_2 Hence the dimensions of the lawn are 9m by 18 m.

_4 9. A rectangular flower bed measures 30 m by 20 m. It is


surrounded by a path of uniform width. The area of the

_6 1
path is of the area of the flower bed. Find the width
y = –x – 3 3
of the path.
_8

20

(b) 5 – 2x – 2x2= 0 30

i.e. y =0
From the graph, the required roots are x = –2.2
and x = 1.2 Solution
(c) (i) 1 – 2x – 2x = 0 2 Let the width of the path be x m.
4 + 1 – 2x – 2x2 = 4 + 0 (30 + 2x)(20 + 2x) – (30)(20) =
1
(30)(20)
5 – 2x – 2x2 = 4 3

i.e. y =4 600 + 100x + 4x – 600 = 200 2

Draw the line y = 4 on the graph in (a). 4x2 + 100x – 200 = 0


Hence, the required roots are x = –1.4 or x2 + 25x – 50 = 0
x = 0.4. x =
–25 ± 25 2 – 4(1)(–50)
2(1)
(ii) 8 – x – 2x2 = 0
–2x2 = x – 8 =
–25 ± 825
5 – 2x – 2x2 = 5 – 2x + x – 8 2
i.e. y = –x – 3 x = 1.86 or x = –26.9
(rejected)
(correct to 3 s.f.)
\ the width of the path is 1.86 m.

99

02 FWS3A(Exp)_ch2.indd 99 12/16/14 12:51 PM


© Star Publishing Pte Ltd. All rights reserved.

10. The distance travelled by a train is 180 km. If the average 12.
Machine A can make 60 donuts in x minutes, where
speed of the train is increased by 10 km/h, the time taken x . 1. Machine B can make 135 donuts in (2x – 1)
by the train would be 15 minutes less. Find the average minutes. The two machines together can make 27 donuts
speed of the train. in 1 minute.
(a) Express, in terms of x, the number of donuts made
Solution by
Let the average speed of the train be x km/h. (i) machine A in 1 minute,
180 15 (ii) machine B in 1 minute.
– 180 =
x x + 10 60 (b) Form an equation in x and show that it reduces to
180( x + 10) – 180x 1 9x2 – 47x + 10 = 0.
=
x ( x + 10) 4 (c) Solve the equation 9x2 – 47x + 10 = 0.
1800(4) = x(x + 10) (d) Find the time that machine A takes to make
x2 + 10x – 7200 = 0 90 donuts.
(x – 80)(x + 90) = 0
x = 80 or x = –90 (rejected) Solution
\ the average speed of the train is 80 km/h. (a) Number of donuts made in 1 minute by
60
(i) machine A =
x
11.
Abel and Sam ran a 42 km marathon. The average speed 135
of Abel was x km/h and the average speed of Sam was (ii) machine B =
2x – 1
3 km/h faster than that of Abel.
60 135
(a) Express, in terms of x, the time taken in hours to (b) + = 27
x 2x – 1
complete the race by
(i) Abel, 60(2x – 1) + 135x = 27x(2x – 1)
(ii) Sam. 120x – 60 + 135x = 54x2 – 27x
(b) Sam completed the race 28 minutes earlier than
54x2 – 282x + 60 = 0
Abel. Form an equation in x and show that it reduces 9x2 – 47x + 10 = 0 (shown)
to x2 + 3x – 270 = 0. (c) 9x2 – 47x + 10 = 0
(c) Solve the equation x2 + 3x – 270 = 0. (x – 5)(9x – 2) = 0
(d) Find Abel’s race time correct to the nearest hour 2
\ x = 5 or x=
and minute. 9
(d) Since x . 1 (given),
Solution 2
x = (rejected).
(a) Time taken to complete the race by 9
42 Hence, machine A can make 60 donuts in 5 minutes.
(i) Adel = h
x \ time taken by machine A to make 90 donuts
5
(ii) Sam =
42
h = 3 90
x+3 60
= 7.5 minutes
42 42 28
(b) – =
x x+3 60
13. A bathtub can hold 1200 litres of water.
42( x + 3) – 42 x 28 (a) A hot water tap delivers water at x litres/min.
=
x ( x + 3) 60 Express the time taken, in minutes, to fill up the
126 3 15 = 7x(x + 3) bathtub.
7x2 + 21x – 1890 = 0 (b) A cold water tap delivers water at (x + 10) litres/
x2 + 3x – 270 = 0 (shown) min. Express the time taken, in minute, to fill up
the bathtub.
(c) x2 + 3x – 270 = 0 (c) The hot water tap alone takes 4 minutes longer to
(x – 15)(x + 18) = 0 fill up the bathtub than the cold water tap. Write
\ x = 15 or x = –18 down an equation in x, and show that it simplifies
(d) Hence, the average speed of Abel was 15 km/h. to x2 + 10x – 3000 = 0.
x = –1.8 (rejected) (d) Solve the equation x2 + 10x – 3000 = 0.
42 (e) Find the time taken by the cold water tap to fill
\ Abel’s race time = h
15 the bathtub.
= 2.8 h
= 2 h 48 min

Chapter 2 More about Quadratic Equations


100

02 FWS3A(Exp)_ch2.indd 100 12/16/14 12:51 PM


© Star Publishing Pte Ltd. All rights reserved.

Solution y
(a) Time taken by the hot water tap to fill up the bathtub
1200
= minutes 400 y = 375
x

(b) Time taken by the cold water tap to fill up the 300
1200
bathtub = minutes
x + 10
200
1200 1200
(c) – =4
x x + 10 100
1200(x + 10) – 1200x = 4x(x + 10)
12 000 = 4x2 + 40x x
4x + 40x – 12 000 = 0
2 O 10 20 30 40 50
x2 + 10x – 3000 = 0 (shown)
_100
(d) x + 10x – 3000 = 0
2

(x – 50)(x + 60) = 0 _200


\ x = 50 or x = –60
y = –225 + 50x – x2
(e) x = –60 (rejected) _300
Hence, the time taken by the cold water tap to fill
the bathtub
1200
= where x = 50 (b) (i)
The maximum profit is $400 and the
50 + 10
= 20 minutes corresponding selling price is $25.
(ii) From the graph,
when y = 0,
14.
Rachel sews some handbags to sell. The profit, $y, from x = 5 and x = 45
selling the handbags at $x each can be modelled by the Hence, the selling prices that yield no profit
equation y = –225 + 50x – x2. or loss are $5 and $45.
(a) Draw the graph of y = –225 + 50x – x 2 for (iii) When profit y = $375, the selling prices x are
0 < x < 50. $20 and $30.
(b) Use your graph to find
(i) the maximum profit which Rachel can make
15. The diagram shows the graph of y = x2 + 4x which passes
and the selling price that yields this maximum
through the origin and a point A on the x-axis.
profit,
y
(ii) the selling prices that yield no profit or loss,
(iii) the selling prices of each handbag if the profit y = x 2 + 4x
is $375.

Solution
(a) y = –225 + 50x – x2
x
A O

x 0 5 10 15 20 25

y –225 0 175 300 375 400


(a) Find the coordinates of A.
(b) State the equation of the line of symmetry of the
x 30 35 40 45 50 graph.

(c) Find the minimum point on the graph.
y 375 300 175 0 –225
Solution
(a) y = x2 + 4x
= x(x + 4)
∴ the x-intercepts of the graph of y = x2 + 4x are
x = 0 and x = – 4.
Hence, coordinates of A are (– 4, 0).

101

02 FWS3A(Exp)_ch2.indd 101 12/16/14 12:51 PM


© Star Publishing Pte Ltd. All rights reserved.

(b) x-coordinate of the minimum point Solution


1 (a) (i) y = x(5 – x)
= (– 4 + 0)
2 The x-intercepts are 0 and 5.
= –2 When x = 0, y = 0(5 – 0)
∴ the line of symmetry is x = –2. =0
(c) When x = –2, The maximum point is at x =
1
(0 – 5)
y = (–2)2 + 4(–2) = – 4 2
∴ the minimum point is (–2, – 4). = 2.5
When x = 2.5, y = 2.5(5 – 2.5)
16. (a) Solve the equation –x2 + 4x – 3 = 0. = 6.25
(b) Express –x2 + 4x – 3 in the form –(x – h)2 + k, y
where h and k are constants.
(c) Hence sketch the graph of y = –x2 + 4x – 3.
(d) State the maximum point of the graph. (2.5, 6.25)

Solution
(a) –x 2 + 4x – 3 = 0 y = x(5 – x)
x2 – 4x + 3 = 0
(x – 1)(x – 3) = 0
∴ x = 1 or x = 3 x
O 2.5 5
(b) –x + 4x – 3 = –(x – 4x) – 3
2 2

= –(x 2 – 4x + 22) + 22 – 3
= –(x – 2)2 + 1 (ii) The line of symmetry of the graph of
y = x(5 – x) is x = 2.5.
(c) y = –x 2 + 4x – 3
= –(x – 2)2 + 1 (b) (i) y = (x + 1)2 – 4
From (a) and (b), we have the x-intercepts of the The minimum point is (–1, – 4).
graph. They are 1 and 3. When x = 0, y = (0 + 1)2 – 4
When x = 0, y = – 02 + 4(0) – 3 = –3 = –3
Hence, the sketch of the graph is as shown in the y
following diagram.
y = (x + 1)2 – 4
y
(2, 1)
1
_1 x
O
x
O 1 2 3 4
_1 _3

(–1, –4)
_2 y = _x 2 + 4x _ 3

_3

(ii) y = (x + 1)2 – 4
(d) The maximum point is (2, 1).
(1) When y = 0,
(e) The gradient at the maximum point is 0. (x + 1)2 – 4 = 0
(x + 1)2 = 4
17. (a) (i)
Sketch the graph of y = x(5 – x).
x + 1 = ± 4
(ii) Write down the equation of the line of
x + 1 = –2 or x + 1 = 2
symmetry of the graph of y = x(5 – x).
\ x = –3 or x=1
(b) (i) Sketch the graph of y = (x + 1)2 – 4.
Hence the graph cuts the x-axis at
(ii) State the coordinates of the points at which
(–3, 0) and (1, 0).
the graph cuts
(1) the x-axis, (2) From (b)(i), the graph cuts the y-axis at
(2) the y-axis. (0, –3).
(iii) Write down the coordinates of the minimum (iii) From (b)(i), the coordinates of the minimum
point of the graph. point of the graph are (–1, – 4).

Chapter 2 More about Quadratic Equations


102

02 FWS3A(Exp)_ch2.indd 102 12/16/14 12:51 PM


© Star Publishing Pte Ltd. All rights reserved.

Go Further Solution
1. A beaker has 400 cm of water. V cm of water is poured
3 3 (a)
When Adam and Betty first meet, the distance
out and replaced by the same amount of vinegar. After covered by Adam = 36 m
stirring well, V cm3 of the first mixture is poured out and by Betty = (L – 36) m.
and replaced by the same amount of vinegar again. The When they meet for the second time, the distance
percentage of vinegar in the final mixture is 36%. Find covered by Adam = L + (L – 12)
the value of V. = (2L – 12) m.
and by Betty = (L + 12) m.
Solution Since they swim in uniform speeds, the ratio of
Percentage of vinegar in the first mixture their distances covered in the two sessions will be
V equal.
= 3 100%
400 36 2 L – 12
\ =
V L − 36 L + 12
= %
4
36(L + 12) = (2L – 12)(L – 36)
Volume of vinegar in the V cm of the first mixture
3

36L + 432 = 2L2 – 72L – 12L + 432
V 2L2 – 120L = 0
= %3V
4 2L(L – 60) = 0
=
V
3V
L = 0 (rejected) or L = 60
400 \ L = 60
V2
= cm3 (b) (i) Swimming speed of Adam =
36
400 20
\ total volume of vinegar in the final mixture = 1.8 m/s
V2
= V – +V L − 36
400 Swimming speed of Betty =
20
 V2 
=  2V –  cm
3
60 – 36
 400  =
20
Since the percentage of vinegar in the final mixture is = 1.2 m/s
36%, we have
2V –
V2 (ii) Time when they meet the second time
400 3 100% = 36% = (2 3 60 – 12) ÷ 1.8 or (60 + 12) ÷ 1.2
400
V2 = 60 s
2V – = 4 3 36 Hence they will meet the second time
400
V2 60 seconds after initial start.
– 2V + 144 = 0
400

V 2 – 800V + 57 600 = 0
(V – 80)(V – 720) = 0
V = 80 or V = 720 (rejected; V should be < 400)
\ V = 80

2. Adam Betty

A Lm B

In the diagram, AB is a lane in a swimming pool which


is L m long. Adam and Betty have swimming practice
back and forth along the lane, each with uniform speed
and turning at the ends without loss of time. They start
at the same instant at opposite ends A and B respectively.
They first meet at a distance of 36 m from A. They meet
for the second time at a distance of 12 m from A.
(a) Find the value of L.
(b) If they first meet 20 seconds after the start, find
(i) the swimming speeds of Adam and Betty in
m/s,
(ii) the time they meet the second time.

103

02 FWS3A(Exp)_ch2.indd 103 12/16/14 12:51 PM


© Star Publishing Pte Ltd. All rights reserved.

3 Linear Inequalities

Class Activity 1
Objective: To explore the addition and subtraction properties of inequalities.
If each box represents an inequality sign “” or “”, what should it be in each case?

1. (a) 3  5, 3 + 6  5 + 6

(b) –9  2, –9 + 3  2 + 3

(c) 7  10, 7 + 8  10 + 8

(d) If a  b, then a + c  b + c

2. (a) 2  8, 2 – 5  8 – 5

(b) –3  –1, –3 – 7  –1 – 7

(c) –12  6, –12 – 4  6 – 4

(d) If a  b, then a – c  b – c

Class Activity 2
Objective: To compare the methods of solving a linear inequality and the corresponding linear equation, and their
solutions.

1. Solve each of the following linear equations and linear inequalities.


(a) 3x + 1 = 16 (b) 3x + 1 , 16
3x + 1 – 1 = 16 – 1
3x + 1 – 1 , 16 – 1
3x = 15
3x , 15
3x 15
= 3x 15
3 3 ,
3 3
\ x =5
\ x ,5

(c) 9 – 7x = 8 (d) 9 – 7x . 8
–9 + 9 – 7x = –9 + 8 –9 + 9 – 7x = –9 + 8
–7x = –1 –7x . –1
–7 x –1 –7 x –1
= ,
–7 –7 –7 –7
1 1
\ x = \ x ,
7 7

Chapter 3 Linear Inequalities


104

03 FWS3A(Exp)_ch3.indd 104 12/16/14 12:52 PM


© Star Publishing Pte Ltd. All rights reserved.

(e) 8x – 3 = 2x + 15 (f ) 8x – 3 . 2x + 15
8x – 2x = 15 + 3 8x – 2x . 15 + 3
6x = 18 6x . 18
\ x =3 \ x .3

(g) 11 – 4x = 5x – 1 (h) 11 – 4x , 5x – 1
– 4x – 5x = –1 –11 – 4x – 5x , –1 – 11
–9x = –12 –9x , –12
–9 x –12 –9 x –12
= .
–9 –9 –9 –9
4 4
\ x = \ x.
3 3

2. With reference to the solutions in Question 1, compare the methods of solving a linear inequality and the corresponding
linear equation.
The manipulation and operations in equations and inequalities are the same. But when both sides of an inequality are multiplied or divided by a

negative number, we have to reverse the inequality sign.

3. What do you realise about the solutions of a linear inequality and the corresponding linear equation?
The solution of a linear equation is a real number but the solution of a linear inequality is a range of real numbers.

105

03 FWS3A(Exp)_ch3.indd 105 12/16/14 12:52 PM


© Star Publishing Pte Ltd. All rights reserved.

Try It! x–4 2x + 1


5. Solve the inequality  . Represent the solution
5 7
Section 3.1 on a number line.
1. It is given that p  0. Is p  2?
Solution
Solution x–4 2x + 1

Since p , 0 5 7
and 0 , 2, 35 3
x–4
> 35 3
2x + 1
\ p , 2. 5 7
7(x – 4) > 5(2x + 1)
7x – 28 > 10x + 5
2. It is given that p  q. Compare the values of the following 7x > 10x + 33
pairs of numbers. –3x > 33
(a) p – 1 and q – 1 33
(b) – 4p and – 4q x<
–3
(c)
1
p and q
1 \ x < –11
5 5
Solution
Solution
(a) Since p , q,
\ p – 1 , q – 1. _ 11 0
(b) – 4p . – 4q
1 1
(c) p, q
5 5 Section 3.3
6. Solve the inequalities 5x – 8  12 and 7x + 3  17.

Section 3.2 Solution


3. Solve the inequality 2x + 3  15. 5x – 8 , 12 and 7x + 3 , 17
5x , 20 and 7x , 14
Solution x , 4 and x,2
2x + 3 , 15
x2
2x + 3 – 3 , 15 – 3
2x , 12 x4
12
x,
2
0 2 4
\ x,6
\ the solution is x , 2.
4. Solve the inequality 3x – 8  7x + 16. Represent the
solution on a number line.
7. Solve the inequalities 6x + 9 < 2x + 13 and
Solution
2x – 8 , 7x + 12.
3x – 8 < 7x + 16
Solution
3x – 8 + 8 < 7x + 16 + 8
6x + 9 < 2x + 13 and 2x – 8 , 7x + 12
3x < 7x + 24
4x < 4 and –5x , 20
3x – 7x < 7x + 24 – 7x
x < 1 and x . –4
– 4x < 24
24
x>
–4 x  –4
\ x > –6
x1

Solution _4 0 1

_6 0 \ the solution is – 4 , x < 1.

Chapter 3 Linear Inequalities


106

03 FWS3A(Exp)_ch3.indd 106 12/16/14 12:52 PM


© Star Publishing Pte Ltd. All rights reserved.

8. Solve the inequalities 5x – 3 > 4 – x > 7 – 4x. 11. A student will get a grade C in an ability test if the mean
score of three examination papers is more than 60 but
Solution less than or equal to 75. Susan scored 57 and 69 in the
5x – 3 > 4 – x > 7 – 4x first two papers respectively. What is the range of the
5x – 3 > 4 – x and 4 – x > 7 – 4x score required in the third paper for Susan to obtain a
5x + x > 4 + 3 and 4x – x > 7 – 4 grade C in the test? The maximum score of each paper
6x > 7 and 3x > 3 is 100.
1
x > 1 and x>1
6 Solution
x  1 61
Let x be Susan’s score in the third paper.
x1 57 + 69 + x
 60 , < 75
3
1 126 + x
1 61  60 , < 75
3
\ the solution is x > 1 .
1 180 , 126 + x < 225
6  54 , x < 99
\ Susan’s required score is in the range 54 , x < 99.
x +1 2x – 5
9. Solve the inequalities x .
2 3

Solution
x +1 2x – 5
 x 
2 3
x +1 2x – 5
, x and  
x,
2 3

x + 1 , 2x and 3x , 2x – 5
x . 1 and   x , –5

x  –5
x1

_5 0 1

\ the given inequalities have no solution.

Section 3.4
10. Saadiah has $70 in her money box. She saves $40 every
month in her money box. What is the minimum number
of complete months required in order for Saadiah to have
more than $500 in her money box?

Solution
Let t be the number of months required.
70 + 40t . 500
40t . 430
t . 10.75
\ there will be more than $500 after 11 months.

107

03 FWS3A(Exp)_ch3.indd 107 12/16/14 12:52 PM


© Star Publishing Pte Ltd. All rights reserved.

Exercise 3.1 Solution


Level 1 (a) If a  b, then a – 6 , b – 6.
1. If a  7, determine whether the following inequalities
are true. (b) If p  q, then p + 7 . q + 7.
(a) a  10 (b) a  2
(c) a + 2  9 (d) – a  –7 (c) If r  s, then 2r < 2s.
x y
Solution (d) If x  y, then – < – .
3 3
(a) Since a , 7
and 7 , 10, Level 2
\ a , 10 is true. 4. Copy and fill in each box with an inequality sign.
(b) When a = 5, (a) If a  b, then
then a , 7 (i) 4a 4b,
but a . 2.
\ a, 2 is false. (ii) 4a + 3 4b + 3.
(c) Since a , 7, (b) If m  n, then
a + 2 , 7 + 2.
\ a + 2 , 9 is true. (i) –m –n,

(d) Since a , 7, (ii) 1 – m 1 – n.


(–1)a . (–1)7
– a . –7 Solution
\ – a , –7 is false. (a) If a  b, then
(i) 4a , 4b,
2. If p  –5, determine whether the following inequalities
(ii) 4a + 3 , 4b + 3.
are true.
(a) p  –3 (b) p  –9 (b) If m  n, then
(c) p + 5  0 (d) –p  –5
(i) –m < –n,
Solution (ii) 1 – m < 1 – n.
(a) When p = – 4,
then p . –5
but p , –3. 5. If p  q, compare the values of the following pairs of
\ p . –3 is false. numbers.
(b) Since p . –5 (a) 2p – 7 and 2q – 7
and –5 . –9, (b) –4 –
1
p and – 4 –
1
q
\ p . – 9 is true. 5 5

(c) Since p . –5, Solution


p + 5 . –5 + 5. (a) If p < q, then
\ p + 5 . 0 is true. 2p < 2q
\ 2p – 7 < 2q – 7
(d) When p = 3,
then p . –5 (b) If p < q, then
but –3 . –5. – p > – q
1 1
\ –p , –5 is false. 3 3
1 1
\ – 4 – p > –4 – q
5 5
3. Copy and fill in each box with an inequality sign.
(a) If a  b, then a – 6 b – 6. Level 3
6. (a) If a  b and c  d, what is the inequality
(b) If p  q, then p + 7 q + 7. relationship between
(i) a + c and b + c,
(c) If r  s, then 2r 2s. (ii) b + c and b + d,
x y
(iii) a + c and b + d ?
(d) If x  y, then – – . (b) If Mr and Mrs Tan’s monthly salaries are more
3 3
than $4000 and $3500 respectively, what can you
say about their total monthly salary?
Chapter 3 Linear Inequalities
108

03 FWS3A(Exp)_ch3.indd 108 12/16/14 12:52 PM


© Star Publishing Pte Ltd. All rights reserved.

Solution Exercise 3.2


(a) (i) a ,b Level 1
\ a + c , b + c.......................... (1)
(ii) c ,d 1. Solve the following inequalities and represent each
\ b + c , b + d..........................(2) solution on a number line.
(iii) From (1) and (2), we have a + c , b + d.
Solution
(b) Mr Tan’s monthly salary . $4000   (a) x – 2 , 7
and Mrs Tan’s monthly salary . $3500 x – 2 + 2 , 7 + 2
\ their total salary . $4000 + $3500 \ x,9
i.e. their total salary . $7500
Solution

7. If 0  x  1, is it true that x2 . x? Explain your answer.


0 9
Solution
If 0  x  1, it is not true that x2 . x. (b) x+3>4
For example, 0 
1
1 x+3–3>4–3
2
2 \ x>1
 1 1 Solution
but  2   .
2

8. If y is positive, must y be greater than 1


? Explain your 0 1 2
y
answer. (c)
x
<2
5
x
Solution 5× <5×2
1 5
If y is positive, it is not necessary that y . . \ x < 10
y

For example, when y = 1


Solution
3
1
then =3
y
1 0 10
and y .
y
(d) 4x . 12
1 1
9. If a  b and c  d, is it necessary that a – c  b – d ? 3 4x . 3 12
4 4
If it is not true, give a counter-example.
\ x.3
Solution Solution
If a , b and c , d, it is NOT necessar y that
a – c , b – d. For example, when a = 8, b = 11, c = 3
and d = 9. 0 3 6
Then a , b and c , d,
but a – c = 8 – 3 = 5, (e) 3x + 5 > 2
and b – d = 11 – 9 = 2. 3x > –3
Here, a – c . b – d. x > –1
Solution
10. If a  b, must a2 be less than b2? If it is not, give a
counter-example.
_1 0
Solution
If a , b, then a2 may not be less than b2. (f ) 7x – 13 , 1
For example, –4 , 1 7x , 14
but (– 4)2 . 12. x , 2
Solution

0 2

109

03 FWS3A(Exp)_ch3.indd 109 12/16/14 12:52 PM


© Star Publishing Pte Ltd. All rights reserved.

(g) 9x – 2 , 4x + 8 (b) 7 – 2(3x – 7) < 6(9 – 2x)


5x , 10 7 – 6x + 14 < 54 – 12x
x , 2 6x < 33
1
Solution x <5
2

Solution
0 2

0 5 12
(h) 6x + 7 > 8x – 5
–2x > –12
x < 6 x 5
(c) + 1 .
Solution 3 6
2x + 6 . 5
2x . –1
0 6 1
x . –
2
1 – 3x Solution
(i) <7
4
1 – 3x < 28
–3x < 27 _1 0
2
x > –9
Solution 2x – 1 x
(d) ,
5 2
_9 0
2(2x – 1) , 5x
4x – 2 , 5x
2
–x , 2
(j) (11 – 9x) . 5x \ x . –2
3
2
3 3 (11 – 9x) . 3 3 5x Solution
3
2(11 – 9x) . 15x
22 – 18x . 15x _2 0
–33x . –22
2
x,
3 2 x 3x 5
(e) + < –
Solution 7 4 4 7
8 + 7x < 21x – 20
–14x < –28
0 2
3 \ x > 2
Solution

Level 2
0 2 4
2. Solve the following inequalities and represent each
solution on a number line.
2(3x + 1) 9( x – 1)
(f ) >
Solution 5 8
(a) 5(2x + 3) . 4(x – 2) – 13 16(3x + 1) > 45(x – 1)
10x + 15 . 4x – 8 – 13 48x + 16 > 45x – 45
6x . –36 3x > –61
1
x . – 6 \ x > –20 3
Solution
Solution

_6 0 _20 1
3 0

Chapter 3 Linear Inequalities


110

03 FWS3A(Exp)_ch3.indd 110 12/16/14 12:52 PM


© Star Publishing Pte Ltd. All rights reserved.

(g)
4x – 5
–4 .
1– x 4. (a) Solve the inequality 29 – 5x  2x + 4.
3 9 (b) Find all the positive integers which satisfy the
3(4x – 5) – 36 . 1 – x inequality.

12x – 15 – 36 . 1 – x
13x . 52 Solution
\ x .4 (a) 29 – 5x . 2x + 4
–7x . –25
Solution
4
x , 3
7
0 4 8
Solution
x –1 4
(h) . (2x + 5) – 1 _1
2 13 0 1 2 3 4

13(x – 1) . 8(2x + 5) – 26

13x – 13 . 16x + 40 – 26 3 74
–3x . 27 (b) The positive integers which satisfy the inequality
x , –9 are 1, 2 and 3.
Solution
Level 3
_18 _9 5. There are some metal spheres of masses 20 g each and
0
a box of mass 150 g. Sam places n spheres into the box.
(i)
x +1

x–1
<
2x – 7 (a) Express, in terms of n, the total mass of the box
2 3 4 and the spheres.
6(x + 1) – 4 (x – 1) < 3(2x – 7) (b) If the total mass of the box and the spheres is more
6x + 6 – 4x + 4 < 6x – 21 than 290 g,
– 4x < –31 (i) form an inequality in terms of n,
3
x>7 (ii) find the smallest value of n.
4
Solution
Solution
(a) Total mass of the box and n spheres
0 = (150 + 20n) g
7 34
(b) (i) 150 + 20n . 290
2x – 1 x +1 x–2
( j) – ,2+ (ii) 150 + 20n . 290
5 3 6
20n . 140
6(2x – 1) – 10(x + 1) , 60 + 5(x – 2)
n . 7
12x – 6 – 10x – 10 , 60 + 5x – 10
Since n is a positive integer, the smallest value of
–3x , 66
n is 8.
x . –22
Solution 6. The maximum loading of electric current on a power
1
board is 13 Amperes (A). A lamp of A is already
_22 0 2
plugged in the power board. Suppose x hairdryers of 3A
are now plugged in the power board.
3. (a) Solve the inequality 5 – 4x  31. (a) Express the total loading on the power board in
(b) Hence write down the largest integer value of x terms of x.
which satisfies the inequality. (b) Form an inequality in terms of x so that the power
board is not overloaded when all the hairdryers are
Solution turned on.
(a) 5 – 4x . 31 (c) Hence find the maximum number of hairdryers that
– 4x . 26 can be plugged in the power board and turned on
1 without overloading it.
x , –6
2
Solution
Solution
1
(a) Total loading =  2 + 3x  A

_7 _6 1 _6
2 1
(b) + 3x , 13
(b) The largest integer value of x is –7. 2

111

03 FWS3A(Exp)_ch3.indd 111 12/16/14 12:52 PM


© Star Publishing Pte Ltd. All rights reserved.

(c)
1
+ 3x , 13 (c) 2x + 5 < 0 and 7x – 1 > 13
2 2x < –5 7x > 14
1
3x , 12 x < –2
1
x>2
2
1 2
x , 4
6

Solution
_2 1 0 2
2
3 4 5
\ there is no solution.
4 61
The maximum number of hairdryers is 4. (d) 5x – 3 , 2 and 4x + 7 > x + 3
5x , 5 3x > – 4
4
x , 1 x>–
7. Write a linear inequality in the form ax + b  cx + d 3
such that the solution is x  8, where a, b, c and d are x  _ 34
constants.
x1
Solution
One example is x + 16 , 2x + 8. _1 1 0 1
3
Then –x , –8.
4
\ x . 8 is the solution. \ the solution is – < x , 1.
3

(e) 6x + 2 > 3x + 11 and 9x – 4 . 5x


Exercise 3.3 3x > 9 4x . 4
x > 3 x.1
Level 1

1. Solve each of the following pairs of simultaneous x3
inequalities. Represent the solution on a number line. x1
(a) x + 1  3 and 3x  15
(b) x – 1  – 4 and 3x – 5  4
0 1 3
(c) 2x + 5  0 and 7x – 1  13
(d) 5x – 3  2 and 4x + 7  x + 3 \ the solution is x > 3.
(e) 6x + 2  3x + 11 and 9x – 4  5x
(f ) 9 – 2x . 7 – x and 11 – 6x  8 – 10x (f ) 9 – 2x , 7 – x and 11 – 6x , 8 – 10x
x 2x
(g) – 1 , 3 and –3 x –x , –2 4x , –3
2 5 3
x –1 x +1 5 – 2x x . 2 x,–
(h)  and 1 4
4 3 7
x  – 43
Solution
x2
(a) x + 1 . 3 and 3x . 15
x.2 x .5
x5 _3 0 2
4
3
x2 \ the solution is x , – .
4

0 2 5 (g)
x
–1,3 and
2x
–3<x
2 5
\ the solution is x . 5.
x–2,6 2x – 15 < 5x
x , 8 –3x < 15
(b) x – 1 < – 4 and 3x – 5 < 4 x > –5
x < –3 3x < 9
x < –3 x<3 x  –5
x8
x  –3
x3 _5 0 8
_3 0 3 \ the solution is –5 < x , 8.
\ the solution is x < –3.

Chapter 3 Linear Inequalities


112

03 FWS3A(Exp)_ch3.indd 112 12/16/14 1:01 PM


© Star Publishing Pte Ltd. All rights reserved.

x –1 x +1 5 – 2x (d) 3x – 5 , 31 , 2x – 7
(h) < and .1
4 3 7 3x – 5 , 31 and 31 , 2x – 7
3(x – 1) < 4(x + 1) 5 – 2x . 7 3x , 36 –2x , –38
3x – 3 < 4x + 4 –2x . 2 x , 12 x . 19
–x < 7 –x . 1 \ there is no solution.
x > –7 x, –1
(e) 3(x – 2) , 5(x + 1) , 3 – x
x  –7
3(x – 2) , 5(x + 1) and 5(x + 1) , 3 – x
x  –1
3x – 6 , 5x + 5 5x + 5 , 3 – x
–2x , 11 6x , –2
_7 _1 0 x . –5
1
x,–
1
2 3
\ the solution is –7 < x , –1. 1 1
\ the solution is –5 ,x,– .
2 3
Level 2
x x–2 3x + 7
2. Solve the following simultaneous inequalities. (f ) < <
3 2 4
(a) 2(4x + 1)  3(5x – 4) and 3(2x – 6)  5(x + 1) x x–2 x–2 3x + 7
3 2 < and <
(b) (x + 2) , x + 5 and x – 4 . (3x + 1) 3 2 2 4
4 3
2x < 3(x – 2) 2(x – 2) < 3x + 7
(c) –5  2x + 1  11 2x < 3x – 6 2x – 4 < 3x + 7
(d) 3x – 5  31  2x – 7
–x < – 6 –x < 11
(e) 3(x – 2)  5(x + 1)  3 – x x > 6 x > –11
x x–2 3x + 7
(f )   \ the solution is x > 6.
3 2 4

Solution 3. Find the integer values of x which satisfy the inequalities


(a) 2(4x + 1)  3(5x – 4) and 3(2x – 6) , 5(x + 1) –2 
5x – 1
 2.
8x + 2  15x – 12 and 6x – 18 , 5x + 5 8
–7x  –14 and x , 23 Solution
x ,2 5x – 1
–2 , <2
8
x2
5x – 1 5x – 1
x  23 –2 , and <2
8 8
–16 , 5x – 1 5x – 1 < 16
0 2 23 –5x , 15 5x < 17
x . –3 x < 3.4
\ the solution is x , 2. \the solution is –3 , x < 3.4.
3 2 Hence, the required integers are –2, –1, 0, 1, 2, 3.
(b) (x + 2) , x + 5 and x – 4  (3x + 1)
4 3

3(x + 2) , 4(x + 5) and 3(x – 4)  2(3x + 1) 4. (a) Solve the inequalities – 6 , 5x – 6 < 36.
3x + 6 , 4x + 20 and 3x – 12  6x + 2 (b) Write down all the prime numbers x which satisfy
–x , 14 and –3x  14 – 6 , 5x – 6 < 36.
2
x  –14 and x , –4
3
Solution
x  –14
x –4 32 (a) –6 , 5x – 6 < 36
–6 , 5x – 6 and 5x – 6 < 36
_14 _4 2
0 , 5x and 5x < 42
0
3 x.0 and x < 8.4
\ the solution is –14 < x < – 4 .
2 x  8.4
3
x0
(c) – 5 , 2x + 1 < 11
–5 , 2x + 1 and 2x + 1 < 11
0 8.4
– 6 , 2x 2x < 10  
x . –3 x< 5 \ the solution is 0 , x < 8.4.
\ the solution is –3 , x < 5.
(b) The prime numbers x which satisfy the inequality
are 2, 3, 5 and 7.

113

03 FWS3A(Exp)_ch3.indd 113 12/16/14 12:52 PM


© Star Publishing Pte Ltd. All rights reserved.

5. Find the smallest integer value of x which satisfies the 8.


A surveyor found that a rectangular playground is
inequalities 2x – 3  4x + 6 and 8x – 1  5x – 7. 20 m long and 13 m wide, corrected to the nearest metre.
Suppose the actual length of the playground is x m and
Solution the breadth is y m.
2x – 3 < 4x + 6 and 8x – 1 . 5x – 7 (a) Write down an inequality for the values of
–2x < 9 3x . – 6 (i) x, (ii) y.
x > –
9
x . –2 (b) Find the range of the values of
2 (i) the perimeter of the playground,
\ the solution is x . –2. (ii) the area of the playground.
Hence, the smallest integer solution is –1.
Solution
(a) (i) 19.5 < x , 20.5
Level 3 (ii) 12.5 < y , 13.5
6. Each blood donor donates 450 cm3 of blood. (b) (i) 19.5 + 12.5 < x + y , 20.5 + 13.5
(a) If there are m blood donors, express the total amount 32 < x + y , 34
of blood donated in terms of m. 2 3 32 < 2(x + y) , 2 3 34
(b) The total amount of blood donated is more than 64 < 2(x + y) , 68
16 000 cm3 but less than 20 000 cm3. Since perimeter is 2(x + y) m, we have
(i) Form an inequality in terms of m. 64 m < perimeter , 68 m.
(ii) Hence find the possible numbers of donors. (ii) 19.5 × 12.5 < xy , 20.5 × 13.5
243.75 < xy , 276.75
Solution
Since area is xy m2, we have
(a) Total amount of blood donated = 450m cm3
243.75 m2 < area , 276.75 m2.
(b) (i) 160 000 , 450m , 20 000
(ii) Hence,
9. Find a possible set of integer values for a, b and c such
16 000 450 n 20 000
, , that the solution to the inequalities ax + 3  2x + b and
50 50 50
cx – 1  5x – 4 is
320 , 9n , 400
5 4
(a) x  –2,
35 , n , 44 (b) –3  x  4.
9 9
\ the possible numbers of donors are 36, 37, Solution
38, ... , 43 and 44. (a) When a = 1, b = 5 and c = 1, the inequalities are
x + 3 . 2x + 5 and x – 1 . 5x – 4
7.
Mr Tan has three $50 notes and n $10 notes in his wallet. –x . 2 – 4x . –3
(a) Express the total amount of money in Mr Tan’s x , –2 x,
3
wallet in terms of n. 4
\ the solution is x , –2.
(b) The total amount of these banknotes is more than
or equal to $260 but less than $310. (b) When a = 1, b = –1 and c = 6, the inequalities are
(i) Form an inequality in terms of n. x + 3 . 2x – 1 and 6x – 1 . 5x – 4
(ii) Hence find the possible values of n. –x . – 4 x . –3
x , 4 x . –3
Solution \ the solution is –3 , x , 4.
(a) Total amount of money
= $50 3 3 + $10 3 n
= $(150 + 10n) Exercise 3.4
Level 1
(b) (i) 260 < 150 + 10n , 310
(ii) Hence, 1. The price of a chocolate bar is $2. Nancy was told by
260 < 150 + 10n and 150 + 10n , 310 her mother that she should not spend more than $13 on
–10n < –110 and 10n , 160 chocolate bars. What is the possible number of chocolate
n > 11 and n , 16 bars Nancy can buy?
\ the solution is 11 < n , 16. Solution
The possible values of n are 11, 12, 13, 14 Let x be the number of chocolate bars that Nancy could buy.
and 15. 2x < 13
13
x <
2
1
i.e.  x < 6
2
\ she could buy 1, 2, 3, 4, 5 or 6 chocolate bars.
Chapter 3 Linear Inequalities
114

03 FWS3A(Exp)_ch3.indd 114 12/16/14 12:52 PM


© Star Publishing Pte Ltd. All rights reserved.

2. A mathematics examination consists of Papers I and II. Level 2


The full score of each paper is 100. Ali scored 57 in 6. A rectangular playground is designed to be x m long and
Paper I. If his total score has to be at least 120 in order y m wide. It is given that
to pass the examination, what should he score in Paper II?
20 < x < 30 and 15 < y < 18.
Solution (a) Express the perimeter of the playground in terms
Let x be Ali’s score in Paper II. of x and y.
57 + x > 120 (b) Find the minimum perimeter of the playground.
x > 63 (c) Find the maximum perimeter of the playground.
The full score of Paper II is 100,
\ 63 < Ali’s score < 100. Solution
(a) The perimeter of the playground = 2(x + y) m

3. Mr Leong has six $10-notes and some $50-notes in his (b) The minimum perimeter of the playground
wallet. If the total value of the notes is less than $400, = 2(20 + 15)
how many $50-notes can Mr Leong possibly have? = 2 3 35
= 70 m
Solution (c) The maximum perimeter = 2(30 + 18)
Let x be the number of $50-notes. = 2 3 48
10 3 6 + 50x , 400 = 96 m
60 + 50x , 400
50x , 340
34 7. In a model toy kit, each brick is 5 cm long. How many
x , bricks can possibly be lined in a row so that the total
5
i.e.   x , 6.8 length is greater than 28 cm but less than 40 cm?
\ the number of $50-notes is 1, 2, 3, 4, 5 or 6.
Solution
Let the number of bricks be x.
4. In a chemical reaction, the mass of a product increases at \ the total length of the bricks = 5x cm
a rate of 11 g per hour. The current mass of the product is Hence,
46 g. After how many hours will the mass of the product
28 , 5x , 40
first exceed 101 g?
28 40
,x,
5 5
Solution
5.6 , x , 8
Let t be the number of hours required.
\ the number of bricks required is 6 or 7.
46 + 11t . 101
11t . 55
t . 5 8. A cylindrical glass has a base area of 50 cm2. Find the
\ after 5 hours, the mass of the product will be more range of possible heights of water in the glass such that
than 101 g. the volume of water in the glass is at least 200 cm3 but
not more than 320 cm3.
5. The mass of Jane was 72 kg. After joining a weight
Solution
management programme, her mass decreased at a rate
Let the height of water in the cylindrical glass be h cm.
of 3 kg per month. Find the least number of complete
Volume of water in the glass
months that Jane has to be on the programme before her
= base area 3 height
mass becomes less than 54 kg.
= 50h cm3
Hence,
Solution
200 < 50h < 320
Let m be the number of months required.
200 320
72 – 3m , 54 < h <
50 50
–3m , –18
4 < h < 6.4
m.6
\ the range of possible heights of water is
\ the least number of complete months needed is 7.
4 cm < height < 6.4 cm.

115

03 FWS3A(Exp)_ch3.indd 115 12/16/14 12:52 PM


© Star Publishing Pte Ltd. All rights reserved.

9. Water at 24 °C is poured into a boiler. The temperature Solution


of the water in the boiler increases by 7 °C per minute. Mean of the maximum temperatures of the 4 states
Find the time range such that the temperature of the =
48.4 + 45.7 + 47.7 + x
water in the boiler is greater than 52 °C but less than 4
73 °C. 141.8 + x
=
4
141.8 + x
Solution Hence, . 47
4
Let t minutes be the required time.
141.8 + x . 188
52 , 24 + 7t , 73
x . 46.2
28 , 7t , 49
4 , t ,7
Level 3
\ the required time range is 4 min , time , 7 min.
13. Company A charges $25 per hour for home maintenance.
Company B charges at the rate of $18 per hour, together
10. A rectangular frame is (2x + 1) cm long and x cm wide. with a basic home visit charge of $35. Find the minimum
If the perimeter of the frame is not less than 38 cm and number of complete hours of a maintenance job such
not greater than 80 cm, find the range of possible values that the cost of engaging company B is lower than that
of x. of company A.

x Solution
Let the duration of the maintenance job be x hours.
2x + 1
Amount charged by company A = $(25x)
Amount charged by company B = $(18x + 35)
If 18x + 35 , 25x,
Solution
then 7x . 35
Perimeter of the frame = 2[(2x + 1) + x]
x .5
= (6x + 2) cm
Hence, the cost of engaging company B is lower than
38 < 6x + 2 < 80
that of company A when company B is engaged for a
36 < 6x < 78
minimum number of 6 hours.
\ 6 < x < 13

14. A car can drive from Jurong East to Changi Airport


11. The sides of a triangle are (x – 8) cm, (x – 11) cm and
along either route X via AYE (Ayer Rajah Expressway)
(x + 4) cm. Find the range of possible values of x.
or route Y via PIE (Pan Island Expressway). Route X is
(Hint: The sum of any two sides of a triangle is greater
1 km shorter than route Y. The average speed of the car
than the third side.)
on route X is 60 km/h and that on route Y is 50 km/h at
a certain time period. Find the shortest driving distance
of route X such that the time taken to drive along route
x _ 11 x_8 X is at least 8 minutes less than that along route Y.

Solution
x+4 Let the driving distance of route X be x km. Hence, route
Y is (x + 1) km.
Solution x
Time taken to drive along route X = hours,
The longest side of the triangle is (x + 4) cm. 60
x +1
Since sum of two sides . the third side, route Y = hours
\ (x – 8) + (x – 11) . x + 4 50
x +1 x 8
2x – 19 . x + 4 – >
50 60 60
Hence, x . 23.
6(x + 1) – 5x > 5 × 8
6x + 6 – 5x > 40
12. Australia had a record hot day on 5 January 2013. The x > 34
maximum temperature in South Australia was 48.4 °C, Hence, the shortest driving distance on route X required
Victoria was 45.7 °C, New South Wales was 47.7 °C is 34 km.
and Queensland was x °C. The mean of the maximum
temperatures of the above 4 states was over 47 °C. Find
the possible values of x.

Chapter 3 Linear Inequalities


116

03 FWS3A(Exp)_ch3.indd 116 12/16/14 12:52 PM


© Star Publishing Pte Ltd. All rights reserved.

15.
The population of Singapore in 2010 was 5.077 million. (c) When x . 0,
Assume that the population of Singapore grows by –x , 0
0.11 million every year.  1   1 
(a) Based on the assumption, find the population of and (–x)  , 0 .
 x 2   x 2 
Singapore
1
(i) in 2011, \ – , 0 is true.
x
(ii) in 2012,
(iii) x years after 2010. (d) When a ,0
(b) Based on the assumption, in which year will the and b , 0,
population of Singapore first reach 6 million? then a + b , 0 + 0.
(c) “The population increases by a fixed number of \ a + b , 0 is true.
people every year.” Is this assumption reasonable?
Give your reason(s). (e) When a = –2 and b = 5,
a , b
1 1
Solution but – ,
2 5
(a) Population of Singapore 1 1
(i) in 2011 = 5.077 + 0.11 i.e. ,
a b.
= 5.187 million 1 1
(ii) in 2012 = 5.077 + 0.11 × 2 \ . is false.
a b
= 5.297 million
(iii) x years after 2010 = (5.077 + 0.11x) million
2. If a  7, state which of the following statements are true.
(b) 5.077 + 0.11x > 6 (a) a  17
0.11x > 0.923 (b) –a  –7
x > 8.39 (c) 3a + 4  25
i.e. the population will first reach 6 million nine (d) a2  49
years after 2010. Hence, the year is 2019.
Solution
(c) The assumption is NOT reasonable. The annual
growth of a population depends on many factors (a) a , 7
like birth rate, mortality rate, movement of people and 7 , 17,
across countries and other social factors. \ a , 17 is true.
(b) a.7
\ (–1)a . (–1)7
Revision Exercise 3 – a . –7
1. State whether each of the following statements is true or \ – a , –7 is false.
false. If it is false, give a numerical example to support (c) a,7
your claim. 3a , 3(7)
(a) If  x  0, then x2  x. 3a + 4 , 21 + 4
1 i.e. 3a + 4 , 25 is true.
(b) If  x  0, then  0.
x
1 (d) Let a = –8.
(c) If  x  0, then –  0.
x Then a , 7
(d) If  a  0 and b  0, then a + b  0. but a2 = (–8)2 = 64 . 49.
\ a2 , 49 is false.
1 1
(e) If  a  b, then  .
a b
3. Solve each of the following linear inequalities and
Solution represent the solution on a number line.
(a) When x = 0.1 . 0, (a) 4x + 9  –11
x2 = 0.01 , x. (b) 15 – 2x  23
\ x2 . x is false. (c) 7(2x – 5)  6(2x + 1)
5x – 8
(b) When x = –2 , 0, (d) <2
11
1 1
then =– , 0.
x 2
1
\ . 0 is false.
x

117

03 FWS3A(Exp)_ch3.indd 117 12/16/14 12:52 PM


© Star Publishing Pte Ltd. All rights reserved.

Solution 7−x 1 – 2x
(c) 
(a) 4x + 9 , –11 4 6
4x , –20 3(7 – x) < 2(1 – 2x)
x , –5 21 – 3x < 2 – 4x
Solution x < –19
(d) (x – 1)2 , (x + 9)2
x2 – 2x + 1 , x2 + 18x + 81
_5 0 –20x , 80
x . –4
(b) 15 – 2x > 23
–2x > 8
x < – 4 5. (a) Solve the inequality 13 – 2x  7.
(b) Find the smallest integer value of x such that
Solution 13 – 2x  7.

Solution
_4 0 (a) 13 – 2x  7
–2x  –6
(c) 7(2x – 5) . 6(2x + 1) x . 3
14x – 35 . 12x + 6 (b) The smallest integer value of x such that
2x . 41 13 – 2x  7 is 4.
1
x . 20
2
Solution 6. (a) Solve the inequalities –12  3x – 8  13.
(b) Write down the smallest and the greatest integers
which satisfy –12  3x – 8  13.
0 20 12
Solution
5x – 8 (a) –12  3x – 8  13
(d) <2
11 –12 + 8  3x  13 + 8
5x – 8 < 22 – 4  3x  21
5x < 30 –1  x  7
1
x < 6 3

Solution

_1 1 0 7
3
0 6
(b) The smallest integer which satisfies the inequality
is –1 and the greatest integer is 6.
4. Solve the following linear inequalities.
x x x
(a) – 2(x – 3)  4 (b) –  –13 7. (a) Solve the equation 6x2 + x – 15 = 0.
5 2 3
(b) Solve the linear inequality 19x + 7  4(4x + 1).
7−x 1 – 2x
(c)  (d) (x – 1)2  (x + 9)2 (c) Which root in (a) satisfies the inequality in (b)?
4 6

Solution
Solution
(a) 6x2 + x – 15 = 0
x
(a) – 2(x – 3) , 4 (3x + 5)(2x – 3) = 0
5
5 3

x – 10(x – 3) , 20 \ x =– or x=
3 2

x – 10x + 30 , 20
–9x , –10 (b) 19x + 7 , 4 (4x + 1)
x. 1
1 19x + 7 , 16x + 4
9 3x , –3
x x x , –1
(b) – > –13
2 3 5
3x – 2x > –78 (c) – , –1
3
x > –78 \ the required root is –
5
.
3

Chapter 3 Linear Inequalities


118

03 FWS3A(Exp)_ch3.indd 118 12/16/14 12:52 PM


© Star Publishing Pte Ltd. All rights reserved.

3x – 2 (c) 13 – (5x + 1) , 2x < 6(7 – x)


8. (a) Solve the inequalities –x   13.
4 13 – (5x + 1) , 2x and 2x < 6(7 – x)
(b) Write down each of the following values which 13 – 5x – 1 , 2x and 2x < 42 – 6x
3x – 2
satisfy –x   13 12 , 7x and 8x < 42
4
5 1
(i) the smallest rational number, 1 ,x and x<5
7 4
(ii) the largest prime number, 5 1
\ the solution is 1 ,x<5 .
(iii) the largest square number. 7 4

Solution 9+x
3x – 2 10. It is given that 5+x< 6 and y = x2 + 3.
(a) –x < < 13 9+x
5
4
(a) Solve  5 + x < 6.
3x – 2 3x – 2 5

–x < and < 13
4 4 (b) Hence, find the range of possible values of y.
– 4x < 3x – 2 and 3x – 2 < 52

–7x < –2 and 3x < 54 Solution
2 9+x

x> and x < 18 (a)
, 5+x<6
7 5
x  18 9+x
, 5+x and 5 + x < 6
2 5
x 7
9 + x , 5(5 + x) and x< 1

9 + x , 25 + 5x and x< 1
2 18 –16 , 4x and x< 1
7
–4 , x and x< 1
\ the solution is
2
< x < 18. \ the solution is – 4 , x < 1.
7
2 (b) y = x2 + 3
(b) (i) The smallest rational number = .
7 The minimum value of y occurs when x = 0,
(ii) The largest prime number = 17. i.e. y = 02 + 3
(iii) The largest square number = 16. =3
When x = – 4,
y = (– 4)2 + 3
9. Solve the following simultaneous linear inequalities.
2x 3x x
= 19
(a) 3(2x – 1)  2(9 + 5x) and – 1– Hence, the range of possible values of y is
5 4 2
(b) –8  5x + 2  37 3 < y , 19.
(c) 13 – (5x + 1)  2x  6(7 – x)
11. The velocity v of a particle at time t is given by the
Solution formula v = u + at.
(a) 3(2x – 1)  2(9 + 5x) (a) Find the value of a when u = 18, v = 30, and t = 4.
6x – 3  18 + 10x (b) When u = 10 and a = 3, find the range of values
– 4x  21 of t such that v is greater than 16 but less than 49.
1
x . –5
4
Solution
2x 3x x
– 1– (a) v = u + at
5 4 2
When u = 18, v = 30, t = 4,
 2 x 3x   x
20 –  20  1 – 30 = 18 + a(4)
 5 4   2 
4a = 12
8x – 15x  20 –10x \ a=3
3x  20
2 (b) When u = 10 and a = 3,
x6
3 v = 10 + 3t
\ the solution is –5
1
x6 .
2 if 16 , v , 49,
4 3
then 16 , 10 + 3t , 49
(b) –8 < 5x + 2 , 37 6 , 3t , 39
–10 < 5x , 35 2 , t , 13
–2 < x , 7 \ the range of time t is 2 , t , 13.

119

03 FWS3A(Exp)_ch3.indd 119 12/16/14 12:52 PM


© Star Publishing Pte Ltd. All rights reserved.

3 Solution
12. A family consumes kg of rice a day. If the family has
10 Let the time taken for a hair cut of a man be x minutes.
15 kg of rice, find the minimum number of days required i.e. the time taken for a hair cut of a lady
for the amount of rice remaining to be less than 4 kg. = (x + 20) minutes
\ the total time taken for the hair cut of 8 men and
Solution 5 ladies = 8x + 5(x + 20)
Let x be the number of days required. = 8x + 5x + 100
15 –
3
x ,4 = (13x + 100) minutes
10
150 – 3x , 40 295 , 13x + 100 < 503
–3x , –110 195 , 13x < 403
110 15 , x < 31
x .
3 \ the least number of whole minutes taken for the hair
i.e. x. 36
2 cut of a man is 16.
3
\ there will be less than 4 kg of rice after 37 days.
Go Further
1. (a) Solve the simultaneous equations in x and y:
13. A recreation club charges a monthly membership fee of 2x + y = 3k – 4   and   x – 2y = k + 1,
$20 and an admission fee of $3 per entry. Cathy spent where k is a constant. Express your solution in
more than $50 on the club in a certain month. Find the terms of k.
minimum number of times she has visited the club in (b) If the solution in (a) satisfies the inequality
that month. x . y . 0, find the possible values of k.

Solution Solution
Let x be the number of times that Cathy has visited the (a) 2x + y = 3k – 4..................... (1)
club. x – 2y = k + 1...................... (2)
20 + 3x . 50 (1) 3 2: 4x + 2y = 6k – 8.................... (3)
3x . 30 (2) + (3): 5x = 7k – 7
x . 10 7
\ the minimum number of times is 11. x= (k – 1)
5
7
Substitute x = (k – 1) into (1)
14. Company A quotes a rental rate for a car at $45 per 5
14
day. Company B quotes a rate of $38 per day, together (k – 1) + y = 3k – 4
5
with a fixed charge of $75. Find the minimum number 14
of complete days of rental required such that the y = 3k – 4 – (k – 1)
5
total charges by Company B are lower than those by
= 3k – 4 –
14
k +
14
Company A. 5 5
1 6

= k–
5 5
Solution 1
Let x be the number of days of rental required. \ y= (k – 6)
5
75 + 38x , 45x 7 1
\ x = (k – 1) and y = (k – 6)
75 , 7x 5 5
7x . 75 (b) x .y.0
5
x . 10 7
(k – 1) .
1
(k – 6) . 0
7
5 5
Hence, the minimum number of complete days required 7 1 1
is 11. (k –1) . (k – 6) and k – 6) . 0
5 5 5

7(k – 1) . k – 6 and k–6 .0
7k – 7 . k – 6
15. The time taken for a hair cut of a man is 20 minutes
6k . 1
less than that of a lady. The total time taken for the hair 1
cut for 8 men and 5 ladies is more than 295 min but not k . and k .6
6
more than 503 min. Find the possible time taken for the \ the possible values of k are k > 6.
hair cut of a man.

Chapter 3 Linear Inequalities


120

03 FWS3A(Exp)_ch3.indd 120 12/16/14 12:52 PM


© Star Publishing Pte Ltd. All rights reserved.

2. Mr Tan is going to buy a new refrigerator for his


family. Model ABC costs $750 and its annual energy
consumption is 670 kWh. Another model PQR of similar
capacity costs $860 and its annual energy consumption is
560 kWh. Assume that the electricity charge is 28 cents
per kWh and the lifespan of a refrigerator is 10 years.
(a) Show that model PQR has the lower total cost of
ownership for its lifespan. What is the difference
in the total cost of ownership between the two
models?
(b) If model ABC has a discount of x % for its price,
what should the value of x be such that the total
cost of ownership of this model is lower than that
of model PQR?

Solution
(a) Total cost of ownership of model ABC
= $750 + $(670 3 0.28 3 10)
= $2626
Total cost of ownership of model PQR
= $(860 + $(560 3 0.28 3 10)
= $2428
Thus, model PQR has the lower total cost of
ownership for its lifespan. (shown)
The difference in the total cost of ownership
between the 2 models
= $2626 – $$2428
= $198
(b) For the total cost of ownership of model ABC to
be lower than that of model PQR, the discount
198
x% . 3 100%
750
\ x . 26.4

121

03 FWS3A(Exp)_ch3.indd 121 12/16/14 12:52 PM


© Star Publishing Pte Ltd. All rights reserved.

Review Exercise 1

1. (a) If 3p 3 4 = 108, find the value of p.  3x 


3 –
1
27x 3

1

1 3

(c) ÷ (81x2y–6) = ÷ (81 x y )


4 4 2 2
(b) Express 1 ÷ x–5 with positive index.  4y  64y 3
(c) Simplify (a3b–4)2(a –2 b5)3. 3

27x 3 y2
= ÷
64y 3
1 1
Solution 81 4 x 2
1 1
(a) 3p 3 4 = 108 27x 3 81 4 x 2
3p 3 4 = 4 3 27 = 3
64y 3 3

3p 3 4 = 4 3 33 y2
7
The value of p is 3. 81x 2
= 9
1
(b) 1÷x =1÷ –5
64y 2
x5
x5
= 1 3
1
= x5 3.
Place Australia Hong Kong Singapore
(c) (a3b–4)2(a –2 b5)3 = a6 b–8 a –6 b15 Population
= a 0 b7 22 683 600 7 136 300 5 312 400
in 2012
= b7
Area in
7 682 300 1104 716
–3 2012 (km2)
2. (a) Evaluate  5
  , expressing your answer as a
2 The above table shows some statistics of Australia, Hong
fraction. Kong and Singapore in 2012.
1
(b) Given that 3 a 3
3
a – a and a ≠ 0,
k
(a) Find the difference in the population of Australia
a
find the value of k. and Singapore, giving your answer in standard form
 3x 
3 –
1
correct to 3 significant figures.
(c) Simplify  4y 
÷ (81x2y–6) 4 . (b) The population density of a place is defined to be
population per square km of area. Find the ratio
Solution of the population density of Hong Kong to that of
–3 Singapore, giving your answer in the form 1 : r.
(a) 

5
 = 1
2  5
3

 2  Solution
(a)
Difference in the population of Australia and
=
1
Singapore
= 22 683 600 – 5 312 400
125
8
8
= 12 371 200
= = 1.74 3 107
125
(b) Population density of Hong Kong
1
(b) 3 a 3
3
a=a
k
=
7 136 300
a 1104
1 1
= 6464 (correct to nearest whole number)

a –1 × a ×a = ak
2 3

1 1
Population density of Singapore
–1 + + =k 5 312 400
2 3 =
716
1
k=– = 7420 (correct to nearest whole number)
6
Ratio = 6464 : 7420
= 1 : 1.15

Review Exercise 1
122

04 FWS3A(Exp)_RevEx1.indd 122 12/16/14 12:53 PM


© Star Publishing Pte Ltd. All rights reserved.

4. (a) Evaluate (2.48 3 106) ÷ (7.75 3 10 –3), giving your (b)


2x + 1

x–7
=0
answer in standard form. x+3 x2 – 9
(b) The wavelength of yellow light is 565 nm. Express
2x + 1

( x – 7)
=0
this wavelength in metre, in standard form. x+3 ( x – 3)( x + 3)
(c) T h e wave l e n g t h o f u l t r av i o l e t l i g h t i s (2 x + 1)( x – 3) – ( x – 7)
( x – 3)( x + 3) =0
4 3 10 –8 m. Find the difference between the
wavelengths of yellow light and ultraviolet light, 2x2 – 6x + x – 3 – x + 7 = 0
giving your answer in standard form. 2x2 – 6x + 4 = 0
x2 – 3x + 2 = 0
Solution (x – 2)(x – 1) = 0
(a) (2.48 3 106) ÷ (7.75 3 10 –3) = 3.2 3 108 x – 2 = 0 or x–1=0
x=2 x=1
(b) 565 3 10 m = 5.65
–7
3 10 m
–7

(c) Difference in wavelength


7. Solve the following equations.
= (5.65 3 10 –7) – (4 310 –8)
(a) 2y2 – 5y – 6 = 0, giving your answers correct to
= 5.25 3 10 –7 m
2 decimal places.
2
(b) 27x = 95x + 4
5. Solve the following equations.
(a) 3(x – 1)2 = 4(x – 1) Solution
(b) (2x – 5)2 = 49 (a) 2y2 – 5y – 6 = 0
a = 2, b = –5, c = –6
Solution
–(–5) ± (–5) 2 – 4(2)(–6) 2
(a) 3(x – 1)2 = 4(x – 1) y =
2(2)
3(x2 – 2x + 1) = 4x – 4
5± 73
3x2 – 6x + 3 = 4x – 4 =
4
3x2 – 10x + 7 = 0
= 3.39 or – 0.89
(3x – 7)(x – 1) = 0
x2
3x – 7 = 0 or x – 1 = 0 (b) 27 = 95x + 4
2
3x = 7 x= 1 33x = (32)5x + 4
3x2
7 3 = 310x + 8
x=
3 3x2 = 10x + 8
= 2
1 3x – 10x – 8 = 0
2

3 (3x + 2)(x – 4) = 0
3x + 2 = 0 or x – 4 = 0
(b) (2x – 5)2 = 49 3x = –2 x=4
2x – 5 = ± 49 x =–
2
2x – 5 = 7 or 2x – 5 = –7 3
2x = 7 + 5 or 2x = –7 + 5
2x = 12 2x = –2 8. (a) Solve the equation 2x(x + 11) = 5(1 + 2x2).
x = 6 x = –1 (b) Solve the inequality 3(2 – x) . 2x + 1.
(c) W hich root in (a) satisfies the inequality
6. Solve the following equations. in (b)?
y+5 3
(a) = y+5 Solution
12
2x + 1 x–7 (a) 2x(x + 11) = 5(1 + 2x2)
(b) – =0 2x2 + 22x = 5 + 10x2
x+3 x2 – 9
8x2 – 22x + 5 = 0
Solution (4x – 1)(2x – 5) = 0
(a)
y+5
=
3 4x – 1 = 0 or 2x – 5 = 0
12 y+5 4x = 1 2x = 5
( y + 5)2 = 36 1 5
x= x=
y2 + 10y + 25 = 36 4 2
y2 + 10y – 11 = 0 = 2
1

( y + 11)( y – 1) = 0 2
y + 11 = 0 or y – 1= 0
y = –11 y=1

123

04 FWS3A(Exp)_RevEx1.indd 123 12/16/14 12:53 PM


© Star Publishing Pte Ltd. All rights reserved.

(b) 3(2 – x) . 2x + 1 Solution


6 – 3x . 2x + 1 (a) –6 ,
7 – 2x
and
7 – 2x
,
3
x–5
–3x – 2x . 1–6 3 3 2
–5x . –5 7 – 2x 3x – 10
–18 , 7 – 2x and ,
x , 1 3 2

2x , 7 + 18 and 14 – 4x , 9x – 30

2x , 25 and – 4x – 9x , –30 – 14
(c) x1
25

x, and –13x , – 44
2
_1 0 1 2 21 3 1 5
1
4 2
x , 12 and x.3
2 13
5
1 x  313
\ The root x = satisfies the inequality in (b).
4
x  12 12

9. (a) Solve the inequality 11 – 5x . 2(x – 6).


(b) Solve the inequality 3(2x + 1) , 4(7 – x). 5
313 12 12
(c) Find the positive integers which satisfy both the
inequalities in (a) and (b). The required solution is 3
5
, x , 12 .
1
13 2

Solution (b) (i) The largest integer is 12.


(a) 11 – 5x . 2(x – 6) (ii) The smallest prime number is 5.
11 – 5x . 2x – 12
–5x – 2x . –12 – 11
–7x . –23 11. A water molecule consists of two hydrogen atoms and
23 one oxygen atom. The mass of one hydrogen atom is
x , 1.674 3 10 –24 g and the mass of one oxygen atom is 2.657
7
3 10 g. Find
–23
2
x , 3 (a) the mass of one water molecule,
7
(b) the mass of 3 3 105 water molecules,
(b) 3(2x + 1) , 4(7 – x)
(c) the number of water molecules in 100 g of water.
6x + 3 , 28 – 4x
6x + 4x , 28 – 3 Solution
10x , 25 (a) Mass of one water molecule
x , 2.5 = (1.674 3 10 –24 3 2) + (2.657 3 10 –23)
1
x , 2 = 2.991 8 3 10 –23 g
2
(b) Mass of 3 3 105 water molecules
(c) x  2 21
= (2.991 8 3 10 –23) × (3 3 105)
x  3 72 = 8.975 4 3 10 –18 g
(c) Number of water molecules
_1 0 1 2 2 21 3 3 72 100 g
=
2.9918 × 10 –23 g
\ T he positve integers 1 and 2 satisfy both = 3.34 3 1024 (correct to 3 s.f.)
inequalities in (a) and (b).

7 – 2x 3 12.
The first 4 terms in a sequence u1, u2, u3, u4, … are given
10. (a) Solve the inequalities –6 , , x – 5. below.
3 2
(b) Write down each of the following values which u1 = 30 + 2 = 3,
7 – 2x 3
u2 = 31 + 4 = 7,
satisfy –6 , , x –5 u3 = 32 + 6 = 15,
3 2
(i) the largest integer, u4 = 33 + 8 = 35.
(ii) the smallest prime number. (a) Write down an expression for u5 and show that
u5 = 91.
(b) Write down an expression for u6 and evaluate it.
(c) Express the nth term, un, in terms of n.
(d) Find the value of u10.
(e) (i) Show that 3n – 1 – 3n – 2 = 2 3 3n – 2.
(ii) Express un – un – 1 in terms of n.

Review Exercise 1
124

04 FWS3A(Exp)_RevEx1.indd 124 12/16/14 12:53 PM


© Star Publishing Pte Ltd. All rights reserved.

Solution (iii) Time taken by machine B to produce 1 shaft


(a) u5 = 34 + 10 = 81 + 10 =
60
= 91 (shown) 12 + 3
= 4 minutes
(b) u6 = 35 + 12 To produce 30 shafts, machine B will take
= 255 4 3 30 = 120 minutes.
(c) un = 3n – 1 + 2n
(d) u10 = 310 – 1 + 2(10) 14.
A cannon ball is fired from the top of a cliff to the
= 39­ + 20 sea. The height, h metres, of the cannon ball above
= 19 703 sea level at time t seconds is given by the function
3n 3n h = 60 + 30t – 5t2 for 0 < t < T, where T seconds is the
(e) (i) 3n – 1 – 3n – 2 = – time that the cannon ball hits the sea.
3 3
2

3(3n ) – 3n (a) Draw the graph of h = 60 + 30t – 5t2 for 0 < t < 8
= using a scale of 1 cm to 1 unit on the horizontal
32
n
3 (3 – 1) axis and 1 cm to 10 units on the vertical axis.
= (b) What is the height of the cliff?
32
= 3n – 2 3 2 (shown) (c) Using the graph, find
(i) the maximum height of the cannon ball and
(ii) un – un – 1 its corresponding time,
= (3n – 1 + 2n) – (3n – 2 + 2n – 2) (ii) the value of T.
= 3n – 1 – 3n – 2 + 2n – 2n + 2 (d) Find the time interval that the cannon ball is at
= 2 3 3n – 2 + 2 least 85 m above sea level.
= 2(3n – 2 + 1)
Solution
(a) h = 60 + 30t – 5t 2
13.
A machine A can produce x shafts in one hour. Another
machine B can produce 3 more shafts than A in one hour. h 60 85 100 105 100 85 60 25 –20
(a) Express, in terms of x, the number of minutes it
takes t 0 1 2 3 4 5 6 7 8

(i) machine A to produce one shaft,
(ii) machine B to produce one shaft.
(b) It takes 1 minute longer for machine A than machine h
B to produce one shaft.
(i) Write down an equation to represent the 120
information and show that it reduces to
x2 + 3x – 180 = 0. 100
(ii) Solve the equation x2 + 3x – 180 = 0.
(iii) Find the time taken to produce 30 shafts by 80
h = 60 + 30t – 5t2
machine B.
60
Solution
(a) (i) Number of minutes taken by machine A 40
60
=
x
20
(ii) Number of minutes taken by machine B
60
= t
x+3 O 1 2 3 4 5 6 7 8
60 60
(b) (i) – =1 _20
x x+3
60(x + 3) – 60x = 1(x)(x + 3)
60x + 180 – 60x = x2 + 3x
x2 + 3x – 180 = 0 (shown) (b)
The height of the cliff is 60 m.
(ii) x2 + 3x – 180 = 0 (c)
(i) From the graph, the maximum height of the
(x + 15)(x – 12) = 0 cannon ball is 105 m and its corresponding
x + 15 = 0   or x – 12 = 0 time is 3 seconds.
x = –15 x = 12 (ii) The value of T is 7.6 seconds.
(d) 1 < t < 5

125

04 FWS3A(Exp)_RevEx1.indd 125 12/16/14 12:53 PM


© Star Publishing Pte Ltd. All rights reserved.

15. The petrol consumption rate of a car is x litres/km


on highways and (x + 0.025) litres/km on city streets.
(a) Express the distance travelled for 40 litres on
highways in terms of x.
(b) Express the distance travelled for 48 litres on city
streets in terms of x.
(c) If the distance in (a) is 16 km more than the
distance in (b), form an equation in x, and show
that it reduces to 80x2 + 42x – 5 = 0.
(d) Solve the equation 80x2 + 42x – 5 = 0.
(e) Hence find the petrol consumption rate, in
litres/km, of the car travelling on city streets.

Solution
(a) Distance travelled for 40 litres on highways
40
= km
x

(b) Distance travelled for 48 litres on city streets.


48
= km
x + 0.025

40 48
(c) – = 16
x x + 0.025
40(x + 0.025) – 48x = 16(x)(x + 0.025)
40x + 1 – 48x = 16x2 + 0.4x
16x2 + 8.4x – 1 =0
160x2 + 84x – 10 =0
80x2 + 42x – 5 = 0 (shown)
(d) 80x2 + 42x – 5 = 0
a = 80, b = 42, c = –5
2
–42 ± 42 – 4(80)(–5)
x =
2(80)
–42 ± 3364

=
160
1 5

= or –
10 8

(e) The petrol consumption rate of the car travelling


on city streets
= 0.1 + 0.025
= 0.125 litres/km

Review Exercise 1
126

04 FWS3A(Exp)_RevEx1.indd 126 12/16/14 12:53 PM


© Star Publishing Pte Ltd. All rights reserved.

4 Conditions of Congruence
and Similarity

Class Activity 1
Objective: To examine what conditions are necessary for congruence.
1. In this activity, you may use a ruler, a protractor and a pair of compasses, or The Geometer’s Sketchpad (GSP), for your
construction.

4 3.5

44° 53°
B C
5

Construct six different triangles, each according to the conditions given in (a) to (f). Label each triangle ABC. In each case,
compare your triangle to those drawn by your classmates. Are the two triangles congruent?
(a) ∠ABC = 44°
(b) BC = 5 cm
(c) ∠ABC = 44° and ∠ACB = 53°
(d) AB = 4 cm and BC = 5 cm
(e) ∠ABC = 44° and BC = 5 cm
(f ) AB = 4 cm, BC = 5 cm and AC = 3.5 cm

2. From your observations of the above drawings, answer the following questions. Can two triangles be congruent if

(a) there is only one pair of equal corresponding angles, Yes/No

(b) there is only one pair of equal corresponding sides, Yes/No

(c) only two pairs of corresponding angles are equal, Yes/No

(d) only two pairs of corresponding sides are equal, Yes/No

(e) there are only one pair of equal corresponding angles and one pair of equal corresponding sides, Yes/No

(f ) three pairs of corresponding sides are equal? Yes/No

127

05 FWS3A(Exp)_ch4.indd 127 12/16/14 12:53 PM


© Star Publishing Pte Ltd. All rights reserved.

Class Activity 2
Objective: To determine what conditions are necessary for congruence.
A
1. In this activity, you may use a ruler, protractor and a pair of compasses, or GSP, 110°
for your construction. 3.4 2.7

Construct five different triangles, each according to the conditions given in (a) 30° 40°
to (e). Label each triangle ABC. In each case, determine whether your triangle is B C
5
congruent to those drawn by your classmates.

Condition Given Congruences (Yes/No)

(a) SAS AB = 3.4 cm, ∠ABC = 30°, BC = 5 cm Yes

(b) ASA ∠ABC = 30°, BC = 5 cm, ∠ACB = 40° Yes

(c) ASS (or SSA) ∠ABC = 30°, BC = 5 cm, AC = 2.7 cm No

(d) AAA ∠BAC = 110°, ∠ABC = 30°, ∠ACB = 40° No

(e) AAS (or SAA) ∠ABC = 30°, ∠BCA = 40°, AC = 2.7 cm No

2. Determine whether each of the above conditions can guarantee the congruence of two triangles.

Class Activity 3
Objective: To construct a right-angled triangle with a given side and the hypotenuse, and examine a condition for
the congruence of right-angled triangles.

B 2 C

1. Draw a triangle ABC in which ∠ABC = 90°, BC = 2 cm and AC = 3 cm, using a ruler and a pair of compasses, or GSP.

2. Determine whether your triangle is congruent to those drawn by your classmates.

3. Can you suggest a condition for two right-angled triangles to be congruent?

Two right-angled triangles are congruent if the hypotenuse and one side of a triangle are equal to those of another triangle.

Chapter 4 Conditions of Congruence and Similarity


128

05 FWS3A(Exp)_ch4.indd 128 12/16/14 12:53 PM


© Star Publishing Pte Ltd. All rights reserved.

Class Activity 4
Objective: To construct two triangles with equal corresponding angles, and determine a condition for similar
triangles.

Do the following using The Geometer’s Sketchpad.

Tasks
(a) Construct a triangle ABC.
(b) Construct another triangle XYZ in which XY // AB, YZ // BC and ZX // CA.
(c) Measure all the angles and the sides of ABC and XYZ.
(d) Calculate the ratios of the sides AB : XY, BC : YZ and CA : ZX.

Questions
1. What can you say about the angles of ABC and XYZ? Explain your answer.

They are the same.

2. Drag the point A or C around and observe the variation in the numerical values of the angles and the sides of the triangles.
What can you say about the angles of the two triangles? What can you say about the ratios of the sides?

The three angles of a triangle are equal to the corresponding angles of the other triangle. The ratios of the sides are always equal.

3. Suggest a condition for the similarity of triangles.

Two triangles are similar if all three pairs of their corresponding angles are equal.

129

05 FWS3A(Exp)_ch4.indd 129 12/16/14 12:53 PM


© Star Publishing Pte Ltd. All rights reserved.

Class Activity 5
Objective: To construct two triangles with sides in proportion, and determine a condition for similar triangles.

Do the following using The Geometer’s Sketchpad.


Tasks
(a) Construct a triangle ABC.
(b) Select ABC and dilate it to XYZ.
(c) Measure all the angles and sides of ABC and XYZ.
(d) Calculate the ratios of the sides AB : XY, BC : YZ and CA : ZX.

Questions
1. Drag the point A or C around and observe the variation in the numerical values of the angles and the sides of the triangles.
What can you say about the three ratios of the sides? What can you say about the angles of the two triangles?

The three ratios of the sides are always equal to 0.4. The three pairs of corresponding angles of the two triangles are always equal.

2. Suggest a condition for the similarity of triangles.

Two triangles are similar if the three pairs of ratios of their corresponding sides are equal.

Chapter 4 Conditions of Congruence and Similarity


130

05 FWS3A(Exp)_ch4.indd 130 12/16/14 12:53 PM


© Star Publishing Pte Ltd. All rights reserved.

Class Activity 6
Objective: To construct two triangles, a pair of equal corresponding angles and two pairs of sides in proportion, and
determine a condition for similar triangles.

Do the following using The Geometer’s Sketchpad.

Tasks
(a) Construct a triangle ABC.
(b) Dilate BC to BZ and BA to BX using the same ratio. Draw the line segment XZ to form XBZ.
(c) Measure all the angles and sides of ABC and XBZ.
(d) Calculate the ratios of the sides AB : XB, BC : BZ and CA : ZX.

Questions
1. Drag the point A or C around and observe the variation in the numerical values of the angles and the sides of the triangles.
What can you say about the three ratios of the sides? What can you say about the angles of the two triangles?

The three ratios of the sides are equal. The three pairs of angles of the two triangles are equal.

2. Suggest a condition for the similarity of triangles.


Two triangles are similar if the ratios of two pairs of their corresponding sides are equal and the pair of included angles of these sides

are equal.

131

05 FWS3A(Exp)_ch4.indd 131 12/16/14 12:53 PM


© Star Publishing Pte Ltd. All rights reserved.

Class Activity 7
Objective: To find the relationship between the ratio of sides and the ratio of areas of similar plane figures.
In this activity, the unit of length is cm. Copy and complete the following table.

Similar plane figures Ratio of sides Ratio of areas

Triangles Ratio of bases Ratio of areas


1
× 8 × 10
= 10 : 20 = 2

16 × 20
16 2
8
= 1:2 = 1:4
10 20

Squares Ratio of sides Ratio of areas


= 12 : 18 = 9 × 15
12 × 20

= 2:3 = 9 : 16
12 18

Rectangles Ratio of breadths Ratio of areas


= 9 : 12 = 9 × 15
12 × 20

9 12
= 3:4 = 9 : 16
15 20

Circles Ratio of radii Ratio of areas


= 15 : 18 =
r × 15 2
r × 18 2
15 18
= 5:6
= 25 : 36

Octagons Ratio of sides Ratio of areas


= a:b = a2 : b2

a b

Chapter 4 Conditions of Congruence and Similarity


132

05 FWS3A(Exp)_ch4.indd 132 12/16/14 12:53 PM


© Star Publishing Pte Ltd. All rights reserved.

Class Activity 8
Objective: To find the relationship between the ratio of sides and the ratio of volumes of two similar solids.
In this activity, the unit of length is cm. Copy and complete the following table.

Similar solids Ratio of sides Ratio of volumes

Cubes Ratio of sides Ratio of volumes


= 5 : 10 = 52 : 102

= 1:2 = 1:8
5 10

Square pyramids Ratio of sides Ratio of volumes


1
× 42 × 6
= 4:6 = 3
1 × 2×
6 9
3

9 = 2:3 = 8 : 27
6

4 6

Cylinders Ratio of sides Ratio of volumes


= 9 : 15 =
r × 62 r × 9
r × 10 2 r × 15

15
9 = 3:5 = 27 : 25
6 10

Spheres Ratio of radii Ratio of volumes

= 4 ra3 : 4 rb3
= a:b 3 3

a b
= a3 : b3

133

05 FWS3A(Exp)_ch4.indd 133 12/16/14 12:53 PM


© Star Publishing Pte Ltd. All rights reserved.

Try It! In nTUV,


Section 4.1 ∠UTV = 180° – ∠VUT – ∠UVT
= 180° – 35° – 100°
1. In the figure, AB = AC, M is the midpoint of BC and = 45°
∠ABM = 60°.
(a) Show that ABM ≡ ACM. (b) nLMN ≡ nTUV (ASA)
(b) Find ∠y. (∠MNL = ∠UVT, LN = TV, ∠MLN = ∠UTV)
A

4. In the figure, AB = 7 cm, CD = 7 cm, ∠CBD = 90°,


∠ADB = 90° and ∠BAD = 65°.
(a) Name the congruent triangles in the figure.
B
60° y
C
(b) Find ∠x.
M C

7
Solution
(a) In nABM and nACM, D x
B
AM = AM (common sides)
AB = AC (given) 65°
7

BM = CM (given) A
\ nABM ≡ nACM (SSS)
(b) ∠ACM = ∠ABM (corr. parts of ≡ ns) Solution
\ ∠y = 60° (a) In nABD and nCDB,
BD = DB (common)
∠ADB = ∠CBD = 90° (given)
2. In the figure, ACE and BCD are straight lines. AC = DC, AB = CD (given)
BC = EC and DE = 8 cm. \ nABD ≡ nCDB (RHS)
(a) Name the triangle that is congruent to ABC.
(b) Find the length of AB. (b) ∠CDB = ∠ABD (corr. parts of ≡ ns)
A D \ ∠x = ∠ABD
= 180° – 90° – 65° (∠ sum of n)
C
8 = 25°

B E Section 4.2
5.
In the figure, PQ // BC, AP = 4 cm, BP = 4 cm,
Solution BC = 6 cm and PQ = x cm.
(a) In nABC and nDEC, (a) Which angle is equal to
∠ACB = ∠DCE (vert. opp. ∠s) (i) ∠ABC,
AC = DC (given) (ii) ∠ACB?
BC = EC (given) (b) Name a pair of similar triangles and give the reason
\ nABC ≡ nDEC (SAS) for the similarity.
(b) AB = DE (corr. parts of ≡ ns) (c) Find the value of x.
A
\ AB = 8 cm
4

3. (a) Find ∠MLN and ∠UTV in the diagram. P


x
Q
(b) State why LMN is congruent to TUV.
4
L T
B C
6
7 7
Solution
100° 100°
M
35°
N V
35°
U
(a) (i) ∠ABC = ∠APQ (corr. ∠s, PQ // BC)
(ii) ∠ACB = ∠AQP (corr. ∠s, PQ // BC)
Solution (b) nAPQ is similar to nABC. (Angle-Angle-Angle
(a) In nLMN, similarity)
∠MLN = 180° – ∠LMN – ∠MNL (∠ sum of n) PQ AP
(c)
= (corr. sides of similar ns)
= 180° – 35° – 100° BC AB
= 45° x 4
=
6 8
\ x = 3
Chapter 4 Conditions of Congruence and Similarity
134

05 FWS3A(Exp)_ch4.indd 134 12/16/14 12:53 PM


© Star Publishing Pte Ltd. All rights reserved.

6. In the diagram, AB = 6 cm, BC = 9 cm, CA = 12 cm, 8. In the diagram, AF // BE // CD, AB = 30 cm, BC = 15 cm,
DE = 8 cm, EF = 12 cm, FD = 16 cm and ∠DEF = 104°. CD = 10 cm, BE = x cm and AF = y cm. Find the values
(a) Show that ABC is similar to DEF. of x and y.
(b) Find ∠y. F
D
A
y D
16 E
12 8
6 x 10
y 104°
B C E F A 30 B 15 C
9 12

Solution
AB 6 3 Solution
(a) = = nABE is similar to nACD. (Angle-Angle-Angle
DE 8 4
BC 9 3 similarity)
= =
EF 12 4 BE AB
CA
\ =
12 3 CD AC
= =
FD 16 4 x 30
AB BC CA
=
\ = = 10 30 + 15
DE EF FD 20
x=
Hence, nABC is similar to nDEF. (Side-Side- 3
Side similarity) = 6
2
3
(b) ∠ABC = ∠DEF (corr. ∠s of similar ns) nCBE is similar to nCAF. (Angle-Angle-Angle
\ ∠y = 104° similarity)
BE CB
\ =
7. In the diagram, AB = 6 cm, BC = 4 cm, CA = 9 cm, AF AB
LM = 9 cm, MN = 6 cm, LN = y cm, ∠ABC = 127° and 20
15
=
3
∠LMN = 127°.
y 15 + 30
(a) Show that ABC is similar to LMN.
y = 20
(b) Find the value of y.
L
Section 4.3
A
y
9. In the diagram, PQ // BC, AQ = 4 cm, QC = 6 cm and
9 9 the area of ABC = 37.5 cm2. Find
6 (a) the ratio of the areas of APQ and ABC,
127°
C
127°
(b) the area of APQ, A
B M N
4 6 (c) the area of BCQP.
4
P Q
Solution
AB 6 2 6
(a) = =
LM 9 3 B C
BC 4 2
= =
MN 6 3 Solution
In nABC and nLMN, (a) AQ : AC = 4 : (4 + 6)
AB BC = 4 : 10
= (proved) =2:5
LM MN
∠ABC = ∠LMN (given) nAPQ is similar to nABC. (Angle-Angle-Angle
\ nABC is similar to nLMN. (Side-Angle-Side similarity)
similarity) \ area of nAPQ : area of nABC = AQ2 : AC2
= 22 : 52
AC AB = 4 : 25
(b) = (corr. sides of similar ns)
LN LM Area of APQ 4
9 2 (b) =
= 37.5 25
y 3 4
27
Area of nAPQ = 3 37.5
\ y = 25
2 = 6 cm 2

= 13.5
(c) Area of BCQP = 37.5 – 6
= 31.5 cm2

135

05 FWS3A(Exp)_ch4.indd 135 12/16/14 12:53 PM


© Star Publishing Pte Ltd. All rights reserved.

10. The areas of two regular hexagons are 25 cm 2 and (b) Ratio of the volumes of the cubes = a3 : b3
64 cm 2. Find the ratio of the corresponding sides of = 53 : 73
these two hexagons. = 125 : 343
Mass of the smaller cube 125
(c) =
Solution Mass of the larger cube 343
Let a : b be the ratio of the corresponding sides of the 125
\ mass of the smaller cube = 686
two hexagons.
3
343
Then, a2 : b2 = 25 : 64 = 250 g
a : b = 25 : 64
=5:8 13. The diagram shows a frustum formed by cutting off a
small cone VCD from the cone VAB of height 8 cm. Find
(a) the length of MN,
11. In the diagram, ABCD is a trapezium with AB // DC, AC (b) the ratio of the area of the upper base to that of the
and BD intersect at E. AE = 10 cm, EC = 6 cm and the lower base of the frustrum,
area of nCDE = 13.5 cm2. Find the area of (c) the volume of the frustum if the volume of the cone
(a) nABE, VCD is 42r cm3.
D C
(b) nBCE.
V
6
E

C 3 D 8
10 N

A B
A 6
M
B
Solution
(a)
nCDE is similar to nABE. (Angle-Angle-Angle
similarity)
Solution
\ area of nCDE : area of nABE = 62 : 102
(a) nVND is similar to nVMB. (Angle-Angle-Angle
= 36 : 100
Area of ABC
similarity)
100
= VN ND
36 36 =
VM MB
100
Area of nABE = 13.5 VN 3
=
3
36
8 6
= 37.5 cm 2
VN = 4
(b) nABE and nBCE are two triangles with the same \ MN = VM – VN
height from B. = 8 – 4
Area of  BCE CE = 4 cm
=
Area of  ABE AE
(b) Area of the upper base = r(3)2
Area of nBCE =
6
3 37.5 = 9r cm2
10 Area of the lower base = r(6)2
= 22.5 cm2 = 36r cm2
Ratio of the areas = 9 : 36
Section 4.4 =1:4
12. The ratio of the areas of the corresponding faces of two (c) Volume of the small cone VCD
crystal cubes is 25 : 49. 1
(a) Find the ratio of the corresponding sides of the = r 3 32 3 4
3
cubes. = 12r cm 3

(b) Find the ratio of the volumes of the cubes. 3


(c) If the mass of the larger cube is 686 g, find the Volume of the larger cone VAB  MB 
=  
Volume of the smaller cone VCD ND 
mass of the smaller cube.
\ volume of the larger cone VAB
3
Solution  6
= 12r 3  
(a)
Let a : b be the ratio of the corresponding sides of 3
the cubes. = 96r cm3
a2 : b2 = 25 : 49 \ volume of the frustum = 96r – 12r
\ a : b = 25 : 49 = 84r cm3
= 5 : 7

Chapter 4 Conditions of Congruence and Similarity


136

05 FWS3A(Exp)_ch4.indd 136 12/16/14 12:53 PM


© Star Publishing Pte Ltd. All rights reserved.

Exercise 4.1 2.
In each of the following, name the pair of congruent
Level 1 triangles and state the reason for the congruence.
(a) A D
1. In each of the following, name the pair of congruent

triangles and state the reason for the congruence.
A
(a) X
B
F
C
9 8 7 E
8

Y Z
(b) G
9 K
B 7 C
L

H
(b) G
N
F M
30°

6 4
6
(c)
P S
U
30°
D E
4 H

K R
Q

(c) L P R T
35°
5 100° (d)
5 100° V
35° A
M N Q
Y
(d) S W
W X
Z
15 Y
15
Solution
7
(a) In nABC and nFED,
T
7
U X ∠CAB = ∠DFE (given)
AB = FE (given)
Solution ∠ABC = ∠FED (given)
(a) In nABC and nYZX, \ nABC ≡ nFED (ASA)
AB = YZ (given) (b) In nGHK and nLMN,
BC = ZX (given) GH = LM (given)
CA = XY (given) ∠HGK = ∠MLN = 90° (given)
\ nABC ≡ nYZX (SSS) GK = LN (given)
(b) In nDEF and nGHK, \ nGHK ≡ nLMN (SAS)
DE = GH (given) (c) In nPQR and nSUT,
∠EDF = ∠HGK (given) PQ = SU (given)
DF = GK (given) QR = UT (given)
\ nDEF ≡ nGHK (SAS) RP = TS (given)
(c) In nLMN and nQPR, \ nPQR ≡ nSUT (SSS)
∠NLM = ∠RQP (given) (d) In nVWX and nAZY,
LM = QP (given) ∠WXV = ∠ZYA = 90° (given)
∠LMN = ∠QPR (given) VW = AZ (given)
\ nLMN ≡ nQPR (ASA) VX = AY (given)
(d) In nSTU and nWXY, \ nVWX ≡ nAZY (RHS)
∠TUS = ∠XYW = 90° (given)
ST = WX (given)
TU = XY (given)
\ nSTU ≡ nWXY (RHS)

137

05 FWS3A(Exp)_ch4.indd 137 12/16/14 12:53 PM


© Star Publishing Pte Ltd. All rights reserved.

3.
In each of the following, determine whether the two (b) E

triangles are congruent. If so, state the reason for


congruence. 4

(a) D
3 F 30°
30°
H
A
F x
3
5
G
35° 35°
B C
5
E
(c) K

(b) G L
4 6 6 36°
4 L P
M
H K M N
7 7 x

N
(c)
P

S
(d) V T
U
45° 9 x
45° T U
Q R 2
2
A 32° 32°
R S
50°
(d) V 9 Y
40° X
9 Solution
(a) (i) nABC ≡ nCDA
W
(ii) AB = CD (given)
Z BC = DA (given)
AC = CA (common)
Solution \ nABC ≡ nCDA (SSS)
(a) The triangles are not congruent. (iii) ∠ADC = ∠CBA (corr. parts of ≡ ns)
\ ∠x = 118°
(b) In nGHK and nLNM,
GH = LN (given) (b) (i) nEFH ≡ nGFH
HK = NM (given) (ii) EF = GF (given)
KG = ML (given) ∠EFH = ∠GFH (given)
\ nGHK ≡ nLNM (SSS) FH = FH (common)
\ nEFH ≡ nGFH (SAS)
(c) The triangles are not congruent.
(iii) GH = EN (corr. parts of ≡ ns)
(d) ∠WVX = 180° – 90° – 40° (∠ sum of n) \ x = 4
= 50°
(c) (i) nKLM ≡ nNPM
∠YZA = 180° – 90° – 50° (∠ sum of n)
(ii) ∠KML = ∠NMP = 90° (given)
= 40°
KL = NP (given)
In nVWX and nAYZ,
KM = NM (given)
∠WVX = ∠YAZ = 50°
\ nKLM ≡ nNPM (RHS)
VX = AZ (given)
(iii) ∠PNM = ∠LKM (corr. parts of ≡ ns)
∠WXV = ∠YZA = 40°
\ ∠x = ∠LKM
\ nVWX ≡ nAYZ (ASA)
= 180° – 36° – 90° (∠ sum of n)
= 54°
Level 2
4. In each of the following diagrams, (d) (i) nRST ≡ nSRV
(i) name the pair of congruent triangles, (ii) ∠SRT = ∠RSV (given)
(ii) give a reason for the congruence in (i), ∠RTS = ∠SVR (given)
(iii) find the unknown angle or side marked x. RS = SR (common)
\ nRST ≡ nSRV (ASA)
(a) D C (iii) ST = RV (corr. parts of ≡ ns)
x
\ x = 9

118°
A B

Chapter 4 Conditions of Congruence and Similarity


138

05 FWS3A(Exp)_ch4.indd 138 12/16/14 12:53 PM


© Star Publishing Pte Ltd. All rights reserved.

5. I n t he diag ra m, t he diagona ls AC a nd BD of (c) Yes. nABD ≡ nAEC


the quadrilateral ABCD intersect at E such that AD = AC (given)
AE = BE and CE = DE. ∠ACE = ∠ADB (proved)
EC = BD (given)
D C
\ nABD ≡ nAEC (SAS)

E 7. In the diagram, ABCD is a square frame and M is the


midpoint of CD. Two strings AM and BM connect A and
B to the point M. Show that
(a) nADM and nBCM are congruent,
A B
(b) the two strings are of equal length.
(a) Prove that nADE is congruent to nBCE. D M C
(b) Prove that nACD is congruent to nBDC.
(c) Name another pair of congruent triangles and prove
their congruence.

Solution
(a) DE = CE (given) A B
∠AED = ∠BEC (vert. opp. ∠s)
AE = BE (given)
Solution
\ nADE ≡ nBCE (SAS)
(a) nADM ≡ nBCM
(b) AC = BD (given) ∠ADM = ∠BCM = 90° (given)
CD = DC (common) AD = BC (given)
AD = BC (given) DM = CM (given)
\ nACD ≡ nBDC (SSS) \ nADM ≡ nBCM (RHS)
(c) AB = AB (common) (b) nADM ≡ nBCM
BD = AC (given) \ AM = BM (corr. sides of ≡ ns)
AD = BC (given)
\ nADB ≡ nBCA (SSS)
8. The diagram shows a rectangular tennis court ABCD.
Show that
Level 3
(a) nABC and nBAD are congruent,
6. In the diagram, BCDE is a straight line, AC = AD and (b) the diagonals AC and BD are equal,
BC = ED. (c) ∠BAC = ∠ABD.
A
D C

A B

B C D E
Solution
(a) Show that ∠ACB = ∠ADE. (a) nABC ≡ nBAD
(b) Prove that nABC is congruent to nAED. ∠ABC = ∠BAD = 90° (given)
(c) Is nABD congruent to nAEC? Give your reason. BC = AD (given)
AB = BA (given)
Solution \ nABC ≡ nBAD (RHS)
(a) ∠ACD = ∠ADC (isos. n)
(b) nABC ≡ nBAD
∠ACB = 180° – ∠ACD (adj. ∠s on a st. line)
\ AC = BD (corr. sides of ≡ ns)
= 180° – ∠ACD
= ∠ACB° (proved) (c) nABC ≡ nBAD
\ ∠BAC = ∠ABD (corr. parts of ≡ ns)
(b) AC = AD (given)
∠ACB = ∠ADE (proved)
CB = DE (given)
\ nABC ≡ nAED (SAS)

139

05 FWS3A(Exp)_ch4.indd 139 12/16/14 12:53 PM


© Star Publishing Pte Ltd. All rights reserved.

9. In the diagram, AB and DC represent two equal and 11. The diagram shows two spots, A and B, on the opposite
parallel rods in an engine. E is the joint of the rods AC banks of a river. Rahmat walks along a bank from B to
and BD. Show that D, then he turns right and walks away from the bank
(a) nABE and nCDE are congruent, to a point E such that A, C and E are on a straight line.
(b) E is the midpoint of AC. BC = DC = 38 m and DE = 40 m.

D C A

B D
A B 38 m C 38 m
40 m

Solution E
(a) nABE ≡ nCDE
(a) Name a pair of congruent triangles in the diagram
∠EAB = ∠ECD (given)
and state the reason for their congruence.
AB = CD (given)
(b) Find the width of the river between A and B.
∠AEB = ∠CED (vert. opp. ∠s)
\ nABE ≡ nCDE (ASA)
Solution
(b) nABE ≡ nBCDE (a) In nABC and nEDC,
AE = CE (corr. sides of ≡ ns) ∠ABC = ∠EDC (given)
\ E is the midpoint of AC. BC = DC (given)
∠ACB = ∠ECD (vert. opp. ∠s)
\ nABC ≡ nEDC (ASA)
10. Huili wants to determine the distance between two points,
A and B, on the edge of a lake. First, she walks from A to (b) AB = ED (corr. parts of ≡ ns)
D passing through M. Next, she goes to point B at the lake. \ AB = 40 m
From point B, she walks to C, passing through M again. \ the width of the river between A and B is
It is found that AM = MD = 56 m, BM = MC = 48 m and 40 m.
CD = 64 m.
(a) Name a pair of congruent triangles in the diagram 12. (a) A A A
and state the reason for the congruence. l2 l2
(b) Find the width of the lake between the two points,
A and B.
l1 l1
P l1
B C B C B C
A B

56 m 48 m A A A
l l2
2
M

48 m 56 m l1 l1
P l1
B C B B
C D
C C
64 m Follow the steps below to bisect ∠ABC using only
a ruler.
Solution 1. Place the ruler along BC and draw the line
(a) In nABM and nDCM, l1.
AM = DM (given) 2. Place the ruler along AB and draw the line
∠AMB = ∠DMC (vert. opp. ∠s) l 2.
BM = CM (given) 3. Let P be the point of intersection of l1 and
\ nABM ≡ nDCM (SAS) l2. Draw the line from B through P. The line
(b) AB = DC (corr. parts of ≡ ns) BP is the angle bisector of ∠ABC.
\ AB = 64 m (b)
With the help of a protractor, check whether BP
is the angle bisector of ∠ABC. Explain why BP is
indeed the angle bisector.
Hint: Consider the perpendiculars from P to AB
and BC.

Chapter 4 Conditions of Congruence and Similarity


140

05 FWS3A(Exp)_ch4.indd 140 12/16/14 12:53 PM


© Star Publishing Pte Ltd. All rights reserved.

Solution 14. Is it possible that 5 parts of one triangle are equal to


(a) A 5 parts of a second triangle even when the two triangles
are not congruent? Illustrate with an example.
N
Solution
P
A
12 8
B M C
B 18 C
(b)
In the figure, PM = PN = width of the ruler. In X
nBMP and nBNP, 18 12
∠BMP = ∠BNP = 90° (given)
Y
BP = BP (common) 27 Z
PM = PN (proved)
\ nBMP ≡ nBNP (RHS) In nABC and nXYZ as shown above,
Hence, ∠MBP = ∠NBP (corr. parts of ≡ ns) ∠BAC = ∠YXZ, ∠ABC = ∠XYZ, ∠ACB = ∠XZY,
i.e. BP is the angle bisector of ∠ABC. AB = XZ and BC = YX.
However, the two triangles are not congruent.

13. Identify the types of quadrilaterals that can be split into


two congruent triangles by one diagonal. For each type, Exercise 4.2
explain why the triangles are congruent.
Level 1
Solution In each of the following, name the pair of similar triangles
D C and state the reason for the similarity.
1. (a) A
65°
D

65°
B
A 67° 48° 67° 48°
B C E F

Consider parallelogram ABCD in the above diagram. (b) G

In nABC and nCDA,


∠BAC = ∠DCA (alt. ∠s, AB // DC) L
AC = CA (common)
∠BCA = ∠DAC (alt. ∠s, BC // AD)
\ nABC ≡ nCDA (ASA) 60° 30°
H K M N
That means, a parallelogram can be split into two
congruent triangles by a diagonal.
(c) S
Since the square, the rhombus and the rectangle are P
4
special parallelograms, therefore all of them can be split
3 3 4
into two congruent triangles by one diagonal. U
Q R 4
3
S T
Y
(d)
V
6
P R 4

35° 35°
W Z A
2 X 3

Q
Solution
(a) In nABC and nDEF,
Besides, a kite like PQRS as shown above, in which ∠BAC = ∠EDF (given)
PQ = PS and QR = SR, also can be split into two congruent ∠ABC = ∠DEF (given)
triangles by a diagonal, as nPQR ≡ nPSR (SSS). \ nABC is similar to nDEF. (Angle-Angle-
Angle similarity)

141

05 FWS3A(Exp)_ch4.indd 141 12/17/14 9:50 AM


© Star Publishing Pte Ltd. All rights reserved.

(b) ∠NLM = 180° – 90° – 30° (∠ sum of n) Solution


= 60° (a) ∠BAC = 180° – 25° – 120°
In nGHK and nNLM, = 35°
∠GHK = ∠NLM (proved) In nABC and nDFE,
∠GKH = ∠NML (given) ∠CAB = ∠EDF (proved)
\  nGHK is similar to nNLM. (Angle-Angle- ∠ACB = ∠DEF (given)
Angle similarity) \  nABC is similar to nDFE. (Angle-Angle-
Angle similarity)
(c) In nPQR and nSTU,
PQ QR RP HG 8
(b)
3
= = = =2 =
ST TU US 4 ML 4
\ 
nPQR is similar to nSTU. (Side-Side-Side
GK
=
9
similarity) LN 5
HK 12
WV 4 2 = =2
(d) = = MN 6
ZY 6 3
HG GK
In nVWX and nYZA, Since  ,
ML LN
WV WX 2 \ the two triangles are not similar.
= =
ZY ZA 3
∠VWX = ∠YZA (c) There is not sufficient information to conclude that

\ 
nVWX is similar to nYZA. (Side-Side-Side the triangles are similar.
similarity) (d) In nVWX and nZYA,
VW 30 5
= =
ZY 18 3
2. In each of the following, determine whether the two WX 20 5
triangles are similar. Give reasons to support your answer. = =
YA 12 3
(a) VW WX
D
\ =
ZY YA
A 35°
∠VWX = ∠ZYA (given)
E 120° \ nVWX is similar to nZYA. (Side-Angle-Side
120°
B
25° similarity)
C

Level 2
F
3. In each of the following,
(b) G (i) name a pair of similar triangles and state the reason
8 L
for the similarity,
9
4 5 (ii) find the unknown x.
H K M N
12 6
(a) E
C
21
(c) P 68 Q
x
30°
36 S B
14
R 27
27 D
30° A
T U
51

(d) (b) C
Y 4
18
V 12 N
Z x
4
A 2
A B
3 M 3
30
X
A 2 D 4
20 (c) C
W
x

B
Chapter 4 Conditions of Congruence and Similarity
142

05 FWS3A(Exp)_ch4.indd 142 12/16/14 12:53 PM


© Star Publishing Pte Ltd. All rights reserved.

(d) A 4. In the diagram, AD ⊥ BC, ∠BAD = ∠ACD.


BD = 9 cm, DC = 16 cm.
10
12 A
P 70°

x Q
6
70°
B C
B C
Solution 9 D 16
(a) (i) In nABE and nCBD,
(a) Show that nABD is similar to nCAD.
∠EAB = ∠DCB (alt. ∠s, AE // DC)
(b) Find the length of AD.
∠ABE = ∠CBD (vert. opp. ∠s)
(c) Find the length of AB.
\ nABE is similar to nCBD. (Angle-
Angle-Angle similarity)
Solution
BC
(ii)=
BD (a) In nABD and nCAD,
BA BE ∠BAD = ∠ACD (given)
x
=
14
∠BDA = ∠ADC (given)
27 21
\  nABD is similar to nCAD. (Angle-Angle-

x = 18
Angle similarity)
(b) (i) In nABC and nAMN,
9 AD
AM 3 1 (b) =
= = AD 16
AB 6 2
AN 4 1 A D2 = 144
= =
AC 6 2 AD = 144
AM AN
\ = = 12 cm
AB AC
The length of AD is 12 cm.
∠BAC = ∠MAN (common)
\ 
n ABC is similar to nAMN. (Side- (c) AB2 = BD2 + AD2 (Pythagoras’ Theorem)
Angle-Side similarity) = 92 + 122
MN AM = 81 + 144
(ii)=
BC AB = 225
2 1 AB = 15
=
x 2 The length of AB is 15 cm.

x =4
(c) (i) In nABC and nDEC, 5. In the diagram, AB // DC, ∠ACB = ∠CDA.
∠BAC = ∠EDC (given) AB = 27 cm, DC = 12 cm and AD = 15 cm.
∠ACB = ∠DCE (common)
12
\  nABC is similar to nDEC. (Angle- D C

Angle-Angle similarity)
DE CD
(ii)= 15
AB CA
x 4
=
5 6
A B
20 27
x=
6
1 (a) Show that nABC is similar to nCAD.
=3
3 (b) Find the length of AC.
(c) Find the length of BC.
(d) (i) In nABC and nAQP,
∠BAC = ∠QAP (common)
Solution
∠ACB = ∠APQ (given)
(a) In nABC and nCAD,
\  nABC is similar to nAQP. (Angle-
∠CAB = ∠DCA (corr. ∠s, AB // CD)
Angle-Angle similarity)
∠ACB = ∠CDA (given)
AQ AP
(ii) = \ nABC is similar to nCAD. (Angle-Angle-
AB AC
Angle similarity)
12 10
=
10 + x 12 + 6
216 = 100 + 10x
x = 11.6

143

05 FWS3A(Exp)_ch4.indd 143 12/16/14 12:53 PM


© Star Publishing Pte Ltd. All rights reserved.

AC 12 7. In the figure, AF // BE // CD, AB = 20 cm, BC = 30 cm,


(b) =
27 AC AF = 25 cm, BE = x cm and CD = y cm.
AC 2 = 324 (a) Name a triangle that is similar to ACF. State the
AC = 18 reason for the similarity.
The length of AC is 18 cm. (b) Find the value of x. D

18 15 (c) Find the value of y.


(c) =
27 BC F
18BC = 405 y
E
BC = 22.5 25
The length of BC is 22.5 cm. x

A 20 B 30 C
6. In the figure, DE // FG // BC, AD = 4 cm, FB = 2 cm,
2
DE = 2 cm, BC = 6 cm, DF = x cm and FG = y cm. Solution
3
(a) State the reason that (a) In nBCE and nACF,
A
ADE is similar to ABC. ∠EBC = ∠FAC (corr. ∠s, AF // BE)
(b) Find the value of x. ∠BCE = ∠ACF (common)
4
(c) Find the value of y. \ nBCE is similar to nACF. (Angle-Angle-
2 32 Angle similarity)
D E
BE BC
x (b) =
AF AC
y
F G x 30
2
=
25 20 + 30
B
6
C
x = 15
(c) Similarly,
Solution nABE is similar to nACD. (Angle-Angle-Angle
(a) In nADE and nABC, similarity)
∠ADE = ∠ABC (corr. ∠s, DE // BC) \
BE
=
AB
∠DAE = ∠BAC (common) CD AC
\ nADE is similar to nABC. (Angle-Angle-
15
=
20
Angle similarity) y 20 + 30
y = 37.5
AD DE
(b) =
AB BC
2
2 8.In the figure, AD // FG, EF = 12 cm, BC = 18 cm,
4
= 3
AD = 12 cm and FG = x cm.
4 +x+2 6
(a) Name two pairs of similar A 12
D

4
=
4
triangles in the figure.
x+6 9
(b) Find the ratio AF : FC.
x+6 =9 (c) Find the value of x.
x = 3 E x
12 F G
(c) Similarly,
nADE is similar to nAFG. (Angle-Angle-Angle B
18 C
similarity) Solution
AD DE (a) In nAEF and nABC,
= ∠AFE = ∠ACB (given)
AE FG
2 ∠EAF = ∠BAC (common)
2
=
4 3 \ AEF is similar to nABC. (Angle-Angle-Angle
4 +3 y similarity)
y=
8
×
7 In nACD and nFCG,
3 4 ∠ACD = ∠FCG (common)

=
14 ∠CAD = ∠CFG (corr. ∠s, AD // FG)
3 \ nACD is similar to nFCG. (Angle-Angle-
2 Angle similarity)
= 4
3

Chapter 4 Conditions of Congruence and Similarity


144

05 FWS3A(Exp)_ch4.indd 144 12/16/14 12:54 PM


© Star Publishing Pte Ltd. All rights reserved.

(b)
AF
=
EF (a) Name two similar triangles in the diagram and
AC BC state the reason for the similarity.
AF 12 (b) Find the height of the tree.
=
AC 18
2
AF = AC Solution
3
(a) In nABM and nCDM,
FC = AC – AF
∠ABM = ∠CDM = 90°
2

= AC – AC ∠AMB = ∠CMD (given)
3
1 \ nABM is similar to nCDM. (Angle-Angle-

= AC
3 Angle similarity)
2 1
\ AF : FC = AC : AC AB
(b) =
BM
3 3
CD DM
= 2 : 1 h 2.5
FG FC =
(c) = 1.8 1.5
AD AC
h =3
x
= 1
The height of the tree is 3 m.
12 3
\ x = 4
11. In the diagram, PQ is the distance across a river. In order
Level 3 to determine it, Lily walks along the paths QRS and ST.
9. B T is a spot such that P, R and T are on a straight line.
QR = 30 m, RS = 20 m and ST = 12 m.
Y
(a) Name two similar triangles in the diagram and
state the reason for the similarity.
1.5
(b) Find the distance PQ.
C
3 A Z 2 X P

In the diagram, a girl 1.5 m tall stands in front of


20 S
a vertical flag pole. The lengths of the shadows of Q
30 R 12
the girl and the pole on the horizontal ground are
2 m and 3 m respectively. Find the height of the pole. T

Solution
Solution
Let h m be the height of the pole.
(a) In nPQR and nTSR,
In nABC and nXYZ,
∠PQR = ∠TSR = 90°
∠BAC = ∠YXZ= 90°
∠PRQ = ∠TRS (vert. opp. ∠s)
∠BCA = ∠YZX  (corr. ∠s, BC // YZ)
\ nPQR is similar to nTSR. (Angle-Angle-
\ nABC is similar to nXYZ. (Angle-Angle-Angle
Angle similarity)
similarity)
PQ QR
AB
=
AC (b) = BC
XY XZ TS
h PQ 30
=
3 =
1.5 2 12 20
h = 2.25
PQ = 18 m
The height of the pole is 2.25 m. \ the distance PQ is 18 m.

10. A
12. The diagram shows a rectangular billiard table ABCD
C in which AB = 2.4 m and BC = 1.2 m. When a ball hits
h
the side AB at E along the path CE, it bounces along the
path EF such that ∠BEC = ∠AEF.
1.8 m
M
D C
B 2.5 1.5 D
F
1.2
Wenrong places a plane mirror M on level ground to 0.8
determine the height of a tree (see the diagram). He A E B
stands at a certain distance so that he can see the top of 2.4
the tree reflected from the mirror. Wenrong’s eye level is
(a) Show that nBEC is similar to nAEF.
1.8 m above the ground. The distances of Wenrong and
(b) If AF = 0.8 m, find the distance AE.
the tree from the mirror are 1.5 m and 2.5 m respectively.

145

05 FWS3A(Exp)_ch4.indd 145 12/16/14 12:54 PM


© Star Publishing Pte Ltd. All rights reserved.

Solution 14. In ABC and DEF, it is given that AB = DE,


(a) In nAEF and nBEC, BC = EF and ∠BCA = ∠EFD. Investigate whether ABC
∠EAF = ∠EBC = 90° is similar to DEF. Give an example to support your
∠AEF = ∠BEC (given) investigation.
\ nAEF is similar to nBEC. (Angle-Angle-
Angle similarity) Solution
AF AE
No. For example, take AB = DE = 3 cm,
(b) = BC = EF = 5 cm, ∠BCA = ∠EFD = 30°. Then by drawing
BC BE
0.8
or by calculation, ∠BAC can be 56.4° and ∠EDF can be

= 123.6°. Therefore, the two triangles are not similar.
1.2
2

=
3
Distance AB = 2.4 m Exercise 4.3
=
AE 2 Level 1
AB 5
1. In each of the following pairs of similar figures, find the
\ AE =
2
3 2.4 unknown area A1 or A2. The units of length and area are
5
cm and cm2 respectively.
= 0.96
(a)
The distance AE is 0.96 m.
A2 = 6
A1
13. (a) In the diagram, AB // CD // EF // GH. PQRS and
3 4
PTUV are straight lines such that PQ = QR = RS.
What can you say about the lengths of PT, TU and (b)
UV ?
A2
(b) Using only a straight edge and a pair of compasses,
A1 = 100
divide a line segment into 3 equal parts.
a 2a

A P
B
(c)
A2
Q T
C D 14
A1 = 64
R U 10
E F

G S V
H
(d)

A1 A2 = 225
Solution
(a) PT = TU = UV 3t
Note: This is the Triangle Intercept Theorem. 5t
(b) R T (e)

Q 3r
2r
A2
A1 = 50

(f )
A B
M N
A1
In the above figure, AB is a given line segment. A2 = 560

The steps of trisecting it are as follows: 12


1. Draw a ray AT which makes an acute angle
with AB. 20
2. Use a pair of compasses to mark 3 equal Solution
segments AP, PQ and QR, on AT. A1
(a)
2
= 32
3. Join B and R. A2 4
4. Draw lines parallel to BR through Q and P A
1 =
9
to cut AB at N and M respectively. 6 16
5. Then, by the result in (a), AM = MN = NB. A1 = 3.375 cm2

Chapter 4 Conditions of Congruence and Similarity


146

05 FWS3A(Exp)_ch4.indd 146 12/16/14 12:54 PM


© Star Publishing Pte Ltd. All rights reserved.

2 Solution
A  a 
(b) 1 =  
A2 2a    x
2
4
1
(a)
  =

100
= 6 16
A2 4 x 1
=

A2 = 400 cm 2 6 2

2
x =3
A  10 
(c) 1 =    x
2
A2 14 (b) =
108
 
25 8 48
64 = x 3
A2 49 =
8 2

A2 = 125.44 cm2 x = 12
2
A  3t 
(d) 1 =    x 
(c) =
2
200
A2 5t  18  648
A 9
1 = =
x 5
225 25
18 9

A1 = 81 cm2 x = 10
2 2
A  2r   x  288
(e) 1 =   (d)   =
A2 3r 18 162
x 4

50
=
4
=
A2 9 18 3
x = 24

A2 = 112.5 cm2
2
A  12 
(f ) 1 =   3. In the diagram, D is a point on BC. BD = 10 cm,
A2 20
A1 9
DC = 15 cm and the area of ABD = 60 cm2. Find
= Area of  ABC
560 25 (a) . A
Area of  ABD

A1 = 201.6 cm2 (b) area of ACD.

2. In each of the following pairs of similar figures, find the B 10 D 15 C


unknown side x. The unit of length is cm.
(a) Solution
(a)
A
16 cm2
4 cm2
x 6 height

(b)
B 10 D 15 C

108 cm2
48 cm2 nABC and nABD are two triangles with the same
height from A.
8 x
Area of  ABD BD
=
Area of  ABC BC

(c)
10
=
63
25 Area of ABC
200 cm2 x
648 cm2 18 10
Area of nABC = 60 ÷
25
= 150 cm2
150 Area of  ABC
\ =
(d) 60 Area of  ABD
5

=
x 2
18
162 cm2 288 cm2
(b) Area of nACD = 150 – 60
= 90 cm2

147

05 FWS3A(Exp)_ch4.indd 147 12/16/14 12:54 PM


© Star Publishing Pte Ltd. All rights reserved.

4. In the diagram, AB // DC, AB = 21 cm, DC = 12 cm, Let the sides of square B be y cm.
and the area of ACD = 72 cm2. Find \ 42 = y2 + y2 (Pythagoras’ Theorem)
area of  ABC 16 = 2y2
(a) . D 12 C
area of  ACD y2 = 8
(b) area of ABC. y = 8 cm
Perimeter of square A = 4 2 cm
Perimeter of square B = 4 8 cm
A B
21 Ratios = 4 2 : 4 8

Solution = 2 : 8
(a) ABC and ACD are two triangles with the same = 1 : 2
height.
(b) Area of square A = ( 2 )2
A C = 2 cm2
Area of square B = ( 8 )2
= 8 cm2
Ratios = 2 : 8
height =1:4
height

Level 2
C A B
12 D 21 6. The radii of two circles are 14 cm and 21 cm. Find the
ratio of

Area of  ABC
=
AB (a) the circumferences of the two circles,
Area of  ACD CD (b) the areas of the two circles.
21
=
12 Solution
=
7 (a) Ratio of the circumferences = 14 : 21
4
=2:3
Area of ABC AB
(b) \ = (b) Ratio of areas of the circles = 22 : 32
72 CD
7 =4:9

Area of ABC = 3 72
4
= 126 cm2 7. The area of a quadrilateral with sides 6 cm, 8 cm, 7 cm
and 5 cm is 40 cm2. Find the area of a similar quadrilateral
5. The diagonals of two squares are 2 cm and 4 cm. Find whose longest side is 6 cm.
the ratio of the
(a) perimeters of the two squares, Solution
(b) areas of the two squares. Let A cm2 be the area of the similar quadrilateral.
2
A  6
=  
Solution 40 8
(a) y
A=
9
3 40
16
= 22.5
x i.e. the required area is 22.5 cm2.
y
4 cm
x 8. In the diagram, AC and ED intersect at B, AB = 20 cm,
2 cm BC = 15 cm, DB = 18 cm, BE = 24 cm, and the area of
ABE = 208 cm2.
A B (a) Name a pair of similar triangles and state the reason
for the similarity.
Let the sides of square A be x cm. (b) Find the area of BCD.
\ 22 = x2 + x2 (Pythagoras’ Theorem)
E
4 = 2x2 24
C
15
x2 = 2
x = 2 cm B
18
20 D
A

Chapter 4 Conditions of Congruence and Similarity


148

05 FWS3A(Exp)_ch4.indd 148 12/16/14 12:54 PM


© Star Publishing Pte Ltd. All rights reserved.

Solution Solution
(a) In nABE and nCBD, (a) nACE is similar to nBCD. (Angle-Angle-Angle
AB 20 4 similarity)
= = 2
CB 15 3 Area of  BCD  BC 
=  
BE 24 4 Area of  ACE AB 
= = 2
BD 18 3 150  BC 
=  AC 
AB BE 150 + 144
\ =
CB BD
BC 25
∠ABE = ∠CBD (vert. opp. ∠s) =
AC 49
\ nABE is similar to nCBD. (Side-Angle-Side 5
similarity)
=
7
2 5
(b)
Area of  BCD
=
 CB 
  BC = AC
Area of  BAE AB  7
2 AB = AC – BC
Area of BCD  3
= 5
208
 
4
= AC – AC
7
\ area of nBCD = 117 cm 2

= AC
2
7
9. In the diagram, DE // BC, AD = 12 cm, BD = 4 cm and \ AB : BC =
2
AC :
5
AC
the area of ADE is 63 cm2. 7 7

(a) State the reason that ADE is similar to ABC. = 2 : 5


(b) Find the area of ABC. A BD
(b) =
BC
(c) Find the area of BCED. AE AC
15 5
12 =
AE 7
AE = 21 cm
D E
4
B C 11. In the diagram, D is a point on AC such that
∠CBD = ∠BAC, AC = 45 cm and BC = 30 cm.
Solution (a) Show that nABC and nBDC are similar.
(a) In nADE and nABC, (b) Find the length of CD. A
∠ADE = ∠ABC (corr. ∠s, DE // BC) (c) Find the value of
∠DAE = ∠BAC (common) area of  BDC
\ nADE is similar to nABC. (Angle-Angle- (i)
area of  ABC 45
Angle similarity) D
area of  BDC
(ii)
Area of  ADE  AD 
2 area of  ABD
(b) =  AB 
Area of  ABC
B 30 C
2
63  12 
=   Solution
Area of ABC 16 (a) In nABC and nBDC,
Area of nABC =
16
3 63 ∠BAC = ∠DBC (given)
9 ∠ACB = ∠BCD (common)
= 112 cm2 \ n  ABC is similar to nBDC. (Angle-Angle-
(c) Area of BCED = 112 – 63 Angle similarity)
= 49 cm2 (b) A

10. In the diagram, AE // BD and BD = 15 cm. The area of


B
BCD = 150 cm2 and the area of ABDE = 144 cm2. 45
(a) Find the ratio AB : BC. 30
(b) Find the length of AE.

E
C 30 B C D
D
BC CD
=
15
AC BC
30 CD
A
B C =
45 30
45CD = 900
CD = 20
The length of CD is 20 cm.
149

05 FWS3A(Exp)_ch4.indd 149 12/16/14 12:54 PM


© Star Publishing Pte Ltd. All rights reserved.

2
Area of  BDC  30  Solution
(c) (i) =  
Area of  ABC 45  (a) BEF is similar to CDF.
900 (Angle-Angle-Angle similarity)
=
2025 Area of  BEF  24 
2
4 =  40 
= Area of CDF
9
2
Area of  BDC 120
(ii) =
900
=  3
 
Area of  ABD 2025 – 900 Area of CDF 5
900
= Area of CDF = 333 3 cm2
1
1125
4
= (b) BDF and BEF are two triangles with the same
5
height from B.
Area of  BCF 5
12. In the diagram, DE // BC, AD : DB = 1 : 2, A =
Area of  BEF 3
and the area of ADE is 24 cm 2. Find Area of BCF 5
the area of D E
=
120 3
(a) ABC, 5
(b) BDE.
Area of BCF = 3 120
3
= 200 cm2
B C
(c) Area of CDF + Area of BCF
1
Solution = 333 3 + 200
2
Area of  ABC  3
(a) = 1
Area of  ADE
 
1 = 533 3 cm2
Area of ABC 9
= Area of parallelogram = 533
1
×2
24 1 3
Area of ABC = 9 3 24 = 1066
2
cm2
= 216 cm2 3

(b) ADE and BDE are two triangles with the same Level 3
height from E. 14.
Two rectangular lawns, A and B, measure 50 m by 40 m
A
and 75 m by 60 m respectively.
height (a) Show that lawn A and lawn B are similar.
D (b) Find the ratio of
E
(i) the diagonals of A and B,
(ii) the areas of A and B.
(c) It takes 20 minutes to mow lawn A. Assuming the
rate of mowing is the same, how much time will
B it take to mow lawn B?

Area of  BDE 2
= Solution
Area of  ADE 1 (a) Ratio of the lengths of the lawns = 50 : 75

Area ofBDE = 2 3 24 =2:3
= 48 cm2 Ratio of the breadths of the lawns = 40 : 60
=2:3
13. In the diagram, ABCD is a parallelogram. BD and CE Since the above two ratios are equal,
intersect at F, AE = 16 cm. EB = 24 cm and the area of \ lawn A and lawn B are similar.
BEF = 120 cm2. Find the area of (b) (i) Ratio of the diagonals = 2 : 3
(a) CDF, (ii) Ratio of the areas = 22 : 32
(b) BCF, =4:9
(c) the parallelogram ABCD. 9
(c) Time taken to mow lawn B = × 20
D C 4
= 45 min

A 16 E 24 B

Chapter 4 Conditions of Congruence and Similarity


150

05 FWS3A(Exp)_ch4.indd 150 12/16/14 12:54 PM


© Star Publishing Pte Ltd. All rights reserved.

15. The diagram shows the face of a cake. The inner heart is (d) Let S m2 be the required cross-sectional area.
filled with strawberry and is surrounded by a chocolate
2
S  3
=  
border of uniform width such that the inner heart is 12 4
similar to the outer heart of the cake. S = 6.75
The heights of the inner and outer hearts \ the required cross-sectional area is 6.75 m2.
10 18
are 10 cm and 18 cm respectively. The
area of the inner heart is 50 cm . Find
2
17. The diagram shows a logo in which ABC is an
the area of the chocolate border. equilateral triangle. P, Q and R are points on the sides
BC, CA and AB respectively such that AR = BP = CQ.
Solution
2
Area of outer heart  18 
=   A
Area of inner heart 10

Area of outer heart = 81


3 50 Q
25
= 162 cm 2
R
\ area of the chocolate border = 162 – 50
= 112 cm2
B P C
16. The diagram shows the vertical cross-section of a (a) Show that
hut. The base CD is 4 m wide and the wall DE is (i) AQ = BR,
2 m high. (ii) AQR and BRP are congruent,
(a) Find ∠BAE. (iii) PQR is an equilateral triangle.
(b) Find the height of A above the ground. (b) If P is the midpoint of BC, find the ratio of the
(c) Find the area of this cross- A area of PQR to that of ABC.
section.
(d) A similar hut has a base of B Solution
135° 135° E
3 m in width. Find the area 2 (a) (i) Let AB = x = BC = AC
of its vertical cross-section. Let AR = y = BP = CQ
C 4 D
\ BR = AB – AR = x – y
Solution AQ = AC – CQ = x – y
(a) Angle sum of a pentagon = 180° 3 (5 – 2) Hence AQ = BR (shown)
= 540°
(ii) AQ = BR (found)
∠BAE = 540° – 135° – 135° – 90° – 90°
∠RAQ = ∠PBR = 60° (given)
= 90°
AR = BP (given)
(b) A \ nAQR ≡ nBRP (SAS)
(iii) nAQR ≡ nBRP (SAS)
Hence, RP = RQ (corr. parts of ≡ ns)
B E
M and RP = RQ = QP
2 (property of equilateral n)
C N D
4 (b) In nPQR,
let BP = PC = x (P is the midpoint)
Since ∠BAE = 90° and ∠ABE = 45°, Since nAQR ≡ nBRP ≡ nQPC,
nABE is a right-angled isosceles triangle. BP = QC = PC = RP = RQ = PQ = x.
AM = BM In nABC,
= 2 m BC = 2BP = 2x
\ height of A above the ground = AM + MN \ AB = AC = BC = 2x
= 2 + 2
PQ
=
QR
=
RP
=
1
= 4 m AB BC AC 2
(ratio of corr. sides of similar ns)
(c) Area of the cross-section 2
Area of PQR  1
= area of nABE + area of BCDE =  
Area of  ABC 2
1
= × 4 × 2  + (4 3 2) 1

2
=
4
= 12 m 2
Ratio of the areas = 1 : 4

151

05 FWS3A(Exp)_ch4.indd 151 12/16/14 12:54 PM


© Star Publishing Pte Ltd. All rights reserved.

18. (a) A (ii) From the result of (a)(ii),


1
area of nAPS = area of nABD
4
R Q 1
area of nQCR = area of nBCD
4

B C \ area of nAPS + area of nQCR


P
1
In the diagram, P, Q and R are the midpoints of = (area of nABD + area of nBCD)
4
the sides of ABC. Show that 1
= area of ABCD
(i) PQR is similar to ABC, 4

(ii) area of PQR =


1
area of ABC. Similarly,
4
area of nBPQ + area of nDRS
(b)
Can the conclusions in (a) be extended to a 1
quadrilateral ABCD? That is, if P, Q, R and S are = area of ABCD
4
the midpoints of the sides of ABCD, \ area of PQRS
(i) is PQRS similar to ABCD? = area of ABCD
1
(ii) is the area of PQRS equal to of the – (area of nAPS + area of nQCR)
4
area of ABCD? – (area of nBPQ + area of nDRS)
 1 1
=  1 – –  area of ABCD
Solution 4 4
(a) (i) In nARQ and nABC, =
1
area of ABCD
2
AR AQ 1
= = \ area of PQRS 
1
area of ABCD.
AB AC 2 4

∠RAQ = ∠BAC (common)
\ nARQ is similar to nABC.
(Side-Angle-Side similarity) Exercise 4.4
RQ AR Level 1
=
BC AB
1. In each of the following pairs of similar solids, find the
1
\ RQ = BC unknown volume V1 or V2.
2
Similarly, PR =
1
CA and PQ =
1
BA. (The unit of length is cm and the unit of volume is cm3.)
2 2 (a)
In nPQR and nABC, V1
PQ QR RP 1 V2 = 180
= = =
AB BC CA 2 4
\ nPQR is similar to nABC.
(Side-Side-Side similarity) 6

Area of PQR  1
2 (b)
(ii) =  
Area of  ABC 2
V1 = 225
1
\ area of nPQR = area of nABC V2
4
10
(b) (i) C
20
R
D (c)
Q V2 = 2680
S V1
8
6

A B
P
(d)
In the figure, ABCD is a quadrilateral. P, Q,
15 18
R and S are the midpoints of its sides.
Using the proof in the first part of (a)(i), we V2
V1 = 900
have
1
RQ = SP = DB
2
1
and PQ = SR = AC.
2
Hence, PQRS is a parallelogram.
\ PQRS is not similar to ABCD.

Chapter 4 Conditions of Congruence and Similarity


152

05 FWS3A(Exp)_ch4.indd 152 12/16/14 12:54 PM


© Star Publishing Pte Ltd. All rights reserved.

Solution Solution
V  4
3
(a) Ratio of the sides = 49 : 36
(a) 1 =  
V2 6 =7:6
V

1
=
8
(b) Ratio of the volumes = 73 : 63
180 27
= 343 : 216
1

V1 = 53 cm3 Volume of larger cube 343
3 (c) Volume of smaller cube
=
3 216
V  10 
(b) 1 =  20  \ volume of larger cube = 648
343
V2 3
216
=
225 1 = 1029 cm3
V2 8

V2 = 1800 cm3
4. The radii of two solid hemispheres are in the ratio 1 : 3.
(a) Find the ratio of the base areas of the two
3
V1  6
(c) =  
V2 8 hemispheres.

V1
=
27 (b) Find the ratio of the volumes of the two hemispheres.
2680 64 (c) If the base area of the larger one is 540 cm2, find

V1 = 1130
5
cm3 the base area of the smaller one.
8
V  15 
3 Solution
(d) 1 =
V2  
18 (a) Ratio of the base areas = 12 : 32
900 125 =1:9

V
=
2 216 (b) Ratio of the volumes = 13 : 33

V2 = 1555.2 cm 3
= 1 : 27
(c) Let S be the surface area of the smaller hemisphere.
2. Copy and complete the following table for the ratios of S 1
measures of similar solids. =
540 9
1
Ratio of areas of S = 540 3
Ratio of Ratio of 9
corresponding = 60 cm2
sides volumes
plane faces

(a) 1:2 5.
A pentagonal prism is 25 cm high and
its base area is 170 cm2.
(b) 4:9
(a) Find the volume of the prism.
(c) 125 : 216 (b) A similar prism is 30 cm high.
25
Find its
(i) base area,
Solution
(ii) volume. 170 cm2
Ratio of areas of
Ratio of Ratio of
corresponding Solution
sides volumes (a) Volume of prism = base area 3 height
plane faces
= 170 3 25
(a) 1:2 1:4 1:8 = 4250 cm3
(b) 2:3 4:9 8 : 27 (b) Let A cm2 and V cm3 be the required base area and
volume respectively.
(c) 5:6 25 : 36 125 : 216 2
(i) A =  30 
 
170 25 
36
Level 2
A = 170 3
25
3. The ratio of the areas of corresponding faces of two = 244.8
cubes is 49 : 36. The required base area is 244.8 cm2.
(a) Find the ratio of the corresponding sides of the two V 2

cubes. (ii) =  30 
4250  25 
(b) Find the ratio of the volumes of the two cubes. 216
V = 4250
(c) If the volume of the smaller cube is 648 cm3, find
3
125
the volume of the larger cube. = 7344
The required volume is 7344 cm3.

153

05 FWS3A(Exp)_ch4.indd 153 12/16/14 12:54 PM


© Star Publishing Pte Ltd. All rights reserved.

6. The diagram shows two similar popcorn cups A and B. Solution


Their heights are 10 cm and 15 cm respectively. Find (a) Ratio of the volumes of two similar solids
the ratio of = a3 : b3
(a) the base area of A to the base area of B, Volume of small solid : volume of big solid
(b) the capacity of A to the capacity of B. = 800r : 2700r
= 8 : 27
= 23 : 33
\ Ratio of heights = 2 : 3
2
base area of larger solid  3
(b) =  
15 base area of smaller solid 2
10 9
base area of lager solid = 3 240
4
= 540 cm2
A
B
Solution Level 3
2
(a)
base area of A
=
 10 
  9. Two similar statues, made of the same material, are of
base area of B 15 heights 15 cm and 20 cm. The mass of the smaller one
=
4 is 405 g. Find the mass of the larger one.
9
The ratio is 4 : 9. Solution
3 3
capacity of A  2 Mass of larger statue  20 
(b) =  3 
=  
capacity of B Mass of smaller statue 15
64
=
8
Mass of larger statue = 405 3
27 27
The ratio is 8 : 27. = 960 g

7. The diagram shows two similar solids, made of the same 10.
The diagram shows two similar barrels A and B. The
material, whose base areas are in the ratio 25 : 49. The ratio of the height of A to the height of B is 3 : 4.
base area of the larger solid is 147 cm2. (a) If the base area of A
(a) Find the base area of is 243 cm2, find the
the smaller one. base area of B.
(b) What is the ratio of the (b) If the capacity of B is
heights of the two solids? 320 litres, find the
(c) If the mass of the smaller capacity of A. A B
solid is 8.25 kg, find the
mass of the larger solid. base base Solution
2
base area of B  4
(a) =  
Solution base area of A 3
(a) Let A cm2 be the base area of the smaller solid. base area of B =
16
3 243
A 25 9
=
147 49 = 432 cm2
A = 75 capacity of A  3
3

The base area of the smaller solid is 75 cm2. (b) =  


capacity of B 4
(b) Ratio of the heights = 25 : 49
capacity of A =
27
3 320
= 5 : 7 64
= 135 l
Mass of smaller solid  5
3

(c) Mass of larger solid =  7 


8.25 11.
In the diagram, A and B are two cakes. They are in the
125

Mass of larger solid = form of two similar circular frustums. The base areas
343
of the bottoms of A and B are 432 cm 2 and 675 cm 2
\ mass of larger solid = 22.638 kg
respectively.
(a) Find the ratio of the height
8. The volumes of two similar solids are 800r cm3 and of A to the height of B.
2700r cm3. Find (b) If the area of the top of B
(a) the ratio of their heights, is 300 cm2, find the
A
(b) the base area of the larger solid, given that the base area of the top of A. B

area of the smaller one is 240 cm2. (c) If the volume of A is 1824 cm , find the volume
3

of B.
Chapter 4 Conditions of Congruence and Similarity
154

05 FWS3A(Exp)_ch4.indd 154 12/16/14 12:54 PM


© Star Publishing Pte Ltd. All rights reserved.

Solution 13. The heights of three similar solid metal cylinders are
base area of A  height of A 
2
6 cm, 8 cm and 10 cm respectively.
(a) =
base area of B  height of B  (a) Find the ratio of the base areas of the cylinders.
432  h1 
2 (b) Find the ratio of the volumes of the cylinders.
= (c) If the cylinders are melted and recast into one
675  h2 
2
similar cylinder, find the height of the cylinder

16
=
 h1  formed.
25  h 
2

42  h1 
2 Solution
2
=  h  (a) Ratio of the heights of the cylinders = 6 : 8 : 10
5
=3:4:5
2

\ Ratio of h1 : h2 = 4 : 5
Ratio of their base areas = 32 : 42 : 52
= 9 : 16 : 25
2
Area of the top of A  4
(b) =  5 
Area of the top of B
(b) Ratio of their volumes = 33 : 43 : 53
16
Area of the top of A = 3 300 = 27 : 64 : 125
25
= 192 cm2 (c) Let H cm be the height of the new cylinder.
3 3
Volume of A Volume of new cylinder H
(c) =
 4
  =  
Volume of B 5 Volume of smallest cylinder 6
3
Volume of B = 1824 ÷
64 27 + 64 + 125 H
125
=  
27 6
= 3562.5 cm3 216 H
3
=  
27 6
12. A cup is in the form of an inverted square pyramid. H
3
  =8
When the depth of water is 10 cm, the volume of water 6
is 187.5 cm3. When x cm3 of water is poured into the
H
=2
cup, the water level rises to 14 cm. 6
(a) Find the value of x. H = 12
(b) Find the increase in the area of the upper water The height of the new cylinder is 12 cm.
surface in the cup.
14. A carpenter cuts a wooden cone into three parts A, B
and C by two planes parallel to the base as shown in the
diagram. The heights of the three parts are equal.
(a) Find the ratio of the base
areas of parts A, B and C. h
A
(b) Find the ratio of the
10
volumes of parts A, B and h
C. B
(c) I f t he volume of t he h

Solution original cone is 540 cm3, C

(a) Let V cm3 be the volume of water in the cup when find the volume of part B.
the depth of water is 14 cm.
V  14 
3 Solution
=
187.5  10  (a) Base area of A : base area of B : base area of C
V = 187.5 ×
343 = 12 : 22 : 32
125 =1:4:9
= 514.5
\ x = V – 187.5 (b) Volume of A : volume of (A + B) :
= 514.5 – 187.5 volume of (A + B + C)
= 327 = 13 : 23 : 33
= 1 : 8 : 27
(b) Increase in the area of water surface \ volume of A : volume of B : volume of C
= 514.5
– 187.5 = 1 : (8 – 1) : (27 – 8)
1
× 14
1
× 10
= 1 : 7 : 19
3 3
7
(c) Volume of part B = 540
= 54 cm2
3
1 + 7 + 19
7
= 540 3
27
= 140 cm3

155

05 FWS3A(Exp)_ch4.indd 155 12/16/14 12:54 PM


© Star Publishing Pte Ltd. All rights reserved.

15. There are some paper cups at a drinking fountain. Each Solution
cup is in the form of an inverted cone of height 12 cm. (a) In nABC and nYXZ,
Determine a method, mathematical or otherwise, to find AB = YX (given)
the water level h cm such that the volume of water is ∠ABC = ∠YXZ (given)
half of the capacity of the cup. BC = XZ (given)
\ nABC ≡ nYXZ (SAS)
(b) ∠X = 180° – 65° – 60° (∠ sum of n)
= 55°
In nPQR and nYZX,
12 ∠PQR = ∠YZX (given)
h QR = ZX (given)
∠PRQ = ∠YXZ (proved)
\ nPQR ≡ nYZX (ASA)

Solution (c) In nPQR and nYZX,


Method 1 QR = ZX (given)
We can find the required height, h cm, by trial and error. ∠QRP = ∠ZXY (given)
Fill a cup full of water. Pour water from this cup into PR = YX (given)
another cup. Adjust the water levels in these two cups \ nPQR ≡ nYZX (SAS)
until they are equal. Then the depth of water in each cup
is the required height. 2. In each of the following, name a pair of congruent
Method 2 triangles and give a reason for the congruence.
Volume of water  h
3 (a) D
=  
Volume of cup 12
3 A C
1  h
=  
2 12
3 B
h 1
=
12 2 (b) A

12
h= 3
B D
2
= 9.52 (correct to 3 s.f.)
\ the height of the water level is 9.52 cm.

C
Revision Exercise 4
(c) A
1. In each of the following, name two triangles which are
congruent and state the reason for the congruence.
(a) P X E B
A C

Z
R
B C D
Q
Y
(d) D C
(b)
A P X
Z
55° 60°
C A B

60° 55° 65°


65° Q R
Y Solution
B (a) In nABC and nADC,
P AC = AC (common)
(c) ∠ACB = ∠ACD (given)
A
4
X BC = DC (given)
2
4 4 2 \ nABC ≡ nADC (SAS)
B C Q R Y Z
2

Chapter 4 Conditions of Congruence and Similarity


156

05 FWS3A(Exp)_ch4.indd 156 12/16/14 12:54 PM


© Star Publishing Pte Ltd. All rights reserved.

(b) In nABC and nADC, Solution


∠BAC = ∠DAC (given) (a) In nABC and nADC,
AC = AC (common) AC = AC (common)
∠BCA = ∠DCA (given) ∠ACB = ∠ACD (given)
\ nABC ≡ nADC (ASA) BC = DC (given)
\ nABC ≡ nADC (SAS)
(c) In nABC and nDEC,
AC = DC (given) (b) AB = AD (corr. parts of ≡ ns)
∠ACB = ∠DCE (vert. opp. ∠s) = 27 m
BC = EC (given)
\ nABC ≡ nDEC (SAS)
5. In each of the following, write down a pair of triangles
(d) In nABD and nCDB, which are similar.
AD = CB (given) (a) X 12
AB = CD (given) A P Z
BD = DB (common) 3.5 4 9
\ nABD ≡ nCDB (SSS) 6 7
10
B
3 C
Q 8 R Y
3. In the figure, ABCD is a rectangle, EF passes through (b) A
the intersecting point, M, of the diagonals AC and BD. X
(a) Is ADM congruent to CBM ?
P
35° R 110°
33° Z
(b) Name the triangle that is D F C
110° 110°
congruent to AEM and M B
37°
C Q Y
state the reason for the (c) Z
9
congruence. A P
45°
(c) Name another pair of A E B
3 4
X
2 6
congruent triangles in the B
45°
C 45°
figure. Q
6 R
Y

Solution Solution
(a) In nADM and nCBM, (a) In nABC and nRPQ,
AM = CM (property of rectangle) AB
=
3.5
=
1
DM = BM (property of rectangle) RP 7 2
AD = CB (opp. sides of rectangle) BC 3 1
= =
PQ
\ nADM ≡ nCBM (SSS) 6 2
CA 4 1
(b) In nAEM and nCFM, = =
QR 8 2
∠EAM = ∠FCM (alt. ∠s, AB // DC) AB BC CA
AM = CM (proved) \ = =
RP PQ QR
∠AME = ∠CMF (vert. opp. ∠s) nABC is similar to nRPQ. (Side-Side-Side
\ nAEM ≡ nCFM (ASA) similarity)
(c) Similar to (a), (b) ∠A = 180° – 37° – 110° (∠ sum of n)
nABM ≡ nCDM. (SSS) = 33°
or In nABC and nZYX,
Similar to (b), ∠BAC = ∠YZX (proved)
nBEM ≡ nDFM. (ASA) ∠ACB = ∠ZXY (given)
\ nABC is similar to nZYX. (Angle-Angle-
4. Minghua determines the distance between two points, Angle similarity)
A and B, of a pond by taking some measurements on (c) In nPQR and nYZX,
land as shown in the diagram. BC = 19 m, AC = 25 m, PQ 4 2
CD = 19 m, AD = 27 m and ∠ACB = ∠ACD. = =
YZ 6 3
(a) Name a pair of congruent QR 6 2
triangles and state the B
= =
A ZX 9 3
reason for the congruence. PQ QR
(b) Find the distance AB.
25 19 \ =
YZ ZX
27
C
∠PQR = ∠YZX (given)
19
\ nPQR is similar to nYZX. (Side-Angle-Side
D similarity)

157

05 FWS3A(Exp)_ch4.indd 157 12/16/14 12:54 PM


© Star Publishing Pte Ltd. All rights reserved.

6. In each of the following, write down a pair of similar (d) In nABC and nACD,
triangles and find the unknown x. AB 8 + 10 3
= =
(a) A (b) AC 12 2
AC 12 3
D E = =
x AD 8 2
4 4 16
C AB AC
\ =
24 15 AC AD
3
B C
∠BAC = ∠CAD (common)
x 3
A B \ nABC is similar to nACD. (Side-Angle-Side
similarity)
D
A
(c) 8
D (d) =
CD AC
E 8 BC AB
12 D 6 12
24
x
12 =
10 6 x 18
A C x = 9
30 B 10 B x C

Solution 7.
In the diagram, ∠A BE = ∠CBD, A E = 12 cm,
(a) In nABC and nCBD, BC = 17 cm and CD = 8 cm.
AB 4 (a) Name a pair of similar triangles and state the reason
=
CB 3 for the similarity. C
AC 4
= (b) Find the length of
CD 3 17
AB AC
(i) BD, 8
\ = (ii) DE,
CB CD D

∠BAC = ∠BCD (given) (iii) AB. E B

\ nABC is similar to nCBD. (Side-Angle-Side


similarity) 12
BD CB
=
BC AB
x 3 A
=
3 4
9
\ x =
4 Solution
=2
1 (a) In nABE and nCBD,
4 ∠ABE = ∠CBD (given)
(b) In nABC and nEDC, ∠AEB = ∠CDB (given)
∠CAB = ∠CED (alt. ∠s, AB // DE) \ nABE is similar to nCBD. (Angle-Angle-
∠ACB = ∠ECD (vert. opp. ∠s) Angle similarity)
\ nABC is similar to nEDC. (Angle-Angle- (b) (i) BD2 = BC2 – CD2 (Pythagoras’ Theorem)
Angle similarity) = 172 – 82
CD CE
= BD = 225
CB CA
x 16 = 15 cm
=
15 24 BE AE
x = 10 (ii) =
BD CD
(c) In nABE and nACD,
BE
=
12
15 8
AB 30 3
= = BE = 22.5 cm
AC 30 + 10 4
AE 24 3 \ DE = 22.5 – 15
= = = 7.5 cm
AD 24 + 8 4
AB AE AB AE
\ = (iii) =
AC AD BC CD
∠EAB = ∠DAC (common) AB 12
\ nABE is similar to nACD. (Side-Angle-Side =
17 8
similarity) AB = 25.5 cm
BE AB
=
CD AC
x 3
=
12 4
x =9
Chapter 4 Conditions of Congruence and Similarity
158

05 FWS3A(Exp)_ch4.indd 158 12/16/14 12:54 PM


© Star Publishing Pte Ltd. All rights reserved.

8. In the diagram, ∠ADC = ∠BAC = 90°, BD = 4 cm and Area of  ABC  AB 


2
(c) =  
DC = 9 cm. Area of  APQ AP 
(a) Show that ABD is similar to CAD. Area of ABC  14 
2
=
(b) Find the length of AD. A 36
 
8
(c) Find the length of AC. Area of nABC = 110.25 cm2
(d) Find the area of ABD.
(d) RC = BC – BR
B C
= BC – PQ (opp. sides of //gram)
4 D 9
= 17.5 – 10
= 7.5 cm
Solution
(e) (i) Similar to the proof in (a),
(a) ∠ABD + ∠BAD = 90° (ext. ∠ of n)
nRCQ is similar to nBAC. (Angle-Angle-
∠CAD + ∠BAD = 90° (given)
Angle similarity)
\ ∠ABD + ∠BAD = ∠CAD + ∠BAD 2
Area of  RCQ  RC 
Hence, ∠ABD = ∠CAD =  
Area of  BCA BC 
In ABD and CAD, 2
∠ABD = ∠CAD (proved)
Area of RCQ
=  7.5 
 
17.5 
∠ADB = ∠CDA (given) 110.25

\ nABD is similar to nCAD. (Angle-Angle- Area of nRCQ = 20.25 cm2


Angle similarity) (ii) Area of BRQP = 110.25 – 36 – 20.25
AD CD = 54 cm2
(b) =
BD AD
AD 9
= 10. In the diagram, ABCD is a quadrilateral, the diagonals
4 AD
AC and BD intersect at E, AD = BC and AC = BD.
AD = 36 2

\ AD = 6 cm D C

(c) AC2 = AD2 + CD2 (Pythagoras’ Theorem) E


= 62 + 92
\ AC = 117 cm A B

(d) Area of nABD =


1
3 BD 3 AD (a) Show that nABC and nBAD are congruent.
2 (b) Is AE equal to BE? Explain briefly.
=
1
3 4 3 6 (c) Name two triangles which are similar, but not
2
congruent.
= 12 cm 2

Solution
9.
In the diagram, PQ // BC, QR // AB, AP = 8 cm, (a) AD = BC (given)
PB = 6 cm, PQ = 10 cm and the area of APQ = 36 cm2. AC = BD (given)
(a) Show that APQ is similar to ABC. AB = BA (common side)
(b) Find the length of BC. A \ nABC ≡ nBAD (SSS)
(c) Find the area of ABC.
(b) ∠CAB = ∠DBA (corr. parts of ≡ ns)
(d) Find the length of RC. 8
\ AE = BE (sides of isos. n)
(e) Find the area of
(i) RCQ, P
10 Q (c) nABE is similar to nCDE.
(ii) BRQP. 6
11. In the diagram, ACE and BCD are straight lines,
B C
R ∠ABC = ∠EDC = 90°.
D
Solution 16
(a) In nABC and nAPQ, A E
∠ABC = ∠APQ (corr. ∠s, PQ // BC) C

∠BAC = ∠PAQ (common) 20 15


\ nABC is similar to nAPQ. (Angle-Angle- B
Angle similarity)
(a) Prove that nABC and nEDC are similar.
BC
(b) = AB (b) If AB = 20 cm, BC = 15 cm and DE = 16 cm,
PQ AP find the length of
BC
= 8+6 (i) CD,
10 8 (ii) AE.

BC = 17.5 cm

159

05 FWS3A(Exp)_ch4.indd 159 12/16/14 12:54 PM


© Star Publishing Pte Ltd. All rights reserved.

Solution Area of  ABC  8


2

(a) In nABC and nEDC, (ii) =  


Area of  ADE 5
∠ABC = ∠EDC = 90° (given) 64
∠ACB = ∠ECD (vert. opp. ∠s) =
25
\ n  ABC is similar to nEDC. (Angle-Angle- Ratio of the areas = 64 : 25
Angle similarity)
DC DE 13.
The diagram shows two similar basins A and B. Their
(b) (i) =
BC BA
base areas are 300 cm2 and 675 cm2 respectively.
DC 16
= (a) Find the ratio of the radius of basin A to that
15 20
of basin B.

DC = 12 cm
(b) If basin A can hold 4.5 kg of water, find the mass
(ii) In nABC, of water that can be held by basin B.
AC 2 = 202 + 152 (Pythagoras’ Theorem)
= 625
AC = 25 cm
In nEDC,
EC 2 = 162 + 122 (Pythagoras’ Theorem) A B
= 400 Solution
EC = 20 cm (a) Ratio of the base areas = 300 : 675
\ AE = AC + CE =4:9
= 25 + 20
Ratio of the base radii = 4 : 9
= 45 cm
=2:3

12. In the diagram, D and E are the points on AC and AB (b) Mass of water held by basin B
3
respectively and ∠ABC = ∠ADE. =
 3
  3 4.5
2
27
A = 3 4.5
8
5 = 15.2 kg (correct to 3 s.f.)
6

E
D 14. The volumes of two similar cylinders are in the ratio
2 343 : 729.
C (a) Find the ratio of the base radii of the cylinders.
B
(b) Find the ratio of the base areas of the cylinders.

(a) Name two similar triangles and give your Solution


reason. (a) Ratio of the base radii = 343 :
3 3
729
(b) If AD = 6 cm, DC = 2 cm and AE = 5 cm, find =7:9
(i) the length of BE,
(ii) the ratio of the area of nABC to the area of (b) Ratio of the base areas = 72 : 92
nADE. = 49 : 81

Solution 15. The ratio of the areas of the corresponding plane faces
(a) ∠ADE = ∠ABC (given) of two similar right square pyramids, made of the same
∠DAE = ∠BAC (common) material, is 16 : 25.
\ nDAE is similar to nBAC. (Angle-Angle- (a) Find the ratio of the heights of the pyramids.
Angle similarity) (b) If the mass of the bigger pyramid is 400 g, find
AD AE the mass of the smaller one.
(i) (b)=
AB AC
6 5 Solution
=
AB 8 (a) Ratio of the heights = 16 : 25
AB = 9.6 cm =4:5
\ BE = AB – AE Mass of smaller pyramid  4
3

= 9.6 – 5 (b) Mass of larger pyramid


=  
5
= 4.6 cm Mass of the smaller one 64
=
400 125
\ mass of smaller pyramid = 204.8 g

Chapter 4 Conditions of Congruence and Similarity


160

05 FWS3A(Exp)_ch4.indd 160 12/16/14 12:54 PM


© Star Publishing Pte Ltd. All rights reserved.

16.
The diagram shows a cup in the form of an inverted cone BH = CH – BC
of base radius 7 cm and height 24 cm. It contains water = y – x
to a depth of 12 cm. AE = BH (found)
(a) Find the volume of the cup. A M 7 B
NE = AN + AE
2 2
(b) Find the volume of the water
in the cup. = BE 2 + BH 2
(c) Find the additional volume D 24 = EH (found)
C N
of water required in order to \ nANE ≡ nBEH (SSS)
increase the depth of water to 12
18 cm. (b) Similarly nDKN ≡ nBEH
(You may leave your answers Let NE = EH = NK = KH = z
  in terms of r.)
V Let ∠EHB = u°
∠BEH = v°
Solution ∠EHB + ∠BEH = 180° – 90° = 90°
1 ∠NEA = ∠EHB = u°
(a) Volume of cup = r3 72 3 24 ∠NEH = ∠NEA + ∠BEH
3
= 392r cm3 = u° + v°
= 90°
Volume of water  12 
2
(b) Volume of cup
=   Similarly for ∠ENK, ∠NKH, ∠KHE,
20
\ EHKN is a square.
Volume of water 1
=
392r 8
(c) NE = y + (y – x)
2 2
\ volume of water = 49r cm3
= y + y – 2xy + x
2 2 2
(c) Let V cm3 be the volume of water in the cup when
the depth is 18 cm. = 2y 2 – 2xy + x 2
3
V
=  
18
Area of EHKN = NE2
392r 24
= (x2 – 2xy + 2y2) cm2
V = 165.375r
\ the required volume of water
= 165.375r – 49r 2. In the diagram, AF, BE and CD are perpendicular
= 116.375r cm3 to the line ABC. AED and CEF are straight lines.
AF = a, CD = b, AB = c, BC = d and BE = x.
Go Further (a) Show that nABE and nACD are similar.
1. In the diagram, ABCD is a square of side x cm. Four (b) Hence express x in terms of b, c and d.
squares of sides y cm, where y . x, are placed along its (c) By considering nBCE and nACF, express x in
sides. terms of a, c and d.
ab
(a) Show that nANE and nBEH are congruent. (d) Hence show that x = .
a+b
(b) What type of quadrilateral is EHKN? Give your F
reasons.
(c) Express the area of EHKN in terms of x and y. D
M L
a
E
b
P y N x

K J A
D c C
C B d

x
Q E A B Solution
(a) ∠ABE = ∠ACD (given)
H I
Let ∠AEB = u°
∠BED = 180° – u° (∠s on a st. line)
F G ∠CDE = 360° – (180° – u°) – 90° – 90°
Solution = u°
(a) AN = BE (sides of ≡ squares) = ∠AEB
In nANE and nBEH, \ nABE is similar to nACD (Angle-Angle-
∠NAE = ∠EBH = 90° Angle similarity)
AE = BE – AB
= y – x

161

05 FWS3A(Exp)_ch4.indd 161 12/16/14 12:54 PM


© Star Publishing Pte Ltd. All rights reserved.

x b
(b) =
c c+d
b
x = (c)
c+d
bc
=
c+d

(c) nBCE is similar to nACF (Angle-Angle-Angle


similarity)
x d

=
a c+d
ad
x =
c+d

(d) From (b), xc + xd = bc


xc – bc = –xd
c(x – b) = –xd
c –x
=
d x–b
From (c), xc + xd = ad
xc = (a – x)d
c a–x
=
d x
–x a–x
\  =
x–b x
–x 2
= (a – x)(x – b)
–x2 = ax – ab – x2 + bx
ax + bx = ab
(a + b)x = ab
ab
x= (shown)
a+b

Chapter 4 Conditions of Congruence and Similarity


162

05 FWS3A(Exp)_ch4.indd 162 12/16/14 12:54 PM


© Star Publishing Pte Ltd. All rights reserved.

5 Coordinate Geometry

Class Activity 1
Objective: To find the length of a line segment given the coordinates of its two end points.

1. (a) The diagram shows two horizontal line segments, AB and CD and 2 vertical line segments EF and GH. Copy and
complete the following table for the lengths of the line segments.

y
F
3

2 A B

H
1
E

x
_4 _3 _2 _1 O 1 2 3 4
_1

_2
G

C _3 D

Line Segment Coordinates of end points Length (units)

AB A( 1 , 2 ), B( 4 , 2 ) 4–1=3
CD C( –3 , –3 ), D( 2 , –3 ) 2 – (–3) = 5

EF E( –2 , 1 ), F( –2 , 3 ) 3–1=2

GH G( 3 , –2 ), H( 3 , 1 ) 1 – (–2) = 3

(b) If the end points of a line segment are A(x1, k) and B(x2, k), where x2  x1, what is the length of AB?
x2 – x1 units

(c) If the end points of a line segment are G(h, y1) and H(h, y2), where y2  y1, what is the length of GH ?
y2 – y1 units

163

06 FWS3A(Exp)_ch5.indd 163 12/16/14 12:56 PM


© Star Publishing Pte Ltd. All rights reserved.

2. y

B
4

C
2

1 A
N

x
_4 _3 _2 _1 O 1 2 3 4 5
_1

_2

P _3 D

Find the lengths of the inclined line segments AB and CD in the above diagram by copying and completing the following.

The coordinates of A, B and N are The coordinates of C, D and P are

A( 1 , 1 ), B( 5 , 4 ) and C( – 4 , 2 ), D( 2 , –3 ) and

N( 5 , 1 ). P( – 4 , –3 ).

AN = 4 units CP = 5 units

BN = 3 units DP = 6 units

AB = AN + BN
CD = CP 2 + DP 2
2 2 2 2

(Pythagoras’ Theorem) = 52 + 62

= 42 + 32
= 61

= 5 ∴ CD = 61 units

∴ AB = 5 units

Chapter 5 Coordinate Geometry


164

06 FWS3A(Exp)_ch5.indd 164 12/16/14 12:56 PM


© Star Publishing Pte Ltd. All rights reserved.

Class Activity 2
Objective: To find the gradient of a line using the coordinates of any two points on the line.
1.

(a) In The Geometer’s Sketchpad, draw a line AB.


(b) Measure the coordinates of the points A(xA, yA) and B(xB, yB) respectively using the commands Measure | Abscissa
(x) and Measure | Ordinate ( y).
yB – yA
(c) Calculate xB – xA, yB – yA and .
xB – xA
(d) Measure the gradient of AB using the command Measure | Slope.
(Note: In The Geometer’s Sketchpad, the term “slope” means the gradient of a line.)
(e) Drag the point B around and observe the changes in the signs and values of the quantities in (c) and (d). What is
the relationship between y B – y A and the gradient of the line AB?
xB – xA
y – yA
Gradient of the line AB = B
xB – x A

(f ) What are some special features of the values of the quantities in (c) and (d) when the line AB is
(i) horizontal?
The values are zero.

(ii) vertical?
The values are undefined.

(g) Discuss the signs of the quantities in (c) and (d) when the point B is on the right of the point A and is
(i) above the point A,
The signs of the quantities are positive.

(ii) below the point A.


The signs of the quantities are negative.

165

06 FWS3A(Exp)_ch5.indd 165 12/16/14 12:56 PM


© Star Publishing Pte Ltd. All rights reserved.

2. In the diagram, A(x1, y1) and B(x2, y2) are two points on the straight line L.
y

B(x2, y2) L
y2
vertical
A(x1, y1) change
y1
horizontal
change

x
O x1 x2

y 2 – y1
(a) Express the vertical change from A to B in terms of y1 and y2.
x2 – x1
(b) Express the horizontal change from A to B in terms of x1 and x2.
y2 – y1

(c) Hence express the gradient of the line L in terms of x1, x2, y1 and y2.
x2 – x1

Chapter 5 Coordinate Geometry


166

06 FWS3A(Exp)_ch5.indd 166 12/16/14 12:56 PM


© Star Publishing Pte Ltd. All rights reserved.

Try It! QR = (4 – 3)2 + (–1 – 2)2


Section 5.1
= 12 + (–3)2
1. Three points are P(–7, – 4), Q(– 4, –2) and R(2, 2).
(a) Find the lengths of PQ, QR and PR. = 10
(b) Show that P, Q and R lie on a straight line. = 3.16 units (correct to 3 s.f.)
RP =
2 2
(–3 – 4) + [0 – (–1)]
Solution
(a) PQ = [– 4 – (–7)]2 + [(–2 – (– 4)]2 = 2
(–7) + 1
2

= 2 2 = 50
3 +2
= 7.07 units (correct to 3 s.f.)
= 13
= 3.61 units (correct to 3 s.f.) (b) PQ + QR = ( 40 ) + ( 10 )
2 2 2 2

QR =
2
(2 – (– 4)] + [2 – (–2)]
2
= 40 + 10
= 50
= 62 + 4 2 RP = ( 50)
2 2

= 52 = 50
\ PQ + QR = RP
2 2 2
= 7.21 units (correct to 3 s.f.)
Hence ∠PQR = 90° (converse of Pythagoras’
PR = 2
(2 + 7) + (2 + 4)
2
Theorem)
= 81 + 36 i.e. nPQR is a right-angled triangle.

= 117 (c) Area of nPQR =


1
3 PQ 3 QR
2
= 10.8 units (correct to 3 s.f.) 1
= 3 40 3 10
(b) PQ + QR – PR = 13 + 52 – 117 2
= 10 units
2
= 0
Hence, P, Q and R lie on a straight line. (d) From the diagram,
the coordinates of the point S = (–2, –3)
2. In the diagram, the vertices of PQR are P(–3, 0),
Q(3, 2) and R(4, –1). 3. The coordinates of a point C are (3, 2). D is a point on
y the y-axis such that CD = 5 units. Find the possible
Q(3, 2) coordinates of the point D.
2

Solution
1
Let (0, k) be the coordinates of the point D.
P(_3, 0) CD = 5
x
_3 _2 _1 O 1 2 3 4
(0 – 3)2 + ( k – 2)2 = 5
_1
9 + (k – 2) = 25
2
R(4, _1)
(k – 2) = 16
2
_2
k – 2 = ± 16
\ k – 2 = – 4 or k – 2 = 4
(a) Find the lengths of PQ, QR and RP. k = –2 or k=6
(b) Show that PQR is a right-angled triangle. \ the possible coordinates of D are (0, –2) and (0, 6).
(c) Find the area of PQR.
(d) If PQRS is a rectangle, from the diagram of PQR, Section 5.2
identify the coordinates of the point S. 4. Find the gradient of the line joining the points
(a) P(–4, 1) and Q(3, 5),
Solution (b) R(0, –3) and S(4, 2).
(a) PQ = [3 – (–3)]2 + (2 – 0)2
Solution
= 2
6 +2 2
(a) Gradient of PQ =
5–1
3 – (– 4)
= 40 4
=
= 6.32 units (correct to 3 s.f.) 7
2 – (–3)
(b) Gradient of RS =
4–0
5
=
4

167

06 FWS3A(Exp)_ch5.indd 167 12/16/14 12:56 PM


© Star Publishing Pte Ltd. All rights reserved.

5. The line joining the points Q(–2, 3) and R(6, –1) intersects (b) Let the equation of L 4 be y = – x + c.
1
the x-axis at the point T. Find 2
(a) the gradient of the line. Since B(4, 1) is on L 4, 1= –
1
(4) + c
(b) the coordinates of T. 2
c=3
1
Solution Hence, the equation of L 4 is y = – x + 3.
–1 – 3 2
(a) Gradient of the line QR =
6 – (–2) 8. Find the equation of the line
= –
1 (a) L 3 that passes through the points P(–2, 3) and
2 Q(2, –1),
(b) Let (k, 0) be the coordinates of T. (b) L 4 that passes through the points R(– 4, 5) and
Gradient of QT = Gradient of QR S(–1, 5).
0–3 1
=– Solution
k – (–2) 2
6= k+2 (a) Gradient of L3 = gradient of PQ
k = 4 =
–1 – 3
\ the coordinates of T are (4, 0). 2 – (–2)
= –1
Let the equation of L3 be y = –x + c.
6. Find the gradient of the line Since P(–2, 3) is on L3, 3 = – (–2) + c
(a) y = 1, (b) x = –3. c=1
y \ the equation of L3 is y = –x + 1.
(b) Gradient of L 4 = gradient of RS
x = _3 5–5
=
–1 – (– 4)
= 0
y=1 \ L 4 is a horizontal line.
1 Since it passes through R(– 4, 5), the equation of
_3 x L 4 is y = 5.
O

9. The equation of a straight line L is 2x – 3y + 4 = 0.


(a) Express the equation in the gradient-intercept form.
(b) Hence state the gradient and the y-intercept of L.
Solution
(a) Since y = 1 is a horizontal line, the gradient of the Solution
line y = 1 is zero. (a) L: 2x – 3y + 4 = 0
(b) Since x = –3 is a vertical line, the gradient of the 3y = 2x + 4
2 4
line x = –3 is undefined. Hence, the gradient-intercept form is y = x + .
3 3
2
(b)    Gradient of L =
3
Section 5.3 4
7. Find the equation of the line y-intercept of L =
3
2
(a) L3 with gradient = and passing through the Section 5.4
5
origin, 10. The diagram below shows a map on a coordinate plane.
1
(b) L 4 with gradient = – and passing through the The coordinates of Singapore Zoo are (8.5, 7.7) and the
2
point B(4, 1). coordinates of Jurong Bird Park are (5, 3).
(a) Find the length of the line segment joining these
Solution two places on the coordinate plane.
(a) y-intercept = 0 (b) Given that 1 unit on the coordinate plane represents
2 2 km, find the shortest distance, in km, between
gradient =
5 these two places.
\ the equation of L3 is
2 Solution
y= x+ 0 (a) Length of the line segment
5
2 = (8.5 – 5) + (7.7 – 3)
2 2
i.e. y= x.
5
= 34.34
= 5.86 units (correct to 3 s.f.)
Chapter 5 Coordinate Geometry
168

06 FWS3A(Exp)_ch5.indd 168 12/16/14 12:56 PM


© Star Publishing Pte Ltd. All rights reserved.

(b)
Given that 1 unit represents 2 km, hence the shortest y
distance between the two places
R(5, 9)
= 34.34 3 2
= 11.7 km (correct to 3 s.f.)

11. The coordinates of four points P, Q, R and S are


P(–3, –1), Q(4, 3), R(5, 9) and S(–2, 5). The lines PR and S(_2, 5)
QS intersect at T. T(1, 4)
(a) Find the equations of PR and QS. Q(4, 3)
(b) Find the coordinates of T.
(c) Show that nPQT and nRST are congruent.
x
Solution O
(a)
–1 – 9
Gradient of PR = P(_3, _1)
–3 – 5
5

=
4
Let the equation of PR be y =
5
x + c. Since (c) PT = 2
(–3 – 1) + (–1 – 4)
2
4
R(5, 9) is on the line,
= 41 units
5
9= (5) +c RT =
2
(5 – 1) + (9 – 4)
2
4

c=
11
= 41 units
4
5 11 QT = (4 – 1)2 + (3 – 4)2
\ the equation of PR is y = x + ............... (1)
4 4

= 10 units
5–3
Gradient of QS =
–2 – 4 ST = (–2 – 1)2 + (5 – 4)2
1
= – = 10 units
3
1 \ PT = RT
Let the equation of QS be y = – x + k. Since
3 QT = ST
Q(4, 3) is on the line, ∠PTQ = ∠RTS (vertically opp. ∠s)

1
3 = – (4) + k Hence nPQT ≡ nRST (SAS)
3
13
k= 12. A boy runs towards a bus stop. His distance, y m, from
3
\ the equation of QS is the bus stop at time t seconds is given in the following
table.
1 13
y=– x+ .......................... (2)
3 3
Time (t s) 0 2 4 6 8
(b) Putting (1) into (2), Distance ( y m) 30 24 18 12 6
5 11 1 13
x+ = – x +
4 4 3 3
15x + 33 = – 4x + 52 (a) Draw the graph of y against t. Describe the
19x = 19 relationship between t and y.
x = 1 (b) Express the equation connecting t and y in the form
Putting x = 1 into (1), y = mt + c, where m and c are constants.
5 11 (c) Find the time when the boy reaches the bus stop.
y = (1) + (d) Interpret the meanings of m and c in the relationship.
4 4
y = 4
\ the coordinates of T = (1, 4)

169

06 FWS3A(Exp)_ch5.indd 169 12/16/14 12:56 PM


© Star Publishing Pte Ltd. All rights reserved.

Solution
(a) The diagram below shows the graph of y against t.
y

30
Distance (m)

20

10

O t
2 4 6 8
Time (s)

The graph shows that the points lie on a straight


line. Hence, t and y have a linear relationship.
(b) The points (0, 30) and (8, 6) are on the straight line
graph.
30 − 6
Gradient of the line =
0−8
= –3
Let the equation of the line be y = –3t + c.
Since (0, 30) is on the line,
c = 30
Hence, the equation connecting t and y is
y = –3t + 30.
(c)
When the boy reaches the bus stop, y = 0.
Putting y = 0 into the equation y = –3t + 30,
0 = –3t + 30
3t = 30
t = 10
Alternatively, the required time can be obtained
from the graph. It is the value of t when the straight
line graph cuts the t-axis.
i.e. the boy reaches the bus stop at time = 10 s.
(d) m = gradient of the straight line graph
= –3
This is the rate of change of distance y with respect
to time t.
Thus, m means the distance between the boy and
the bus stop is decreasing at a rate of 3 m/s.
c = y-intercept of the straight line graph
= 30
This is the value of y when t = 0.
Thus, c means the initial distance between the boy
and the bus stop is 30 m.

Chapter 5 Coordinate Geometry


170

06 FWS3A(Exp)_ch5.indd 170 12/16/14 12:56 PM


© Star Publishing Pte Ltd. All rights reserved.

Exercise 5.1 Level 2


Level 1 3. In each of the following sets of points,
1. In each of following, find the length of the line segment (i) find the lengths of AB, BC and AC,
with the given end points. (ii) determine whether the points A, B and C lie on a
(a) A(3, 2), B(7, 2) (b) C(4, 5), D(4, –3) straight line.
(c) O(0, 0), E(–3, 4) (d) G(3, 1), H(8, 13) (a) A(– 4, 3), B(–2, 2), C(0, 1)
(e) K(–2, 5), L(–7, 6) (f ) P(4, –3), Q(–1, –7) (b) A(–3, –2), B(1, –1), C(3, 0)

Solution Solution

(a) AB = (7 – 3)2 + (2 – 2)2 (a) (i) AB = (–2 + 4)2 + (2 – 3)2


= 4 units = 5
= 2.24 units (correct to 3 s.f.)
(b) CD = (4 – 4) + (–3 – 5)
2 2

= 8 units BC =
2 2
(0 + 2) + (1 – 2)

(c) OE = (–3 – 0) + (4 – 0)
2 2 = 5
= 5 units = 2.24 units (correct to 3 s.f.)

(d)  GH = (8 – 3) + (13 – 1)
2 2 AC = (0 + 4)2 + (1 – 3)2
= 13 units = 20
= 4.47 units (correct to 3 s.f.)
(e) KL = (–7 + 2)2 + (6 – 5)2
= 26 (ii)
AB + BC – AC = 5 + 5 – 20
= 5.10 units (correct to 3 s.f.) = 0
\ AB + BC = AC
(f ) PQ =
2 2
(–1 – 4) + (–7 + 3) Hence, A, B and C lie on a straight line.
= 41
(b) (i)
AB = (1 + 3)2 + (–1 + 2)2
= 6.40 units (correct to 3 s.f.)

= 17
2. A(1, 3) and B(2, 6) are points on a coordinate plane. = 4.12 units (correct to 3 s.f.)
(a) Find the lengths of the line segments OA, OB and
AB, where O is the origin. BC = (3 – 1)2 + (0 + 1)2
(b) Show that the points O, A and B lie on a straight
= 5
line.
(c) What is the relationship between the point A and = 2.24 units (correct to 3 s.f.)
the line segment OB?
AC = (3 + 3)2 + (0 + 2)2
Solution
= 40
(a)
OA = 2
(1 – 0) + (3 – 0) 2 = 6.32 units (correct to 3 s.f.)
= 10 (ii)
AB + BC – AC = 17 + 5 – 40
= 3.16 units (correct to 3 s.f.) ≠0
\ AB + BC ≠ AC
OB = (2 – 0)2 + (6 – 0)2 Hence, A, B and C do not lie on a straight
= 40 line.
= 6.32 units (correct to 3 s.f.)
4. The vertices of ABC are A(–1, –2), B(1, 8) and C(–5,

AB = (2 – 1)2 + (6 – 3)2
4). Show that ABC is an isosceles right-angled triangle.
= 10
= 3.16 units (correct to 3 s.f.) Solution

(b)
OA + AB – OB = 10 + 10 – 40 AB = (–1 – 1)2 + (–2 – 8)2
= 0 = 104 units
\ OA + AB = OB
BC = (–5 – 1)2 + (4 – 8)2
Hence, O, A and B lie on a straight line.
= 52 units
(c) Since OA = AB, A is the midpoint of OB.

171

06 FWS3A(Exp)_ch5.indd 171 12/16/14 12:56 PM


© Star Publishing Pte Ltd. All rights reserved.

(c)
The y-coordinates of A and B are the same, thus
AC = (–5 + 1)2 + (4 + 2)2
AB is a horizontal line.
= 52 units The perpendicular distance from C to AB,
From above, AC = BC, i.e. two sides of nABC are equal CP = 3 – (–1)
in length. = 4 units
BC + AC = ( 52 ) + ( 52 )
2 2 2 2 1
\ area of nABC = 3 AB 3 CP
= 52 + 52 2

= 104 =
1
3 4 3 4
2
AB = ( 104 )
2 2
= 8 units
2
= 104
\ BC + AC = AB (d) Let BN be the perpendicular from B to CA.
2 2 2

Hence, ∠ACB = 90° (converse of Pythagoras’ Theorem) 1


Area of nABC = CA 3 BN
\ nABC is an isosceles right-angled triangle.
3
2
1
8 = 3 5 3 BN
2
5. The vertices of ABC are A(–2, –1), B(2, –1) and BN = 3.2
C(1, 3). Find \ the required length is 3.2 units.
(a) the lengths of AB, BC and CA,
(b) the perimeter of ABC,
6. The vertices of PQR are P(– 4, 4), Q(4, 6), and
(c) the area of ABC,
R(5, 2).
(d) the length of the perpendicular from B to CA.
(a) Find the lengths of PQ, QR and RP.
(b) Show that PQR is a right-angled triangle.
Solution
(c) Find the area of PQR.
(a) y (d) State the coordinates of the point S if PQRS is a
rectangle.
C(1, 3)
3
Solution
(a) PQ = (–4 – 4) + (4 – 6)
2 2
2
= 68
= 8.25 units (correct to 3 s.f.)
1
N
QR = (5 – 4) + (2 – 6)
2 2

x = 17
_2 _1 O 1 2
= 4.12 units (correct to 3 s.f.)
_1
B(2, _1) RP = (–4 – 5) + (4 – 2)
2 2
A(_2, _1) P
= 85
= 9.22 units (correct to 3 s.f.)
AB =
2 2
(–2 – 2) + ( – 1 + 1)
(b) PQ + QR = ( 68 ) + ( 17 )
2 2 2 2

= 16 = 68 + 17
= 4 units = 85
RP = ( 85 )
2 2

BC = (2 – 1)2 + ( – 1 – 3)2 = 85
\ PQ + QR = RP
2 2 2
= 17
Hence ∠PQR = 90° (converse of Pythagoras’
= 4.12 units (correct to 3 s.f.)
Theorem)
CA = (–2 – 1) + ( – 1 – 3)
2 2
\ PQR is a right-angled triangle. (shown)
= 25
1
= 5 units (c) Area of nPQR = 3 PQ 3 QR
2
(b) Perimeter of nABC = AB + BC + CA =
1
3 68 3 17
2
= 4 + 17 + 5
= 17 units
2

= 13.1 units (correct to 3 s.f.)

Chapter 5 Coordinate Geometry


172

06 FWS3A(Exp)_ch5.indd 172 12/16/14 12:56 PM


© Star Publishing Pte Ltd. All rights reserved.

(d) y 10. The vertices of KLM are K(3, 13), L(8, 2) and
M(3, 3). Find
Q(4, 6) (a) the length of LK,
6
(b) the perpendicular distance from M to LK,
P(_4, 4) (c) the coordinates of the point Q if KLQM is a
4
parallelogram.

2 R(5, 2) Solution
(a) LK = (3 – 8)2 + (13 – 2)2
_4 S _2 x
O 2 4 6 = 146
= 12.1 units (correct to 3 s.f.)
On the diagram, construct a line through R and (b) y
parallel to PQ, and a line through P and parallel
to QR. The point of intersection of these two lines 15 K(3, 13)
is the point S where PQRS is a rectangle. From the
diagram, the coordinates of the point S = (–3, 0).
10 P
7. The distance between the points P(–3, 2) and Q(1, k) is
5 units. Find the possible values of k. 5
M(3, 3) L(8, 2)
Solution x
Given PQ = 5 units O 2 4 6 8

(–3 – 1)2 + (2 – k )2 = 5 _5
(– 4) + (2 – k) = 5
2 2

(2 – k) = 25
2
_10 Q
2–k=± 9
2–k=3 or 2 – k = –3
\ k = –1 or k=5
KM is a vertical line as both K and M have the
same x-coordinates, x = 3.
8. A(3, 4) and B(–3, 2) are points on a coordinate plane.
Length of KM = 13 – 3
Find the coordinates of a point C on the x-axis such that
= 10 units
AC = BC.
Perpendicular distance from L to KM
Solution =8–3
Let (k, 0) be the coordinates of the point C. = 5 units
AC = BC Area of nMKL =
1
3 10 3 5
2
(3 – k ) + (4 – 0) = (–3 – k ) + (2 – 0)
2 2 2 2
= 25 units
2

(3 – k) + 16 = (–3 – k) + 4
2 2

12k = 12 Let MP be the perpendicular distance from M to LK.


\ k =1 Area of nMKL =
1
× LK × MP
Hence, the coordinates of C = (1, 0). 2
1
25 = × 146 × MP
2
9. Two points A(– 4, 5) and B(7, 2) are given. Find the
25 3 2
coordinates of a point C on the y-axis which is equidistant \ MP =
146
from the points A and B.
= 4.14 units (correct to 3 s.f.)
Solution
(c) Use the similar method as in Question 6(d) to obtain
Let (0, h) be the coordinates of the point C.
the point Q. From the diagram, the coordinates of
AC = BC
the point Q = (8, –8).
(–4 – 0) + (5 – h ) = (7 – 0)2 + (2 – h )2
2 2

16 + (5 – h) = 49 + (2 – h)
2 2

16 + 25 – 10h + h = 49 + 4 – 4h + h
2 2

6h = –12
\ h = –2
Hence, the coordinates of C = (0, –2).

173

06 FWS3A(Exp)_ch5.indd 173 12/16/14 12:56 PM


© Star Publishing Pte Ltd. All rights reserved.

Level 3 Area of nOCN =


1
3 1 3 2
2
11. In the diagram, AOC and BOD are straight lines. = 1 unit
2

(a) Find the lengths of OA, OB, OC and OD.


3
(b) Name a triangle which is congruent to AOB and Area of nOCD = +2–1
2
state the reason for the congruence. 1
= 2 units
2
(c) Find the area of OCD. 2
y

3
B 12. The vertices of a quadrilateral ABCD are A(5, 4),
B(–3, 2), C(– 4, –3) and D(4, –1).
2 (a) Show that AB = DC and AD = BC.
(b) Determine whether ∠ABC = 90°.
1 A (c) What type of quadrilateral is ABCD?

x Solution
_2 _1 O 1 2
_1 (a) AB = (–3 – 5)2 + (2 – 4)2
C
= 68 units
_2
DC =
2 2
(4 + 4) + (–1 + 3)
_3
D = 68 units
\ AB = DC (shown)

AD = (4 – 5) + (–1 – 4)
2 2
Solution
(a) The coordinates of A, B, C and D are (2, 1), (1, 3), = 26 units
(–2, –1) and (–1, –3) respectively. 2 2
BC = (–4 + 3) + (–3 – 2)
OA = (2 – 0)2 + (1 – 0)2 = 26 units
= 5 \ AD = BC (shown)
= 2.24 units (correct to 3 s.f.) 2 2
(b) AC = (– 4 – 5) + ( – 3 – 4)

OB = (1 – 0)2 + (3 – 0)2
= 130 units
= 10
AB + BC = ( 68 ) + ( 26 )
2 2 2 2
= 3.16 units (correct to 3 s.f.)
= 68 + 26

OC = (–2 – 0) + (–1 – 0)2 = 94
2

AC = ( 130 )
2 2
= 5
= 130
= 2.24 units (correct to 3 s.f.)
\ AB + BC ≠ AC
2 2 2


OD = (–1 – 0) + (–3 – 0)2 Hence ∠ABC ≠ 90°.
2

= 10 (c) From (a), we have AB = DC and AD = BC.


= 3.16 units (correct to 3 s.f.) \ the opposite sides of the quadrilateral ABCD
(b) In nAOB and nCOD, is equal and parallel, but the adjacent sides
OA = OC (found) AB ≠ BC and AB ≠ AD.
∠AOB = ∠COD (vert. opp. ∠s) Hence, the quadrilateral ABCD is a parallelogram.
OB = OD (found)
\ nAOB ≡ nCOD (SAS) 13. The line y = 2x + 6 cuts the x-axis at A and the y-axis
(c) Draw perpendiculars CN and DM from the points at B. Find
C and D to the x-axis. (a) the length of AB,
1
(b) the shortest distance of O to AB, where O is the
Area of nODM = 3133 origin (0, 0).
2
3
= units
2
2
1
Area of trapezium CNMD = 3 (1 3 3) 3 1
2
= 2 units
2

Chapter 5 Coordinate Geometry


174

06 FWS3A(Exp)_ch5.indd 174 12/16/14 12:56 PM


© Star Publishing Pte Ltd. All rights reserved.

Solution
(c) BC = (0 + 2)2 + (–6 – 9)2
(a) y = 2x + 6
When the line cuts the x-axis at A, y = 0. = 40 units
\ 0 = 2x + 6 Let the length of the perpendicular from A to BC
x = –3 be h units.
\ the coordinates of A = (–3, 0). Area of nABC = 15 units
When the line cuts the y-axis at B, x = 0. 1
3 BC h = 15
\ y = 2(0) + 6
3
2
y =6
1
h = 15
3 40 3
\ the coordinates of B = (0, 6). 2
15 3 2
h=
Hence, AB =
2 2
(–3 – 0) + (0 – 6) 40

= 45 = 4.74 units (correct to 3 s.f.)
= 6.71 units (correct 3 s.f.) Hence, the length of the perpendicular from A to
BC is 4.74 units. (correct to 3 s.f.)
1
(b) Area of nAOB = 3 3 3 6
2
= 9 units
2
15. The diagram represents the locations of four places A,
Let h be the shortest distance of O to AB. B, C and D on a map with a scale of 1 unit representing
Area of nAOB = 9 units 2 km. The positive y-axis points to the North direction.
1 A man drives along the roads AB, BC and CD from A
3 AB h =9
to D.
3
2
1 y
3 45 3 h =9
2
932 C(2, 4)
h = D(4, 4)
45
h = 2.68 units (correct to 3 s.f.)

14. The graph of y = x – x – 6 cuts the x-axis at A and B,


2

where the x-coordinate of A is greater than that of B.


The y-intercept of the graph is at C. Find
(a) the coordinates of A, B and C,
(b) the area of ABC, A(0, 1) B(2, 1)
(c) the length of the perpendicular from A to BC.
x
Soluton O
(a) y=x –x–6
2

When the graph cuts the x-axis, (a) What is the distance travelled by the man?
y=0 (b) If he could drive directly from A to D, by how
\ x – x – 6 = 0
2 many km would his journey be shortened?
(x + 2)(x – 3) = 0 (c) Find ∠DAB.
x = –2 or x = 3
Since the x-coordinate of A is greater than that of Solution
B, the coordinates of A = (3, 0) and B = (–2, 0). (a)
Total length on the map = AB + BC + CD
At C, x=0 =2+3+2
\ y=0 –0–6
2 = 7 units
y = –6 Given map scale: 1 unit represents 2 km
\ the coordinates of C = (0, –6). \ total distance travelled by the man
=732
(b) Length of AB = 5 units = 14 km
OC = 6 units
(b) Length of AD on the map = (4–0) + (4 – 1)
2 2
1
\ area of nABC =
3 AB 3 OC
2 = 25

1
= 3 5 3 6 = 5 units
2 \ distance AD = 5 3 2
= 15 units
2
= 10 km
If the man drives directly from A to D, his journey
will be shortened by 14 – 10 = 4 km.

175

06 FWS3A(Exp)_ch5.indd 175 12/16/14 12:56 PM


© Star Publishing Pte Ltd. All rights reserved.

(c) y In the second case,


(a – 1) = (–3) and (b – 2) = 4
2 2 2 2

C(2, 4) D(4, 4) a – 1 = –3 b–2=4


a = –2 and b=6
\ the coordinates of another possible point P = (–2, 6).
Other possible cases may be (a – 1) = 5 and (b – 2) = 20,
2 2

(a – 1) = 10 and (b – 2) = 15 etc.
2 2

A(0, 1)
B(2, 1) N
Exercise 5.2
x Level 1
O
1. In each of the following, find the gradient of the line
passing through the given pair of points.
Draw the perpendicular, DN, from D to AB (a) A(3, 4), B(6, 7)
produced. (b) C(–5, 8), D(–2, 5)
In nDAN, (c) E(2, 3), F(2, –5)
DN = 3 units (d) G(–7, –1), H(3, –1)
(e) S(a , 3ka), T(b , 3kb)
2 2
and AN = 4 units
(f ) V(2ht, –ht ), W(2hs, –hs )
2 2
DN
tan ∠DAB =
AN

=
3 Solution
4 7–4
\ ∠DAB = 36.9° (correct to 1 d.p.) (a) Gradient of AB =
6–3
=1
16. Given a point G(1, 2), find the coordinates of two possible (b) Gradient of CD =
5–8
–2 – (–5)
positions of a point P(a, b) in the second quadrant such
= –1
that the length of PG = 5 units.
–5 – 3
(c) Gradient of EF =
2–2
Solution
y =
–8
0
P(a, b) \ gradient of EF is undefined.
–1 – (–1)
(d) Gradient of GH =
3 – (–7)
5
=0
2 3ka – 3kb
G(1, 2) (e) Gradient of ST =
a –b
2 2

3k (a – b )
x =
O 1 (a + b )(a – b )
3k
=
a+b
If P(a, b) is a point in the 2nd quadrant, the value of a
– ht 2 – (– hs 2 )
is negative and that of b is positive. (f ) Gradient of VW = 2 ht – 2 hs
PG = 5 units
h( s 2 – t 2 )
= 2 h(t – s )
(a –1)2 + (b – 2)2 = 5
( s + t )( s – t )
(a – 1) + (b – 2) = 5
2 2 2
= –2( s − t )
By using the Pythagorean Triplet 3 + 4 = 5 , we can
2 2 2
s+t
= –
consider the following two possible cases: 2
(– 4) + 3 = 5
2 2 2

and (–3) + 4 = 5
2 2 2

In the first case,


(a – 1) = (– 4) and (b – 2) = 3
2 2 2 2

a – 1 = – 4 b–2=3
a = –3 and b=5
\ the coordinates of one possible point P = (–3, 5).

Chapter 5 Coordinate Geometry


176

06 FWS3A(Exp)_ch5.indd 176 12/16/14 12:56 PM


© Star Publishing Pte Ltd. All rights reserved.

2. Find the gradients of the line segments AB, CD, EF and 5. The points A(–5, –3), B(–2, k) and C(4, 3) lie on a straight
GH in the diagram. line. Find the value of k.
y
C(2, 6)
Solution
6 Gradient of AB = gradient of AC
k – (–3) 3 – (–3)
A(– 4, 4)
4
B(2, 5) =
–2 – (–5) 4 – (–5)
E(3, 2) k +3 2
2 =
G(–6, 1) H(–1, 1) D(6, 2) 3 3

x
k + 3 = 2
_6 _4 _2 O 2 4 6 k = –1
_2
6. The line segment joining P(–6, 7) and Q(5, –2) intersects
_4 F(3, –3) the y-axis at the point R. Find the coordinates of R.

Solution
Solution Let the coordinates of R be (0, r).
5–4 Gradient of PR = gradient of PQ

Gradient of AB = r –7 –2 – 7
2 – (– 4) =
=
1 0 – (– 6) 5 – (– 6)
r –7 –9
6
=
2–6 6 11
Gradient of CD =
6–2 11r – 77 = –54
= –1 11r = 23
Since EF is vertical, gradient of EF is undefined. 23
r=
1–1 11
Gradient of GH =
–1 – (– 6)
\ the coordinates of R are  0, 2
1
= 0 .
11 

3. The vertices of ABC are A(–5, –3), B(4, 0) and 7. The gradient of a line AB is –2, where A is the point
C(–1, 3). Find the gradients of the sides of the triangle. (1, –8) and B is a point on the x-axis.
(a) Find the coordinates of B.
Solution (b) Find the length of AB.
Gradient of AB =
0+3 (c) If C is another point on the x-axis such that
4+5 AC = AB, find the coordinates of C.
=
1 (d) Find the gradient of AC.
3
3–0
Gradient of BC = Solution
–1 – 4
3
(a) Let the coordinates of B be (h, 0).
= – Gradient of AB = –2
5
–8 – 0
Gradient of CA =
–3–3 1– h
= –2
–5 – (–1)
8
=
3
1– h
=2
2
1–h =4
\ h = –3
Level 2 \ the coordinates of B = (–3, 0)
4. The gradient of the line passing through A(2, 3) and
B(k, 7) is
2
. Find the value of k. (b) Length of AB = 2
(–3 – 1) + [0 – (–8)]
2
3
= 80
Solution
2 = 8.94 units (correct to 3 s.f.)
Gradient of AB =
3
7–3 2
=
k –2 3
12 = 2k – 4
2k = 16
k=8

177

06 FWS3A(Exp)_ch5.indd 177 12/16/14 12:56 PM


© Star Publishing Pte Ltd. All rights reserved.

(c) Let the coordinates of C be (c, 0). (b) y

AC = AB C(2, 6)
B(5, 6)

(1 – c ) + (–8 – 0) = 2 2
80
(1 – c) + 64 = 80
2

(1 – c) = 16
2

1 – c = ± 16 A(5, 2)
1 – c = 4 or 1 – c = – 4
c = –3 or c=5 x
Since the point (–3, 0) is B, the coordinates of O T D(3, 0) S

C = (5, 0).
Draw the perpendicular, AS, from A to OD produced
–8 – 0
(d) Gradient of AC = (i.e. the x-axis).
1– 5
In nAOS,
=
2
AS
tan ∠AOD =
OS
8. In the diagram, the vertices of the quadrilateral OABC 2
=
are the origin O, A(5, 2), B(5, 6) and C(2, 6). D is the 5
point (3, 0). Similarly, draw the perpendicular, CT, from C to
(a) Find the gradients of OA, AB, BC and OC. the x-axis.
(b) Find tan ∠AOD and tan ∠COD. In nCOT,
(c) What is the relationship between the gradient of CT
tan ∠COD =
OA and tan ∠AOD? OT
(d) Find the size of ∠AOC. =
6
2
y
=3
C(2, 6) B(5, 6)

(c) Gradient of OA = tan ∠AOD

(d) ∠AOC = ∠COD – ∠AOD


A(5, 2) –1  2
=
tan (3) – tan
–1
 5 
x
O D(3, 0) ≈ 71.57° – 21.80°
= 49.8° (correct to 1 d.p.)

Solution
9. The equation of the graph of a parabola is
(a)
2–0
Gradient of OA = y = 4x + 12x + 9.
2
5–0
2
(a) Show that the graph touches the x-axis and find
= the coordinates of the point of contact, A.
5
6–2 (b) Find the coordinates of the point of intersection,
Gradient of AB = B, of the graph with the y-axis.
5–5
4 (c) Find the gradient of AB.
= (undefined) (d) Sketch the graph of the parabola.
0
6–6
Gradient of BC = Solution
5–2
0 (a) When y = 0,
= 4x + 12x + 9 = 0
2
3
=0 (2x + 3)(2x + 3) = 0
2x + 3 = 0
6–0
Gradient of OC = \ x = –1
1
0–3 2
=3 The equation has only one root, thus the graph
touches the x-axis at only one point. (shown)
The coordinates of the point of contact,
A =  –1 , 0  .
1
 2 

Chapter 5 Coordinate Geometry


178

06 FWS3A(Exp)_ch5.indd 178 12/16/14 12:56 PM


© Star Publishing Pte Ltd. All rights reserved.

(b) When x = 0, Gradient of CD =


1–2
y = 4(0) + 12(0) + 9
2 –1 – 3
=9 =
1
4
\ the coordinates of the point B = (0, 9). 1 – (–2)
Gradient of DA =
9–0 –1 – (–2)
(c) Gradient of AB = = 3
( )
0 – –1
1
2
(b) From (a), we have
=6
gradient of AB = gradient of CD,
(d) y and gradient of BC = gradient of DA.
y = 4x2 + 12x + 9 \ the gradients of the opposite sides of a
parallelogram are equal.

10 12. The vertices of a rectangle PQRS are P(–2, –3), Q(4, 0),
9 B(0, 9)
R(2, 4) and S(– 4, 1).
(a) Find the gradients of all four sides of PQRS.
5 (b) What can you say about the gradients of the adjacent
sides of a rectangle?
x
_3 _2 _ 1 1 _1 O Solution
2
0 – (–3)
A( _1 1 , 0)
2 (a) Gradient of PQ =
4 – (–2)
1
=
2
4 –0
Level 3 Gradient of QR =
2–4
10. The line joining the points E(k, – 6) and F(–5, 4k) has a = –2
k
gradient of – . Find the possible values of k. 1–4
3 Gradient of RS =
–4 – 2
1
Solution =
k 2

Gradient of EF = – –3 – 1
3 Gradient of SP =
4 k − (–6) –2 – (– 4)
k
–5 – k
=– = –2
3
4k + 6 k 1
=– (b) Gradient of PQ 3 gradient of QR = 3 (–2)
–(5 + k ) 3 2
3(4k + 6) = k(5 + k) = –1
12k + 18 = 5k + k The product of the gradients of any 2 adjacent sides
2

k – 7k – 18 =0 of a rectangle is equal to –1.


2

(k + 2)(k – 9) =0
\ k = –2 or k=9
13. In the diagram, OA and OB represent two escalators in
a shopping mall and the unit of the coordinate plane is
11. The vertices of a parallelogram ABCD are A(–2, –2), in metres. The point A(20, 5) is at the second level and
B(2, –1), C(3, 2) and D(–1, 1). the point B(k, 9) is at the third level. The gradient of OB
(a) Find the gradients of all four sides of ABCD. is 0.3. Find
(b) What can you say about the gradients of the opposite (a) the height of the third level,
sides of a parallelogram? (b) the angle between OA and the horizontal,
(c) the value of k,
Solution (d) the difference in the length of the two escalators.
–1 – (–2)
(a) Gradient of AB = y
2 – (–2)
1 B(k, 9)
=
4
2 – (–1)
Gradient of BC = A(20, 5)
3–2
= 3 x
O

179

06 FWS3A(Exp)_ch5.indd 179 12/16/14 12:56 PM


© Star Publishing Pte Ltd. All rights reserved.

Solution 15. In the diagram, a vertical pole OP is supported by two


(a)
B(k, 9) represents a point at the third level, that is wires, AB and AC, of equal lengths. The gradient of AB
its y-coordinate will represent the height. is
4
and the coordinates of B are (–75, 0), where the
\ the height of the third level is 9 m. 3
unit of length in the coordinate plane is centimetre. Find
(b) Draw the perpendicular, AN, from A to the x-axis. (a) the coordinates of the point A,
tan ∠AON =
5 (b) the length of AB,
20 (c) the coordinates of the point C,
∠AON = 14.0° (correct to 1 d.p.) (d) the gradient of AC.
\ the angle between OA and the horizontal is y
14.0°.
(c) Given that gradient of OB = 0.3, P
9–0
\ k–0
= 0.3 A

0.3k = 9
\ k = 30
x
(d) The difference in the length of the 2 escalators, B O C
OB – OA
= (30 – 0)2 + (9 – 0)2 – (20 – 0)2 + (5 – 0)2
Solution
= 981 – 425 (a) Let the coordinates of A be (0, a).
= 10.7 m (correct to 3 s.f.) 4
Gradient of AB =
3
a–0 4
14. Filbert Street in San Francisco, USA, is one of the steepest =
0 – (–75) 3
streets in the world. A section of the street, which is a 4
65.9 m long, descends a vertical distance of 19.8 m. Find =
75 3
(a) the horizontal distance covered by this section of a = 100
the street, \ the coordinates of A are (0, 100).
(b) the gradient of this section of the street, correct to
3 decimal places, (b) AB = (–75–0)2 + (0 – 100)2
(c) the angle of inclination of this section of the street = 125 cm
to the horizontal.
(c) The points B and C are symmetrical about the
y-axis since AB = AC (given)
Solution
\ the coordinates of C are (75, 0).
(a)
0 – 100
65.9 (d) Gradient of AC =
19.8 75 – 0
4
= –
3
x

Let x m be the horizontal distance covered. 16. Given a point P(4, –2), find two possible pairs of
Using Pythagoras’ Theorem, coordinates of a point Q such that the gradient of PQ
x + 19.8 = 65.9
2 2 2
is – 3 .
5
x=
2 2
65.9 – 19.8
Solution
= 3950.77 Let (a, b) be the coordinates of Q.
= 62.9 (correct to 3 s.f.) Gradient of PQ = –
3

\ the horizontal distance covered by this section b – (–2)


5
3
of the street is 62.9 m. \ a–4
=–
5
b+2 3
(b) Required gradient = 19.8 =–
3950.77 a–4 5
= 0.315 (correct to 3 d.p.) Some possible coordinates of Q can be obtained by:
Case 1: b + 2 = –3 and a – 4 = 5
19.8
(c) sin i = b = –5 and a=9
65.9
\ Q = (9, –5)
i = 17.5° (correct to 1 d.p.)
Case 2: b + 2 = –6 and a – 4 = 10
\ the angle of inclination of this section of the
b = 1 and a = –1
street to the horizontal is 17.5°.
\ Q = (–1, 1)
Chapter 5 Coordinate Geometry
180

06 FWS3A(Exp)_ch5.indd 180 12/16/14 12:56 PM


© Star Publishing Pte Ltd. All rights reserved.

Case 3: b + 2 = –6 and a – 4 = 10 3. Write down the equation of the line that passes through
b = –8 and a = 14 the two given points.
\ Q = (14, –8) (a) A(–3, 5), B(0, 7)
Note: –
6
is an equivalent fraction of – .
3 (b) C(–2, 11), D(13, – 4)
10 5 (c) E(4, 3), F(4, 9)
(d) P(–3, –2), Q(2, –3)
(e) R(–5, –6), S(8, –6)
Exercise 5.3 (f ) T(4, –3), V(–3, –17)
Level 1
1. For each of the following, write down the equation of Solution
the line given its gradient m and y-intercept c. 7–5
(a) Gradient of AB =
(a) m = 2, c = 3 0 – (–3)
2
(b) m=– ,c=7
2
=
3
7
(c)
2
m = –3, c = –5 Let the equation of the line be y = x + c. Since
3
4
(d) m= , c=0 B(0, 7) is on the line,
9
2
7 = (0) + c
Solution 3
(a) The equation of the line is y = 2x + 3. C =7
2
(b) The equation of the line is y = –
2
x + 7. \ the equation of the line AB is y = x + 7.
7 3
(c) The equation of the line is y = –3x – 5. (b) Gradient of CD =
–4 – 11
13 – (–2)
4
(d) The equation of the line is y = x + 0. = –1
9
i.e. y =
4
x. Let the equation of the line be y = –x + d. Since
9 C(–2, 11) is on the line,
11 = – (–2) + d
2. For each of the following, write down the equation of d =9
the line that passes through the given point A and has \ the equation of the line CD is y = –x + 9.
gradient m. 9–3
(c) Gradient of EF =
(a) m = 3, A(1, 4) 4–4
(b) m = –2, A(3, –5) 6
=
(c) m = 0, A(–3, 4) 0

(d) m=–
4
, A(–2, –7) \ gradient of EF is undefined. EF is a vertical
5 line. Since E(4, 3) is on the line, the equation
of the line EF is x = 4.
Solution
–3 – (–2)
(a)
Let the equation of the line be y = 3x + c. (d) Gradient of PQ =
2 – (–3)
Since A(1, 4) is on the line, 4 = 3(1) + c 1
c=1
=–
5
\ the equation of the line is y = 3x + 1. 1
Let the equation of the line be y = – x + k. Since
(b) Let the equation of the line be y = –2x + d. 5
Q(2, –3) is on the line,
Since A(3, –5) is on the line, –5 = –2(3) + d
1
d=1 –3 = – (2) + k
5
\ the equation of the line is y = –2x + 1. 13
k= –
(c) Let the equation of the line be y = 0x + h, 5
1 13
i.e. y = h. \ the equation of the line PQ is y = – x – .
5 5
Since A(–3, 4) is on the line, 4 = 0(–3) + h – 6 – (– 6)
h=4 (e) Gradient of RS =
8 – (–5)
\ the equation of the line is y = 4. 0
=
4 13
(d) Let the equation of the line be y = – x + k.
5 =0
Since A(–2, –7) is on the line,
4
–7 = – (–2) +k \ RS is a horizontal line.
5 Since R(–5, –6) is on the line, the equation of the
k=–
43 line RS is y = – 6.
5
4 43
\ the equation of the line is y =– x– .
5 5

181

06 FWS3A(Exp)_ch5.indd 181 12/16/14 12:56 PM


© Star Publishing Pte Ltd. All rights reserved.

–17 – (–3) Level 2


(f ) Gradient of TV = –3 − 4
5. Find the equations of the lines L1, L2, L3, L 4 and L5 in
= 2 the following diagram.
Let the equation of the line be y = 2x + h. Since
T(4, –3) is on the line y
–3 = 2(4) + h L2
4
h = –11
L1
\ the equation of the line TV is y = 2x – 11. 3
L5
L3

2
4. For each of the following equations,
(i) express it in the gradient-intercept form, 1
(ii) write down the gradient and the y-intercept of the
line. _3 _2 _1 O 1 2 3 4
x

(a) 2x + y – 3 = 0 _1
(b) x – 4y – 8 = 0
(c) 3x + 7y + 6 = 0 _2 L4
x y
(d) + =1 _3
5 3

Solution
(a) (i) 2x + y – 3 = 0
\ the gradient-intercept form is
y = –2x + 3. Solution
(ii) Gradient of the line = –2 The line L1 passes through (0, 0) and (3, 3).
y-intercept of the line = 3 Gradient of L1 =
3–0
3–0
(b) (i) x – 4y – 8 = 0 = 1
4y = x – 8
1
y-intercept of L1 = 0
\ y= x –2 \ the equation of L1 is y = x.
4
is the gradient-intercept form of the line. The line L2 passes through (0, 3) and (2, 4).
1
(ii) Gradient of the line = Gradient of L2 =
4 –3
4 2–0
y-intercept of the line = –2
1
=
(c) (i) 3x + 7y + 6 = 0 2
7y = –3x – 6 y-intercept = 3
3 6
\ y= – x – \ the equation of L2 is y =
1
x + 3.
7 7 2
is the gradient-intercept form of the line.
The line L3 passes through (0, –2) and (–2, 1).
3
(ii) Gradient of the line = – 1 – (–2)
7 Gradient of L3 =
6 –2 – 0
y-intercept of the line = – 3
7 = –
y x 2
(d) (i) + =1
3 5 y-intercept = –2
y x
=– +1 \ the equation of L3 is y = – x – 2.
3
3 5 2
3
\ y= – x+3 L4 is a horizontal line that passes through (1, –2).
5
is the gradient-intercept form of the line. The equation of L4 is y = –2.
(ii) Gradient of the line = –
3 L5 is a vertical line that passes through (4, 0).
5
The equation of L5 is x = 4.
y-intercept of the line = 3

Chapter 5 Coordinate Geometry


182

06 FWS3A(Exp)_ch5.indd 182 12/16/14 12:56 PM


© Star Publishing Pte Ltd. All rights reserved.

6. A line L cuts the x-axis at A(2, 0) and the y-axis at i.e.


6
= –2
B(0, 6). The point O is the origin. Find k
(a) the equation of L, Hence, k = –3
(b) the coordinates of C where the line L intersects the 2
(b) Gradient of the line = –
line y = –3, k
(c) the equation of the line passing through B and –2
=
having the same gradient as OC. –3
2
=
Solution 3
6–0
(a) Gradient of L = 0–2 9. A line L1 passes through A(2, –5). The gradient of L1
= –3 is equal to that of the line L2: 3x –7y + 6 = 0. Find the
Since the y-intercept of L = 6, the equation of the equation of the line L1.
line L is y = –3x + 6.
Solution
(b) Putting y = –3 into the equation of the line L,
L2: 3x – 7y + 6 = 0
–3 = –3x + 6
7y = 3x + 6
3x = 9
x=3 3
y=
x+
6
7 7
\ the coordinates of C = (3, –3)
\ gradient of L2 = .3
(c) Gradient of the required line = gradient of OC 7

=
–3 – 0 Hence, gradient of L1 = 3 .
3– 0 7
= –1 Let the equation of L1 be y = 3
x + c.
Let the equation of the line be y = –x + c. Since 7
the line passes through B(0, 6), its y-intercept = 6. Since A(2, –5) is on L1, –5 = 3
(2) + c
\ the equation of the required line is y = –x + 6. 7
41
c=–
7
3
7. The gradient of the line 6x + ky – 9 = 0 is – . Find \ the equation of L1 is y = 3
x–
41
.
4
(a) the value of k, 7 7
(b) the y-intercept of the line. 10.
4
The gradient of a line L1 is – . It cuts the y-axis at the
5
Solution same point as the line L2: 7x + 4y – 12 = 0. Find the
(a) 6x + ky – 9 = 0 equation of the line L1.
ky = – 6x + 9
6 9 Solution
y=– x+ L2: 7x + 4y – 12 = 0
k k
6 4y = –7x + 12
\ gradient = –
k 7
6 3
y=– x+3
i.e. – =– 4
k 4 \ y-intercept of L1 = y-intercept of L2
Hence, k=8 =3
4
(b) y-intercept of the line =
9 Hence, the equation of L1 is y = – x + 3.
k 5
9
= 11. Three points A(– 4, 3), B(–6, –5) and C(2, 1) are given.
8
A line AD has the same gradient as the line BC and it
cuts the y-axis at D.
8. The y-intercept of the line 2x + ky – 6 = 0 is –2. Find
(a) Find the equation of the line BC.
(a) the value of k,
(b) Find the equation of the line AD.
(b) the gradient of the line.
(c) Find the coordinates of D.
Solution
(d) What is the relationship between the lines AD and
(a) 2x + ky – 6 = 0 BC ?
ky = –2x + 6
2 6
y =– x+
k k
6
\y-intercept =
k

183

06 FWS3A(Exp)_ch5.indd 183 12/16/14 12:56 PM


© Star Publishing Pte Ltd. All rights reserved.

Solution Solution
1 – (–5)
(a) Gradient of BC = (a) Gradient of AB =
–2 – 1
2 – (–6) – 4 – (– 4)
3
= =
–3
4 0
Let the equation of the line be y =
3
x + c. Since \ gradient of AB is undefined.
4 AB is a vertical line and its equation is x = – 4.
C(2, 1) is on the line,
5 – (–2)
1=
3
(2) +c Gradient of BC =
2 – (– 4)
4
7
c= –
1 =
6
2
3 1 7
\ the equation of the line BC is y = x – Let the equation of BC be y = x + c.
4 2 6
(b) Gradient of AD = gradient of BC Since C(2, 5) is on the line,
7

=
3 5= (2) + c
4 6
3
Let the equation of the line AD be y = x + d. c=
8
4
Since A(– 4, 3) is on the line, 3
7 8

3
3 = (– 4) + d \ the equation of BC is y = x+ .
4 6 3
d=6 Gradient of CA =
5–1
3 2 – (– 4)
\ the equation of the line AD is y = x + 6. 2
4 =
(c)
Point D is where the line AD cuts the y-axis. 3
From (b),
2
Let the equation of CA be y = x + k.
3
y-intercept of the line AD = 6
Since C(2, 5) is on the line,
\ the coordinates of D are (0, 6).
2
y 5= (2) +k
(d) 3
11
k=
8 D(0, 6) 3
2 11
\ the equation of CA is y = x+ .
3 3
6 11
y= 3x (b) y-intercept of AC =
4 + 6 3
\ the required coordinates are  0, 3
2
4  .
3
A(_4, 3) (c)
2 y
C(2, 1)

x N 5 C(2, 5)
_6 _4 _2 O 2
_2
y = 43 x _ 1
2

_4

A(_4, 1) 1
B(_6, _5) _6 h
_4 _2 x
O 2

_2
From the diagram, AD // BC. B(_4, _2)

Level 3
AB = 1 – (–2)
12. The vertices of ABC are A(– 4, 1), B(– 4, –2) and = 3
C(2, 5). Find Height, CN, from C to AB = 2 – (– 4)
(a) the equations of the lines AB, BC and AC, =6
(b) the coordinates of the point where the line AC cuts 1
the y-axis, \ area of nABC = 3 3 3 6
2
(c) the area of ABC, = 9 units
2

(d) the perpendicular distance from A to BC.


Chapter 5 Coordinate Geometry
184

06 FWS3A(Exp)_ch5.indd 184 12/16/14 12:56 PM


© Star Publishing Pte Ltd. All rights reserved.

(c) AB is a horizontal line,


(d) BC = (2 + 4)2 + (5 + 2)2
i.e. gradient of AB = 0.
= 85 units Since C(–5, 4) is on the line, the equation of the
Let h units be the perpendicular distance from A required line is y = 4.
to BC. (d) Line in (b):
Consider the area of nABC, we have y = –2x + 4................. (1)
3
1
85 3 h= 9 Line in (c):
2
18
y = 4............................ (2)
\ h= Putting (1) into (2),
85
–2x + 4 = 4
= 1.95 units (correct to 3 s.f.) –2x = 0
The perpendicular distance from A to BC is x=0
1.95 units. From (2),
y=4
13. The diagram shows the points A(–3, 0), B(2, 0) and \ the coordinates of the point of intersection = (0, 4).
C(–5, 4).
14. The equation of the graph of a parabola is y = 2x – 3x – 1.
2
y
C(_5, 4)
The graph cuts the y-axis at A. The point B(k, 8), where
k . 0, is on the graph. Find
(a) the coordinates of A,
(b) the value of k,
x
(c) the equation of the line AB,
A(_3, 0) O B(2, 0) (d) the coordinates of the point at which the line AB
cuts the x-axis.

Find Solution
(a) the equation of the line BC, (a) When the graph cuts the y-axis at A,
(b) the equation of the line that has the same gradient x=0
as AC and passes through B, Putting x = 0 into the equation of the graph,
(c) the equation of the line through C and parallel to y = 2(0) – 3(0) – 1
2

AB, y = –1
(d) the coordinates of the point of intersection of the \ the coordinates of A = (0, –1)
lines in (b) and (c).
(b) Since B(k, 8) is on the graph,
8 = 2k – 3k – 1
2
Solution
2k – 3k – 9 = 0
2
4–0
(a) Gradient of BC = –5 – 2 (2k + 3)(k – 3) = 0
3
= –
4 k=– or k=3 (rejected as k . 0)
7 2
4 \ k = 3
Let the equation of the line be y = – x + c. Since
(c)
7
Coordinates of B = (3, 8)
B(2, 0) lies on the line –1 – 8
4 Gradient of AB =
0 = – (2) + c 0–3
7
= 3
8
c= Let the equation of the line be y = 3x + c. Since
7
\ the equation of the line BC is y = – x +
4 8
. A(0, –1) lies on the line,
7 7 –1 = 3(0) + c
(b) Gradient of the required line c = –1
= gradient of AC \ the equation of the line AB is y = 3x – 1.
4–0
= (d) When the line AB cuts the x-axis,
–5 – (–3)
y=0
= –2
Putting y = 0 into the equation of AB,
Let the equation of the line be y = –2x + d.
0 = 3x – 1
Since B(2, 0) lies on the line,
3x = 1
0 = –2(2) + d
1
d =4 x=
3
\ the equation of the required line is y = –2x + 4. \ the coordinates of the required point =
1 
 , 0 
3

185

06 FWS3A(Exp)_ch5.indd 185 12/16/14 12:56 PM


© Star Publishing Pte Ltd. All rights reserved.

Exercise 5.4 Solution


Level 1 (a) Gradient of PB = gradient of AB
0–6 –6 – 6
1. Three points A(–2, 1), B(4, 3) and P(3, – 4) are given. =
k –7 1–7
Show that AP = BP. –6
=2
k –7
Solution k – 7 = –3
AP = (–2 – 3)2 + (1 + 4)2 k=4
= 50 units (b) P = (4, 0)

BP = AP = (4 – 1) + (0 + 6)
2 2 2 2
(4 – 3) + (3 + 4)
= 50 units = 45 units
Hence, AP = BP PB = (7 – 4) + (6 – 0)
2 2

= 45 units
2. Three points A(– 4, 7), B(1, 5) and C(6, 3) are given. Show \ AP = PB
that
(a) A, B and C lie on a straight line.
(b) AB = BC. 4. The vertices of a quadrilateral ABCD are A(0, 5),
B(3, 3), C(8, 4) and D(2, 8). Show that the gradient of
Solution AB is equal to the gradient of DC.
(a) y
Solution
8
A(_4, 7) Gradient of AB =
5–3
0–3
2
6 = –
B(1, 5) 3
4–8
4 Gradient of DC =
8–2
C(6, 3) = –
2
2 3
\ the gradient of AB is equal to the gradient of DC.

_4 _2 x
O 2 4 6 5. The diagram represents the cross-section of a funnel
AOB. The y-axis is its line of symmetry and the unit of
the length of both axes is cm. The coordinates of B are
AB = (–4 – 1) + (7 – 5) (8, 15).
2 2

= 29 units (a) State the coordinates of A.


(b) Find the length of OA.
BC = (1 – 6) + (5 – 3) (c) Find the equations of the lines OA and OB.
2 2

= 29 units (d) What is the relationship between the gradients of


OA and OB ?
AC = (–4 – 6) + (7 – 3)
2 2

= 116 units y

AB + BC – AC = 29 + 29 – 116 A B(8, 15)


= 0
\ AB + BC = AC
Hence A, B and C lie on a straight line.
(b) From (a),
AB = BC
= 29 units O
x

3. The line segment joining A(1, –6) and B(7, 6) intersects


the x-axis at P(k, 0).
(a) Find the value of k.
(b) Show that AP = PB.

Chapter 5 Coordinate Geometry


186

06 FWS3A(Exp)_ch5.indd 186 12/16/14 12:56 PM


© Star Publishing Pte Ltd. All rights reserved.

Solution 1.5 – 0.5


Gradient of AD =
(a)
The points A and B are symmetrical about the 4 –0
y-axis. =
1
\  the coordinates of A = (–8, 15) 4
y-intercept of the line AD is at A,
(b) OA = (–8 – 0) + (15 – 0) i.e. = 0.5
2 2

= 17 cm \ the equation of AD is y =
1
x
1
+ .
15 – 0 4 2
(c) Gradient of OA =
–8 – 0 (c) Gradient of BC = gradient of AD
15
= –
8
Level 2
Since the line OA passes through the origin O, its
y-intercept = 0 7. The diagram shows nABC and nPQR whose vertices
15 are A(–6, 2), B(–1, 3), C(1, 7), P(–2, –2), Q(3, –1) and
\ the equation of OA is y = –
x. R(5, 3).
8
15 – 0 (a) Find the lengths of the sides of nABC.
Gradient of OB =
8–0 (b) Find the lengths of the sides of nPQR.
=
15 (c) Show that nABC and nPQR are congruent.
8 y
y-intercept of OB = 0 C(1, 7)
15
\ the equation of OB is y = x.
8
(d) Gradient of OA = – (gradient of OB)
R(5, 3)
6. In the diagram, AD and BC represent two handrails along B(_1, 3)
A(_ 6, 2)
a staircase. AB and DC are two vertical poles of 0.5 m
long. The unit of the scale on both axes is metre. B is x
O
the point (0, 1) and C is the point (4, 2).
(a) Find the equation of CD. Q(3, _1)
(b) Find the equations of the lines BC and AD. P(_2, _2)
(c) What is the relationship between the gradients of
BC and AD?
y
Solution
C
2 (a) AB = (–1 + 6) + (3 – 2)
2 2

1.5 = 26
D
B = 5.10 units (correct to 3 s.f.)
1
BC = (–1 – 1) + (3 – 7)
2 2
0.5 A
= 20
O 4
x
= 4.47 units (correct to 3 s.f.)

AC = (–6 – 1) + (2 – 7)
2 2

Solution = 74
(a) CD is a vertical line and it passes through C(4, 2). = 8.60 units (correct to 3 s.f.)
\ the equation of CD is x = 4.
(b) PQ = (–2 – 3) + (–2 + 1)
2 2
2–1
(b) Gradient of BC =
4 –0 = 26
1
= = 5.10 units (correct to 3 s.f.)
4
The line BC cuts the y-axis at B(0, 1), QR = (3 – 5) + (–1 – 3)
2 2

hence the y-intercept = 1


1 = 20
\ the equation of BC is y = x + 1.
4 = 4.47 units (correct to 3 s.f.)
CD = 0.5 m
\ D = (4, 2 – 0.5) PR = (–2 – 5) + (–2 – 3)
2 2

= (4, 1.5)
= 74
Similarly, BA = 0.5 m,
= 8.60 units (correct to 3 s.f.)
\ A = (0, 1 – 0.5)
= (0, 0.5)

187

06 FWS3A(Exp)_ch5.indd 187 12/16/14 12:56 PM


© Star Publishing Pte Ltd. All rights reserved.

(c) In nABC and nPQR, Putting (1) into (2),


AB = PQ (found) 3 1 1 4

x+ =– x+
BC = QR (found) 4 4 3 3
AC = PR (found)
9x + 3 = – 4x + 16
Hence, nABC ≡ nPQR (SSS) 13x = 13
x = 1
8. In the diagram, the coordinates of A, B, C and D are Putting x = 1 into (1),
A(–7, –5), B(4, 0), C(5, 4) and D(–5, 3). The lines AC and y = (1) +
3 1
BD intersect at E. 4 4
= 1
(a) Find the equations of AC and BD.
(b) Find the coordinates of E. \ the coordinates of E = (1, 1)
(c) Show that nADE and nCBE are similar.
(c) AE =
2 2
(–7 – 1) + (–5 – 1)
y
C(5, 4) = 10 units
D(_5, 3)
DE =
2 2
(–5 – 1) + (3 – 1)
E

= 40 units
x
O B(4, 0)
CE = (5 – 1) + (4 – 1)
2 2

= 5 units

BE = (4 – 1) + (0 – 1)
2 2

A(_7, _5)
= 10 units
AE 10
= =2
Solution CE 5
–5 – 4
(a) Gradient of AC = =
DE 40
= 4
=2
–7 – 5 BE 10 1
3

= AE DE
4 \ =
3 CE BE
Let the equation of the line be y = x + c. Since
4 ∠AED = ∠CEB (vertically opp. ∠s)
C(5, 4) is on the line, Hence, nADE and nCBE are similar.
4 =
3
(5) +c (ratio of corr. sides and included ∠ are equal)
4
AD 68
c=
1 Alternatively: = = 2.
4 CB 17

\ the equation of AC is y =
3
x + .
1 Hence, nADE and nCBE are similar.
4 4 (ratios of 3 pairs of corr. sides are equal)
3–0
Gradient of BD =
–5 – 4
1 9. In the diagram, A(4, 6) represents the position of a ship,

=–
3 B(28, 11) represents the position of a pier on an island.
Let the equation of the line be y = – x + d.
1 The unit of the scale on each axis is kilometre. The ship
3 steers along the line AB and reaches B after 2 hours. Find
Since B(4, 0) is on the line, (a) the equation of the line of the course of the ship,
1 (b) the distance between the ship and the pier,
0 = – (4) + d
3 (c) the speed of the ship.
4
d=
3 y
1 4
\ the equation of BD is y = – x + .
3 3
18

(b) From (a),


12
3 1
AC: y= x+ ..................... (1)
4 4 B(28, 11)
6
1 4 A(4, 6)
BD: y=– x + .................. (2)
3 3
x
O 5 10 15 20 25 30 35

Chapter 5 Coordinate Geometry


188

06 FWS3A(Exp)_ch5.indd 188 12/16/14 12:56 PM


© Star Publishing Pte Ltd. All rights reserved.

Solution 7–4
(c) Gradient of EF =
11 – 6 –1 – 0
(a) Gradient of AB =
28 – 4 = –3
=
5 Let the equation of the line be y = –3x + c.
24 Since the line EF meets the y-axis at E, the
Let the equation of the line be y =
5
x + c. Since y-intercept = 4.
24 \ the equation of EF is y = –3x + 4.
A(4, 6) is on the line,
5
6 =(4) + c Level 3
24
31
11. The vertices of OAB are the origin O, A(–15, 0) and
c= B(0, 8).
6
(a) Find the equation of the line AB.
\ the equation of the required line is (b) Find the equation of the perpendicular bisectors,
y=
5
x +
31
. (i) L1, of OA, and
24 6 (ii) L2, of OB.
(c) Suppose G is the intersecting point of L 1
(b) AB = (28 – 4) + (11 – 6)
2 2
and L2.
= 601 (i) Find the coordinates of G.
= 24.5 units (correct to 3 s.f.) (ii) Show that G lies on the line AB.
\ the distance between the ship and the pier is (d) Find the lengths of OG, AG and BG.
24.5 km. (e) What is the relationship between G and OAB?

601 y
(c) Speed of the ship =
2
= 12.3 km/h (correct to 3 s.f.) B(0, 8)

10. The diagram shows a simplified model of the cross-


x
section of a chair on a coordinate plane. The scale is A(_15, 0) O
1 unit represents 10 cm. The points A, B, C, D, E and
F are A(–1, 0), B(4, 0), C(4, 4), D(3, 4), E(0, 4) and
F(–1, 7). Find Solution
(a) the length of the leg BD, (a) Gradient of AB =
8–0
(b) the equation of the seating level EC, 0 – (–15)
8
(c) the equation of the backrest EF of the chair. =
15
y
y-intercept of AB = 8
8
8 \ the equation of the line AB is y = x + 8.
F 7 15
(b) (i) Midpoint of OA =  – 0 
15
6 ,
2
5
E D The perpendicular bisector L1 of OA is a
4 C
vertical line passing through  – 0  .
15
3 ,
2
2
1
1 \ the equation of L1 is x = –7 .
A B x 2
_5 _4 _3 _2 _ O
1_ 1 2 3 4 5
1 (ii) Midpoint of OB = (0, 4)
_
2 The perpendicular bisector L2 of OB is a
horizontal line passing through (0, 4).
Solution \ the equation of L2 is y = 4.
(a) BD =   1
2 2
(4 – 3) + (0 – 4) (c) (i) The coordinates of G are  –7 , 4  .
2
= 17 (ii) Gradient of AG =
4–0
15
= 4.12 units (correct to 3 s.f.) –
2
– (–15)
Given that 1 unit represents 10 cm, 8
=
\ the length of the leg BD = 41.2 cm 15
\ gradient of AG = gradient of AB
(b)
Since E and C have the same y-coordinate, 4, the
Hence, A, G and B lie on the same straight
line EC is a horizontal line.
line.
\ the equation of EC is y = 4.
i.e. G is on the line AB.

189

06 FWS3A(Exp)_ch5.indd 189 12/16/14 12:56 PM


© Star Publishing Pte Ltd. All rights reserved.

2 y
 15
(d) OG = –
 2
– 0  + (4 – 0)2 = 8.5 units
R A(6.2, 7.4)
2
 15

AG =  – + 15  + (4 – 0) = 8.5 units
2
2

 15
2 S

BG =  – – 0  + (4 – 8)2 = 8.5 units B(1, 4.8)
2

(e) From (d), OG = AG = BG.


T C(6, 2)
\ G is the midpoint of the side AB of nOAB and
G is equidistant from the vertices of nOAB,
i.e. G is the circumcentre of nOAB. O
x

12. The diagram below shows a map of Bermuda Triangle


RT = (7.4 – 2) 3 400
where many mysterious incidents such as sudden loss of
= 2160 km
aeroplanes have occurred. It is found between Bermuda,
RS = (7.4 – 4.8) 3 400
Florida and Puerto Rico. One unit on the map represents
= 1040 km
400 km. The vertices of the triangle are A(6.2, 7.4), B(1, 4.8)
ST = (4.8 – 2) 3 400
and C(6, 2). Find
= 1120 km
(a) the actual distance, in km, between B and C,
\ the area of the Bermuda Triangle
(b) the area of the Bermuda Triangle, in km , using
2
= area of trapezium ARTC – area of ARSB
the diagram.
– area of BSTC
1 1
y
= (2480 + 2400) 3 2160 – (2480 + 400)
2 2
1
8 Bermuda   × 1040 – (400 + 2400) 3 1120
A 2
7
= 2 204 800 km
2

6
Florida
Bermuda
Gulf of 5 B Triangle 13. A hotel will cater for a party at the cost of $30 per person
Mexico
4 plus a basic charge of $200. Let $y be the total cost of
3
catering for x persons.
(a) Copy and complete the following table.
2 C
Puerto Rico
Number of
1 Caribbean Sea 20 40 60 80 100
persons (x)
x
O 1 2 3 4 5 6 7 Total cost ($y)

(b)
Write a linear equation in terms of x and y.
(c)
Without drawing the graph of the linear equation
Solution
in (b),
(a) BC = (1 – 6) + (4.8 – 2)
2 2
(i) state the gradient of the line and interpret its
= 32.84 units meaning.
\ the actual distance between B and C (ii) state the y-intercept of the line and interpret
= 32.84 3 400 its meaning.
≈ 2292.2 Solution
= 2290 km (correct to 3 s.f.) (a) Number of
(b)
From the points A, B and C, draw the perpendiculars 20 40 60 80 100
persons (x)
AR, BS and CT respectively to the y-axis. Hence,
AR = 6.2 units Total cost ($y) 800 1400 2000 2600 3200
= 6.2 3 400 km
= 2480 km (b) y = 30x + 200
BS = 1 3 400 (c) (i) Gradient of the line = 30
= 400 km It represents the cost of catering per person.
CT = 6 3 400 (ii) y-intercept of the line = 200
= 2400 km It represents the basic charge of catering the
party.

Chapter 5 Coordinate Geometry


190

06 FWS3A(Exp)_ch5.indd 190 12/16/14 12:56 PM


© Star Publishing Pte Ltd. All rights reserved.

14. A bottle of 500 cm solution was administered to a (d)


m represents the rate at which the volume of solution
3

patient using intravenous infusion. The following table in the bottle is charging over time.
shows the volume, V cm , of the solution remaining in m = –60 means that the volume of solution decreases
3

the bottle after t hours. by 60 cm in every one hour.


3

c is the value of V when t = 0. Thus, c = 500 means


Time (t hours) 1 2 3 4 5 that there was 500 cm of solution in the bottle
3

Volume (V cm )
3
440 380 320 260 200 initially.

(a) Draw the graph of V against t. Describe the Revision Exercise 5


relationship between t and V. 1. The vertices of ABC are A(–1, 2), B(1, 5) and C(4, 3).
(b) Express the equation connecting t and V in the (a) Find the lengths of the sides AB, BC and CA.
form V = mt + c, where m and c are constants. (b) What type of triangle is ABC?
(c) Find the time when the bottle of the solution was (c) Find the perpendicular distance from B to AC.
empty. (d) Find the coordinates of the point at which the line
(d) Interpret the meanings of m and c in the relationship. AC cuts the x-axis.
Solution
Solution
(a) The diagram below shows the graph of v against t.
v
(a)
AB = (1 – (–1)2 + (5 – 2)2
= 13
500 = 3.61 units (correct to 3 s.f.)

BC = (4 – 1)2 + (3 – 5)2
= 13
400 = 3.61 units (correct to 3 s.f.)

CA = (–1 – 4) + (–7 + 3)2


2

= 26
= 5.10 units (correct to 3 s.f.)
300
(b) Since AB = BC,
\  nABC is isosceles.
AB + BC = 13 + 13
2 2

= 26
200 \ AB + BC = CA
2 2 2

∠ABC = 90° (converse of Pythagoras’
O t Theorem)
1 2 3 4 5
Hence, nABC is a right-angled isosceles triangle.
(c) Let h units be the perpendicular distance from B
All the points lie on a straight line. Hence, t and v to AC.
have a linear relationship. Considering the area of nABC,
1 1
(b) The points (1, 440) and (5, 200) are on the straight 3 h 3 AC = 3 AB 3 BC
2 2
line graph.
200 – 440 h 3 26 = 13 3 13
Gradient of the line = 13
5–1 h=
=–60 26

Let the equation of the line be v = –60(5) + c. Since = 2.55 (correct to 3 s.f.)
(5, 200) is on the line \ the required distance is 2.55 units.
200 = –60(5) + c (d) Let T(t, 0) be the point that AC cuts the x-axis.
c = 500 Gradient of AT = gradient of AC
Hence, the equation connecting t and v is 0–2 3–2
v = –60t + 500. =
t – (–1) 4 – (–1)
(c) When the bottle of the solution was empty, v = 0.
–2
=
1
\ –60t + 500 = 0 t +1 5
60t = 500 –10 = t + 1
t =8
1 t = –11
3 1 i.e. the required point is (–11, 0).
Hence, the required time is 8 hours, i.e. 8 h 20 min.
3

191

06 FWS3A(Exp)_ch5.indd 191 12/16/14 12:56 PM


© Star Publishing Pte Ltd. All rights reserved.

2. A line L passes through A(–2, 3) and its gradient is .


1 Let the equation of the line be
2 y=x+c
Find
Since B(1, 4) is on the line,
(a) the equation of L,
4=1+c
(b) the value of k if the point B(4, k) lies on L,
c=3
(c) the equation of the line through B and having the
\ the equation of AB is y = x + 3.
same gradient as OA.
(c) When the line AB cuts the x-axis, y = 0.
Solution \ 0=x+3
(a) Let the equation of L be x = –3
y=
1
x +c \ the coordinates of the required point = (–3, 0)
2
Since A(–2, 3) is on L, (d)
Gradient of the required line
1 = gradient of AB
3= (–2) +c = 1
2
c= 4 Since the line passes through the origin, y-intercept
1 = 0.
\ the equation of L is y = x + 4.
2 \ the equation of the required line is
y=x+0
(b) If B(4, k) lies on L,
i.e. y=x
1
k= (4) +4
2
\ k = 6 4. The vertices of ABC are A(–2, 4), B(–2, –3) and
C(4, 7). Find y
(c) Gradient of the required line (a) the length of AC, C(4, 7)
= gradient of OA (b) the gradient of AB,
=
3–0 (c) the equation of BC,
–2 – 0 (d) tan ∠ABC. A(_2, 4)
3
= –
2
Let the equation of the line be
3
y= – x + k O
x
2
Since the line passes through B(4, 6),
3
6=
– (4) + k B(_2, _3)
2
k = 12
Solution
\ the equation of the required line is
3 (a) AC = (–2 – 4)2 + (4 – 7)2
y = – x + 12.
2 = 45
= 6.71 units (correct to 3 s.f.)
3. Two points A(–5, –2) and B(1, 4) are given. Find
(a) the length of AB, (b)
–3 – 4
Gradient of AB =
(b) the equation of AB, –2 – (– 2)
(c) the coordinates of the point where the line segment
=
7
AB cuts the x-axis, 0

(d) the equation of the line passing through the origin \ the gradient of AB is undefined.
and having the same gradient as AB. –3 – 7
(c) Gradient of BC =
–2 – 4
5
Solution
=
3
(a) AB = (–5 – 1)2 + (–2 – 4)2 Let the equation of the line be
= 72 y=
5
x +c
3
= 8.49 units (correct to 3 s.f.)
Since C(4, 7) is on the line,
–2 – 4
(b) Gradient of AB =
–5 – 1 5
7= (4) +c
= 1 3
1
c=
3
5 1
\ the equation of BC is y = x + .
3 3

Chapter 5 Coordinate Geometry


192

06 FWS3A(Exp)_ch5.indd 192 12/16/14 12:56 PM


© Star Publishing Pte Ltd. All rights reserved.

(d) Draw the perpendicular CN from C to BA produced. 3– 2


(b) Gradient of CD =
nCNB is a right-angled triangle. –1 – 0

y
= –1
Since the line CD cuts the y-axis at C(0, 2),
the y-intercept = 2
N C(4, 7) \ the equation of the line CD is y = –x + 2.
(c)
The line AB: y = x + 4 cuts the y-axis at B and the
x-axis at A. Let the coordinates of B be (0, b) and
A(_2, 4) A be (a, 0).
Hence b=0+4
b=4
i.e. B = (0, 4)
and 0=a+4
O
x a = –4
\ A = (– 4, 0)
Length of BC = 4 – 2
= 2 units
B(_2, _3) Perpendicular distance, h, from D to BC = 1 unit
1
\ Area of nBCD = 3 BC 3 h
2
From the diagram, 1
= 2 1
the coordinates of N = (–2, 7)
3 3
2
Hence, = 1 unit
2

CN = 4 – (–2)
(d) Area of quadrilateral AOCD
= 6 units
= area of nBOA – area of nBCD
and BN = 7 – (–3)
1
= 10 units = × OA × OB – 1
2
In nCNB, 1
CN = ×4×4–1
tan ∠ABC = 2
BN = 7 units
2

6

=
10

=
3 6. In the diagram, A is the point (0, –10) and D is the point
5 (0, 6). CD is a horizontal line. The line AC cuts the x-axis
5
at B and its gradient is .
2
5. In the diagram, the line AB: y = x + 4 and the line passing (a) Write down the equations of CD and AC.
through C(0, 2) intersect at D. The x-coordinate of D (b) Find the coordinates of B and C.
is –1. Find
(c) Given that the length of AC = d units, find the
(a) the coordinates of D,
value of d.
(b) the equation of the line CD,
(d) Find the shortest distance from D to AC.
(c) the area of BCD, y
(d) the area of the quadrilateral AOCD.
y D(0, 6)
y=x+4 C

B x
O B
D 5
m=
C(0, 2) 2

A(0, _10)
_1 x
A O

Solution
(a)
Let the coordinates of D be (–1, p). Since D(–1, p) Solution
is on the line AB, (a) CD is a horizontal line and D is (0, 6).
y = x + 4, \ the equation of CD is y = 6.
p = –1 + 4 5
Since gradient of AC = and y-intercept = –10,
p=3 2
5
\ the coordinates of D are (–1, 3). the equation of the line AC is y = x – 10.
2

193

06 FWS3A(Exp)_ch5.indd 193 12/16/14 12:56 PM


© Star Publishing Pte Ltd. All rights reserved.

(b) Let the coordinates of B be (b, 0).


BG =
2
(1 – 4) + (2 – 1)
2
Since B(b, 0) is on the line AC,
0 =
5
b – 10 = 10
2 = 3.16 units (correct to 3 s.f.)
b = 4
i.e. the coordinates of B are (4, 0).
CG = (1 – 5) + (2 – 4)2
2

Let the coordinates of C be (k, 6). = 20


Since C is on the line AC,
5
= 4.47 units (correct to 3 s.f.)
6=k – 10
2
DG = (1 – (–2)2 + (2 – 3)2
32
k =
5 = 10
i.e. the coordinates of C are  6 , 6  .
2
5 = 3.16 units (correct to 3 s.f.)
(c) In right-angled nACD, 4–0 1
AC = AD + DC (Pythagoras Theorem)
2 2 2 (b) Gradient of AC = =
–5 – (–3) 2
AC = AD 2 + DC 2 2–0 1
Gradient of AG = =
2 1 – (–3) 2
d = 16 2 +  32 
 
5 Since gradient of AC = gradient of AG and A is the
\ d = 296.96 common point.
\ A, G and C lie on a straight line.
1 32
(d) Area of nACD = 3 16 3 3–1 1
2 5 (c) Gradient of BD = =–
= 51.2 units
2 –2 – 4 3
Let h units be the shortest distance from D to AC. Gradient of BG =
2–1
=–
1
1 1–4 3
Area of nACD = 3 h 3 AC Since gradient of BD = gradient of BG and B is the
2
1 common point, B, G and D lie on a straight line
51.2 = 3 h 3 296.96
2
(d) G is the point of intersection of AC and BD, and
\ h = 5.94 (correct to 3 s.f.) G bisects the line segments AC and BD.
Hence, the shortest distance from D to AC is
5.94 units. (e) ABCD is a parallelogram.

7. The vertices of a quadrilateral ABCD are A(–3, 0), 8. In the diagram, the coordinates of the points A and C
B(4, 1), C(5, 4) and D(–2, 3). The point G(1, 2) is inside are (6, 0) and (–2, 0) respectively. The gradient of the
ABCD. 2
line AB is – .
(a) Find the lengths of AG, BG, CG and DG. 3
(b) Do the points A, G and C lie on a straight line? (a) Find the equation of the line AB.
(c) Do the points B, G and D lie on a straight line? (b) Find the equation of the line BC.
(d) From the results in (a), (b) and (c), what can you (c) Determine whether ABC is a right-angled triangle.
say about G? (d) Find the area of ABC.
(e) What type of quadrilateral is ABCD? y

Solution
B
y
m=_ 2
3
6
C(5, 4) C(_2, 0)
D(_2, 3) 4 x
O A(6, 0)
G(1, 2)
2
A(_3, 0) B(4, 1)
_4 _2 x Solution
O 2 4 6
2
(a) Let the equation of the line AB be y = – x + h.
3
2
(a)
AG = (1 – (–3)2 + (2 – 0)2 Since A(6, 0) is on AB, 0= – (6) + h
3

= 20 h= 4
2
= 4.47 units (correct to 3 s.f.) \ the equation of the line AB is y = – x + 4.
3

Chapter 5 Coordinate Geometry


194

06 FWS3A(Exp)_ch5.indd 194 12/16/14 12:56 PM


© Star Publishing Pte Ltd. All rights reserved.

(b) y-intercept of the line AB = 4


(b) AC = [0 – (–2)]2 + (6 – 0)2
\ the coordinates of B are (0, 4).
4 –0 = 40
Gradient of BC =
0 – (–2) = 6.32 units (correct to 3 s.f.)
=2 6–0
y-intercept of BC = 4 (c) Gradient of BC = = –2
0–3
\ the equation of the line BC is y = 2x + 4. \ the equation of BC is y = –2x + 6.
(c) AB = (6 – 0)2 + (0 – 4)2 (d) Area of nOAC : area of nOBC
1 1
= 52 units = 3 2 3 6: 3 3 3 6
2 2
BC = 2
(–2 –0) + (0 – 4)
2
=2:3
= 20 units
CA = [6 – (–2)] 10. The number, y, of sheets of paper in the output tray of a
= 8 units printer in t minutes is given in the following table.
AB + BC = ( 52 ) + ( 52 )
2 2 2 2
t 1 2 3 4 5
= 72
AC = 8 = 64
2 2
y 25 45 65 85 105
AB + BC ≠ AC
2 2 2

\ ∠ABC ≠ 90° (a) Draw the graph of y against t, and show that t and
i.e. nABC is not a right-angled triangle. y have a linear relationship.
1 (b) Express the equation connecting t and y in the form
(d) Area of nABC = 3 AC 3 BO
2 y = mt + c, where m and c are constants.
=
1
3 8 3 4 (c) Using the equation in (b), find the number of sheets
2
in the tray when t = 7.
= 16 units
2
(d) Interpret the meanings of m and c in the relationship.
9. The diagram shows a quadratic graph y = –x + x + 6
2
Solution
which cuts the x-axis at A and B, and the y-axis at C. Find (a) The diagram below shows the graph of y against t.
(a) the coordinates of A, B and C,
y
(b) the length of AC,
(c) the equation of the line BC,
(d) the ratio of the areas of OAC and OBC. 100
y

C
y = _x2 + x + 6

50

x
A O B
t
O 1 2 3 4 5


Solution
(a) y = –x + x + 6 ............... (1)
2
The graph shows that the points lie on a straight
Putting x = 0 into (1), line. Hence, t and y have a linear relationship.
y = –6 + 0 + 6
2
(b) The points (1, 25) and (5, 105) are on the graph of
=6 the straight line.
\  the coordinates of C are (0, 6). 105 – 25
Putting y = 0 into (1), Gradient of the line =
5 –1
0 = –x + x + 6
2
= 20
x –x–6=0
2
Let the equation of the line be y = 20t + c. Since
(x + 2)(x – 3) = 0 (1, 25) is on the line,
x = –2 or x = 3 25 = 20(1) + c
\ the coordinates of A are (–2, 0). c=5
the coordinates of B are (3, 0). Hence the equation connecting t and y is
y = 20t + 5

195

06 FWS3A(Exp)_ch5.indd 195 12/16/14 12:56 PM


© Star Publishing Pte Ltd. All rights reserved.

(c) When t = 7, (c) Area of nOAB =


1
3 OA 3 OB
y = 20(7) + 5 2
= 145 =
1
3 52 3 13
i.e. there are 145 sheets in the tray when t = 7. 2
2
= 13 units
(d)
m represents the rate at which the sheets of paper
(d) From (a),
are landed in the output tray of the printer in
sheets/min, i.e. this rate is 20 sheets/min. gradient of the line AB = –
4
7
c is the value of y when t = 0. vertical change 4
Thus, c represents the initial number of sheets of i.e. =–
horizontal change 7
paper in the output tray and the initial number is
A
5 sheets.

4
Go Further
1. In the diagram, the coordinates of A and B are (– 4, 6)
7 B
and (3, 2) respectively. The line AB cuts the y-axis at C.
(a) Find the coordinates of C. Hence, tan a =
4
(b) Show that nOAB is a right-angled triangle. 7
(c) Find the area of nOAB. i.e. a =
–1  4 
tan  
(d) Find the angle that the line AB makes with the 7
positive direction of the x-axis. i = 180° – tan
–1 4
(adj. ∠s on a st. line)
 
y 7
= 150.3° (correct to 1 d.p.)
A(_ 4, 6)
Hence the angle that the line AB makes with the
positive direction of the x-axis is 150.3°.
C

B(3, 2)
2. In the diagram, the vertices A and B of nABC are
x
(–2, –3) and (4, 5) respectively. The equation of the side
O BC is 4x – 9y + 29 = 0. The gradient of the side AC is
4
– . Find
3
(a) the equation of the side AC,
Solution
(a) Let the coordinates of C be (0, k).
(b) the coordinates of C,
Gradient of BC = gradient of BA (c) the area of nABC,
k –2 6–2 (d) sin ∠BAC, expressing your answer
= –4 – 3 as a fraction.
0–3 y

k –2 4
=
–3 –7 B(4, 5)


7k – 14 = 12 4x – 9y + 29 = 0
7k = 26
5
k = 3 C
7

3 
5
\ the coordinates of C =  0, O
x
7
m=_
4
3
(b)
OA = (– 4 – 0) + (6 – 0)
2 2 2
A(–2, –3)
=52
OB = (3 – 0) + (2 – 0)
2 2 2

= 13
AB = (– 4 – 3) + (6 – 2)
2 2 2
Solution
=65 (a) Let the equation of AC be y = – x + c. Since
4
OA + OB = 52 + 13
2 2 3
A(–2, –3) is on the line,
=65
4
i.e. OA + OB = AB
2 2 2
–3 = – (–2) + c
3
Hence ∠AOB = 90° (converse of Pythagoras
17
Theorem) c = –
3
\ nOAB is a right-angled triangle. \ the equation of AC is y = – x –
4 17
.
3 3

Chapter 5 Coordinate Geometry


196

06 FWS3A(Exp)_ch5.indd 196 12/16/14 12:57 PM


© Star Publishing Pte Ltd. All rights reserved.

(b) Point C is the point of intersection of the lines (d) In nBTA,


4 17
AC: y =– – x . ....... (1) tan ∠BAT =
8
3 3
and 6
–1  4
BC: 4x – 9y + 29 = 0......................(2) \ ∠BAT = tan  
3
Putting (1) into (2),
 4 17 
In nCSA,
4x – 9  – x –  + 29 = 0
3 3 4
tan ∠CAS =
4x + 12x + 51 + 29 =0 3
16x = –80 –1  4
\  ∠CAS = tan  
x = –5 3
Putting x = –5 into (1), Note: There is no necessity to evaluate these values,
4 17
y = – (–5) – tan
–1 4
  , now as it will cause inaccuracy
3 3 3
=1 in the final answer.
\ the coordinates of C = (–5, 1)
Line SAT is a straight line.
Draw perpendiculars from A, B and C to the axes –1 4 –1 4
to complete a rectangle BRST. \ ∠BAC = 180° – tan   – tan  
3 3
y (adj. ∠s on a st. line)
Hence,
sin ∠BAC = sin 180° – tan –1  4  – tan –1  4  
R(_5, 5) 5   3  3
B(4, 5) 24
=
25

C(_5, 1)

_5 x
O 4

S(_5, _3) _3 T(4, _3)


_ _
A( 2, 3)

(c) Area of nABC


= Area of rectangle BRST – Area of nBRC
– Area of nCSA – Area of nBTA
1
= (9 3 8) –  × 4 × 9  – 1
 × 3 × 4 
2 2
1

–  × 6 × 8 
2
= 24 units
2

197

06 FWS3A(Exp)_ch5.indd 197 12/16/14 12:57 PM


© Star Publishing Pte Ltd. All rights reserved.

6 Functions and Graphs

Class Activity 1
Objective: To explore the characteristics of graphs of the power functions y = axn, where n = 0, 1, 2 and 3.
1. Using The Geometer’s Sketchpad or other graphing software, draw the graphs of the following functions and complete the
table.
Function Observe the graph and describe it

(a) y = 3x0 horizontal straight line passing through (0, 3)

(b) y = 3x straight line passing through the origin which slope upwards from left
to right

(c) y = 3x2 parabola which opens upwards with its minimum point at the origin.
The y-axis is its line of symmetry

(d) y = 3x3 curve which slopes upwards from left to right, passes through the
origin. Situated in the 1st and 3rd quadrants

(e) y = –3x0 horizontal straight line passing through (0, –3)

(f) y = –3x straight line passing through the origin which slopes downwards from
left to right

(g) y = –3x2 parabola which opens downwards with its maximum point at the origin
and has the y-axis as the line of symmetry

(h) y = –3x3 curve which slopes downwards from the 2nd quadrant to the 4th
quadrant and passes through the origin

2. Describe the graph of y = ax0, where a is a real number.


The graph of y = ax0, where a is a real number, is a horizontal straight line passing through the y-axis at the point (0, a).

3. In general, what are the properties of the graph of y = ax, where a is a real number?
The graph of y = ax, where a is a real number, is a straight sloping line passing through the origin. When a > 0, the line slopes upwards from left to
right. When a < 0, the line slopes downwards from left to right.

4. What can you say about the graph of y = ax2, where a is a real number?
The graph of y = ax2, where a is a real number, is a parabola with its turning point at the origin and has the y-axis as the line of symmetry. When

a > 0, the graph opens upwards and when a < 0, the graph opens downwards.

Chapter 6 Functions and Graphs


198

07 FWS3A(Exp)_ch6.indd 198 12/16/14 12:57 PM


© Star Publishing Pte Ltd. All rights reserved.

1
5. (a) Sketch the graphs of y = 4x3 and y = – x3 on the same axes.
2

y = 4x3
30

20

10

_3 _2 _1 x
O 1 2 3
_10

y = _ 12 x3
_20

_30

(b) Does each of the above graph possess symmetry?


1 3
Each of the graph y = 4x3 and y = – x is symmetrical about the origin.
2

(c) Describe how the coefficient of x3 affects the shape of the function y = ax3 when
(i) a . 0.
When a . 0, the graph of y = ax3 lies in the 1st and 3rd quadrants. The curves get closer to the y-axis when the value of a gets larger.

(ii) a , 0.
When a , 0, the graph of y = ax3 lies in the 2nd and 4th quadrants. The curves get closer to the y-axis when the numerical value of a

gets larger.

199

07 FWS3A(Exp)_ch6.indd 199 12/16/14 12:57 PM


© Star Publishing Pte Ltd. All rights reserved.

Class Activity 2
Objective: To explore the characteristics of graphs of the power functions y = axn, where n = –1 and –2.
1 4 –1 –5
1. (a) Using The Geometer’s Sketchpad, draw the graphs of y = , y= , y= and y =
x x x x
on the same axes.

y y

y= 1
x
y= 4
x

O x x
O

Chapter 6 Functions and Graphs


200

07 FWS3A(Exp)_ch6.indd 200 12/16/14 12:57 PM


© Star Publishing Pte Ltd. All rights reserved.

y y

y = _ 1x y = _ 5x

O x x
O

a
(b) Describe the shape of the graph of y = for a . 0.
x
a
The graphs of y = for a . 0 lies in the 1st and 3rd quadrants. The graph consists of two branches, one for x . 0 and one for x , 0.
x

When x is close to 0, the graph is close to the y-axis but does not touch it. The graph approaches the x-axis for large numerical values of x but

does not touch it. The graph is symmetrical about the origin.

a
(c) Describe the shape of the graph of y = for a , 0.
x
a
The graphs of y = for a , 0 lies in the 2nd and 4th quadrants. The graph consists of two branches, one for x . 0 and one for x , 0.
x

When x is close to 0, the graph is close to the y-axis but does not touch it. The graph approaches the x-axis for large numerical values of x but

does not touch it. The graph is symmetrical about the origin.

1 3 –1 –4
2. (a) Using The Geometer’s Sketchpad, draw the graphs of y = , y= , y= and y = on the same axes.
x2 x2 x x2

y y

1
y= 3
x2 y=
x2

x x
O O


201

07 FWS3A(Exp)_ch6.indd 201 12/16/14 12:57 PM


© Star Publishing Pte Ltd. All rights reserved.

y y

x x
O O

y = _ 12 y= _ 4
x x2


a
(b) Describe the shape of the graph of y = 2 for a . 0.
x
a
The graphs of y = for a . 0 lies in the 1st and 2nd quadrants. The graph consists of two branches, one for x . 0 and one for x , 0.
x2

When x is close to 0, the graph is close to the y-axis but does not touch it. The graph approaches the x-axis for large numerical values of x but

does not touch it. The graph is symmetrical about the y-axis.

a
(c) Describe the shape of the graph of y = 2 for a , 0.
x
a
The graphs of y = for a , 0 lies in the 3rd and 4th quadrants. The graph consists of two branches, one for x . 0 and one for x , 0.
x2

When x is close to 0, the graph is close to the y-axis but does not touch it. The graph approaches the x-axis for large numerical values of x but

does not touch it. The graph is symmetrical about the y-axis.

Chapter 6 Functions and Graphs


202

07 FWS3A(Exp)_ch6.indd 202 12/16/14 12:57 PM


© Star Publishing Pte Ltd. All rights reserved.

Class Activity 3
Objective: To explore the characteristics of exponential functions y = kax, where k is a constant and a is a positive
integer.

6
Graphs of Exponential Functions
x
4 y=5

–15 –10 –5 5 10 15

–2
x
y = –2
–4

–8

1. Using The Geometer’s Sketchpad, draw the graphs of y = 2x, y = 5x, y = 8x, y = –2x and y = –3(5x) on the same screen.

y = 2x
y=8 x

4
y = 5x
g(x) = 8x
h(x) = 5x
2
q(x) = 2x
r(x) = –3x
s(x) = –2x

-10 -5 5 10 15

-2

y = –2x

-4

y = –3(5)x

-6

-8

203

07 FWS3A(Exp)_ch6.indd 203 12/16/14 12:57 PM


© Star Publishing Pte Ltd. All rights reserved.

2. Observe the graphs of y = 2x, y = 5x and y = 8x.


(a) What are the coordinates of the common point of the graphs?
(0, 1)

(b) Explain how you can get the coordinates of the common point.
Putting x = 0 into each equation. For instance, put x = 0 into y = 2x, we have
y = 20
y=1
Hence, the point is (0, 1).

(c) Describe the general shape of these graphs.


These graphs are graphs of increasing functions,

i.e. the values of y increases as x increases. The graphs pass through (0, 1) and lies above the x-axis. They do not touch the x-axis but get

close to the x-axis when x is a large negative number.

3. What is the relationship between the graphs of y = 2x and y = –2x?


The graphs of y = 2x and y = –2x are reflections of each other about the x-axis. The graph of y = 2x is above the x-axis and that of y = –2x is

below the x-axis.

4. What is the relationship between the graphs of y = 5x and y = –3(5x)?


The graph of y = 5x is above the x-axis while the graph of y = –3(5x) is below the x-axis and it is closer to the y-axis than y = 5x.

5. For the graph of y = kax,


(a) what are the coordinates of the point where the graph cuts the y-axis?
When the graph of y = kax cuts the y-axis, x = 0. Hence, y = ka0 = k. The graph of y = kax cuts the y-axis at the point (0, k).

(b) does the graph cut the x-axis?


No, the graph of y = kax does not cut the x-axis.

(c) what is the shape of the graph?


The graph of y = kax is a smooth curve which rises rapidly for positive values of x when k > 0 and falls rapidly for positive values of x when

k < 0. It does not cut the x-axis but will get very close to the x-axis when x is a large negative number. The graph of y =kax cuts the x-axis at

(0, k).

Chapter 6 Functions and Graphs


204

07 FWS3A(Exp)_ch6.indd 204 12/16/14 12:57 PM


© Star Publishing Pte Ltd. All rights reserved.

Class Activity 4
Objective: To match and justify sketches of graphs with their respective functions.
A. y B. y C. y
(2, 32) (2, 16)
(2, 2)

x x
O O

1
x
O

D. y E. y F. y

x (1, 4)
2 O

x (1, _3)
O
_1 O x
3

G. y H. y I. y
(2, 8)
x
O
x
O
2
(1, _3)
O
x (_1, _2)

J. y K. y L. y
x
O
3 _1
x
_
( 1, 2) O
(1, _3)
x
O (1, _2)

Tasks
Work in groups to match the correct functions with the above sketches of graphs and provide appropriate reasons.
1. y = (x + 1)2 + 2 2. y = –(x – 3)(x + 1) 3. y = 2x0
4. y = –3x 5. y = –2x2 6. y = 4x3
7. y = 4x–1 8. y = –3x–2 9. y = –2x3
10. y = 4x 11. y = –3x 12. y = 2(2x)

205

07 FWS3A(Exp)_ch6.indd 205 12/16/14 12:57 PM


© Star Publishing Pte Ltd. All rights reserved.

Graphs Functions Reasons

Graph of cubic function y = ax3, a . 0


A 6
y = 4x3: when x = 2, y = 4(23) = 32

Graph of exponential function y = kax, k . 0, a . 1


B 10 Passes through (0, 1) \ k = 1
y = 4x: when x = 2, y = 42 = 16

Graph of y = ax–1, a . 0
C 7
y = 4x–1: when x = 2, y = 4
=2
2

Graph of constant function y = ax0


D 3
y = 2x0: passes through (0, 2)

Graph of y = ax–2, a , 0
E 8 –3
y = –3x–2: when x = 1, y = 2
= –3
1

Graph of quadratic function, passes through (–1, 0) and (3, 0) with maximum point at
F 2
(1, 4): y = –(x – 3)(x + 1)

Graph of exponential function y = kax, k . 0, a . 1


Passes through (0, 2)
G 12
\ k=2
y = 2(2x): when x = 2, y = 2(22) = 8

Graph of quadratic function y = ax2, a , 0


H 5
y = –2x2: when x = –1, y = –2(–1)2 = –2

Graph of linear function y = ax, a , 0


I 4
y = –3x: when x = 1, y = –3(1) = –3

Graph of quadratic function, passes through (0, 3) and minimum point is at (–1, 2):
J 1 y = (x + 1)2 + 2
When x = 0, y = (0 + 1)2 + 2 = 3

Graph of cubic function y = ax3, a , 0


K 9
y = –2x3: when x = 1, y = –2(13) = –2

Graph of exponential function y = kax, k , 0, a . 1 cuts the y-axis at (0, –1)


L 11 \ k = –1
y = –3x: when x = 1, y = –31 = –3

Chapter 6 Functions and Graphs


206

07 FWS3A(Exp)_ch6.indd 206 12/16/14 12:57 PM


© Star Publishing Pte Ltd. All rights reserved.

Class Activity 5
Objective: To explore the gradient of a curve at various points on the curve.


18

Gradient of a Curve
16

14

12

10

8 Slope AB = 12.66
6 Q
1 3 S
4
y= ·x 3·x2 + 5
2
2

4 3 2 1 1 2 3 4 5 6 7 8

B
2

A 4

P 10

12 R

Tasks
1 3
1. Using The Geometer’s Sketchpad and choosing appropriate scales, draw the curve y = x – 3x2 + 5.
2

2. Mark the points P, Q, R and S on the curve, where Q is the maximum point and R is the minimum point of the curve.

3. Mark two points A and B on the curve. Draw the line AB and find its gradient.

4. Drag the point B towards the point A to make them so close that AB becomes the tangent to the curve at A.

5. Select the line AB and use animation to show the movement of the line along the curve PQRS. Observe the variation of the
gradient of the line.

Questions
1. Based on your observation, describe the change in the gradient of the tangent to the curve at a point A as the point A moves
(a) from P to Q,
The gradient of the tangent to the curve at a point A as the point A moves from P to Q decreases in magnitude. The gradient is positive.

(b) from Q to R,
The gradient of the tangent to the curve at a point A as the point A moves from Q to R increases and then decreases in magnitude. The

gradient is negative.

(c) from R to S.
The adjacent of the tangent to the curve at a point A as the point A moves from R to S increases in magnitude and its gradient is positive.

2. What is the gradient of the tangent to the curve at


(a) the point Q?
the point Q is zero.

(b) the point R?


the point R is zero.

207

07 FWS3A(Exp)_ch6.indd 207 12/16/14 12:57 PM


© Star Publishing Pte Ltd. All rights reserved.

Class Activity 6
Objective: To examine and make sense of graphs in a variety of contexts.
Eight graphs and eight scenarios are shown below. Match each graph with a scenario it represents.
Graphs
A. B.

O O

C. D.

O O

E. F.

O O

G. H.

Scenarios
Scenario 1 has been done for you.
Volume of petrol (litres)

1. The petrol in an oil tank of a car was replenished steadily from 10 litres to 60
60 litres in 25 seconds.
Answer: Graph A

10
O 25
Time (Seconds)

Chapter 6 Functions and Graphs


208

07 FWS3A(Exp)_ch6.indd 208 12/16/14 12:58 PM


© Star Publishing Pte Ltd. All rights reserved.

2. A car decelerated steadily from 60 km/h to a stop in 20 seconds.

Answer: Graph F
60

Speed (km/h)
O 20
Time (s)

3. The mass of a newborn baby increases from 3.5 kg to 6.5 kg in


12 months. It increases more rapidly in the first few months.
6.5
Answer: Graph D

Mass (kg)
3.5

O 12
Time (months)

4. When walking on a treadmill, a man burnt his energy steadily. He


burnt 80 calories of energy in 20 minutes.
80
Energy (calories)
Answer: Graph B

O 20
Time (min)

Graph B

5. When a piece of meat was taken out from a freezer, its temperature
increased from –15 °C to 10 °C in 2 hours.

Answer: Graph G
Temperature (°C)

10

O 2
Time (h)

_15

209

07 FWS3A(Exp)_ch6.indd 209 12/16/14 12:58 PM


© Star Publishing Pte Ltd. All rights reserved.

6. A cup of water cooled down from 90 °C to 25 °C in 40 minutes.

Answer: Graph C 90

Temperature (°C)
25

O 40
Time (min)

7. The water level in a burette increases steadily from 5 cm to 20 cm


in 10 seconds, and then decreases steadily from 20 cm to 0 cm in
15 seconds.
20
Answer: Graph H

Water level (cm)


5

O 10 25
Time (s)

8. The heart rate of a runner increased from 70 beats/min to


120 beats/min in 2 minutes. The heart rate was kept at 120 beats/
min for the next 8 minutes of running. Then he stopped running 120
Heart rate (beats/min)

and the heartbeat resumed to 70 beats/min after 5 minutes.

Answer: Graph E 70

O 2 10 15
Time (min)

Chapter 6 Functions and Graphs


210

07 FWS3A(Exp)_ch6.indd 210 12/16/14 12:58 PM


© Star Publishing Pte Ltd. All rights reserved.

Try It! (b) Both the graphs of y = –x3 and y = –2x3 lie in
Section 6.1 the second and fourth quadrants, and they both
1 pass through the origin. For each value of x, the
1. (a) Draw the graphs of y = x3, y = –x3 and y = –2x3 corresponding value of y of y = –2x3 is twice that
2
for –3 < x < 3 on the same axes, using the scale of y = –x3. Hence the graph of y = –2x3 is above
of 2 cm to 1 unit on the x-axis and 1 cm to 10 units the graph of y = –x3 for x . 0 and is below the
on the y-axis. graph of y = –x3 for x , 0. The graph of y = –2x3
(b) What is the relationship between the graphs of is steeper than the graph of y = –x3.
y = –x3 and y = –2x3? 1
(c) The graph of y = x3 lies in the first and third
(c) Describe the difference between the graphs of 2
1 quadrants while the graph of y = –x3 lies in the
y = x3 and y = –x3. second and fourth quadrants. The graph of y = –x3
2
1
Solution is steeper than the graph of y = x3.
2
(a) Table of values for the functions: 12
2. Draw the graph of y = –
for –5  x  5 using a scale
x
of 1 cm to 1 unit on the x-axis and 1 cm to 5 units on
x –3 –2 –1 0 1 2 3
the y-axis.
1
y= x3 –13.5 – 4 –0.5 0 0.5 4 13.5 Solution
2
12
y=–
x
y = –x 3
27 8 1 0 –1 –8 –27
x –5 –4 –3 –2 –1 –0.5
y = –2x 3
54 16 2 0 –2 –16 –54
y 2.4 3 4 6 12 24
y

x 0.5 1 2 3 4 5
60
y = _2x3 y –24 –12 –6 –4 –3 –2.4
50
y

40 30

20
30
10
20
1 3 _6 _4 _2 O x
y= 2x 2 4 6
_ 10
10 y = _ 12
x
_ 20
_2 x
O 2 4 _ 30
_10

_20 3. Draw the graph of y = – 5


for –5  x  5 using a scale
x2
of 1 cm to 1 unit on the x-axis and 1 cm to 5 units on
_30 y = _x3
the y-axis.

_ 40 Solution
5
y=– 2
x
_50
x –5 –4 –2 –1 –0.5
_60 y –0.2 –0.31 –1.25 –5 –20

x 0.5 1 2 4 5
y –20 –5 –1.25 –0.31 –0.2

211

07 FWS3A(Exp)_ch6.indd 211 12/16/14 12:58 PM


© Star Publishing Pte Ltd. All rights reserved.

y Solution
(a) y = x3 – 4x
_6 _4 _2 O x
2 4 6 x –3 –2 –1 –0.5 0.5 1 2 3
y=_ 5
x2 y –2.89 –1.75 0 3.5 4.5 2 2.25 3.11
_ 10
y

6 y = x + 12
_ 20 x

y=3 4

_3 _2 _1 O x
1 2 3
Section 6.2 _2
4. (a) Draw the graph of the function y = x3 – 4x for
–3  x  3 using a scale of 2 cm to 1 unit on the _4
x-axis and 2 cm to 5 units on the y-axis.
(b) Use your graph to solve the equation x3 – 4x = 10, _6
giving your answer correct to 1 decimal place.

Solution
(a) y = x3 – 4x (b) (i) From the graph, the coordinates of the turning
point are (1.25, 1.9).
x –3 –2 –1 0 1 2 3 (ii) x+
1
–3 =0
x2
y –15 0 3 0 –3 0 15 1
x+ =3
x2
Draw the line y = 3 on the graph in (a).
y
From the graph, the solutions of
15 1
y = x3 _ 4x x + 2 – 3 = 0 are x = –0.5, 0.65 and
x
10 y = 10 2.9 (correct to 1 d.p.)
5
6. Two variables x and y are related by the equation
50
_3 _2 _1 O x y = 12 – 2x – .
1 2 3 x
_5 The table below shows some values of x and the
corresponding values of y, correct to 2 decimal places
_ 10
where necessary.
_ 15
x 2 3 4 5 6 7 8

y –17 –10.67 –8.5 p –8.33 –9.14 –10.25

(b) Draw the line y = 10 on the graph in (a). (a) Find the value of p.
From the graph, the solution of x3 – 4x = 10 is (b) Using a scale of 1 cm to represent 1 unit on the
x = 2.8. x-axis and 2 cm to represent 5 units on the y-axis,
draw the
1
5. (a) Draw the graph of the function y = x + for graph of y = 12 – 2x –
50
for 2  x  8.
x2 x
–3  x  3 using a scale of 2 cm to 1 unit on the (c) Use your graph to find, for 2  x  8,
x-axis and 1 cm to 1 unit on the y-axis. (i) the maximum value of y,
(b) Use your graph to find (ii) the range of values of x for which
(i) the coordinates of the turning point, 50
1 12 – 2x – > –10.
(ii) the solutions of the equation x + 2 – 3 = 0. x
x 50
(d) Solve the equation 17 + x – = 0, where x . 0,
x
by drawing a suitable straight line to the graph
in (b).

Chapter 6 Functions and Graphs


212

07 FWS3A(Exp)_ch6.indd 212 12/16/14 12:58 PM


© Star Publishing Pte Ltd. All rights reserved.

Solution The x-coordinates of the points of intersection of


50
(a) y = 12 – 2x – the graph of y = 12 – 2x –
50
and y = –5 – 3x are
x x
50
When x = 5, y = 12 – 2(5) – the solutions of the equation 17 + x –
50
= 0.
5 x
= –8
From the diagram, x = 2.6 (correct to 1 d.p.).
\ the value of p = –8.
(b) The diagram below shows the graph of Section 6.3
y = 12 – 2x –
50
. 7. Draw the graph of y = 2x for –3  x  3 by using a
x
scale of 1 cm to represent 1 unit on the x-axis and 1 cm
y to represent 2 units on the y-axis.

O x Solution
1 2 3 4 5 6 7 8
y = 2x

x –3 –2 –1 0 1 2 3
_5
y 0.125 0.25 0.5 1 2 4 8

_8 y
_10 8
y = _10 y = 2x
6

4
_15
2
y = 12 _ 2x _ 50
x y = _5 _ 3x x
_ _2 _1 O
3 1 2 3
_20

8. The number of yeast cells in a cup of water at time t


(c) (i) From the graph, the maximum value of y hours after the first observation is given by
is –8. y = 60(2t ).
50 (a) Find the number of yeast cells in the cup
(ii) 12 – 2x – > –10 (i) when it was first observed,
x
i.e. y > –10 (ii) 2 hours later.
Draw the line y = –10 on the graph in (b). (b) Draw the graph of y = 60(2t ) for 0  t  4 using
a scale of 2 cm to represent 1 hour on the t-axis
From the graph, the line y = –10 intersects
50
and 1 cm to represent 100 cells on the y-axis.
the graph of y = 12 – 2x – at x = 3.2 and (c) Estimate the time when the number of yeast cells
x
7.8. (correct to 1 d.p.) is 400.
Hence, the range of values of x for which
50 Solution
12 – 2x – > –10 is 3.2 < x < 7.8.
x (a) (i) y = 60(2t )
(d) 17 + x –
50
=0 When t = 0,
x
50 y = 60(20)
17 – 5 + x – 3x – = –5 – 3x = 60
x
12 – 2x –
50
= –5 – 3x When first observed, the number of yeast cells
x
in the cup is 60.
i.e. y = –5 – 3x
(ii) When t = 2,
Draw the line y = –5 – 3x on the graph in (b).
y = 60(22)
x 0 3 5 = 240

Two hours later, the number of yeast cells in
y –5 –14 –20
the cup is 240.

213

07 FWS3A(Exp)_ch6.indd 213 12/16/14 12:58 PM


© Star Publishing Pte Ltd. All rights reserved.

(b) y = 60(2t ) 10. An open rectangular box has a square base of side
x cm and height h cm. The volume of the box is
t 0 1 2 3 4 100 cm3.
y 60 120 240 480 960
h

y
x
x
1000 y= 60(2t)
(a) (i) Express h in terms of x.
800
Number of yeast cells

(ii) Let A cm2 be the total external surface area


400
600 of the box. Show that A = x2 + .
x
y = 400
400 (b) Using a scale of 2 cm to represent 1 cm, draw a
horizontal axis for 3  x  10.
200
Using a scale of 1 cm to represent 5 cm2, draw a
t vertical axis for 90  A  150.
O 1 2 3 4 400
Time (hours) On your axes, draw the graph of A = x2 + for
x
3  x  10.
(c) Draw the horizontal line at y = 400. The estimated (c) Use your graph to find
time is 2.7 hours. (i) the values of x for which the total external
surface area is 120 cm2,
Section 6.4 (ii) the gradient of the curve at x = 4,
9. (a)
Draw the graph of y = x2 – 1 for –3  x  3 by (iii) the dimensions of the box which has the least
using a scale of 1 cm to 1 unit on the x-axis and possible total external surface area, giving
1 cm to 2 units on the y-axis. your answers correct to 1 decimal place.
(b) Estimate the gradient of the graph at the point
where Solution
(i) x = –1, (a) (i) Volume = x2h
(ii) x = 0, = 100
(iii) x = 2. \ h=
100
x2
Solution (ii) A = x2 + 4hx
(a) y=x –1 2

= x2 + 4 3
100
3 x
x2
x –3 –2 –1 0 1 2 3 \ A = x2 +
400
x
y 8 3 0 –1 0 3 8 400
(b) A = x2 +
x
y
x 3 4 5 6 7 8 10
8 y = x2 _ 1
A 142 116 105 103 106 114 140
6

4 A
140 A = x2 + 400
x
2
130
External Surface Area (cm2)

_3 _2 _1 O x
1 2 3 A = 120
120
_2
110

–2 100
(b) (i) Gradient at (–1, 0) = = –2
1
(ii) Gradient at (0, –1) = 0 90
4
(iii)  Gradient at (2, 3) = =4 O x
1 3 4 5 6 7 8 9 10
Length of side (cm)

Chapter 6 Functions and Graphs


214

07 FWS3A(Exp)_ch6.indd 214 12/16/14 12:58 PM


© Star Publishing Pte Ltd. All rights reserved.

(c) (i) Draw the line A = 120 on the graph in (b). 12. The diagram below shows a simplified distance-time
The required values of x are x = 3.8 and graph model of Hafiz who ran a 100-metre race.
x = 8.6.
(ii) Draw the tangent to the curve at x = 4. C
100
–34
Gradient of curve at the point = = –17
2
80
(iii) From the graph, A is minimum when x = 5.8.

Distance (m)
B
100
Hence,  
h= 2
= 2.97 (correct to 3 s.f.) 60
5.8
Therefore, the required dimensions of the box 40
are 5.8 cm 3 5.8 cm 3 2.97 cm.
20

11. The following is a conversion table between miles and A


O
kilometres. 2 4 6 8 10 12
Time (seconds)

Miles (x miles) 2 4 6 8 10 (a)


How long did Hafiz take to complete the race?
Kilometres ( y km ) 3.22 6.44 9.66 12.88 16.10 (b)
State the time interval in which Hafiz’s speed is
not constant.
(a)
Draw the graph of y against x using a scale of (c) Find the speed of Hafiz
2 cm to represent 2 miles on the x-axis and 2 cm (i) during the first 10 seconds,
to represent 5 km on the y-axis. (ii) when the time is 11.2 seconds.
(b) From the graph, find the equivalent of (d) Find the average speed for the complete race.
(i) kilometres in 9 miles,
Solution
(ii) miles in 5 kilometres.
(a) Time taken by Hafiz to complete the race = 12 s
Solution (b) Hafiz’s speed is not constant during the interval
(a) 10 s to 12 s.
(c)

C
15 100
Kilometres (y km)

80 B S
Distance (m)

10
60 (10, 60)

5 40
T (8, 28)
20

0 A
2 4 6 8 10
Miles (x miles) O 2 4 6 8 10 12
Time (seconds)
(b) (i) 9 miles ≈ 14.5 km (i)
Hafiz completed 68 m in the first 10 s since
(ii) 5 km ≈ 3.2 miles the coordinates of B are (10, 68).
68 – 0
\ speed during the first 10 s =
10 – 0
= 6.8 m/s
(ii) The speed when time = 11.2 s is given by
the gradient of the curve BC at that time.
Draw the tangent ST to the curve at S where
t = 11.2. Points (8, 28) and (10, 60) are on
the tangent.
\ the required speed = gradient of ST
60 – 28
=
10 – 8
= 16 m/s
215

07 FWS3A(Exp)_ch6.indd 215 12/16/14 12:58 PM


© Star Publishing Pte Ltd. All rights reserved.

(d) Average speed for the complete race (ii) Regina arrived at P at 10.40 a.m.
100 (iii) Reading from the graph, Jacintha and Regina
=
12 met at 9.43 a.m. at a place 14.3 km from P.
= 8.33 m/s (correct to 3 s.f.)
(d) From the given diagram, Jacintha’s speed is
20 km/h between 9 a.m. and 10 a.m.,
13. The distance-time graph below represents Jacintha’s 0 km/h between 10 a.m. and 10.20 a.m.
journey between two places P and Q along a road. and 30 km/h between 10.20 a.m. and 11 a.m.
Q is 20 km from P. The diagram below shows Jacintha’s speed-time
graph.
20
Distance from P (km)

30
10 Jacintha

Speed (km/h)
O 20
9 a.m. 10 a.m. 11 a.m.
Time

(a)
Describe briefly Jacintha’s jouney as shown in the
above graph.
10
(b) Find Jacintha’s average speed for the whole journey.
(c) At 9.20 a.m., Regina started travelling from Q to
P along the same road. She cycled at a constant
speed of 15 km/h.
(i) Draw Regina’s distance-time graph on the O 9 am
given diagram. 10 am 11 am
(ii) When did Regina arrive at P? Time
(iii) When and where did Jacintha meet Regina?
(d) Draw the corresponding speed-time graph of
Jacintha’s journey.
14. The diagram below shows the speed-time graphs of two
Solution cyclists, C and D.
(a) Jacintha cycled from P to Q at a uniform speed
of 20 km/h for one hour. She then stayed at Q for
20 minutes. After that, she travelled back to P at a
8
uniform speed of 30 km/h. She reached P at 11 a.m.
(b) Jacintha’s average speed for the whole journey 6
C
Speed (m/s)

20 + 20
= = 20 km/h
2 4

(c) (i) The diagram below shows the distance-time


2 D
graph of Regina. It is formed by joining
the points A and B by a straight line. Point O
A refers to the start of Regina’s journey at 1 2 3 4 5
Time (seconds)
9.20 a.m. Point B refers to Regina’s distance
from Q after 1 hour.
(a) Find the accelerations of cyclists C and D at
time = 2 seconds.
20 A
Distance from P (km)

(b) Find the total distance travelled by cyclist C during


the first 4 seconds.
Jacintha
10 Regina


9 a.m. 10 a.m. 11 a.m.

Time
Chapter 6 Functions and Graphs
216

07 FWS3A(Exp)_ch6.indd 216 12/16/14 12:58 PM


© Star Publishing Pte Ltd. All rights reserved.

Solution The diagram below is the required speed-time graph.


(a) When time = 2 s, v
6–2
acceleration of cyclist C =
5
= 0.8 m/s2 10
(2, 9.2)

8
v = 6t _ t2

Speed (m/s)
P(0, 9.4) 6

8
4
T
Distance (m/s)

6
2
Q(4, 4.4) (0, 1.2)
4 t
O 1 2 3 4 5 6
Time (seconds)
2

(b) When t = 1,
O 1 2 3 4 5 acceleration of the car
Time (seconds) = gradient of the tangent at (t = 1)
9.2 – 1.2
=
2–0
When time = 2 s, = 4 m/s2
acceleration of cyclist D
= gradient of the tangent at T 16. The following diagram shows the graph of the parking
fee, $y, against the park duration, t hours, of a car park
9.4 – 4.4
=
0–4
= –1.25 m/s2 in an HDB estate.
y
(b) Total distance travelled by cyclist C during the first
4 seconds 3
= area of the trapezium under the graph of C
1
= 3 (2 + 5.2) 3 4
2
Parking fee ($)

= 14.4 m 2

15. The speed, v m/s, of a car at t seconds is given by


v = 6t – t2 for 0 < t < 6. 1
(a) Draw the speed-time graph of the car for 0 < t < 6
using a scale of 2 cm to represent 1 second on the
t-axis and 1 cm to represent 1 m/s on the v-axis.
(b) Estimate, from the graph, the acceleration of the O t
1 2 3
car when t = 1. Time (hours)

Solution (a) Find the parking fee when the parking duration is
(a) V = 6t – t 2 (i) 0.3 hour, (ii) 0.8 hour,
(iii) 1.5 hours.
t 0 1 2 3 4 5 6 (b) Find the equation of the graph for 2 , t < 2.5.
v 0 5 8 9 8 5 0
Solution
(a) (i) When t = 0.3, y = 0.5.
i.e. the required parking fee is $0.50.
(ii) When t = 0.8, y = 1.
i.e. the required parking fee is $1.00.
(iii) When t = 1.5, y = 1.5.
i.e. the required parking fee is $1.50.
(b) The equation of the graph is y = 2.5.

217

07 FWS3A(Exp)_ch6.indd 217 12/16/14 12:58 PM


© Star Publishing Pte Ltd. All rights reserved.

17. The given graph shows the variation of the temperature, (c) Between the time t = 1 and t = 3, the temperature
y °C, of a greenhouse against time, t hours. of the greenhouse is decreasing at a constant rate
from 23 °C to 20 °C.
y
This is represented by the straight line BC with a
negative gradient.
23
(d) The graph for 1 < t < 3 is represented by the
line BC.
Temperature (°C)

22
23 – 20
Gradient of BC =
21 1–3
= –1.5
20
Let the equation of the line BC be y = –1.5t + c.
Since C(3, 20) is on the line,
t
O 1 2 3 4 20 = –1.5(3) + c
Time (hours) c = 24.5
Hence, the equation of the graph for 1 < t < 3 is
(a) State the temperature of the greenhouse when y = –1.5t + 24.5.
(i) t = 0,
(ii) t = 4. (e) Putting t = 2.7 into the equation in (d).
(b) Find the gradient of the graph when t = 0.5 and y = –1.5(2.7) + 24.5
interpret its meaning. = 20.45
(c) What do you think is happening between the time Hence when t = 2.7, the temperature of the
t = 1 and t = 3? greenhouse is 20.45 °C.
(d) Find the equation of the graph for 1 < t < 3.
(e) Using the equation in (d), find the temperature of 18. The diagrams below show the vertical uniform cross-
the greenhouse when t = 2.7. sections of two water tanks C and D. The dimensions
are in metres. Both water tanks have the same length
Solution perpendicular to the cross-section. Water is pumped into
(a) these tanks at a constant rate.
y

23 1 1
Temperature (°C)

22
1 1

21 1 1 1 1
Tank C Tank D
20
The graph below shows the height of water from the
t bottom of one of the water tanks against time as the
O 1 2 3 4 tank is being filled.
Time (hours)

(i) When t = 0, 2
the temperature = 20 °C
(ii) When t = 4, 1.5
Height (m)

the temperature = 23 °C
1
(b) When t = 0.5,
gradient of the graph = gradient of AB 0.5
23 – 20
=
1–0 O 5 10 15 20 25 30 35
= 3 Time (min)
Since the gradient is the rate of change of y over
t, this means when t = 0.5 hours, the temperature
in the greenhouse is increasing at a constant rate
of 3 °C/hour.

Chapter 6 Functions and Graphs


218

07 FWS3A(Exp)_ch6.indd 218 12/16/14 12:58 PM


© Star Publishing Pte Ltd. All rights reserved.

Solution
(a) The graph represents the filling of tank D. For
the height of the first 1 m, the graph is a straight
line. That means the rate of increase in height is
constant. Thus, the vertical cross-sectional area of
the water tank is uniform for the first 1 m in height
from the bottom.
When the height is between 1 m and 2 m, the graph
is a curve with increasing gradient. This means the
rate of increase in the height of water increases
with the height and it is not a constant. Thus the
cross-sectional area of the water tank decreases as
the height increases.
(b)

2
Tank C
Tank D
Height (m)

1.5

0.5

O 5 10 15 20 25 30 35
Time (min)

For the height of the first 1 m, the cross-sectional


1
area of tank C  2 × 20 × 10 = 100 m 3  is half

that of tank D (20 3 10 = 200 m 3). Thus tank
C takes half the length of time taken by tank D
to fill up to the height of 1 m. Also, the area of
water surface of tank C is increasing as the height
increases for the first 1 m, hence it has decreasing
rate of increase in the height of water. Therefore
for the first 1 m in height, the graph for tank C is
a curve with decreasing gradient.
For the height between 1 m and 2 m, the
cross-sectional area of the tank C (10 3 (10 + 10)
= 200 m3) is the same as that of tank D in its first
1 m of height and is uniform as height increases.
Hence tank C takes 20 minutes to fill up the height
from 1 m to 2 m. The graph is a straight line with
a constant gradient.

219

07 FWS3A(Exp)_ch6.indd 219 12/16/14 12:58 PM


© Star Publishing Pte Ltd. All rights reserved.

Exercise 6.1 (c) y=


1
x2 and y = –3x2 are quadratic functions. Their
2
Level 1
graphs are parabolic curves with the y-axis as the
1. Sketch each of the following pairs of functions on the line of symmetry and passing through the origin.
same diagram.
3
y
(a) y = 2x0 and y = – x0
2
(b) y=
3
x and y = –2x 10 y= 1 2
2x
4
1
(c) y= x2 and y = –3x2 _3 _2 _1 x
2 O 1 2 3

Solution _10
(a) y = 2x0
=2 _20
3 y = _ 3x2
y = – x 0
2
_30
3

=–
2

These are constant functions and their graphs are


horizontal lines.
y 2. Draw the graph of each of the following functions for
–3 < x < 3 using a scale of 2 cm to 1 unit on the x-axis
and your choice of an appropriate scale on the y-axis.
2 y = 2x° 3 3
(a) y= x
2
1
(b) y= – x3
x 4
O
_
y= 3 x°
2 Solution
_
3 3
2 (a) y= x3
2

3 x –3 –2 –1 0 1 2 3
(b) y= x and y = –2x are linear functions. The graphs
4
y –40.5 –12 –1.5 0 1.5 12 40.5
are sloping straight lines passing through the origin.
y
3
y
y= 4x

4 40 3 3
y= 2x
3
2 30

_4 _2 x 20
O 2 4
_2 10

_4 x
y = _2x _3 _2 _1 O 1 2 3
_10

_20

_30

_ 40

Chapter 6 Functions and Graphs


220

07 FWS3A(Exp)_ch6.indd 220 12/16/14 12:58 PM


© Star Publishing Pte Ltd. All rights reserved.

(b) y = – x3
1 y
4

x –3 –2 –1 0 1 2 3 y= 6
x
10
y 6.75 2 0.25 0 –0.25 –2 –6.75
5

y _4 _3 _2 _1 O x
1 2 3 4
_5
8
_ 10
y = _ 41 x3
6

8
4 (b) y=–
x

2 x –4 –3 –2 –1 –0.5
y 2 2.67 4 8 16
x
_3 _2 _1 O 1 2 3
x 0.5 1 2 3 4
_2
y –16 –8 –4 –2.67 –2

_4 y

15
_6
10
_8
5

_4 _3 _2 _1 O x
1 2 3 4
_5
3. Using a scale of 1 cm to 1 unit on the x-axis and your y=_ 8
x
_ 10
choice of an appropriate scale on the y-axis, draw the
graphs of the following functions for – 4  x  4. _ 15
6
(a) y=
x
8
(b) y=–
x 6
(c) y=
(c) y= 1 x
x2
10 x –4 –3 –2 –1 –0.5
(d) y= – 2
x
y 0.06 0.11 0.25 1 4
Solution

6 x 0.5 1 2 3 4
(a) y=
x
y 4 1 0.25 0.11 0.06
x –4 –3 –2 –1 –0.5
y
y –1.5 –2 –3 –6 –12
4
1
x 0.5 1 2 3 4 3 y=
x2

y 12 6 3 2 1.5 2

1

_4 _3 _2 _1 O x
1 2 3 4

221

07 FWS3A(Exp)_ch6.indd 221 12/16/14 12:58 PM


© Star Publishing Pte Ltd. All rights reserved.

(b) y=–
10 5. Sketch each of the following pairs of functions on the
x2 same diagram.
1 3
x –4 –3 –2 –1 –0.5 (a) y = x3 and y = x
3
y –0.63 –1.11 –2.5 –10 – 40 (b) y = –x3 and y = –3x3

Solution
x 0.5 1 2 3 4
(a)
y – 40 –10 –2.5 –1.11 –0.63
y

y y = x3
30

_4 _3 _2 _1 O x
1 2 3 4
20
_ 10 1 3
y= 3x
_ 20 y = _ 10 10
x2
_ 30
_3 _2 _1 x
O 1 2 3
_ 40
_10

_20
Level 2
4. The graph of the function y = ax passes through the point _30
(6, 4).
(a) Find the value of the constant a.
(b) Sketch the graph of the function for –2  x  6.
(c) Find the gradient of the graph.

Solution (b)
(a) Since (6, 4) is a point on the graph
of y = ax, y
4 = a(6)
2 30
a =
3

2
(b) y = x 20
3

x –2 0 3 6 10

y –1.33 0 2 4
_3 _2 _1 O
x
1 2 3
y
_10
y= 2x
4 3

_20
2
y = _3x3
_2 x _30 y = x3
O 2 4 6
_2

4
(c) Gradient of the graph =
6
2
=
3

Chapter 6 Functions and Graphs


222

07 FWS3A(Exp)_ch6.indd 222 12/16/14 12:58 PM


© Star Publishing Pte Ltd. All rights reserved.

6. The graph of the function y = kx3 passes through the y


point (–2, 16). 6
(a) Find the value of the constant k. y=x
(b) Draw the graph of the function for –2  x  2 4
using a scale of 2 cm to 1 unit on the x-axis and
1 cm to 5 units on the y-axis. 2
1
y= x
Solution _6 _4 _2 O x
2 4 6
(a) Since (–2, 16) is a point on the graph of
_2
y = kx3,
16 = k(–2)3 _4
k = –2
_6
(b) y = –2x3
x –2 –1 0 1 2
y 16 2 0 –2 –16 (b) The points of intersection are (1, 1) and (–1, –1).
1
(c) The graph of y = is symmetrical about the line
y x
y = x.
15
y = _ 2x 3 8. (a) Copy the given coordinate plane and sketch the
10 graph of y = – 2 on it.
4
x
5 (b) If (k, –9) is a point on the graph, find the possible
values of k.
x y
_2 _1 O 1 2
_5
2
_ 10

_ 15
x
–2 O 2

–2
1
7. (a)
Draw the graphs of the functions y = x and y =
x
on the same axes for –5  x  5 using a scale of
1 cm to represent 1 unit on each axis. Solution
(b) State the coordinates of the intersecting points of (a) y=–
4
the two graphs. x2
1
(c) Describe the shape of the graph of y = with When x = 2, y=
–4
x 2
2

respect to the line y = x. = –1


–4
Solution When x = –2, y =
(–2) 2
(a) y=x = –1
y
x –5 0 5
y –5 0 5 2

1
y= x
x2 _2 O 2
_1
(_2, _1) (2, _1)
x –5 –4 –2 –1 –0.5 _2
y –0.2 –0.25 –0.5 –1 –2
_4
_
y= 4
x 0.5 1 2 3 4 x2

y 2 1 0.5 0.25 –2

223

07 FWS3A(Exp)_ch6.indd 223 12/16/14 12:58 PM


© Star Publishing Pte Ltd. All rights reserved.

(b) If (k, –9) is on the graph, y


4
–9 = –
k2
9=
4 15
k2

k2 =
4 10
9
y= 4
k=±
4 5 x
9
2 2 x
\ k = – or k= O 1 2 3 4 5
3 3

9. The distance, y m, that a marble covers is directly


11. The force of attraction, F, between two pieces of magnets
proportional to the time, t seconds, of rolling. The marble
rolls 6 m in 4 seconds. is given by the function F = 1002 where d is the distance
d
(a) Express y in terms of t. between the magnets, and F and d are in appropriate
(b) Sketch the graph of y against t for t > 0. units.
(a) Draw the graph of the function for 0  d  10 using
Solution
a scale of 1 cm to represent 1 unit on the d-axis
(a) y = kt, where k is a constant. and 1 cm to represent 50 units on the F-axis.
When t = 4, y = 6 (b) Find the percentage change in F when d increases
6 = k(4) from 5 units to 10 units.
3
k= (c) Describe the change in F when the value of
2
3
d approaches zero.
Hence, y= t
2
(b) y

y= 3
2t
3 Solution
(a) F= 100
d2

d 0.5 1 2 5 10
t F 400 100 25 4 1
O 2
F
10. The area of a rectangular poster is 4 m2. The dimensions
400
of the poster are x m by y m.
(a) Express y as a function of x. 300
(b) Draw the graph of the function for 0  x  5 using F = 1002
d
a scale of 2 cm to represent 1 m on the x-axis and 200
1 cm to represent 1 m on the y-axis.
100
Solution
d
(a) xy = 4 O 2 4 6 8 10
∴  y =
4
x
4 (b) When d = 5, F = 4.
(b) y= When d = 10, F = 1.
x
Decrease in F = 4 – 1 = 3.
x 0.25 0.5 1 2 3 4 3
∴ percentage decrease in F = × 100%
4
y 16 8 4 2 1 0.8 = 75%
(c) When d approaches zero, F increases to a very large

number. We say that F approaches to infinity.

Chapter 6 Functions and Graphs


224

07 FWS3A(Exp)_ch6.indd 224 12/16/14 12:58 PM


© Star Publishing Pte Ltd. All rights reserved.

12. A filter is in the form of an inverted cone. When water (c) m


is poured into the filter, the depth, d cm of water and the
mass, m g, of water have the following corresponding 400 m = 41 d 3
values.

Mass of water (g)


d 2 5 10 12 300

m 2 31.25 250 432


200

100

d
O 2 4 6 8 10 12
d
Depth of water (cm)

It is known that m = kd n, where k is a constant and n is 13. Are the functions y =


2x
and y = 2 equivalent? Explain
a positive integer. x
(a) Determine the value of n. briefly, with diagrams if necessary.
(b) Find the mass of water when the depth is 15 cm.
(c) Sketch the graph of m against d for d > 0. Solution
2x
m y =
x
= 2 when x ≠ 0
Mass of water (g)

but it is undefined at x = 0.
y = 2 is a constant function and it is defined for all real
numbers x.
2x
O d ∴ y = and y = 2 are not equivalent functions.
x
Depth of water (cm) 2x
The diagram below shows the graph of y = . It has a
x
hole at (0, 2).
Solution
(a) m = kdn y
y = 2x
Putting d = 2 and m = 2: x

2 = k(2n).....................(1)
Putting d = 10 and m = 250:
250 = k(10n)...................(2)
250 k (10 n )
(2) ÷ (1): = n
2 k (2 ) x
125 = 5n –2 –1 O 1 2 3
5n = 53
\ n =3
(b) Hence, m = kd 3 Exercise 6.2
Putting d = 2 and m = 2:
2 = k(2)3 Level 1
1 1. Draw the graphs of the following functions for
k=
4 –3  x  3.
\ m =
1 3
d (a) y = x + 2
4 (b) y = x2 – 4
When d = 15, (c) y = x3 + 5
1
m = (15)3 (d) y=3–
1
4 x
= 843.75
\ the required mass of water is 843.75 g.

225

07 FWS3A(Exp)_ch6.indd 225 12/16/14 12:58 PM


© Star Publishing Pte Ltd. All rights reserved.

1
Solution (d) y=3–
(a) y=x+2 x

x –3 –2 –1 –0.5 0.5 1 2 3
x –3 0 3
y 3.33 3.5 4 5 1 2 2.5 2.67
y –1 2 5

y
y 5

6
y=x+2 4
4 3
2
2 y = 3 _ 1x
_3 _2 _1 O x
1 2 3 1
_2
_3 _2 _1 O x
1 2 3


2. Draw the graphs of the following functions for –3  x  3.
(b) y = x2 – 4 (a) y = 2x – x2 (b) y = x3 + x
(c) y = 4x – x3 (d) y = x2 + 1
x –3 –2 –1 0 1 2 3 x2
Solution
y 5 0 –3 –4 –3 0 5
(a) y = 2x – x2

y x –3 –2 –1 0 1 2 3
6 y –15 –8 –3 0 1 0 –3
y = x2 _ 4
4 y
2 5

_3 _2 _1 O x x
1 2 3 _3 _2 _1 O 1 2 3
_2 _5

_4 _ 10

_ 15
y = 2x _ x 2

(c) y = x3 + 5
(b) y = x3 + x
x –3 –2 –1 0 1 2 3
x –3 –2 –1 0 1 2 3
y –22 –3 4 5 6 13 32
y –30 –10 –2 0 2 10 30

y y
y = x3 + 5 30
30 y = x3 + x

20 20

10 10

x _3 _2 _1 O x
_3 _2 _1 O 1 2 3 1 2 3
_ 10
_ 10

_ 20 _ 20

_ 30

Chapter 6 Functions and Graphs
226

07 FWS3A(Exp)_ch6.indd 226 12/16/14 12:58 PM


© Star Publishing Pte Ltd. All rights reserved.

(c) y = 4x – x3 y

15
x –3 –2 –1 0 1 2 3 y = 2x 3 _ x + 2

y 15 0 –3 0 3 0 –15 10

5
y y=3
15 _3 _2 _1 O x
1 2 3
_5
10
_ 10
5 y = 4x _ x 3

_3 _2 _1 O x
1 2 3 (b) Draw the line y = 3 on the graph in (a).
_5 Hence, the solution of 2x3 – x + 2 = 3 is x = 1.
_ 10
4. (a) Draw the graph of y = –x3 + 3x + 2 for –2  x  3
_ 15 using a scale of 2 cm to 1 unit on the x-axis and
2 cm to 5 units on the y-axis.
(b) Find the turning points of the graph.
(d) y=x + 2 1
x2 Solution
(a) y = 2x3 – x + 2
x –3 –2 –1 –0.5 –0.3 0.3
x –2 –1 0 1 2 3
y 9.11 4.25 2 4.25 11.02 11.02
y 4 0 2 4 0 –16
x 0.5 1 2 3
y
y 4.25 2 4.25 9.11 5 y = _ x 3 + 3x + 2

y _3 _2 _1 O x
1 2 3
_5

15 _ 10
y = x2 + 1
x2
10 _ 15

5

x (b) The maximum point of the graph is (1, 4).
_3 _2 _1 O 1 2 3 The minimum point of the graph is (–1, 0).

Level 2 5. (a)
Draw the graph of y = x3 + 4x2 + 4x for –3  x  1
using a scale of 2 cm to 1 unit on the x-axis and 1
3. (a) Draw the graph of y = 2x3 – x + 2 for –2  x  2
cm to 1 unit on the y-axis.
using a scale of 2 cm to 1 unit on the x-axis and
(b) Use your graph to solve the equation
2 cm to 5 units on the y-axis.
x3 + 4x2 + 4x = –1.
(b) Use your graph to solve the equation 2x3 – x + 2 = 3.
Solution
Solution
(a) y = x3 + 4x2 + 4x
(a) y = 2x – x + 2
3

x –3 –2 –1 0 0.5 1
x –2 –1 0 1 2
y –3 0 –1 0 3.125 9
y –12 1 2 3 16

227

07 FWS3A(Exp)_ch6.indd 227 12/16/14 12:58 PM


© Star Publishing Pte Ltd. All rights reserved.

y y

10 4

8
y = x3 + 4x2 + 4x 2
6
y=x

4
_2 _1 x
O 1 2
2

_3 _2 _1 x
O 1 2 _2
y = _1
_2 y = x3 _ 2x _ 1
x2


_4
_4

(b) Draw the line y = –1 on the graph in (a). _6


The solutions of the equation
x3 + 4x2 + 4x = –1
are x = –2.6, x = –1 and x = – 0.4.

6. (a) Draw the graph of y = x3 – 2x – 1


for –2  x  2
x2 (b) The turning point on the graph is (–1, 0).
using a scale of 2 cm to 0.5 unit on the x-axis and (c) Draw the line y = x on the graph in (a).
1 cm to 1 unit on the y-axis.
The solutions of the equation
(b) Find the coordinates of the turning point of the
graph. x3 – 2x – 12 = x
x
(c) Use your graph to solve the equation are x = –1.7, x = – 0.75 and x = 1.8.
1
x3 – 2x – = x.
x2
1
7. (a) Draw the graph of y = x + 3 – for
Solution x
–5  x  4 using a scale of 1 cm to 1 unit on each
(a) x3 – 2x – 1
axis.
x2
(b) Use your graph to solve the equation
1
x –2 –1.5 –1 –0.5 –0.4 x + 1 – = 0.
x
y –4.25 –0.819 0 –3.125 –5.51 Solution
1
(a) y=x+3–
x
x 0.4 0.5 1 1.5 2
y –6.99 –4.88 –2 –0.07 3.75 x –5 –4 –3 –2 –1
y –1.8 –0.75 0.33 1.5 3


x –0.5 0.5 1 2 4
y 4.5 1.5 3 4.5 6.75

Chapter 6 Functions and Graphs


228

07 FWS3A(Exp)_ch6.indd 228 12/16/14 12:58 PM


© Star Publishing Pte Ltd. All rights reserved.

y (b) y=– x+4 y


1
2
10

Concentration of medicine (units)


y = _0.2t3 + 5t

y=6
6

t
O 1 2 3 4 5
Time (hours)

6
y=x x 0 2 4
4
y 4 3 2
2
1
y= 1
x
Draw the line y = – x + 4 on the graph in (a).
_6 _4 _2 O x 2
2 4 6
2 1
_2 (c) The solutions of x + = – x + 4 are the
x 2
_4 x-coordinates of the points of intersection of the
two graphs.
_6 The solutions are x ≈ 0.7 and x = 2.

Level 3
1
(b) x+1– =0 9. The function
x
Adding 2 to both sides, y = – 0.2t3 + 5t for 0  t  5
1 gives the approximate concentration y units of a medicine
x+3– =2
x found in a person’s blood t hours after taking a pill
Draw the line y = 2 on the graph in (a). containing the medicine.
The solutions of x + 1 –
1
=0 (a) Draw the graph of y = –0.2t3 + 5t for 0  t  5
x by using a scale of 2 cm to represent 1 hour on the
are x = –1.6 and x = 0.6.
t-axis and 2 cm to represent 2 units on the y-axis.
2 (b) Estimate from the graph,
8.
1
(a) Draw the graph of y = x + for  x  5 (i) the maximum concentration of the medicine
x 2
using a scale of 2 cm to 1 unit on each axis. and the corresponding time, for 0  t  5,
(b)
1
Draw the line y = – x + 4 on the same diagram (ii) the times at which the concentration is 6 units.
2
in (a).
Solution
(c) Use the graphs to solve the equation
(a) y = – 0.2t3 + 5t
2 1
x + =– x + 4.
x 2
t 0 1 2 3 4 5
Solution
2 y 0 4.8 8.4 9.6 7.2 0
(a) y=x+
x
y
x 0.5 1 1.5 2 3 4 5
y 4.5 3 2.83 3 3.67 4.5 5.4 10
y = _0.2t3 + 5t
Concentration of medicine (units)


y
8

6 y=6
6
2
y=x+ x
5
4

4
2

3
y = _ 12 x + 4
t
O 1 2 3 4 5
Time (hours)
2
The maximum concentration is 9.6 units when (b) (i)
1
y

10
Concentration of medicine (units)

y = _0.2t3 + 5t

the time is 2.9 hours after taking the pill.


y=6
6

t
O 1 2 3 4 5
Time (hours)

Note: y = 9.6222 when t = 2.9.


x (ii) Draw the line y = 6 on the graph in (b).
O 1 2 3 4 5
The required times are 1.3 hours and
4.2 hours after taking the pill.

229

07 FWS3A(Exp)_ch6.indd 229 12/16/14 12:58 PM


© Star Publishing Pte Ltd. All rights reserved.

1
10. (a) Draw the graph of the function y = 4 – 2 for Solution
x 1  20 
–3  x  3 using a scale of 2 cm to 1 unit on each axis. (a) y = 30 – x2 –
5  x 
(b) The cross-section of a bench is in the shape bounded
1  20 

1
by the graph of y = 4 – 2 , the x- and y-axes, and When x = 5, y =  30 – 52 –
x 5  5 
the lines x = 3 and y = 4. Shade the region on your = 0.2
graph in (a) which shows the required cross-section. \ the value of p is 0.2.
(b)
Solution
1 y
(a) y=4–
x2
3
x –3 –2 –1 –0.5 –0.4
y 3.89 3.75 3 0 –2.25 2

y=1
x 0.4 0.5 1 2 3 1

y –2.25 0 3 3.75 3.89


x
O 1 2 3 4 5 6
y
y=4 _
4 1

y = 15 (30 _ x2 _ 20
x )
3 x=3 _2

2
y = 4 _ 12
x
1
(c) (i) Draw the line y = 1 on the graph in (b).
x
_3 _2 _1 O 1 2 3 The solutions of the equation
_1
1 
– 52 –
20 
= 1 are x = 0.8 and x = 4.5.
 30 5 
5 
_2 (ii) The greatest value of y is 3.2 and the
corresponding value of x is 2.15.

(b) The shaded region in the above diagram is the
required cross-section. 12. T he d iag r a m shows t he g r aph of a f u nct ion
y = ax3 + bx2 + cx, where a, b and c are constants.
Give a possible equation such that its graph has a shape
11. Two variables x and y are connected by the equation similar to that shown in the diagram.
1  20 
y= 30 – x2 – . y
5  x  y = ax3 + bx2 + cx
Some corresponding values of x and y, correct to
2 decimal places where necessary, are given in the x
O
following table.
x 0.5 1 2 3 4 5 6
y –2.05 1.8 3.2 2.87 1.8 p –1.87 Solution
Equation of the form y = ax(x – h)(x – k), where a, h
(a)
Calculate the value of p. and k are positive constants, have graphs of this shape.
(b)
Using a scale of 2 cm to represent 1 unit on each A possible equation is y = x(x – 1)(x – 2). Its sketch is
axis, plot the points given in the table and join them as shown below.
with a smooth curve on a coordinate plane.
(c) Using your graph, for 0  x  6 find
y = x(x – 1)(x – 2)
(i) the values of x such that
1  20  (0.4, 0.384)
30 – x2 – =1
5  x 
x
(ii) the greatest value of y and its corresponding O 1 2
value of x.
(1.6, –0.384)

Chapter 6 Functions and Graphs


230

07 FWS3A(Exp)_ch6.indd 230 12/16/14 12:58 PM


© Star Publishing Pte Ltd. All rights reserved.

13. The variables x and y are connected by the equation (c) Draw the line y = 1 on the graph in (a). The solutions
1 1
y= x(x2 – 10). Some corresponding values of x and y of the equation x(x 2 – 10) = 1 are x = –2.8,
6 6
are given in the following table. x = – 0.6 and x = 3.4.
x –3 –2 –1 0 1 2 3 4 (d) (i) 2x + y = 1
y = –2x + 1
y 0.5 2 1.5 0 –1.5 –2 –0.5 p
x –1 0 1
(a) Find the value of p.
(b) Using a scale of 2 cm to 1 unit on each axis, plot y 3 1 –1
the points in the table and join them with a smooth
curve for –3  x  4. (ii) The two graphs intersect at x = 1.45.
(c) Use the graph to find the solutions of the equation (iii) The value found in (ii) is a solution of the
1
x(x2 – 10) = 1 for –3  x  4. equation.
6 1
(d) (i)
On the same axes, draw the graph of x(x2 – 10) = –2x + 1
6
2x + y = 1 for –2  x  2. x3 – 10x = –12x + 6
(ii) Write down the x-coordinate of the point
x + 2x – 6 = 0
3

where the two graphs intersect. Comparing with x3 + bx2 + cx + d = 0, we


(iii) The x-coordinate in (ii) is a solution of the have b = 0, c = 2 and d = –6.
equation x3 + bx2 + cx + d = 0, where b, c
and d are constants. Find the values of b, c
and d. Execise 6.3
Level 1
Solution
1 1. (a) Copy and complete the following table for y = 5x.
(a) y= x(x2 – 10)
6
When x = 4, x –2 –1 0 1 2 3
y =
1
3 4(42 – 10) y
6
=4 (b) Draw the graph of y = 5x for –2  x  3 using a
\ the value of p is 4. scale of 2 cm to 1 unit on the x-axis and 2 cm to
(b) 25 units on the y-axis.
y Solution
(a) y = 5x
5
x –2 –1 0 1 2 3
2x + y = 1
y 0.04 0.2 1 5 25 125
4 2_
y= 1
6 x(x 10)
(b)
3 y
125
y = 5x
2
100
y=1 75
1
50
_3 _2 _1 x
O 1 2 3 4 25
_1 x
_2 _1 O 1 2 3
_2

_3

231

07 FWS3A(Exp)_ch6.indd 231 12/16/14 12:58 PM


© Star Publishing Pte Ltd. All rights reserved.

2. (a) Copy and complete the following table for y = –2x. Level 2
4. (a) Draw the graph of y = 2(3x) for –2  x  2 using
x –2 –1 0 1 2 3
a scale of 2 cm to 0.5 unit on the x-axis and 2 cm
y to 5 units on the y-axis.
(b) Use your graph to find the value of x when
(b) Draw the graph of y = – 2x for –2  x  3 using a 2(3x) = 10.
scale of 2 cm to 1 unit on the x-axis and 1 cm to
1 unit on the y-axis. Solution
(a) y = 2(3x)
Solution
(a) y = –2x x –2 –1 0 1 2
y 0.22 0.67 2 6 18
x –2 –1 0 1 2 3
y –0.25 –0.5 –1 –2 –4 –8 y

20
y y = 2(3 x)
(b)
15
_2 _1 O x
1 2 3 y = 10
_2 10
y = _ 2x
_4 5

_6 _ 2 –1.5 _ 1 –0.5 O x
0.5 1 1.5 2
_8
(b) Draw the line y = 10 on the graph in (a). The
solution of 2(3x) = 10 is x = 1.45.
3. (a) Draw the graph of y = 10x for –1  x  1 using
a scale of 2 cm to 0.5 unit on the x-axis and 1 cm 5. (a) Draw the graph of y = –5(4x) for –2  x  2 using
to 1 unit on the y-axis. a scale of 2 cm to 0.5 unit on the x-axis and 1 cm
(b) If 10x = 8, find the value of x from the graph. to 10 units on the y-axis.
(b) Use your graph to find the value of x when
Solution –5(4x) = –60.
(a) y = 10x
Solution
x –1 –0.5 0 0.5 0.8 1
(a) y = –5(4x)
y 0.1 0.32 0 3.16 6.31 10
x –2 –1 0 1 2
y y –0.3 –1.3 –5 –20 –80

y = 10x y
10

_ 2 –1.5 _ 1 –0.5 O x
y=8 0.5 1 1.5 2
8
_ 20 y = _ 5(4 x)

6 _ 40

_ 60 y = _ 60
4
_ 80

2

x (b) Draw the line y = –60 on the graph in (a).


_1 _0.5 O 0.5 1 The solution of –5(4x) = – 60 is x = 1.8.

(b) Draw the line y = 8 on the graph in (a).


The solution of 10x = 8 is x = 0.9.

Chapter 6 Functions and Graphs


232

07 FWS3A(Exp)_ch6.indd 232 12/16/14 12:58 PM


© Star Publishing Pte Ltd. All rights reserved.

6. (a) Draw the graph of y = –3(5x) for –1.5  x  1 using Solution


a scale of 2 cm to 0.5 unit on the x-axis and 2 cm (a) y = 30(4t)
to 5 units on the y-axis. (i) When t = 0, y = 30(40)
(b) Draw the graph of y = 2x – 1 on the same diagram = 30
in (a). \ the population of frogs when t = 0 is 30.
(c) Hence use the graphs to solve the equation (ii) When t = 2, y = 30(42)
2x – 1 + 3(5x) = 0 = 480
\ the population of frogs when t = 2 is 480.
Soluton
(a) y = –3(5x) (b) y = 30(4t)

x –1.5 –1 –0.5 0 0.5 1 t 0 1 2 2.5 3 3.5 4

y –0.268 –0.6 –1.34 –3 –6.71 –15 y 30 120 480 960 1920 3840 7680

y y

x 8000 y = 30(4t)
_ 1.5 _1 _0.5 O 0.5 1

y = 2x _ 1 _5 6000

Population of frogs
_ 10 4000

y = _3(5x)
_15 2000

y = 1000

(b) y = 2x – 1
t
x –1 0 1 O 1 2 3 4
Number of years
y –3 –1 1

Draw the line y = 2x – 1 on the graph in (a). (c) Draw the line y = 1000 on the graph in (b). Hence,
(c) The solution of the equation 2x – 1 + 3(5x) = 0 is the population of frogs is 1000 after 2.55 years.
x = – 0.35.
8.
The asset, y (in thousand dollars), of a company after
Level 3 t years in business is given by the function y = 50(3t )
7. The population, y, of frogs in a river can be modelled for 0  t  3.
by the function (a) Find the asset of the company
y = 30(4t), (i) initially,
where t is the number of years after the first observation. (ii) after 3 years in business.
(a) Find the population of frogs when (b) Draw the graph of y = 50(3t ) for 0  t  3 using
(i) t = 0, (ii) t = 2. a scale of 2 cm to 0.5 unit on the t-axis and 2 cm
(b) Draw the graph of y = 30(4t) for 0  t  4 using to 250 units on the y-axis.
a scale of 2 cm to represent 1 year on the t-axis (c) Estimate from the graph the time when the asset
and 2 cm to represent 2000 frogs on the y-axis. would just reach $1 million.
(c) Estimate from the graph the time when the Solution
population of frogs is 1000. (a) (i) y = 50(3t )
When x = 0, y = 50(30)
= 50
∴ the initial asset of the company is $50 000.

233

07 FWS3A(Exp)_ch6.indd 233 12/16/14 12:58 PM


© Star Publishing Pte Ltd. All rights reserved.

(ii) When x = 3, y = 50(33) y


= 1350
∴ the asset after 3 years is $1 350 000. 3000
y = 3(2t)
(b) y = 50(3t )
t 0 1 2 3 2.5
y 50 150 450 1540 779 2500

y
2000

Number of neutrons
1500

1250
Asset (thousand $)

y = 1000 1500
1000

750
y = 50(3 t)
500 1000

250

O t
0.5 1 1.5 2 2.5 3 500 y = 400
Number of years

(c) Draw the line y = 1000 on the graph in (b).
Hence, the asset just reaches $1 million after
2.7 years. t
O 2 4 6 8 10
Time (milliseconds)
9. In a nuclear fission process, the number of neutrons, (c) Draw the line y = 400 on the graph in (b). From
y, released in t milliseconds is given by the function the graph, the number of neutrons produced is 400
y = 3(2t). in 7.1 milliseconds.
(a) Find t he number of neut rons released in
10 milliseconds. 10. (a) Draw the graphs of the exponential function y = 2x
(b) Draw the graph of y = 3(2t) for 0  t  10 using and the quadratic function y = x2 on the same axes
a scale of 1 cm to 1 unit on the t-axis and 2 cm to for –2  x  5 using a scale of 2 cm to 1 unit on
500 units on the y-axis. the x-axis and 2 cm to 5 units on the y-axis.
(c) Estimate from the graph the time when the number (b) Use the graphs to solve the equation 2x = x2.
of neutrons produced is 400. (c) Compare the shapes of the two graphs.
Solution
Solution
(a) y = 3(2t) (a) x –2 –1 0 1 2 3 4 5
When t = 10, y = 3(210) y = 2x 0.25 0.5 1 2 4 8 16 32
= 3072
The number of neutrons released in 10 milliseconds y=x 2
4 1 0 1 4 9 16 25
is 3072. y
y = 2x
(b) y = 3(2t)
30
t 0 1 2 3 4 5
25
y 3 6 12 24 48 96 y = x2
20

t 6 7 8 9 10 15
y 192 384 768 1536 3072
10

x
–2 –1 O 1 2 3 4 5
Chapter 6 Functions and Graphs
234

07 FWS3A(Exp)_ch6.indd 234 12/16/14 12:58 PM


© Star Publishing Pte Ltd. All rights reserved.

(b) The solutions of 2x = x2 are the x-coordinates of (d) y


the points of intersection of the graphs y = 2x and
3
y = x2.
∴ the solutions are x = – 0.8, x = 2 and x = 4.
A 2

(c) 1. Both graphs are above the x-axis; y = x just


2

1
touches the x-axis at one point only.
2. The graph of y = x2 is symmetrical about the
x
y-axis and has a minimum point at (0, 0). _2 _1 O 1
3. The graph of y = 2x is increasing and has
neither minimum nor maximum point. It has
the horizontal asymptote y = 0. Solution
4. The graph of y = 2x is above the graph of (a) y
y = x2 when – 0.8  x  2 and x  4.
3

2 A
Execise 6.4
1
Level 1
1. By drawing a tangent, estimate the gradient of each of O
x
1 2 3
the following curves at the point A.
(a) y
1.4
3
Gradient at A =
1
= 1.4
2 A

1 (b) y

x
3
O
1 2 3
2 A

1
(b) y
x
O 1 2 3
3

2 A –2
Gradient at A =
1
1
= –2
x
O 1 2 3
(c) y

3
(c) y A
2
3
1
A
2
_2 _1 O x
1
1

x Gradient at A = 0
_2 _1 O 1

235

07 FWS3A(Exp)_ch6.indd 235 12/16/14 12:58 PM


© Star Publishing Pte Ltd. All rights reserved.

(d) y y

3 y = x3
8

A 2
6
1

x 4
_2 _1 O 1
2 (0, 2)
–2
Gradient at A =
1
= –2 _2 _1 x
O 1 2
_2
2. (a)
Draw the graph of y = 2x2 + 1 for –3  x  3 using
a scale of 2 cm to 1 unit on the x-axis and 2 cm to (_2, _4) _4
5 units on the y-axis.
(b) By drawing a tangent, find the gradient of the curve
at the point _6
(i) A(0, 1),
(ii) B(2, 9). _8

Solution
(a) y = 2x + 1 2

x 3 –2 –1 0 1 2 3 (b) (i) The tangent at (0, 0) is the x-axis.


y 19 9 3 1 3 9 19 Gradient of the curve at x = 0 is 0.
(ii) The tangent at x = –1 passes through the
y points (0, 2) and (–2, – 4).
–4 – 2
20 Gradient of the curve at x = –1 is = 3.
–2 – 0
y = 2x 2 + 1
15 2
4.
1
(a) Draw the graph of y = for  x  4 using
x 2
10 B(2, 9) a scale of 2 cm to 1 unit on both axes.
5 (b) By drawing a tangent, find the gradient of the curve
A(0, 1) at the point A(2, 1).
_3 _2 _1 O x
1 2 3 Solution
2
(a) y=
x
(b) (i) Gradient of the curve at A(0, 1) = 0
(ii) Gradient of the curve at B(2, 9) =
8 x 0.5 1 2 3 4
1
=8 y 4 2 1 0.67 0.5

y
3. (a)
Draw the graph of y = x3 for –2  x  2 using
4
a scale of 2 cm to 1 unit on the x-axis and 2
y=
1 cm to 2 units on the y-axis. 3
x

(b) By drawing a tangent, find the gradient of the curve


at the point where 2
(i) x = 0,
(ii) x = –1. 1

Solution x
O 1 2 3 4
(a) y=x 3

x –2 –1 0 1 2 (b) Gradient of the curve at (2, 1) =


–1
2
y –8 –1 0 1 8 = –
1
2

Chapter 6 Functions and Graphs


236

07 FWS3A(Exp)_ch6.indd 236 12/16/14 12:58 PM


© Star Publishing Pte Ltd. All rights reserved.

Level 2 Solution
5. (a) Draw the graph of y = x2 + x for –3  x  2 using (a) y = 1 – x3
the scale of 2 cm to 1 unit on the x-axis and 1 cm x –2.5 –2 –1 0 1 2 2.5
to 1 unit on the y-axis.
(b) Find the minimum point of the graph. y 16.6 9 2 1 0 –7 –14.6

(c) Estimate the gradient of the graph at the minimum
point.
y
Solution y = 1 _ x3
15
(a) y = x2 + x
10
x –3 –2 –1 0 1 2
y 6 2 0 0 2 6 5

B(_ 1,2) A(0,1)
_3 _2 _1 O x
1 2 3
y _5
y = x2 + x 6
_ 10
4
_ 15
2

_3 _2 _1 O x
1 2
( 0.5, 0.25)_ 2
_ _ (b) (i) At A(0, 1). the tangent is horizontal.
∴ gradient of the curve at A(0, 1) = 0.

(ii) The tangent at B(–1, 2) meets the curve again
at (2, –7).
(b) The graph cuts the x-axis at (–1, 0) and (0, 0). ∴ gradient of the curve at B(–1, 2)
–7 – 2
∴ the minimum point is at x =
1
(–1 + 0) =
2+1
2
= –3
= – 0.5.
When x = – 0.5 (c) The required point of intersection is (2, –7).
y = (– 0.5)2 + (– 0.5)
= – 0.25 7. (a) Draw the graph of the curve y = x 2 +
1
for
∴ the minimum point is (– 0.5, – 0.25). 1
x2
 x  3 using a scale of 2 cm to 0.5 unit
(c) The tangent line at the minimum point is horizontal. 2
∴ gradient at the minimum point = 0. on the x-axis and 2 cm to 2 units on the y-axis.
(b) Find the minimum point of the curve.
(c) Estimate the gradient of the curve at
6. (a)
Draw the graph of the curve y = 1 – x 3 for (i) the minimum point,
–2.5  x  2.5 using a scale of 2 cm to 1 unit on (ii) the point where x = 2.
the x-axis and 2 cm to 5 units on the y-axis. (d) Use your graph to solve the equation
(b) By drawing a tangent, find the gradient of the curve 1
at the point x2 + = 3.
x2
(i) A(0, 1),
Solution
(ii) B(–1, 2).
1
(c) The tangent to the curve at B meets the curve at (a) y = x2 +
x2
another point. Find the coordinates of that point.
x 0.5 1 1.5 2 2.5 3
y 4.25 2 2.69 4.25 6.41 9.11

237

07 FWS3A(Exp)_ch6.indd 237 12/16/14 12:58 PM


© Star Publishing Pte Ltd. All rights reserved.

y Solution
(a) y = x(9 – x2)
10
x –3.5 –3 –2 –1 0 1 2 3 3.5
y 11.4 0 –10 –8 0 8 10 0 –11.4
y = x2 + 1
x2
y
8 (3, 8)
10
y = x(9 _ x 2)
5

_4 _3 _2 _1 O x
6 1 2 3 4
_5

_ 10

4
(b) The maximum point on the curve is (1.7, 10.4).
y=3 The minimum point on the curve is (–1.7, –10.4).
–10
(c) At x = 2.5, gradient of the curve =
1
2 = –10
Note: The exact value is –9.75.
(d) The gradient of the curve is positive when
–1.7  x  1.7.
(1, 0.5)
5
O
x (e) (i) y= x
1 2 3 2
x –2 0 2
y –5 0 5
(b) The minimum point is (1, 2).

(c) (i)
The tangent at the minimum point is a The line is drawn on the diagram in (a).
horizontal line. (ii) 2x(9 – x2) = 5x
\ the gradient of the curve at the minimum
x(9 – x2) =
5x
point is 0. 2
(ii) The tangent to the curve at the point where The solutions of this equation are the
x = 2 passes through (3, 8) and (1, 0.5). x-coordinates of the points of intersection of
5x
\ the gradient of the curve at x = 2 is the graphs y = x(9 – x2) and y = . Hence,
2

8 – 0.5
= 3.75. the solutions are x = –2.5, x = 0 and x = 2.5.
3–1

(d) Draw the line y = 3 on the graph in (a). Level 3


Hence, the solutions of the equation x2 +
1
=3 9. When x printers are sold, the profit $y made by a computer
x2 retailer is given by the equation y = k(x – 10)(90 – x),
are x = 0.6 and x = 1.6. where k is a constant and 0  x  90. It is known that
the profit is $600 for 30 printers sold.
8. (a) Draw the graph of the curve y = x(9 – x 2) for (a) Find the value of k.
–3.5  x  3.5 using a scale of 2 cm to 1 unit on (b) Hence, draw the graph of y = k(x – 10)(90 – x)
the x-axis and 2 cm to 5 units on the y-axis. for 0  x  90 by using a scale of 2 cm to represent
(b) Find the turning points on the curve. 10 printers on the x-axis and 2 cm to represent
(c) Find the gradient of the curve at the point where $200 on the y-axis.
x = 2.5 by drawing a tangent. (c) By drawing a tangent, find the gradient of the graph
(d) Write down the interval of values of x for which at the point where x = 30.
the gradient of the curve is positive. (d) (i) Find the number of printers that should be
5 sold to get the maximum profit.
(e) (i) Draw the line y = x on the diagram in (a). (ii) What is the profit per printer when the profit
2
(ii) Hence use your graphs to solve the equation is the maximum?
2x(9 – x2) = 5x. (e) (i) Draw the line y = 12x on the same axes.
(ii) Hence find the number of printers sold such
that the profit per printer is $12.
Chapter 6 Functions and Graphs
238

07 FWS3A(Exp)_ch6.indd 238 12/16/14 12:58 PM


© Star Publishing Pte Ltd. All rights reserved.

Solution 10. In the diagram, ABCD is a rectangular poster in which


(a) y = k(x – 10)(90 – x) the rectangle PQRS is the printing area. The printing
When x = 30, y = 600. area is 300 cm2. The top and bottom margins are 4 cm
∴ 600 = k(30 – 10)(90 – 30) wide, and the left and right margins are 5 cm. Let PQ
600 = 1200k be x cm and the area of ABCD be y cm2.
1
k=
2 D C
S 4 R
1
(b) y= (x – 10)(90 – x) We
2
Singapore!
x 0 10 20 30 40 5
Q
5
P 4
A B
y – 450 0 350 600 750

(a) Express each of the following in terms of x
x 50 60 70 80 90 (i) the length of AB,
(ii) the length of BC.
y 800 750 600 350 0
(b) Hence show that y = 8x + 3000
+ 380.
x
y
y = 12x
(c) The table below shows some values of x and the
corresponding values of y, correct to the nearest
800
integer.
600
x 14 16 18 20 22 24 26
400 p 696 691 690 692 697 703
y
(x _ 10)(90 _ x)
Profit ($)

1
y= 2
200
(i) Find the value of p.
x
O 10 20 30 40 50 60 70 80 90 (ii) Using a scale of 2 cm to 2 cm, draw the x-axis
_ 200 for 14 < x < 26.
Number of printers Using a scale of 2 cm to 5 cm 2, draw the
_ 400 y-axis for 685 < y < 710.
On your axes, draw the graph of
y = 8x + 3000
+ 380.
x
(c) At x = 30, gradient of the graph =
200 (d)
By drawing a tangent, find the gradient of the graph
10 at x = 18.
= 20 (e) Use your graph to find
(d) (i) The maximum point on the graph is (i) the range of values of x for which the area of
(50, 800). the poster is at most 700 cm2,
∴ the required number of printers is 50. (ii) the value of x for which the area of the poster
(ii) The required profit per printer = $800 ÷ 50 is the least.
= $16
(e) (i) The line y = 12x passes through the points Solution
(0, 0) and (50, 600). It is drawn on the diagram (a) (i) AB = (x + 10) cm
in (a). (ii) RS = (area of PQRS) ÷ PQ
(ii) When x printers were sold with a profit of =
300
$12 each, the total profit = $12x. x
∴ x is a solution of the equation \ BC =
 300
+ 8  cm
1  x 
(x – 10)(90 – x) = 12x.
2 (b) Area of ABCD = AB 3 BC
From the graph,  300
the required number of printers sold is 15 y = (x + 10)  + 8 
x 
or 61.
y = 300 + 8x + 3000
+ 80
x
\ y = 8x + 3000
+ 380 (shown)
x

239

07 FWS3A(Exp)_ch6.indd 239 12/16/14 12:58 PM


© Star Publishing Pte Ltd. All rights reserved.

(c) (i) When x = 14, Exercise 6.5


3000
y = 8(14) + + 380 Level 1
14
= 706.3 (correct to 4 s.f.) 1. The following is a conversion table between kilograms
\ the value of p is 706. (correct to the and pounds.
nearest integer) Kilograms (x kg) 2 4 6 8 10

(ii) Pounds ( y lb ) 4.4 8.8 13.2 17.6 22.0

y (a)
Draw the graph of y against x using a scale of
1 cm to 1 kg on the x-axis and 2 cm to 5 lb on
710 the y-axis.
(b) Find, from the graph,
(i) the number of pounds in 5 kg,
(ii) the number of kilograms in 15 lb.
3000
y = 8x + x + 380
705 Solution
(a) The diagram below shows the graph of y against x.

y = 700 20
700
15
Area (cm2)

Pounds (y 1b)
10

695 5
(15, 695)

0 2 4 6 8 10
Kilograms (x kg)

690
(b) (i) 5 kg ≈ 11 lb
(ii) 15 lb ≈ 6.8 kg
(21, 687)

2. The diagram shows a simplified distance-time graph


685
model of Kah Seng’s and Rahim’s performances in a
200-m race. Kah Seng allowed Rahim an advantage of
x a certain distance.
O 14 16 18 20 22 24 26
Length (cm)

200
(d)
The tangent to the graph at x = 18 passes through
(15, 695) and (21, 687).
Distance (m)

\ the gradient of the graph at x = 18 is Rahim


687 – 695 100
= –1.33 (correct to 3 s.f.)
21 – 15 Kah Seng
34
Note: The exact value is – = –1.26.
27
(correct to 3 s.f.) O
10 20 30
(e) (i) Draw the line y = 700 on the graph in (c). Time (seconds)
The line intersects the curve at x = 15 and
x = 25. For the area to be at most 700 cm2, (a) What was the advantage, in terms of distance, given
the range of values of x is 15 < x < 25. to Rahim?
(ii) The area of the poster is the least when (b) Find Kah Seng’s speed in the race.
x = 19.4. (c) When and where did Kah Seng overtake Rahim?
(d) Who won the race? How much faster, in seconds,
was the winner?

Chapter 6 Functions and Graphs


240

07 FWS3A(Exp)_ch6.indd 240 12/16/14 12:58 PM


© Star Publishing Pte Ltd. All rights reserved.

Solution 4. The diagram shows the speed-time graphs of two cars, A


(a) Distance advantage given to Rahim = 50 m and B, which started from the same point along a road.
(Note: This is the difference in their initial distances.)
200
(b) Speed of Kah Seng in the race = 20
24
1 A
= 8 m/s

Speed (m/s)
3
B
(c) Kah Seng overtook Rahim after 18 seconds where 10
Kah Seng was 150 m away from his starting point.
(d) Kah Seng won the race by 3 seconds.
O
5 10 15 20 25
3. The diagram shows the distance-time graph of Time (seconds)
Mrs Brown’s journey.
(a) Find the accelerations of ca r A and B at
time = 20 seconds.
A (b) Find the difference between the speed of A and the
1000
speed of B after they have travelled for 10 seconds.
800
Distance from P (m)

Solution
20 – 0
600 (a) Acceleration of A =
25
400 = 0.8 m/s2
Acceleration of B when time is 20 seconds
200
12 – 0
=
B 20
O
10 20 30 = 0.6 m/s2
Time (minutes)
(b) After 10 seconds,
(a)
Find Mrs Brown’s speed during the period difference between speed of A and speed of B
represented by =8–6
(i) OA, = 2 m/s
(ii) AB.
(b) Find Mrs Brown’s average speed for the whole
journey. 5. The diagram shows the speed-time graph for the journey
(c) Describe briefly Mrs Brown’s journey as shown in of a trolley.
the graph.

Solution 6
(a) (i) Speed of Mrs Brown during OA
Speed (m/s)

1000 4
=
12
1 2
= 83 m/min
3
O
(ii) Speed of Mrs Brown during AB 2 4 6 8 10
Time (seconds)
1000
=
18
5 (a) Find the initial speed of the trolley in km/h.
= 55 m/min (b) Find the acceleration of the trolley during
9
(i) the first 4 seconds,
(b) Average speed of Mrs Brown for the whole journey (ii) the last 4 seconds.
1000 3 2
= (c) It is known that the distance travelled by an object
30
2
during a time interval is equal to the area bounded
= 66 m/min by its speed-time graph and the time axis in that
3
time interval. Find the total distance travelled by
(c) Mrs Brown travelled for 12 minutes at a uniform the trolley during the 10 seconds.
1
speed of 83 m/min to a place 1000 m from her
3
starting point P. She then returned to the starting
5
point at a uniform speed of 55 m/min in 18 minutes.
9

241

07 FWS3A(Exp)_ch6.indd 241 12/16/14 12:58 PM


© Star Publishing Pte Ltd. All rights reserved.

Solution 7. In a spreadsheet software, there is an absolute value


(a) Initial speed of the trolley = 5 m/s function (ABS) y of x whose graph is as shown below.
3600 y
=53 km/h
1000
= 18 km/h 3

(b) (i) Acceleration during the first 4 seconds 2


5–5
=
4 1
= 0 m/s2
x
(ii) Acceleration during the last 4 seconds _3 _2 _1 O 1 2 3
2–4
=
4
= – 0.5 m/s2
(c) Total distance travelled by the trolley during the
10 seconds
1
+ 2) 3 6  (a) Find the value of y when
= (4 3 5) +  2 3 (5
  (i) x = 2, (ii) x = 0,
= 41 m (iii) x = –1.7, (iv) x = –2.8.
(b) Express y in terms of x when
(i) x > 0, (ii) x , 0.
6. In a spreadsheet software, there is a SIGN function y of
x whose graph is as shown below. Solution
(a) (i) When x = 2, y = 2
y
(ii) When x = 0, y = 0
1.5 (iii) When x = –1.7, y = 1.7
(iv) When x = –2.8, y = 2.8
1 (b) (i) When x > 0, y = x
(ii) When x , 0, y = –x
0.5

Level 2
x
_3 _2 _1 O 1 2 3 8. The following is a conversion table between temperature
_0.5 in degrees Celsius and temperature in degrees Fahrenheit.

_1 Celsius (x °C) 20 40 60 80 100


Fahrenheit ( y °F) 68 104 140 176 212
_1.5
(a)
Draw the graph of y against x using a scale of
2 cm to represent 20 °C on the x-axis and 2 cm to
Find the value of y when represent 50 °F on the y-axis.
(a) x = 1, (b) Read, from the graph,
(b) x = 2.8, (i) the corresponding value in °F for 0 °C,
(c) x = 0, (ii) the corresponding value in °C for 80 °F.
(d) x = –1.5. (c) Find an equation connecting x and y.

Solution
Solution
(a) The diagram below shows the graph of y against x.
(a) When x = 1, y = 1
(b) When x = 2.8, y = 1
(c) When x = 0, y = 0 200
(d) When x = –1.5, y = –1
Fahrenheit (y °F)

150

100

50

0 20 40 60 80 100
Celsius (x °C)
Chapter 6 Functions and Graphs
242

07 FWS3A(Exp)_ch6.indd 242 12/16/14 12:58 PM


© Star Publishing Pte Ltd. All rights reserved.

(b) (i) 0°C ≈ 32°F The taxi started from rest and accelerated at 1.5 m/s2
(ii) 80°F ≈ 27°C for 10 seconds to a speed v m/s. It maintained the speed
212 – 32 of v m/s for 18 seconds and then decreased its speed
(c) Gradient of the graph =
100 uniformly to zero. The total distance travelled was 495 m.
9 (a) Find the value of v.
=
5 (b) Find the value of T.
9
\ equation of the graph is y = x + 32. (c) Find the retardation of the taxi just before it came
5
to rest.
9. The diagram shows the distance-time graph for a journey (d) Find the average speed for the whole journey.
by a cyclist. (The area enclosed under a speed-time graph of an
object represents the total distance travelled by the
object in that time period.)
C
40
Solution
30 v–0
(a) = 1.5
Distance (km)

10
A B
20 v = 15

10 (b) Total distance travelled


= area under the speed-time graph
1
O
1 2 3 495 = (18 + T) 3 15
2
Time (hours)
18 + T = 66
(a) Find the speed of the cyclist in his last 1.5 hours T = 48
of the journey. (c) Acceleration of taxi just before it came to rest.
(b) Describe the cyclist’s journey as shown in the graph. =
0 – 15

(c)
48 – 28
Find the average speed of the cyclist for the whole
journey. = –0.75 m/s2
i.e. the required retardation is 0.75 m/s2.
Solution (d) Average speed for the whole journey
(a) Speed of cyclist in his last 1.5 hours of the journey 495
=
40 – 20
= 48
1.5 5
= 10 or 10.3125 m/s
1 16
= 13 km/h
3

(b) The cyclist travelled to A, 20 km from his starting 11. The diagram shows the distance-time graph of Ada’s
point, at a uniform speed of 20 km/h for one hour. journey along a road joining two cities, P and Q, which
He then rested for half an hour before continuing are 80 km apart.
1
his journey to C at a uniform speed of 13 km/h.
3
He took 1.5 hours for the last 20 km of his journey. 80
He has taken 3 hours to complete his whole journey
of 40 km.
Distance from P (km)

60

(c) Average speed of cyclist for the whole journey Ada


40
40
=
3
20
1
= 13 km/h
3
O 1 2 3
Time (hours)
10. The diagram below shows the speed-time graph of a taxi
for a time period of T seconds. (a)
How long was Ada at rest?
(b)
Estimate the speed of Ada when the time = 3.
(c)
John travelled at a uniform speed of 60 km/h from
Speed (m/s)

v Q to P. He set off one hour later than Ada.


(i) Draw John’s distance-time graph on the same
0
diagram.
10 28 T
Time (seconds) (ii) When and where did Ada meet John?

243

07 FWS3A(Exp)_ch6.indd 243 12/16/14 12:58 PM


© Star Publishing Pte Ltd. All rights reserved.

Solution (c)
It is known that the distance travelled by an object
(a) The horizontal part of the graph indicates that Ada during a time interval is equal to the area bounded
was at rest. She was at rest for 30 minutes. by its speed-time graph and the time axis in that
time interval.
(b) Draw the tangent to the curve at the point where
(i) Complete the following table for the distance
the time = 3 hours.
travelled by the trolley at different times.
The tangent passes through the points (2.2, 80) and
(2.5, 60). t (s) 0 1 2 3 4 5 6
80 – 60
\ gradient of the tangent = 2.2 – 2.5 Distance
2 (m)
= –66
3
2 (ii) Sketch the distance-time graph of the trolley
Hence, the required speed is 66 km/h.
3 for the first 6 seconds.
Note: T he negative sign indicates that Ada is
Solution
travelling from Q to P at this time.
(a) Maximum speed of the trolley = 4 m/s
(c) (i) The diagram below shows the distance-time =43
3600
km/h
graphs of John and Ada. 1000
= 14.4 km/h
4–0
(b) (i) Acceleration during OA =
2
80 = 2 m/s2
4–4
(ii) Acceleration during AB =
Distance from P (km)

60 3
= 0 m/s2
John Ada
40 0–4
(iii) Acceleration during BC =
1
= – 4 m/s2
20
(c) (i)
t (s) 0 1 2 3 4 5 6

O 1 2 3 Distance
0 1 4 8 12 16 18
Time (hours) (m)

(ii) Ada met John at a place 56 km from P at (ii)  
the time of 1 hour 24 minutes after she had
started her journey. y

12. The diagram shows the speed-time graph of a trolley


during a journey of 6 seconds. 20

A B
16
4
Distance (m)

3 12
Speed (m/s)

2
8
1
4
C
O
1 2 3 4 5 6
Time (seconds) x
O 1 2 3 4 5 6
(a)
Find the maximum speed of the trolley in km/h. Time (seconds)
(b)
Find the acceleration of the trolley during the period
represented by
(i) OA, (ii) AB, (iii) BC.

Chapter 6 Functions and Graphs


244

07 FWS3A(Exp)_ch6.indd 244 12/16/14 12:58 PM


© Star Publishing Pte Ltd. All rights reserved.

13. The speed, v m/s, of a car at time t seconds is given by At C, x = 120 . 80.
v = t 2 – 2t + 3 for 0 < t < 5. y = basic connection charge + connection charge
(a) Draw the speed-time graph of the car for 0 < t < 5 for next 40 minutes
using a scale of 2 cm to represent 1 second on the  0.2625
= 19.95 + × (120 – 80) 
t-axis and 2 cm to represent 5 m/s on the v-axis.  100 
(b) Find, from the graph, the acceleration of the car
= 20.055
when t = 3. (b) At the point A, x = 0,
i.e. there is zero connection time.
Solution
Hence, the y-coordinate at the point A represents
(a) v = t 2 – 2t + 3
the connection charge for the first 80 minutes is
t 0 1 2 3 4 5 6 $19.95.
20.055 – 19.95
v 0 1 4 8 12 16 18 (c) Gradient of BC =
120 – 80
= 0.002 625
The gradient of BC represents the connection
20 v = t2 _ 2t + 3
charge rate, in $/min, after the first 80 minutes is
$0.002 625/min (this is equivalent to 0.2625 cents/
Speed (v m/s)

min).

10
15. The basic fare for a taxi is as follows:

Flag-Down (inclusive of 1st km or less) $p


0 1 2 3 4 5 Every 400 m thereafter or less up to 10 km $q
Time (t seconds)

(b) When t = 3, The diagram below shows the graph of basic fare,
$y, against the distance, x km, of a journey for
acceleration of the car
0 , x < 3.
= gradient of the tangent (at t = 3)
14 – 6 y
= 5–3
5
= 4 m/s2

14. A mobile phone plan charges $19.95 per month with 80


Basic fare ($)

minutes of free connection time. The connection charge 4


after the first 80 minutes is 0.2625 cent per minute. The
following graph shows the monthly charge, $y, against
the connection time, x minutes, for 0 < x < 120.
3
y
x
C O 1 2 3
Monthly charge ($)

Distance (km)
A

B

(a) Find the values of p and q.


x
(b) What is the basic fare for a journey of
O
80
Connection time (minutes)
120 (i) 0.6 km?
(ii) 2.8 km?
(a) Find the coordinates of A, B and C on the graph.
(b) Interpret the meaning of the y-coordinate of the Solution
point A. (a) From the graph, when 0 < x ≤ 1, the value of
(c) Find the gradient of BC and interpret its meaning. y = 3.40.
\ p = 3.40
Solution For x > 1, each part of the step graph is 0.4 units
(a) Coordinates of A = (0, 19.95), B = (80, 19.95) and in length and goes higher by 0.2 units each.
C = (120, 20.055) This means for every 0.4 km, i.e. 400 m, the fare
increases by $0.20.
\ q = 0.20

245

07 FWS3A(Exp)_ch6.indd 245 12/16/14 12:58 PM


© Star Publishing Pte Ltd. All rights reserved.

(b) (i) The basic fare for a journey of 0.6 km is Let $y be the income tax for $x chargeable income.
$3.40. (a) Find the value of y when
(ii) The basic fare for a journey of 2.8 km is (i) x = 18 000, (ii) x = 27 000,
$4.40. (iii) x = 36 000, (iv) x = 60 000.
(b) The diagram below shows the graph of y against
16. A rectangular aquarium tank is 40 cm long, 25 cm wide x for 0 < x < 80 000.
and 32 cm high. The tank is cleaned by siphoning water y
out of tank. The graph below shows the change in height
3500 D
of water in the tank against time during the course of
siphoning. 3000
y
2500

Income tax ($)


30 2000
25
Height (cm)

1500
20
15 1000
10 C
500
5 B
A
x O x
O 2 4 6 8 20 40 60 80
Time (min) Chargeable income in thousands ($)

(a) State the initial height of water in the tank. (i) State the coordinates of A, B, C and D.
(b) What is the volume of water in the tank after 8 (ii) Find the equation of the line CD.
minutes? (iii) What does the gradient of the line CD
(c) Find the gradient of the graph. represent?
(d) Interpret the meaning of the gradient of the graph.
Solution
Solution (a) (i)
The value of y when x = 18 000 is $0.
(a) The initial height of water is 30 cm. (ii) The value of y when x = 27 000 is
(b)
After 8 minutes, the height of water in the tank is $0 + 2% of $7000 = $140
10 cm. (iii)The value of y when x = 36 000 is the sum
\ the volume of water = 40 3 25 3 10 of the tax on the first $30 000 and 3.5% of
= 10 000 cm3 the remaining
10 – 30 $(36 000 – 30 000) = $200 + 3.5% 3 $6000
(c) Gradient of the graph = = $410
8–0
= –2.5 (iv) The value of y when x = 60 000 is the sum
of the tax on the first $40 000 and 7% of the
(d) This means the height of water in the tank is
remaining
decreasing at a constant rate of 2.5 cm/min.
$20 000 = $550 + 7% 3 $20 000
= $1950
Level 3
17. The following table is part of the income tax rates (b) (i) Coordinates of A = (20, 0), B = (30, 200),
for resident individuals in Singapore in the Year of C = (40, 550) and D = (80, 3350).
Assessment 2012. (ii) Gradient of the line CD =
3350 – 550
80 – 40
Gross Tax = 70
Chargeable Income Rate (%) Let the equation of the line be y = 70x + c.
Payable ($)
Since C(40, 550) is on the line,
First $20,000 0 0 550 = 70(40) + c
Next $10,000 2 200 c = –2250
Hence, the equation of CD is y = 70x – 2250.
First $30,000 – 200
Next $10,000 3.5 350 (iii) The gradient of the line CD represents the rate
of income tax for every $1000 of chargeable
First $40,000 – 550 income between $40 000 and $80 000, i.e.
Next $40,000 7 2800 $70 is taxed on every $1000 of chargeable
income between $40 000 and $80 000. This
rate is 7%.

Chapter 6 Functions and Graphs


246

07 FWS3A(Exp)_ch6.indd 246 12/16/14 12:58 PM


© Star Publishing Pte Ltd. All rights reserved.

18. A bucket has a capacity of p cm 3. Water is injected 19.


The diagram shows a container
into the bucket at the rate of q cm3 per min. When the which consists of cylinders A and
B
bucket is full, all water will be poured out and water is B. Cylinder A has a base radius of 12
injected into it again. Let V cm3 be the volume of water 10 cm and a height of 6 cm.
5
in the bucket at time t minutes after the first observation. Cylinder B has a base radius
Assume that the pouring time is negligible. The diagram of 5 cm and a height of 12 cm. 6
10
below shows the graph of V against t for 0 < t , 10. (a) Find the volume of A
cylinder
v (i) A,
A C (ii) B.
50
Leave your answers in terms of r.
(b) Water flows into the container at a constant rate of
Volume of water (cm3)

40
200r cm3 per minute.
30
(i) How long will it take to fill up cylinder A?
(ii) How long will it take to fill up the whole
20 container?
(iii) Draw the graph of d against t to show the
10 relationship between the height of the water,
d cm, measured from the bottom of the
O
B t container and the time, t minutes, as the
5 10
Time (minutes) container is being filled.

Solution
(a) What is the initial volume of water in the bucket? (a) (i) Volume of cylinder A = r 3 102 3 6
(b) Find the values of p and q from the graph. = 600r cm3
(c) Find the gradient of the line BC. What does it
(ii) Volume of cylinder B = r 3 52 3 12
represent?
= 300r cm3
(d) Find the equation of the line BC.
(b) (i) Time taken to fill up cylinder A
600r
Solution =
(a) Initial volume of water in the bucket = 0 cm . 3 200r
= 3 min
(b) From the graph,
p = 50 (ii) Time taken to fill up the whole container
600r + 300r
and q =
50
=
5 200r

= 10 = 4.5 min

(c) Gradient of the line BC (iii) The diagram shows the graph of d against t.
50 – 0
=
10 – 5 y
= 10
It represents the rate at which water is injected
into the bucket in cm3/min, i.e. 10 cm3 of water is 18
injected into the bucket in 1 minute.

(d)
Let the equation of the line BC be V = 10t + c. 15
Depth of water (cm)

Since B(5, 0) is on the line,


0 = 10(5) + c 12
c = –50
Hence, the equation of BC is V = 10t – 50. 9

0 t
1 2 3 4 5
Time (minutes)

247

07 FWS3A(Exp)_ch6.indd 247 12/16/14 12:58 PM


© Star Publishing Pte Ltd. All rights reserved.

Revision Exercise 6 2. Which of the following graphs represent the graph of


1. (a) The diagram is a sketch of the graph of y = x , n (a) y = –x + 1, (b) y = x3,
1
where n is a constant. State a possible value of n. (c) y=– ?
x
y y y
A. B.
n
y=x

x x
x O O
O

C. y D. y

(b) Write down a possible equation for the graph shown


in the diagram. x x
O O

y
Solution
(a) Graph D
1
(b) Graph A
x
O (c) Graph B

3. The following graph shows the local postage rate, $y,


(c) The point (1, 1) is marked on the given diagram.
of small parcels of next day delivery within the city
Copy the diagram and sketch the graph of y = 12 business district against the mass, m kg, of the parcel.
x
on it. y
y
16

14
x G H
O
12
E F
Rate ($)

10
C D

Solution A B
6
(a) n = –1, –3, –5, ...
(b) The general equation of the graph is y = ax for any O 10 20 30
m
a . 1. Some possible equations are y = 2x, y = 3x Mass (kg)
and y = 5.6x.
(a) Find the postage rate for a parcel of mass
(c) y
(i) 3.5 kg,
(ii) 20 kg.
1
(b) Find the equation of the line GH.
y=
x2 (c) If John has to send two small parcels of masses
2.5 kg and 8 kg to Amy, which way of packaging
can provide him the lowest delivery cost? What is
(_1, 1) (1, 1) the cost then?

x Solution
O
(a) (i) The postage rate for a parcel of 3.5 kg is $10.
(ii) The postage rate for a parcel of 20 kg is $12.
(b) The equation of the line GH is y = 14.
(c) To achieve the lowest delivery cost, John should
pack both parcels together as one parcel.
The cost will be $12.

Chapter 6 Functions and Graphs


248

07 FWS3A(Exp)_ch6.indd 248 12/16/14 12:58 PM


© Star Publishing Pte Ltd. All rights reserved.

4. The table below shows some corresponding values of (ii) The roots of the equation
x and y, where y = x(x + 2)(x – 3). x(x + 2)(x – 3) = 2x
are x = –2, x = – 0.2 and x = 3.6.
x –2.5 –2 –1 0 1 2 3 4
y p 0 4 0 –6 –8 0 24
5. Chris took 3 trips by MRT trains in a morning. The
diagram below shows the balance, $y, in his Ezlink
(a)
Find the value of p.
(b)
Using a scale of 2 cm to 1 unit on the x-axis and card against the time, t a.m.
2 cm to 5 units on the y-axis, draw the graph of y
y = x(x + 2)(x – 3) for –2.5  x  4.
(c) From the graph, find 8

(i) the turning points of the graph,


(ii) the gradient of the graph at the point (3, 0)
by drawing a tangent.
6
(d) (i) Draw the graph of y = 2x for –2.5  x  4
on the same axes.
(ii) Hence use your graphs to solve the equation

Balance ($)
x(x + 2)(x – 3) = 2x. 4

Solution
(a) y = x(x + 2)(x – 3)
When x = –2.5, 2
y = –2.5(–2.5 + 2)(–2.5 – 3)
= –6.875
∴ the value of p is –6.875.
t
(b)
y
O 8 a.m. 9 a.m. 10 a.m. 11 a.m.
Time

25
y = x(x + 2)(x _ 3)
20 (a) When did he exit an MRT station in the first trip?
How much was the fare of the trip?
15 (b) Which trip had the lowest fare? How much was the
fare?
10
Solution
5 y = 2x
(a) In the first trip, Chris exited the MRT station at
x 8.30 a.m.
_2 _1 O 1 2 3 4 His fare was $8 – $5.80 = $2.20
_5
(b) The second trip had the lowest fare,
_ 10 The fare was $5.80 – $4.40 = $1.40.

6. (a) Copy and complete the following table of


(c) (i)  The minimum point on the graph is cor r esp ond i ng va lues of x a nd y, where
20
(1.8, –8.2). y = x2 + . Give your answers correct to 1 decimal
x
The maximum point on the graph is
place where necessary.
(–1.1, 4.1).
15 x 0.5 1 2 3 4 5 6 7
(ii) Gradient at the point (3, 0) =
1
y
= 15
(d) (i)
y = 2x
(b) Taking 2 cm to represent 1 unit on the x-axis and
x –2.5 –2 –1 0 1 2 3 4
2 cm to represent 10 units on the y-axis, draw
20
the graph of y = x 2 + for 0.5  x  7.
y 0.18 0.25 0.5 1 2 4 8 16 x
(c) By drawing a tangent, find the gradient of the graph
The graph of y = 2x is shown on the diagram at x = 1.
in (b).

249

07 FWS3A(Exp)_ch6.indd 249 12/16/14 12:58 PM


© Star Publishing Pte Ltd. All rights reserved.

(d) Use your graph to find the solutions of the equation (f ) (i) y = –8x + 40
20
(i) x + 2
= 30, x 0 2 5
x
(ii) x2 +
20
= 10. y 40 24 0
x
(e) Use your graph to find the minimum value of The line y = –8x + 40 is drawn on the diagram
x2 +
20
for 0.5  x  7. in (b).
x
(ii) The required equation is
(f ) (i) Draw the graph of the straight line
20
y = –8x + 40 on the same axes. x2 + = –8x + 40
x
(ii) Write down, but do not simplify, an equation
in x which has the x-coordinates of the
intersecting points of the two graphs as 7.
A piece of wire 72 cm long is cut into 12 pieces to form
solutions. the frame of a cuboid x cm by x cm by h cm as shown.
(a) Express h in terms of x.
Solution (b) Let y cm3 be the volume of the cuboid. Show that
20 y = 18x2 – 2x3.
(a) y = x2 + (c) Draw the graph of y = 18x2 – 2x3 for
x
0  x  9 by using a scale of 2 cm to represent
x 0.5 1 2 3 4 1 unit on the x-axis and 2 cm to represent
50 units on the y-axis.
y 40.3 21 14 15.7 21
(d) By drawing a tangent, find the gradient of the graph
at x = 7.
x 5 6 7 (e) Use the graph to find the values of x for which the

y 29 39.3 51.9 volume of the cuboid is
(i) 150 cm3,
(ii) at its maximum.
(b) y

60

50 y = x 2 + 20
x h

40
y = 30
30 x

x
20
y = 10
10 Solution
y = _ 8x + 40
(a) Consider the total length of the edges of the frame.
x 8x + 4h = 72
O 1 2 3 4 5 6 7
2x + h = 18

∴  h = 18 – 2x
39 – 21 (b) y = x 2h
(c) At x = 1, gradient of the graph =
0–1 = x 2(18 – 2x)
= –18 ∴  y = 18x 2 – 2x 3
(d) (i) Draw the line y = 30 on the graph in (b).
(c) y = 18x 2 – 2x 3
20
The solutions of x + 2
= 30 are
x x 0 1 2 3 4
x = 0.7 and x = 5.1.
(ii) Draw the line y = 10 on the graph in (b). y 0 16 56 108 160

The line does not meet the graph of
y = x2 +
20
. x 5 6 7 8 9
x

20 y 200 216 196 128 0
Thus, the equation x 2 + = 10 has no
x
solutions.
(e) (2.15, 13.9) is the minimum point on the graph.
∴ the minimum value is 13.9.

Chapter 6 Functions and Graphs


250

07 FWS3A(Exp)_ch6.indd 250 12/16/14 12:58 PM


© Star Publishing Pte Ltd. All rights reserved.

(b) y
y

_5 _4 _3 _2 _1 x
(5.7, 250) O
250

200
y = 150 (8.1, 150) (_1.8, _1.6)
150 _2

100
y = 18x2 _ 2x3
50
_4
x
O 1 2 3 4 5 6 7 8 9
y = _5

(d) At x = 7, gradient of the graph _6


250 – 150
=
5.7 – 8.1
= – 41.7 (correct to 3 s.f.)
Note: The exact value is – 42.
_8
(e) (i) Draw the line y = 150 on the graph in (c). y = 3x _ 12
x2
+7
The required values of x are
x = 3.8 and x = 7.75.
(ii) The maximum point of the graph is at x = 6.
∴ the required value of x is 6. _9 (0, _9)

8. Two quantities x and y are related by the equation


y = 3x – 12
+ 7. The following table shows some
x2 (c) (i) The maximum value of y is –2 and the
values of x and the corresponding values of y, correct to corresponding value of x is –2.
two decimal places where necessary.
(ii) 3x – 12
+ 7 . –5
x –5 –4 –3 –2 –1 x2
i.e. y . –5
y p –5.75 –3.33 –2 –8
Draw the line y = –5 on the graph in (b).
(a)
Find the value of p. The line intersects the curve at x = –3.7 and
(b)
Using a scale of 2 cm to represent 1 unit on the x = –1.2.
x-axis and 1 cm to represent 1 unit on Hence the range of values of x such that
the y-axis, draw the graph of y = 3x – 122 + 7
x 3x – 12
+ 7 . –5 is –3.7 < x , –1.2.
for –5  x  –1. x2
(c) Using your graph, find
(d) The tangent to the graph at the point where
(i) the maximum value of y and the corresponding
x = –1.5 passes through (–1.8, –1.6) and (0, –9).
value of x,
(ii) the range of possible values of at x such that Hence, the required gradient
–9 – (–1.6)
3x – 12
+ 7 . –5 for –5  x  –1. =
x2 0 – (–1.8)
(d) By drawing a tangent, find the gradient of the graph = – 4.11 (correct to 3 s.f.)
at the point where x = –1.5.

Solution
(a) y = 3x – 12
+7
x2
12
When x = –5, y = 3(–5) – (–5)
2 +7
= –8.48
\ p = –8.48

251

07 FWS3A(Exp)_ch6.indd 251 12/16/14 12:58 PM


© Star Publishing Pte Ltd. All rights reserved.

9. The cost, $y, of making a plate of radius x cm is given (c)


b
by the function y = ax + 2
, where a and b are constants y
x
It is known that when x = 2, y = 110 and when x = 12,
y = 90. 120
(a) Find the values of a and b.
y = 12 x2 + 216
(b) The table below shows some corresponding values 100
x

of x and y.
x 2 4 6 8 10 12 80
(12, 77)

Cost ($)
y 110 62 54 59 p 90 y = 70

60
Find the value of p.
(c) Using a scale of 1 cm to represent 1 cm on the 40
x-axis and 1 cm to represent $10 on the y-axis, (4, 40)
b
draw the graph of y = ax2 + for 2  x  12. 20
x
(d) Using your graph, find the radius of the plate
(i) which gives the minimum cost, x
O 2 4 6 8 10 12
(ii) when the cost is $70.
(e) By drawing a tangent, find the gradient of the graph Radius (cm)
at the point where x = 8. What does this gradient
represent? (d) (i) The minimum point of the graph is (6, 54).
\ the radius of the plate which gives the
Solution
minimum cost is 6 cm.
b
(a) y = ax2 + (ii) Draw the line y = 70 on the graph in (c). The
x
Putting x = 2, y = 110: x-coordinates of the points of intersection of
b the curve and the line are x = 3.4 and x = 9.8.
4a + = 110 Hence, the radius of the plate when the cost
2
8a + b = 220....................(1) is $70 is 3.4 cm or 9.8 cm.

Putting x = 12, y = 90: (e) The tangent to the graph at the point where x = 8
b passes through (4, 40) and (12, 77).
144a + = 90 77 – 40
12 Hence, the required gradient =
1728a + b = 1080................. (2) 12 – 4
(2) – (1): 1720a = 860 = 4.625
1 The gradient of this graph measures the rate of
a = change of the cost $y with respect to the radius
2
1 x cm.
Putting a = into (1):
2 Thus when x = 8, the cost is increasing at
 1 $4.625/cm.
8   + b = 220
2
b = 216 10. A rectangular box for juice has a
Hence, a =
1
and b = 216. square base of sides x cm and a
2
height of h cm. The capacity of the
1 2 216 box is 400 cm3. Let y cm2 be the h
(b) y= x +
2 x total surface area of the box.
When x = 10, (a) Show that
400 x
y =
1
(10)2 +
216 (i) h= , x
2 10 x2
1600
= 71.6 (ii) y = 2x2 + .
x
\ p = 71.6 (b) The table below shows some values of x and the
corresponding values of y correct to the nearest
integer.
x 4 5 6 7 8 9 10 11
y 432 370 339 327 328 340 360 p

Chapter 6 Functions and Graphs


252

07 FWS3A(Exp)_ch6.indd 252 12/16/14 12:58 PM


© Star Publishing Pte Ltd. All rights reserved.

(i) Find the value of p. (c) Draw the line y = 350 on the graph in (b). The
(ii) Using a scale of 2 cm to represent 1 cm, draw values of x for which y = 350 are x = 5.55 and
the x-axis for 4  x  11. x = 9.55.
Using a scale of 2 cm to represent 50 cm2,
(d) The tangent to the graph at the point where x = 5
draw the y-axis for 300  y  450.
passes through (4.3, 400) and (6, 325).
On your axes, plot the points given in the 325 – 400
table and join them with a smooth curve. \ the required gradient =
6 – 4.3
(c) Use your graph to find the values of x for which = – 44.1 (correct to 3 s.f.)
y = 350. Note: The exact value is – 44.
(d) By drawing a tangent, find the gradient of the graph
at the point where x = 5. (e) (i) From the graph, y is the least when x = 7.4.
(e) Use your graph to find (ii) The smallest possible total surface area is the
(i) the value of x for which y is the least, least value of y, i.e. 325 cm2.
(ii) the smallest possible total surface area of the
box.
11. A container is being filled with water at constant rate
from a burette. Initially the container is empty. The height
Solution
of the water in the container is h cm after t seconds.
(a) (i) Volume of box = 400 cm3
(a) If the container is a cylindrical beaker, sketch the
i.e. x2 h = 400
graph of h against t in the diagram below.
400

\ h=
x2 h
(ii) Total surface area of box,
y = 2x2 + 4xh
 400 
y = 2x2 + 4x   h
x2 
1600
\ y = 2x2 +
x t
O
1600
(b) (i) y = 2x + 2
x
When x = 11, (b) If the container is a test tube consisting of a
1600 cylindrical body and a hemispherical end, sketch
y = 2(11)2 + the graph of h against t in the diagram below.
11
= 387 (correct to the nearest integer) h
\ p = 387
(ii)

y
h

450 t
O

y = 2x2 + 1600
x
Total surface area (cm2)

400 Solution
(4.3, 400)
(a)
h

y = 350
350

(6, 325)
300

x
O 4 5 6 7 8 9 10 11 t
O
Length of side (cm)

253

07 FWS3A(Exp)_ch6.indd 253 12/16/14 12:58 PM


© Star Publishing Pte Ltd. All rights reserved.

(b) h (d) Distance between their homes = 200 m


(e) Peter arrived at the school first.
(f ) They met after 2 minutes at a place 300 m from
Mary’s home. They also met after 6.8 minutes at
a place 880 m from Mary’s home.

Go Further
1. The diagram shows a mathematical model of a road
top of the arch of a bridge. The equations of the road
t x (200 – x )
O and the arch are y = 70 and y = + 10 respectively,
200
where 0 < x < 200 and the unit of the length is metre.
The x-coordinate is the horizontal distance from the left
12. Peter’s and Mary’s homes are located along the same road support OC. The y-coordinate is the vertical distance
that leads to their school, which is 1000 m away from above sea level.
Mary’s home. The diagram below shows the distance- (a) Express the equation of the arch in the form
time graphs of their journeys from home to school on a y = a(x – h)2 + k, where a, h and k are constants.
certain day. (b) What is the least distance between the arch and
the road?
x (200 – x )
1000 (c) Draw the graph of y = + 10.
200
Distance from Mary’s home (m)

800
(d) Using the graph, find the gradient of the arch at
Mary the point where x = 50.
600 Peter y
y = 70 Road
400 C B

200 F
Arch
x (200 _ x)
O 2 4 6 8 y= + 10
200
Time (t minutes)
E G
(a) Find Mary’s speed during the first 4 minutes. O A x
(b) Find Mary’s average speed for the whole journey.
(c) Estimate the speed of Peter when the time = 6 minutes. Solution
(d) What is the distance between Peter’s home and x (200 – x )
(a) y= + 10
Mary’s home? 200
(e) Who arrived at the school first? 1
= (200x – x2) + 10
(f ) When and where did Peter and Mary meet on their 200
way to the school? 1
= – (x2 – 200x) + 10
200
Solution 1
(a) Mary’s speed during the first 4 minutes = – [(x – 100)2 – 1002] + 10
200
600
= = –
1
(x – 100)2 + 60
4 200
= 150 m/min
(b) Maximum point F of the equation of the arch
Mary’s speed during the last 4 minutes
= (100, 60)
400
= \ the least distance between the arch and the road
4
= 70 – 60
= 100 m/min
= 10 m
1000
(b) Mary’s average speed = x (200 – x )
8 (c) y= + 10
= 125 m/min 200

(c) \ the required speed x 0 40 80 100 120 160 200


= gradient of tangent PQ
840 – 0 y 10 42 58 60 58 42 10
=
7 – 2.8
= 200 m/min

Chapter 6 Functions and Graphs


254

07 FWS3A(Exp)_ch6.indd 254 12/16/14 12:58 PM


© Star Publishing Pte Ltd. All rights reserved.

y Putting a = –2 into (1):


–2 – b + 6 = 0
60 (76, 60) \ b =4
_
x(200 x)
Hence, the equation of C1 is
y= + 10
y = –2x2 + 4x + 6.
50 200

(iii) y = –2[x2 – 2x – 3]
40 = –2[(x – 1)2 – 12 – 3]
= –2[(x –1)2 – 4]
30 = –2(x – 1)2 + 8
\ the coordinates of the turning point M of
C1 are (1, 8).
20
Since the coefficient of x2 is negative, M is a
maximum turning point.
10 (0, 22)
(b) (i)
Si nc e t he cu r ve C 2 ha s a m i n i mu m
x
point at N(1, –2), its equation can be
O 50 100 150 200 expressed as y = a(x – 1) 2 – 2, where
a . 0.
Since B(3, 0) is on C2,
0 = a(3 – 1)2 – 2
(d) Draw the tangent to the curve at the point where 4a – 2 = 0
x = 50. 1
The tangent passes through the points (0. 22) and a =
2
(76, 60).
1
Hence, the equation of C2 is y = (x – 1)2 – 2
60 – 22 2
Hence, the required gradient = 1 2 3
76 – 0 i.e. y = x –x–
1 2 2
=
2
(ii) M(1, 8) and N(1, –2)
Distance between M and N
2. (a)
A quadratic curve, C1, intersects the x-axis at
A(–1, 0) and B(3, 0), and the y-axis at D(0, 6). Find = (1 – 1)2 + (–2 – 8)2
(i) the equation of the line of symmetry of C1,
(ii) the equation of the curve C1 in the form = 100
y = ax2 + bx + c, = 10 units
(iii) the coordinates of the turning point M of C1.
Is M a maximum or minimum point? (c) The diagram below shows the sketch graphs of C1
(b) Another quadratic curve, C2, also intersects the and C2.
x-axis at points A and B. The curve C2 has the
minimum point at N(1, –2). Find y
(i) the equation of the curve C2 in the form
y = ax2 + bx + c, M(1, 8)
C2
(ii) the distance between M and N. y= 1 2 _x_ 3
2x 2
(c) On the same axes, sketch the graphs of C1 and C2.
D(0, 6)

Solution
(a) (i) The equation of the line of symmetry of C1
–1 + 3
is x = i.e. x = 1. 3
2
(ii)
Since the curve C1 cuts the y-axis at D(0, 6),
the equation of C1 will be y = ax2 + bx + 6.
Since A(–1, 0) and B(3, 0) are on C1, A B x
_2 _1 O 1 2 3 4
0 = a(–1)2 + b(–1) + 6
a – b + 6 = 0......................... (1)
C1
0 = a(32) + b(3) + 6 N(1, _2)
_3 y = _2x2 + 4x + 6
9a + 3b + 6 = 0......................... (2)
(1) × 3:3a – 3b + 18 = 0......................... (3)
(2) + (3): 12a + 24 = 0
\ a = –2

255

07 FWS3A(Exp)_ch6.indd 255 12/16/14 12:58 PM


© Star Publishing Pte Ltd. All rights reserved.

Review Exercise 2

1. In the diagram, ABCD is a rectangle, E is a point on AB, 2.


In the diagram, ABC is an equilateral triangle of side 5
and AC and DE intersect at F. cm, and ADEB and ACFG are squares.
(a) Prove that nAEF and nCDF are similar. (a) Find ∠CAE.
(b) If AE : EB = 3 : 2, find the ratio of (b) Find the length of AE.
(i) the area of nAEF to the area of nCDF, (c) Prove that
(ii) the area of nADF to the area of nCDF. (i) nACE and nABF are congruent,
(ii) BF = CE.
A D D G

F A

E E F


B C
B C
Solution
Solution (a) In nABC,
(a) ∠EAF = ∠DCF (alt. ∠s, AE // DC) ∠ABC = ∠BAC = ∠ACB = 60° (∠s of equilateral n)
∠AEF = ∠CDF (alt. ∠s, AE // DC) In square ABED,
∠AFE = ∠DFC (vert. opp. ∠s) ∠BAD = 90° (∠ of a square)
\ nAEF is similar to nCDF. (Angle-Angle- \ ∠BAE = 45°
Angle similarity) Hence, ∠CAE = 45° + 60°
2 = 105°
Area of n AEF  3
(b) (i) =
Area of nCDF
 
5 (b) AE 2 = AB2 + BE 2 (Pythagoras’ Theorem)
9 = 52 + 52
=
25 = 50
The ratio is 9 : 25. AE = 50
(ii) nADF and nCDF share a common height = 7.07 cm (correct to 3 s.f.)
from D.
(c) (i) AC = AB (sides of equilateral n)
A D ∠CAE = ∠BAF (found)
height AE = AF (found)
\ nACE ≡ nABF (SAS)
(ii) BF = CE (corr. sides of ≡ ns)
F
3. In the diagram, A and B are two similar cylindrical cans.
The ratio of the base areas of A and B is 18 : 32.
(a) Find the ratio of the capacities of A and B.
(b) If the capacity of can B is 768r cm3 and the
C height of can A is 9 cm, find the base radius
Area of n ADF 9 of can A.
=
Area of nCDF 25
3
=
5
The ratio is 3 : 5.

A B

Review Exercise 2
256

04 FWS3A(Exp)_RevEx2.indd 256 12/16/14 1:02 PM


© Star Publishing Pte Ltd. All rights reserved.

Solution 5. Two points A(–4, 1) and B(8, –7) are given. Find
Base area of A 18 (a) the length of AB,
(a) =
Base area of B 32
(b) the equation of the line AB.
9
=
16
Solution
9
= (a) Length of AB = [8 – (–4)] + (–7 – 1)
2 2
16
3 = 208
=
4 = 14.4 units
2
Capacities of A  3
=   (b) gradient of AB =
–7 – 1
Capacities of B 4 8 – (–4)
27
= =
–8
64 12
The ratio is 27 : 64. 2
=
3
Capacity of A 27
(b) = 2
768r 64 Equation of AB: y – 1 = – (x – (– 4))
3
27
Capacity of A = 3 768r 2 8
64 y=– x – +1
= 1018 (correct to 4 s.f.) 3 3

rr2 h = 1018 y=–


2
x –
5
3 3
r2 = 36
r = 6 cm
6. The line L: 2x – 3y + 12 = 0 cuts the x-axis at A and the
4. In the diagram, a right square pyramid P is cut into two y-axis at B. Find
parts, X and Y, by a plane parallel to the base. Each slant (a) the coordinates of A and B,
edge of P, being 7 cm long, is cut into two parts of 4 cm (b) the distance AB,
and 3 cm. Find the ratio of (c) the gradient of the line,
(d) the area of nAOB, where O is the origin.
(a) the diagonals of the bases of P and X,
(b) the base areas of P and X, Solution
(c) the volumes of X and Y. (a) When y = 0,
2x – 3(0) + 12 = 0
4 2x + 12 = 0
2x = –12
7 X
x = –6
P Coordinates of A = (–6, 0)
3 When x = 0,
2(0) – 3y + 12 = 0
Y
–3y + 12 = 0
3y = 12
y=4
Solution
Coordinates of B = (0, 4)
(a) The ratio of the diagonals of the bases of P and X
= 7 : 4.
(b) Distance AB = [0 – (–6)] + (4 – 0)
2 2
2
base area of P  7
(b) =   = 52
base area of X 4
49 = 7.21 units
=
16 0–4
(c) gradient =
The ratio is 49 : 16. –6 – 0
4
 7 Volume of P
3
=
(c)
  = 6
4 Volume of X
2
343 =

= 3
64

Volume of Y = 343 – 64 1
(d) Area of nAOB = 6 4
= 279 cm3
3 3
2
\ The ratio of volumes = 64 : 279 = 12 units 2

257

04 FWS3A(Exp)_RevEx2.indd 257 12/16/14 1:02 PM


© Star Publishing Pte Ltd. All rights reserved.

7. Two points A(–5, –2) and B(1, 4) are given. Find (b) y
(a) the coordinates of the point C on the y-axis such
that AC = BC.
(0, 10)
(b) the coordinates of the point D that the line segment 10
AB cuts the x-axis. y = x2 – 6x + 10

Solution
5
(a) Let the coordinates of C be (0, y).

Length AC = (–5 – 0) + (–2 – y )


2 2 (3, 1)
x
= 25 + ( y – 4y + 4)
2
O 2 4 6
= y 2 – 4y + 29
(c) From the graph in (b),
Length BC = (1 – 0) + (4 – y )
2 2
Coordinates of the minimum point = (3, 1)
= Equation of the line of symmetry: x = 3
1+ (y 2 – 8 y + 16)
(d) x2 – 6x + 10 = 0
= y – 8 y + 17
2
(x – 3)2 + 1 = 0
AC = BC x – 3 = –1
y2 – 4y + 29 = y2 – 8y + 17 x has no real roots.
12y = –12
y = –1 9. On the axes provided, sketch the graphs of
Coordinates of C = (0, –1) (a) y = x3,
y
(b) Let the coordinates of D be (x, 0).
4 – (–2)
gradient of AB =
1 – (–5)
1
6
=
6
=1 x
_1 O 1
Equation of AB: y – 4 = 1(x – 1)
y–4=x–1 _1
y = x + 3................. (1)
Substituting y = 0 into (1),
0=x+3
x = –3
The coordinates of D are (–3, 0). 1
(b) y =
x
y
8. (a) Express x – 6x + 10 in the form (x – h) + k, where
2 2

h and k are constants.


(b) Sketch the graph of y = x2 – 6x + 10. 1

(c) State the coordinates of the minimum point and the


equation of the line of symmetry of the graph.
(d) Hence solve the equation x2 – 6x + 10 = 0. _
1 O 1
x

Solution
2 2 _1
 6  6
(a) x2 – 6x + 10 = x2 – 6x +   –   + 10
2 2
= (x – 3)2 + 1

Review Exercise 2
258

04 FWS3A(Exp)_RevEx2.indd 258 12/16/14 1:02 PM


© Star Publishing Pte Ltd. All rights reserved.

Solution 11.
In the diagram, ABCD is a parallelogram, P and Q are
y
(a) points on AB and CD respectively such that PD bisects
∠ADC and QB bisects ∠ABC.
1 (a) Prove that
(i) ∠ADP = ∠CBQ,
(ii) nADP and nCBQ are congruent,
_1 x (iii) BP = QD.
O 1
(b) Is P the midpoint of AB? Explain briefly.

_1 D Q C


(b) y A P B

Solution
(a) (i) ∠ADC = ∠ABC (opp. ∠s of a //gram)
1 1
1 ∠ADC = ∠ABC
2 2
PD and QB are bisectors of ∠ADC and
∠ABC. (given)
_1 x
O 1 \ ∠ADP = ∠CBQ

(ii) In nADP and nCBQ,
_1 ∠ADC = ∠ABC (proved)
AD = CB (opp. sides of //gram)
\ nADP ≡ nCBQ (ASA)
_2 (iii) AP = CQ (corr. sides of ≡ ns)
AB = DC (opp. sides of //gram)
\ AB – AP = DC – CQ
i.e. BP = QD
10. (a) Sketch the graphs of y = 2x and y = 2 – x on the (b) We can extend the lengths of AB and DC of the
same axes. parallelogram ABCD while keeping ∠ADC and
(b) Hence state the number of solutions of the equation ∠ABC unchanged. In such case, AP remains
2x = 2 – x. unchanged and PB can be longer than AP, Thus,
P is not necessarily the midpoint of AB.
Solution
(a) y
12. In the diagram, BCE is a straight line, AB // DC and
AC // DE.
4 (a) Prove that
(i) ∠BAC = ∠CDE,
3 y = 2x (ii) nABC and nDCE are similar.
(b) If AB = 35 cm, CD = 25 cm and BE = 42 cm,
2 find the length of BC.
A
1
D

x
_2 _1 O 1 2
_1 y=2_x
B C E

(b) There is only one solution.

259

04 FWS3A(Exp)_RevEx2.indd 259 12/16/14 1:02 PM


© Star Publishing Pte Ltd. All rights reserved.

Solution (d) Equation of the line having same gradient as A:


(a) (i) ∠CDE = ∠ACD = ∠BAC (alt. ∠s, AC // DE, y = – x + c
3
AB // DC) 4
(ii) ∠BAC = ∠CDE (found) When passing through C,
∠ABC = ∠DCE (corr. ∠s)
3
6 = – (–2) + C
\ nABC is similar to nDCE. (Angle- 4
Angle-Angle similarity) 6 =
3
+C
2
35 AB
(b) = (ratio of corr. sides of similar ns) C = 4
1
25 DC 2
7 1
= \ Equation of the line: y = – x + 4
3
5 4 2
BC 7
\ =
CE 5
BC 7
14. In the diagram, the curve y = (3 – x)(x + k) cuts the
= x-axis at points A and E, and cuts the y-axis at C(0, 6).
42 – BC 5
The equation of the line is y = mx + b. It cuts the y-axis
5BC = 294 – 7BC
at B(0, 2). The curve and the line intersect at A and D.

12BC = 294
Find
BC = 24.5 cm
(a) the values of constants b, k and m,
(b) the coordinates of the point E, where the curve cuts
13. The points A, B and C are (–5, 2), (3, –4) and (–2, 6) the negative part of the x-axis,
respectively. (c) the equation of the line of symmetry of the curve,
(a) Find the lengths of AB and BC. (d) the coordinates of the other point of intersection,
(b) Show that AB is perpendicular to AC. D, of the line and the curve.
(c) Find the area of nABC.
y
(d) Find the equation of the line passing through
C and having the same gradient as AB. C(0, 6)
y = (3 _ x)(x + k)
Solution
(a) Length AB = (–5 – 3) + [2 – (–4)]
2 2
D
B(0, 2)
= 100
E A
= 10 units O
x

y = mx + b
Length of BC = [3 – (–2)]2 + (– 4 – 6)2
= 125
= 11.2 units Solution
–4 – 2 (a) b is the y-intercept of the line AD.
(b) Gradient of AB = \ b = 2
3 – (–5)
6 Substituting c(0, 6) into y = (3 – x)(x + k)
=– 6 = (3 – 0)(0 + k)
8
3 6 = 3(k)
= – k=2
4
6–2 When y = 0,
Gradient of AC =
–2 – (–5) (3 – x)(x + k) = 0
4 3 – x = 0 or x + k = 0

=
3 x=3 x = –k
The gradient of AB is negative reciprocal of the Coordinates of A = (3, 0)
gradient of AC.
Gradient of AB =
2–0
\ AB is perpendicular to AC. (shown) 0–3
2
= –
(c) Length AC = [–5 – (–2)] + (2 – 6)
2 2
3
2
= 25 Equation of AB: y = – x+2
3
= 5 units 2
1 \ m=–
Area of nABC = 3 5 3 10 3
2
= 25 units2

Review Exercise 2
260

04 FWS3A(Exp)_RevEx2.indd 260 12/16/14 1:03 PM


© Star Publishing Pte Ltd. All rights reserved.

(b) y = (3 – x)(x + k) (c) By drawing a tangent, find the gradient of the graph
0 = (3 – x)(x + k) at the point where x = – 4.
\ 3 – x = 0 or x + 2 = 0 (d) Using the graph, solve the equation
x = 3 x = –2 8
1 x2 = + 10 for –6 < x , 0.
\ Coordinates of E = (–2, 0) 3 x

(c) x-coordinate of the line of symmetry


Solution
3 – (–2)
=3– (a) When y = p, x = –5,
2
1 8
=3–
5 p = – (–5)2 + + 10
2 3 (–5)
25 8
=
1

=– – + 10
2 3 5
Equation of the line of symmetry of curve: x =
1
= 0.07 (correct to 2 d.p.)
2
2 (b)
(d) (3 – x)(x + 2) = – x + 2
3 (_2.6, 5.6) y
2

3x + 6 – x – 2x = – 2
x +2
3
5
2
6 – x2 + x = – x +2
3
5 4
4 – x2 + x =0
3

x2 –
5
x –4=0 3
3
3x2 – 5x – 12 = 0
2

(3x + 4)(x – 3) = 0
3x + 4 = 0 or x – 3 = 0
3x = – 4  x= 3 (rejected) 1
4 (_5, 0.6)
x=–
3
x
4 _6 _5 _4 _3 _2 _1 O
When x = – ,
3
2  4 _1
y= – –  +2
3  3
8

= +2 _2
9
26

=
9 _3
8

=2
9
_4
 4
2 
8
Coordinates of D =  – ,
3 9
_
5
y = _ 13 x2 + 8
x + 10
15. Two variables x and y are related by the equation
_
1 8 6
y = – x2 + + 10. Some values of x and the
3 x
corresponding values of y, correct to 2 decimal places,
are given in the following table.

x –6 –5 –4 –3 –2 –1 –0.5 (c) Gradient of the graph =


5.6 – 0.6
–2.6 – (–5)
y –3.33 p 2.67 4.33 4.67 1.67 –6.08 5
=
3.6
(a) Find the value of p. =
25
(b) Using the scale of 2 cm to 1 unit on the x-axis and 12
1 2 8
1 cm to 1 unit on the y-axis, draw the graph of (d) x = + 10
3 x
1 8
y = – x + 2
+ 10 for – 6 < x < – 0.5. 0 = – x2 +
1 8
+ 10
3 x
3 x
Draw the line y = 0 on the graph. The solutions
are x = –5 and x = – 0.8.

261

04 FWS3A(Exp)_RevEx2.indd 261 12/16/14 1:03 PM


© Star Publishing Pte Ltd. All rights reserved.

Review Exercise 2
262

04 FWS3A(Exp)_RevEx2.indd 262 12/16/14 1:03 PM

You might also like